Sei sulla pagina 1di 221

Only about 1/3 of the number of pages in a typical LSAT Guide

Book is devoted to LOGICAL REASONING.

THIS IS THE FIRST BOOK DEDICATED ENTIRELY


TO THIS SUBJECT!

Again, the other guide books usually explain WHY A


PARTICULAR CHOICE IS THE CORRECT ANSWER TO THE GIVEN
QUESTION ON AN EX POST FACTO REASONING.

They do not guide you step-by-step on HOW TO ARRIVE AT


THE CORRECT ANSWER TO THE GIVEN QUESTION BY YOURSELF,
WHICH IS WHAT YOU ARE EXPECTED TO DO WHEN YOU TAKE
THE TEST.

This book teaches you just that - THE STEP-BY-STEP


PROCESS OF ARRIVING AT THE CORRECT ANSWER, that too IN
THE QUICKEST POSSIBLE TIME!

AND WITH AUTHENTIC EXAMPLES OF THE SAME


PATTERNS AND LEVELS OF DIFFICULTY AS THOSE
ASKED IN LSAT!

THAT IS WHY THIS IS


THE ULTIMATE GUIDE
TO LOGICAL REASONING
FOR LSAT!!
CONTENTS
Chapter 1: Introduction 3

Chapter 2: 'Conclusion' Questions 7

Chapter 3: 'Inference' Questions 23

Chapter 4: 'Assumption' Questions 34

Chapter 5: 'Weakens' Questions 50

Chapter 6: 'Strengthens' Questions 76

Chapter 7: 'Completion' Questions 92

Chapter 8: 'Evidence Analysis' Questions 96

Chapter 9: 'Relevance' Questions 100

Chapter 10: 'Argumentation' Questions 105

Chapter 11: 'Paradox' Questions 110

Chapter 12: 'Analogy' Questions 114

Chapter 13: 'Multiple' Questions 116

ANSWERS & ANALYSIS

Chapter 2: 'Conclusion' Questions 128

Chapter 3: 'Inference' Questions 142

Chapter 4: 'Assumption' Questions 149

Chapter 5: 'Weakens' Questions 160

Chapter 6: 'Strengthens' Questions 185

Chapter 7: 'Completion' Questions 198

Chapter 8: 'Evidence Analysis' Questions 201

Chapter 9: 'Relevance' Questions 203

Chapter 10: 'Argumentation' Questions 206

Chapter 11: 'Paradox' Questions 209

Chapter 12: 'Analogy' Questions 212

Chapter 13: 'Multiple' Questions 213


LSAT - LOGICAL REASONING
Chapter 1
INTRODUCTION
The multiple-choice sections of Law School Admission Test (LSAT) feature 3 different types of questions –
Reading Comprehension, Analytical Reasoning and Logical Reasoning.
The following are the numbers of questions under each type:
Reading Comprehension: 26 or 27
Analytical Reasoning: 24
Logical Reasoning (2 sections): 25 or 26 each
There will be an additional section in each test containing questions from any of the above 3 types. Known
as the experimental or the variable section, the purpose of this additional section is to pretest questions for
possible inclusion in future tests. Your performance in this section is not taken into consideration for your
score. You will, however, not be told which among the 5 sections is the variable section, and you will have to
answer each of them with equal seriousness.
You are allowed 35 minutes to answer each section in LSAT..
Your score in LSAT is given in the range 120 to 200.

This book deals with the Logical Reasoning questions in LSAT.

These questions are all of ‘multiple choice’ type.

The purpose of these questions is NOT to test your proficiency in any specific subject such as mathematics,
physics, economics, psychology, politics etc.
What they seek to do broadly is to test your capacity to understand, analyze and evaluate arguments.
A specific question may be framed so as to test your capacity to recognize the point of an argument, recognize
the conclusions that may be drawn from an argument, recognize the assumption on which an argument is based,
evaluate an argument or a counterargument, analyze evidence and to recognize the logical pattern of an argument.

Each question starts with a short passage which may contain some facts, some opinions, some assumptions
or some conclusions, and is followed by a question that has a bearing on what is contained in that passage. You
have to choose the best answer to the question from among the five given choices.

The following are some typical questions under Critical Reasoning in LSAT.

1. Many people argue that tobacco advertising plays a crucial role in causing teen-agers to start or continue
smoking. In Norway, however, where there has been a ban on tobacco advertising since 1975, smoking is at least
as prevalent among teen-agers as it is in countries that do not ban such advertising.
Which of the following statements draws the most reliable conclusion from the information above?
(A) Tobacco advertising cannot be the only factor that affects the prevalence of smoking among teen-agers.
(B) Advertising does not play a role in causing teen-agers to start or continue smoking.
(C) Banning tobacco advertising does not reduce the consumption of tobacco.
(D) More teen-agers smoke if they are not exposed to tobacco advertising than if they are.
(E) Most teen-agers who smoked in 1975 did not stop when the ban on tobacco advertising was implemented.

2. Currently people in the United States eat, on the average, 1,431 pounds of food per year, 35 pounds more
than in 1980. This increase is, at least in part, because people between the ages of 15 and 64 have accounted for
an increasing share of the population.
Which of the following can be properly inferred from the passage above?
(A) More than half of the current population of the United States is between the ages of 15 and 64.
(B) The population has risen since 1980.
(C) Children below the age of 15 require, on the average, more food than do people over the age of 64.
(D) Before 1980, children below the age of 15 outnumbered people between the age of 15 and 64.
(E) Individuals between the ages of 15 and 64 consume, on the average, more food than do those younger or
older.
American Education Aids - LSAT - Logical Reasoning
3. Child’s World, a chain of toy stores, has relied on a “supermarket concept” of computerized inventory
control and customer self-service to eliminate the category of sales clerks from its force of employees. It now
plans to employ the same concept in selling children’s clothes.
The plan of Child’s World assumes that
(A) supermarkets will not also be selling children’s clothes in the same manner
(B) personal service by sales personnel is not required for selling children’s clothes successfully
(C) the same kind of computers will be used in inventory control for both clothes and toys at Child’s World
(D) a self-service plan cannot be employed without computerized inventory control
(E) sales clerks are the only employees of Child’s World who could be assigned tasks related to inventory
control

4. Caterpillars of all species produce an identical hormone called “juvenile hormone” that maintains feeding
behavior. Only when a caterpillar has grown to the right size for pupation to take place does a special enzyme
halt the production of juvenile hormone. This enzyme can be synthesized and will, on being ingested by immature
caterpillars, kill them by stopping them from feeding.
Which of the following, if true, most strongly supports the view that it would not be advisable to try to eradicate
agricultural pests that go through a caterpillar stage by spraying crop lands with the enzyme mentioned above?
(A) Most species of caterpillar are subject to some natural predation
(B) Many agricultural pests do not go through a caterpillar stage
(C) Many agriculturally beneficial insects go through a caterpillar stage
(D) Since caterpillars of different species emerge at different times, several sprayings would be necessary
(E) Although the enzyme has been synthesized in the laboratory, no large-scale production facilities exist as
yet

5. Corporations exist to make money for their shareholders. Corporations have no money to give away because
any excess funds belong to the shareholders. Therefore, corporations should not contribute to charities.
Which of the following most seriously weakens the reasoning in the argument above?
(A) Managers are being trained to consider the ethical, moral, and social effects of their decisions in addition
to the economic factors
(B) Some of the largest corporations in the country are known to be major contributors to philanthropic
organizations
(C) Shareholders tend to make larger individual donations to charities than do people who hold no shares in
corporations
(D) Most corporations contribute solely to large, well-known, and highly organized charitable organizations
rather than to individuals in need
(E) The favourable publicity and tax advantages gained by corporate charity often result in an increase in
profits greater than the actual costs of making the donations

6. According to an independent computer-industry analyst, the new Regent microcomputer is of high quality,
is fast, and costs less than any currently existing competing model. It is reasonable to conclude, therefore, as the
manufacturer’s prospectus does, that the Regent will quickly establish itself as a fast-selling, low-priced alternative
to currently available microcomputers.
Which of the following, if true, would LEAST weaken the argument above?
(A) Many retailers already carry one or more low-priced microcomputer models and are disinclined to carry
another.
(B) Several faster and lower-period models of microcomputers will soon be introduced by other computer
manufacturers.
(C) The Regent Corporation’s microcomputer can be used in conjunction with higher-priced microcomputers
manufactured by other companies.
(D) Most of those individuals and companies that could be expected to make up the potential market for the
Regent microcomputer have already filled their microcomputer needs.
(E) The independent computer-industry analyst whose assessment was incorporated in the prospectus has
used measures of quality that are not universally accepted by the computer-buying public.

7. The people who are most at risk of contracting a particular type of influenza are those who have never
previously contracted that type of influenza and have never been vaccinated for it. Cases of type A influenza
have been recently diagnosed again in the United States. Type A influenza was last found in the United States in
1950.
4
American Education Aids - LSAT - Logical Reasoning
Which of the following groups of people have the LEAST risk of contracting type A influenza?
(A) People who were not vaccinated for any type of influenza in 1950 and did not contract any type of influ-
enza
(B) People who have previously contracted a different type of influenza, but not type A
(C) People who were vaccinated for a different type of influenza in 1950, but not for type A
(D) People who had type A influenza in 1950, but were not vaccinated for it
(E) People born after 1950 who were exposed to different types of influenza, but who never contracted any
type of influenza

8. The United States imprisons more people, both in absolute numbers and per capita, than any other civi-
lized Western society. But it has a higher crime rate than any of them. This shows that imprisonment contributes
to a high incidence of crime.
Which of the following is the most serious weakness in the argument above?
(A) It may be confusing cause and effect
(B) It lacks substantiating statistics
(C) It fails to differentiate among various classes of crime
(D) It rests on the assumption that all Western societies are civilized
(E) It compares phenomena that may not be comparable

9. It is true that it is against international law to sell plutonium to countries that do not yet have nuclear
weapons. But if United States companies do not do so, companies in other countries will.
Which of the following is most like the argument above in its logical structure?
(A) It is true that it is against the police department’s policy to negotiate with kidnappers. But if the police
want to prevent loss of life, they must negotiate in some areas.
(B) It is true that it is illegal to refuse to register for military service. But there is a long tradition in the
United States of conscientious objection to serving in the armed forces.
(C) It is true that it is illegal for a government official to participate in a transaction in which there is an
apparent conflict of interest. But if the facts are examined carefully, it will clearly be seen that there was no
actual conflict of interest in the defendant’s case.
(D) It is true that it is against the law to burglarize people’s homes. But someone else certainly would have
burglarized that house if the defendant had not done so first.
(E) It is true that company policy forbids supervisors to fire employees without two written warnings. But
there have been many supervisors who have disobeyed this policy.

10. Which of the following best completes the passage below?


The computer industry’s estimate that it loses millions of dollars when users illegally copy programs without
paying for them is greatly exaggerated. Most of the illegal copying is done by people with no serious interest in
the programs. Thus, the loss to the industry is much smaller than estimated because .........
(A) many users who illegally copy programs never find any use for them
(B) most of the illegally copied programs would not be purchased even if purchasing them were the only way
to obtain them
(C) even if the computer industry received all the revenue it claims to be losing, it would still be experiencing
financial difficulties
(D) the total market value of all illegal copies is low in comparison to the total revenue of the computer
industry
(E) the number of programs that are frequently copied illegally is low in comparison to the number of programs
available for sale

You can notice that questions under this category are of various types:
Example 1: What is the conclusion that can be logically drawn from the given information?
Example 2: What is the inference that can be drawn from the given information?
Example 3: What is the assumption that has been made while coming to a particular conclusion based on
a set of facts?
Example 4: Which choice, if true, further strengthens the conclusion that has been drawn based on the
given facts?
Example 5: Which choice, if true, weakens the conclusion that has been drawn based on the given facts?
Example 6: Which of the choices least weakens the conclusion in the passage?
5
American Education Aids - LSAT - Logical Reasoning
Examples 7 and 8: These expect you to evaluate the information in the given passages in order to choose the
correct answer to the questions given under them.
Example 9: This asks you to identify the choice that has the same argumentative structure as the given
passage.
Example 10: This asks to complete the given passage logically by choosing the most appropriate phrase from
among the given choices.

(You may try to answer the questions by yourself even before studying the lesson that follows. These questions
are answered and analysed in the appropriate chapters later.)

There can be no theoretical ‘lesson’ on Logical or Critical Reasoning, because it is not a ‘subject’ such as
mathematics, physics or economics, but is a ‘skill’ that you must have developed over a period of years.

The examples in the following chapters, grouped under different heads, will help you hone that skill and
enable you to answer the Logical Reasoning questions in LSAT correctly when you sit for it.

Answering the questions on Logical Reasoning involves not merely the spotting of the correct choice, but
also the discarding of the wrong choices.
While analysing the questions in order to arrive at the correct answers, we shall also point out why the other
choices are wrong.
We shall also point out the usual types of errors that are built into the wrong choices under each type of
questions.

For an effective understanding of our analysis of each question in this volume, you will have to
refer back to the questions themselves repeatedly. In order to make this process convenient for you,
we have organized the answers and analysis to these questions as a separate volume.

6
Chapter 2
'CONCLUSION' QUESTIONS
‘Conclusion questions’ in the Logical Reasoning section ask Example 2
you to identify the conclusion that can be logically arrived at In the normal progression of life, people die approximately
on the basis of the information in the given passage. in order of birth. Therefore, when a younger person
predeceases an older person, that death may be especially
The ‘conclusion questions’ may be worded in many different painful to the survivor, who feels that a law of nature has
ways such as: been violated.
(i) Which of the following conclusions can be properly drawn The argument above best supports which of the following
from the statements above? conclusions?
(ii) The statements above best support the conclusion that (A) When children die before their parents, the loss is
.... perhaps more intensely felt than when the parents are the
(iii) If the statements above are true, which of the following first to die.
must also be true? (B) It is perfectly normal for children whose parents
(iv) From the information above, it can be properly predecease them to feel little grief.
concluded that .... (C) Laws of nature are merely general descriptions, and
(v) The author of the above passage argues that .... as such they offer little consolation to those whose loved ones
(vi) Which of the following is best supported by the passage predecease them.
above? (D) When the older of two parents dies first, the grief of
the surviving parent and of the children is likely to be
We shall first give you a number of typical examples of especially intense.
‘conclusion questions’ and also analyse them for your benefit (E) Parents should not feel guilty of neglect when their
to guide you on how such questions should be tackled. children die before them.
Try to answer these questions by yourself first before
reading the analyses below. Analysis
What the statement says is that when a younger person
Example 1 dies, the grief of the older person who has survived would be
During the month of July in City X, the humidity was greater than if the older person had died first leaving the
always 80 percent or higher whenever the temperature was younger person to mourn his death.
75½ F or higher. Temperatures during that month ranged from (A) is the only suitable illustration of this statement, and
65½ to 95½ F. is the answer.
If the statements in the passage above are true, it can be The given statement does not mean that children will not
concluded which of the following CANNOT be an accurate grieve at all when their parents die - what it means is that
report of a temperature and humidity reading of City X in the grief of parents when the children die is greater than the
July? grief of children when the parents die. So, (B) is not a valid
(A) 77½F, 81% conclusion that can be drawn from the passage.
(B) 76½F, 80% The given statement says that a death is 'especially painful'
(C) 75½F, 79% to the survivor when the law of nature (of an older person
(D) 74½F, 78% dying before a younger person) is violated. This implies that
(E) 73½F, 77% when an older person dies before a younger person, the
grieving survivor consoles himself /herself with the realisation
Analysis that what has happened is in consonance with a law of nature
The information given is that, whenever the temperature and could not have been avoided. (C) contradicts this
in City X was 75½ or higher, the humidity was at least 80%. statement of the author, and is therefore not a conclusion
We have, in effect, been asked to spot that choice which from it.
contradicts this information. (D) directly contradicts the view of the author, and is not
(A) and (B) conform to the given information, and can be the answer.
discarded. (E) is too general a statement which may not be applicable
(C) contradicts this information and is the answer. to all cases of deaths of children. (For example, in a case where
The given passage deals with only those cases where the the doctor says that the ailing child had been brought too
temperature is 75° F or higher. (D) and (E) refer to the days late to him and that he could have saved its life if only it had
when the temperature was lower than 75°, and are irrelevant been brought to him a day earlier, the bereaved parents may
to the information in the passage. well feel guilty of neglect.) In any case, (E) goes beyond the
So, we should choose (C) as the answer. scope of what is stated in the passage which talks only about
(Note that the question in this case itself is stated in the grief and not about neglect.
negative. It asks you not to spot the choice which conforms to (Note that, among the choices, (B) and (E) are wrong
the given conditions, but the choice which violates the given because they extend the scope of the author's premise much
conditions. Also note that two of the answer choices are wrong beyond what he implies. Choices (C) and (D) contradict what
because they conform to the given conditions, and two others the author says in the passage.)
are wrong because they are irrelevant to the question. This
will be the general pattern of how the wrong choices
are constructed in such questions.)
7
American Education Aids - LSAT - Logical Reasoning
Example 3 drivers over the age of 25. So, (B) is not a conclusion that is
Macklin County’s mandatory driver-training program for warranted by the information in the narrative.
fifteen-year-old high school students was discontinued ten The passage merely implies that youngsters who had
years ago, and since then private driver-training program undergone a driver-training program are less likely to get
enrollment has increased only slightly. During the same involved in accidents than are those who have not undergone
period the county’s accident rate increased by 12 percent for such a program. Choice (C) which states that 'a majority of
drivers between the ages of sixteen and twenty-five, even youngsters who take some type of driver-training program
though traffic conditions changed little. do not get involved in accidents at all' goes much beyond the
Which of the following conclusions is best supported by the scope of what is stated in the passage, and is wrong.
information above? The narration specifically says that there had been little
(A) Graduates of public driver-training programs tend to change in the traffic conditions during the period covered by
be safer drivers than graduates of private driver-training the survey. (D), which refers to the contribution of changes
programs tend to be. in traffic conditions to the accident rates, is irrelevant to the
(B) Drivers between the ages of sixteen and twentyfive, on information in the given passage, and is not the answer.
average, have higher accident rates than do drivers over the (Note that, in this example, (A), (B) and (C) are wrong
age of twenty-five. because they go beyond the scope of what is stated in the
(C) The majority of people who take some type of driver- passage, while (D) is wrong because it is irrelevant to the
training program do not become involved in an accident when information in the passage.)
they are between the ages of sixteen and twenty-five
(D) Changes in traffic conditions affect the accident rate Example 4
of drivers over the age of twenty-five more than they do that A group of subjects saw a film of two cars colliding.
of drivers between the ages of sixteen and twenty-five Immediately afterward, half of the group were asked a set of
(E) As a group, drivers between the ages of sixteen and questions about the cars “bumping” into one another, while
twenty-five become involved in fewer accidents if they the other half were asked the same questions, but with the
participate in a driver-training program than if they do not. verb “smash” substituted for “bump”. In later descriptions of
the filmed collision, subjects in the latter half were more likely
Analysis to remember seeing broken glass.
(Mandatory means legally required or compulsory) The The experiment described above best supports which of the
first part of the first sentence gives the information that the following conclusions about eye-witness testimony?
Macklin County’s compulsory driver-training program for 15- (A) A witness who is agitated at the time of an event is
year old students was discontinued ten years ago. likely to give less accurate testimony than is a calm witness.
The second part of the sentence adds that, since then, (B) A witness’s perception of an event will be distorted if
enrollment in the (voluntary) private driver-training program inflammatory language is used by the questioner.
in the County has shown only a slight increase. (C) The manner in which a witness is questioned after an
The second sentence says that, during the last ten years event can influence the recollection of the witness.
(after the discontinuance of the compulsory requirement), the (D) Most eyewitness testimony can be assumed to contain
County’s accident rate for drivers between the ages of 16 and inaccurate elements.
25 increased by 12%, even though traffic conditions had (E) Special questioning techniques can be developed that
changed little. will be more conducive to eliciting accurate information from
We have been asked to identify the choice that forms the witnesses than traditional techniques have been.
conclusion best supported by the given information.
The obvious conclusion that can be drawn from the given Analysis
information is that, if there is a compulsory driver-training (In this question, the word ‘subjects’ in the first line means
program for 15 year old students, drivers between the ages ‘persons’) What the narrative states is that a number of
of 16 and 25 get involved in fewer accidents. persons were first shown a film depicting the collision of two
In other words, drivers between the ages of 16 and 25 who cars. Subsequently, they were divided into two groups, and
had undergone a driver-training program get involved in those in the first group were given a questionnaire in which
fewer accidents than those who had not undergone such a the word 'bump' was used, while those in the second group
program. were given the same questionnaire with a single variation -
It is (E) which says this, and is the answer. the word 'bump' having been substituted by the word 'smash'.
(A) would have been correct if the private driver-training The narrative then states that the persons in the second
programs had attracted all the teenagers who would have group, for whom the word ‘smash’ was suggested, were more
otherwise been trained by the compulsory public driver likely to remember seeing glass having been broken during
training program. But the second part of the first sentence the collision.
of the narrative specifically states that private driver-training What this means is that though two witnesses had seen
program had attracted very few additional enrollments during the same event, their recollection of that event can be
this period, implying that many of those who had been influenced by the manner in which questions are put to them.
involved in subsequent accidents had not undergone any So, (C) is the answer.
training programs at all. So, (A) is not a conclusion that is (A) talks of two different witnesses one of whom was
warranted by the information in the narrative. agitated at the time of seeing the event while the other was
The narration compares the accident rates involving only calm. But the passage talks of a single group of persons who
drivers between the ages of t6 and 25 before and after the were all shown the same film of two cars colliding. This single
discontinuance of the mandatory driver-training program for group was only subsequently divided into two groups just for
the 15-year old students. It does not compare the accident the sake of being given two different questionnaires
rates for drivers younger than 25 with the accident rates for containing just one significant variation. There is no evidence
8
American Education Aids - LSAT - Logical Reasoning
in the passage that the first of these groups had been agi- conclude that swimming in these lakes can cause respiratory
tated even while seeing the film while the other had remained and intestinal illnesses. (You cannot conclude that swimming
calm. So, (A) is irrelevant to the information in the passage. in these lakes will cause these illnesses because, after all,
Even assuming that the word 'smash' is more inflammatory the increase in percentage of swimmers who are affected is a
than the word 'bump', the result of the experiment described mere 3%, and 93% of the swimmers are found to be not
in the passage is that those to whom the word 'smash' was suffering from these diseases.)
suggested were more likely to remember seeing broken glass, But you do not know how many beaches there are along
implying that they would be more accurate than the other the shores of these two Great Lakes, how many thousands of
group for whom the word 'bump' had been suggested. So, (B) persons are using these beaches and whether 10 beaches and
contradicts what is stated in the passage, and is not the 8,000 people are large enough and representative samples
answer. (If the last sentence in the passage were, "In later from which a conclusive inference can be drawn about all
descriptions of the filmed collision, subjects in the latter half those who swim in these lakes from all the other beaches along
were more likely to imagine seeing broken glass", this choice their shores. (For example, if these 10 beaches are located
would have been the correct answer.) near factories which pollute the water in the region of the
The statement in (D), "Most eyewitness testimony can be lake near them, the phenomenon may be restricted to the
assumed to contain inaccurate elements", is a sweeping water near these beaches alone, and not to the entire
generalisation which cannot follow as a necessary conclusion waterspread.)
from a single experiment that is described in the passage. Therefore we can, at best, conclude from the given data
(All 'conclusion' questions will be based on the narration of a that there is some evidence of a cause and effect relationship
single event having a limited scope, and any such sweeping between swimming in the two lakes and the incidence of
generalisation in one of the choices is likely to render it wrong.) respiratory and intestinal diseases (implying that a further
(E) extends the scope of the result of the experiment much detailed study may have to be undertaken before such a
beyond what is specifically stated in it. So, though what is relationship could be established as true).
stated in (E) may be true, it does not follow immediately from (D) is what makes such a tentative statement, and is the
the given passage, and is not the answer. answer.
(Note that, among the wrong choices in this example, (A) Even the sample study shows that, between non-swimmers
is irrelevant to the information in the passage, (B) contradicts and swimmers in these lakes, the difference in the incidence
the given information, (D) is an unwarranted and sweeping of respiratory and gastrointestinal illnesses is only 3% and
generalisation and (E) extends the scope of the given that, even among the swimmers, nearly 93% are not affected
information beyond what can be reasonably inferred.) by these illnesses. Nor does the passage indicate that these
illnesses are serious in nature, as otherwise the affected
Example 5 persons will not be continuing with their swimming.
In a study of more than 8,000 people using ten beaches on Therefore, at least for a large percentage of the people,
two of the Great Lakes, ecologists from the University of swimming in these lakes does not constitute a risk at all , and
Toronto determined that the rate of respiratory and the question of people underestimating the risk does not arise.
gastrointestinal illness among people who had been So, (A) is a sweeping generalisation not warranted by the
swimming was 69.6 per 1000, whereas the respiratory and information in the passage.
gastrointestinal illness rate among those who had not been The sample study was conducted only among people who
swimming was only 29.5 per 1,000. were swimming from ten beaches on two of the three Great
Which of the following conclusions can be most properly Lakes. The findings of this study have no relevance to
drawn from the data above? swimmers in other lakes. (B) seeks to extend the scope of what
(A) People tend to underestimate the risks of swimming is stated in the passage beyond what immediately follows from
in these lakes. it, and is not the answer.
(B) Respiratory and gastrointestinal illnesses occur at a The study that is referred to was limited only to two specific
higher rate as a result of swimming in either of these lakes illnesses, and cannot throw any light on whether or not any
than they do as a result of swimming in any other lake. other illness is associated with swimming in these two Great
(C) Illnesses of kinds other than respiratory and Lakes. (C) also seeks to extend the scope of what is stated in
gastrointesinal are not likely to be associated with swimming the passage beyond what immediately follows from it, and is
in either of these two lakes. not the answer.
(D) The association between swimming in these lakes and (E) talks generally of 'diseases that swimming may cause'
respiratory and gastrointestinal illness is some evidence for whereas the study was restricted to only two diseases -
a causal relationship between them. respiratory and gastrointestinal. So, (E) is in the nature of a
(E) A large percentage of the people who swim in these sweeping generalisation which is not warranted by the
lakes are immune to the diseases that swimming may cause. information in the passage.
(Note that, among the wrong choices, two are sweeping
Analysis and unwarranted generalisations and two try to extend the
(The Great Lakes of USA are Lake Superior, Lake Michigan scope of what is stated in the passage.)
and Lake Huron.) The passage says that a sample study of
8,000 persons using ten beaches of two of the Great Lakes Example 6
showed that nearly 7% of the swimmers among them had “The hardest known mineral is diamond; the second
respiratory and gastrointestinal illnesses, while only 3% of hardest is corundum.
the non-swimmers among them had these illnesses. We have A mineral can be scratched only by minerals as hard as or
been asked to spot that choice which can be a logical harder than itself.”
conclusion that can be validly drawn from this study. If the statements above are true, which of the following must
From the given information, you may be tempted to be true of a known mineral that can be scratched by corundum?
9
American Education Aids - LSAT - Logical Reasoning
I. The mineral is not diamond. (A) S weighs the same as U weighs
II. The mineral is not corundum. (B) S is heavier than T
III. The mineral is not as hard as corundum. (C) T is heavier than Q
(A) I only (D) T is heavier than U
(B) III only (E) U is heavier than M
(C) I and II only
(D) I and III only Analysis
(E) I, II, and III (Questions of the above type are easier to answer if you
Analysis rewrite the descriptive sentences using the mathematical signs
(This is a special type that is found occasionally among '>' to mean 'greater than' and '<' to mean 'less than'.)
Logical Reasoning / Critical Reasoning questions. ) For this question, let us use the ‘>’ symbol to denote
The given passage means that a piece of corundum can ‘heavier than’, and the symbol ‘<‘ to denote ‘lighter than’.
scratch (i) another piece of corundum, or (ii) a piece of another Then, the first statement can be rewritten as M > Q, M <
mineral which is not as hard as corundum, but (iii) it cannot R.
scratch diamond. Since using two different symbols can lead to confusion,
So, if a known mineral can be scratched by corundum, we rewrite the above information using only one symbol '>': M >
can conclude that (i) it is not diamond; (ii) it may be another Q and R > M. These two can be combined as R > M > Q. (i)
piece of corundum or (iii) it is a mineral which is not as hard The second statement can be written as S > Q and S > R.
as corundum. (ii)
Note that the question that is asked is ‘Which of the three (These cannot be combined because we do not know whether
choices must be true?’ Q is heavier or lighter than R.)
So, of the three given possibilities, I must necessarily be The third statement can be written as U > Q and U > R.
true; II need not be true, because the piece could very well be (iii)
corundum; and III also need not be true, because the piece (We cannot combine these two because we do not know
could be corundum itself. whether Q is heavier or lighter than R.)
So, it is only I which must be true. Since U > R according to (iii), we can combine (i) and (iii)
It is (A) which states this, and is the answer. to write U > R > M > Q.
From (i), (ii) and (iii), we can deduce that each of S, U and
Example 7 R is heavier than M.
M is heavier than Q, but it is lighter than R. So, if we know that T is heavier than any one of these three,
S is heavier than Q and it is also heavier than R. then we can conclude that T is heavier than M also.
U is heavier than Q and it is also heavier than R. (D) says that T is heavier than U, and is the answer.
If the statements above are true, one can conclude with You can easily see that none of the other choices will enable
certainty that T is heavier than M if one knows in addition us to conclude that T > M.
that

You can notice from the examples discussed above that even the wrong choices given underneath a question
are not arbitrarily worded, but follow a specific pattern.
Therefore, while evaluating each of the answer choices, ask yourself the questions:
(i) Does this choice contradict a specific information in the passage?
(ii) Does the choice extend the scope of what is stated in the passage beyond what can be immediately
inferred?
(iii) Does the choice make a sweeping generalisation from a single instance referred to in the passage?
(iv) Is the statement in this choice irrelevant to the information in the passage?
If the answer to any of these questions is 'yes', that choice is bound to be a wrong answer, and can
be discarded.

Most of the Logical Reasoning questions can be answered mentally without your having to do any rough
calculations on paper.
But, for answering some questions (such as those involving the use of ‘<‘ or ‘>’ signs, or a little bit of arithmetic),
you may have to use a pencil and a paper. You will be given blank sheets for this purpose in the examination
hall.

Logical Reasoning questions are carefully and tightly worded. Each word used in them, whether it occurs in
the initial narration or in the question or in the answer choices, is carefully chosen. You must, therefore, read
every word carefully and understand it before attempting to answer the question.
Questions in LSAT should not only be answered correctly, but should be answered within the given
time limit. Try to answer each set of 10 questions below within 20 minutes. Later, after you get used to
them, you should try to answer them within 15 minutes.
After answering each set, verify your answers with the correct answers in the companion volume, and also
read and understand the analysis underneath before attempting to answer the next set of questions.
10
American Education Aids - LSAT - Logical Reasoning
SET 1
1. A package is never accepted for delivery by the delivery agers. Beginning at age 35, the stored calcium is used without
service unless it is within the established size limits. All being replenished. The more bone mass an individual has,
packages accepted for delivery by the delivery service have a the less likely it is that the individual will develop
return address. osteoporosis, a disease that makes bones brittle, especially
If the statements above are true, which of the following must in people over age 50.
also be true? Which of the following is a conclusion that can be properly
(A) The delivery service charges more for heavier packages drawn from the statements above?
than for lighter packages. (A) People who steadily increase the amount of calcium
(B) The delivery service will always accept for delivery a they consume from age 35 on are less likely to suffer
package that is within the established size limits. osteoporosis than are people who do not steadily increase the
(C) If a package is within the established size limits and amount of calcium they consume from age 35 on.
has a return address, it will be accepted for delivery by the (B) People should steadily increase the amount of calcium
delivery service no matter how heavy the package is. they consume from ages 35 to 50 to prevent osteoporosis.
(D) A package that is not within the established size limits (C) People who consume the recommended levels of calcium
but has a return address is never accepted for delivery by the during their teen years are less likely to suffer osteoporosis
delivery service. than are people who increase their consumption of calcium
(E) The delivery service does not charge for packages that to the recommended levels between the ages of 35 and 50.
must be returned to the sender. (D) People who are at least 50 years old and who suffer
from osteoporosis can be cured by immediately increasing
2. Each increase of 1 percent in real disposable personal the amount of calcium they consume.
income per capita will increase the share of the electorate for (E) People who are at least 50 years old and who suffer
an incumbent by about 2.2 percentage points, other things from osteoporosis did not consume calcium when they were
being equal. Since 1952 there has been a decline in real teen-agers
disposable income during only one presidential election year.
The incumbent lost that election. 5. Carol is shorter than Juan, but she is taller than Ed.
Which of the following conclusions can be properly drawn Sandra is shorter than Juan, and she is shorter than Ed.
from the statements above? Wallie is taller than Sandra, but shorter than Juan.
(A) When an incumbent runs for office, he or she is likely If the statements above are true, one can validly conclude
to win. that Bill is shorter than Carol if it is true that
(B) Political parties should take care to put forth a (A) Carol is equal in height to Wallie
candidate who seems prosperous. (B) Wallie is equal in height to Bill
(C) Presidential candidates should put their greatest efforts (C) Bill is taller than Sandra, but shorter than Wallie
into improving their public image. (D) Bill is shorter than Juan, but taller than Ed
(D) Because a presidential campaign requires the (E) Wallie is taller than Bill, but shorter than Ed
expenditure of large amounts of money, it frequently
impoverishes a candidate and his or her supporters. 6. Clay absorbs radiation with time, releasing it only when
(E) The outcome of a presidential election is substantially heated. By heating a clay sculpture and measuring the
affected by factors other than the ideological positions of the radiation it releases, experts can determine to within a
candidates. century when the sculpture was last heated. The original
firing of the finished sculpture might be the occasion of that
3. A group of scientists studying sound patterns in certain most recent heating.
monkeys found that the adult monkeys had distinct alarm Experts who obtain the year A.D. 1450 as an estimate for a
calls for eagles, leopards, and snakes. The infant monkeys, given sculpture using the method described above would
however, used the eagle alarm for any flying animal, the thereby most seriously undermine any claim that the sculpture
leopard alarm for any walking animal, and the snake alarm was made in
for any long, thin animal. (A) A.D. 1000
Which of the following is the most logically defensible (B) A.D. 1400
explanation of the monkeys’ behavior? (C) A.D. 1450
(A) Neither the adult monkeys nor the infant monkeys (D) A.D. 1500
observed in the experiment had speech of any sort (E) A.D. 1900
(B) The infant monkeys had yet to learn to distinguish
one kind of animal from another within the three groups 7. “To be a good debater, one must be intelligent. Some
(C) The infant monkeys mimicked the adult monkeys with good debaters, however, are also contentious, and contentious
no understanding of the significance of the three different persons are always boring”
sounds Which of the following conclusions can be properly drawn
(D) The infant monkeys had not yet learned to make the from the statements above?
three alarm calls as well as the adult monkeys could (A) All good debaters are boring
(E) Only the adult monkeys recognized that eagles, (B) All contentious persons are good debaters
leopards, and snakes posed dangers to them (C) Only good debaters are contentious
(D) Some intelligent persons are boring
4. Calcium consumed in recommended levels is stored by (E) Most intelligent persons are boring
bones and so helps increase bone mass, especially for teen-

11
American Education Aids - LSAT - Logical Reasoning
8. “Dryden makes deliveries only if he has his employer’s (A) The speed limit alone is probably not responsible for
permission. Dryden’s employer permits Dryden to make the continued reduction in highway deaths in the years after
deliveries alone during day-light hours, but permits him to 1974
make deliveries after dark, only if both Stan and Estey are (B) People have been driving less since 1974
with him.” (C) Driver-education courses have been more effective since
Which of the following conclusions can be logically inferred 1974 in teaching drivers to drive safely
from the statements above? (D) In recent years highway patrols have been less effective
(A) If it is dark and Estey is with Dryden, then Dryden in catching drivers who speed
will make deliveries. (E) The change in the speed limit cannot be responsible
(B) If it is dark and Dryden is making deliveries, then Stan for the abrupt decline in highway deaths in 1974
is with him.
(C) If Stan is not with Dryden, then Dryden is not making 10. The country’s continued existence as a nation depends
deliveries. on its taking active measures to preserve national security.
(D) If Dryden is making deliveries, then Stan and Estey Some of the measures may not be morally acceptable, but
are with him. they must be undertaken, nevertheless.
(E) If Dryden is making deliveries alone, then it is before If the statements above are true, it follows that
6.00 p.m (A) It is impossible to act morally in foreign policy.
(B) Each country is morally obliged to defend its own
9. After the national speed limit of 55 miles per hour was
security.
imposed in 1974, the number of deaths per mile driven on a
(C) Active measures to preserve national security must be
highway fell abruptly as a result. Since then, however, the
average speed of vehicles on highways has risen, but the immoral.
number of deaths per mile driven on a highway has continued (D) The pursuit of continued national existence is morally
to fall. justifiable.
Which of the following conclusions can be properly drawn (E) Adherence to morality is less important than ensuring
from the statements above? the country’s security.

SET 2
1. Women generally do not receive lower pay for doing (C) In fact, their parents’ low expectations for them often
precisely the same work as men. Rather, regardless of their are a major factor inducing young people to use marijuana
skills, women have disproportionately high representation heavily
in jobs traditionally paying lower wages. If jobs paid according (D) In other words, heavy use may be a symptom of
to the skills and education required by them, this de facto depression and low self-esteem rather than a cause
segregation would not exist. (E) Moreover, some studies indicate that heavy users of
If the statements above are true, which of the following is marijuana experience a broadening of interests and a gain of
most likely to be true? motivation
(A) Some jobs performed by more men than women pay
higher wages than jobs requiring comparable skills, but 3. The university constitution defines the powers of two
performed by more women than men governing bodies. The general council, which currently retains
(B) Regardless of sex or occupation, a person should be authority over constitutional matters, consists of all the
paid an amount sufficient to support his or her household university’s living alumni. The twenty-member senate decides
(C) As men enter a field, earnings tend to rise, not only for routine matters by majority vote. A new proposal suggests
men but also for the women in the field making a unanimous vote in the senate sufficient to change
(D) Little percentage gain has occurred in the number of the constitution.
women holding jobs traditionally held by men If the statements above are true, which of the following must
(E) Generally accepted methods are available for follow on the basis of them?
evaluating the level of skill required in, and hence the (A) The proposal will take effect only if it receives the
appropriate pay for, any job traditionally held by women senate’s unanimous support.
(B) Currently each member of the senate has power of veto
2. Marijuana has been blamed for causing apathy, loss of over any proposal that comes before that body.
motivation, narrowing of interests, and similar signs of (C) Adopting the proposal would allow passage of
depression in people who use it heavily. There is no evidence constitutional changes without the general council’s approval.
that the drug does indeed produce such an “a motivational (D) Any of the university’s alumni can fill any one of the
syndrome”. On the contrary, young people who are depressed twenty seats on the senate.
and have low expectations for themselves - and whose parents (E) Adapting the new proposal would increase the voice of
expect little of them - are more likely than others to make the alumni in the conduct of university affairs.
heavy use of marijuana.
Which of the following would provide the most logical 4. The greater the division of labor in an economy, the
concluding sentence for the paragraph above? greater the need for coordination. This is because increased
(A) Therefore, it can be concluded that marijuana does not division of labor entails a larger number of specialized
produce an amotivational syndrome producers, which results in a greater burden on managers
(B) Hence, it can be concluded that the evidence concerning and, potentially, in a greater number of disruptions of supply
the amotivational syndrome is at best inconclusive and production. There is always more division of labor in

12
American Education Aids - LSAT - Logical Reasoning
market economies than in planned economies. belts.
If all of the statements above are true, then which of the
following must also be true? 8. Most canvases used by artists in the pre-industrial
(A) Disruptions of supply and production are more frequent United States were imported from Europe, but the wooden
in planned economies than in market economies. stretchers on which the canvas was mounted were always
(B) There are more specialized producers in planned made from American wood. Retailers of art supplies, both in
economies than in market economies. the United States and Europe, often stamped blank canvases
(C) The need for coordination in market economies is with their names and addresses. Completed paintings brought
greater than in planned economies. from Europe to America were frequently taken off their
(D) A manager’s task is easier in a market economy than European stretchers for shipping and remounted on American
in a planned economy. stretchers after their arrival.
(E) Division of labor functions more effectively in market A cultural historian could legitimately use the information
economies than in planned economies. in the passage above in establishing that a painting of that
era done on
5. “Investigators studying the facial expressions of children (A) canvas sold by a European retailer and mounted on a
who are born both blind and deaf have found that certain stretcher of American wood is probably an American painting
patterns of muscular movement, such as those involved in (B) canvas sold by an American retailer and mounted on a
smiling, are similar in form to the patterns of muscular stretcher of American wood is probably an American painting
movement in the expressions shown by children without these (C) canvas sold by an American retailer and not mounted
handicaps.” on a stretcher is probably not an American painting
Which of the following conclusions can most reliably be (D) unmarked canvas mounted on a stretcher of American
drawn from the above information? wood is probably an American painting
(A) Facial expressions are not part of the language of (E) unmarked canvas that is not mounted on a stretcher
gesture that is taught by each society to its children is probably not an American painting
(B) The facial expressions found to be similar are the same
in human beings everywhere. 9. For a local government to outlaw all strikes by its
(C) The facial expressions found to be similar are not workers is a costly mistake, because all its labor disputes
always learned by imitation of sight or sound. must then be settled by binding arbitration, without any
(D) Facial expressions denoting positive emotions, such negotiated public-sector labor settlements guiding the
as smiling, are more easily learned than those denoting arbitrators. Strikes should be outlawed only for categories of
negative emotions. public-sector workers for whose services no acceptable
(E) The children who smiled were classifiable as normal substitute exists.
for educational purposes. The statements above best support which of the following
conclusions?
6. “Joe: All Americans like ice-cream. (A) Where public-service workers are permitted to strike,
James: No, that is not true, I know a couple of Americans contract negotiations with those workers are typically settled
who just love chocolates”. without a strike.
From the above conversation, we can say that James has (B) Where strikes by all categories of public-sector workers
misunderstood Joe to mean are outlawed, no acceptable substitutes for the services
(A) Only Americans like ice-cream provided by any of those workers are available.
(B) Joe does not like ice-cream (C) Binding arbitration tends to be more advantageous for
(C) Americans like only ice-cream public-service workers where it is the only available means
(D) America does not produce chocolates of settling labor disputes with such workers.
(E) Those who like ice-creams will not like chocolates (D) Most categories of public-sector workers have no
counterparts in the private sector.
7. “If a car was built after 1965, it has harness-style seat (E) A strike by workers in a local government is unlikely
belts in the front seats”. to be settled without help from an arbitrator.
From which of the following choices does the statement
above logically follow? 10. If it is true that the streets and the sidewalks are wet
(A) Harness-style seat belts first appeared in cars in the whenever it is raining, which of the following must also be
late 1950’s. true?
(B) No seat belts other than harness-style seat belts were I. If the streets and sidewalks are wet, it is raining.
used in front seats of cars after 1965, but, all cars built after II. If the streets are wet but the sidewalks are not wet, it
1965 were required to have seat belts in the front seats. is not raining.
(C) Some cars built before 1965 have harness-style seat III. If it is not raining, the streets and sidewalks are not
belts in the front seats. wet.
(D) Seat belts were not required in cars built before 1965, (A) I only
but many people who had observed the benefits of seat belts (B) II only
on racing cars had them installed. (C) III only
(E) All cars built after 1960 were required to have seat (D) I and II only
belts in the front seats, but not necessarily harness-style seat (E) II and III only

13
American Education Aids - LSAT - Logical Reasoning
SET 3
1. Dormitories range from two to six stories in height. If a the amount of information the historian has available
dormitory room is above the second floor, it has a fire escape. (E) a historian has an obligation to be factually accurate,
If the statements above are true, which of the following must but accuracy is not the historian’s most notable
also be true? accomplishment
(A) Second-floor dormitory rooms do not have fire escapes.
(B) Third-floor dormitory rooms do not have fire escapes. 5. “The “plain English” law requires that a contractual
(C) Only dormitory rooms above the second floor have fire agreement be stated in nontechnical language, if the parties
escapes. to the agreement are not represented by attorneys.
(D) Fourth-floor dormitory rooms have fire escapes The Rent Stabilization code legally requires a contractual
(E) Some two story dormitories do not have fire escapes. agreement for renewing a housing lease to be upon the same
terms and conditions as the expiring lease.”
2. Although aspirin has been proven to eliminate moderate Which of the following conclusions can most properly be
fever associated with some illnesses, many doctors no longer drawn from the information above?
routinely recommend its use for this purpose. A moderate (A) If a contractual agreement for an expiring housing lease
fever stimulates the activity of the body’s disease-fighting is stated in technical language and cannot be stated in
white blood cells and also inhibits the growth of many strains nontechnical language, a landlord and a tenant not
of disease-causing bacteria. represented by attorneys cannot satisfy all legal requirements
If the statements above are true, which of the following in renewing the lease.
conclusions is most strongly supported by them? (B) Since the two legal requirements are in conflict and
(A) Aspirin, an effective painkiller, alleviates the pain and cannot be reconciled, the provisions of the “plain English”
discomfort of many illnesses law must be suspended for contractual agreements to renew
(B) Aspirin can prolong a patient’s illness by eliminating housing leases.
moderate fever helpful in fighting some diseases (C) When the renewal of a housing lease becomes necessary,
(C) Aspirin inhibits the growth of white blood cells, which it will be necessary for both landlord and tenant to employ
are necessary for fighting some illnesses attorneys to renegotiate the terms of the lease and to
(D) The more white blood cells a patient’s body produces, formulate them in “plain English”.
the less severe the patient’s illness will be (D) Contractual agreements to renew housing leases
(E) The focus of modern medicine is on inhibiting the cannot be upon the same terms and conditions as the expiring
growth of disease-causing bacteria within the body leases, if the provisions of the “plain English” law are to be
enforced.
3. The city’s public transportation system should be (E) Contractual agreements to renew housing leases must
removed from the jurisdiction of the municipal government, be restated in nontechnical language.
which finds it politically impossible either to raise fares or to
institute cost-saving reductions in service. If public 6. Superficially, college graduates in 1982 resemble college
transportation were handled by a private firm, profits would graduates of 1964; they are fairly conservative, well dressed,
be vigorously pursued, thereby eliminating the necessity for and interested in tradition; they respect their parents. But
covering operating costs with government funds. there is a deep-seated difference; a majority of the members
The statements above best support the conclusion that of the class of 1982 who were surveyed in their freshman
(A) the private firms that would handle public year stated that making a good income was an important
transportation would have experience in the transportation reason for their decision to go to college.
industry The statements in the passage above, if true, best support
(B) political considerations would not prevent private firms which of the following conclusions?
from ensuring that revenues cover operating costs (A) The concerns of college graduates of 1964 were
(C) private firms would receive government funding if it superficial compared to the financial worries of college
were needed to cover operating costs graduates of 1982.
(D) the public would approve the cost-cutting actions taken (B) Fewer than half the students of the class of 1964
by the private firm declared as freshmen that they entered college in order to
(E) the municipal government would not be resigned to increase their earning potential.
accumulating merely enough income to cover costs (C) Educational background did not play as significant a
part in determining income in 1964 as it does in 1982.
4. Praising a historian for factual accuracy in describing (D) A majority of the members of the class of 1964 revised
events is like praising an architect for using well-seasoned their reasons for attending college between their freshman
timber or properly mixed concrete in a building. year and college graduation.
The author of this statement is arguing that (E) College graduates of 1964 were actually less
(A) nonhistorians can appreciate well-written historical conservative than college graduates of 1982.
accounts, but they cannot judge the accuracy of a historian’s
version of events 7. To produce seeds, plants must first produce flowers. Two
(B) history is analogous to architecture in that both kinds of tarragon plants. Russian tarragon and French
disciplines build on foundations laid by other professions tarragon, look very similar except that Russian tarragon
(C) a good historian must have a thorough knowledge of produces flowers and French tarragon does not. The leaves
auxiliary sciences that help establish facts about events of Russian tarragon, however, lack the distinctive flavor that
(D) the credibility of a historian’s argument depends on makes French tarragon a desirable culinary herb.

14
American Education Aids - LSAT - Logical Reasoning
If the information presented is true, which of the following wear a lap seat belt alone.
can most reliably be concluded on the basis of it?
(A) As a decorative plant, French tarragon is more desirable 9. As soon as any part of a person’s conduct affects
than Russian tarragon prejudicially the interests of others, society has jurisdiction
(B) The flowers of Russian tarragon plants are probably over it, and the question of whether the general welfare will
not favorable or will not be promoted by interfering with it becomes open
(C) Plants that grow from seeds sold in a packet labeled to discussion. If a person’s conduct does not affect prejudicially
“tarragon” are not French tarragon the interests of others, it should not come under the
(D) There are no other kinds of tarragon besides Russian jurisdiction of society in the first place.
tarragon and French tarragon. The author in the passage above argues that
(E) Garden plants that have flavorful leaves generally do (A) society is independent of the actions of individuals
not produce flowers (B) the general welfare of a society is promoted when a
person’s conduct benefits others
8. Wearing either a lap seat belt or a shoulder-and-lap seat (C) conduct that does not infringe on the interests of others
belt protects passengers from the major types of injuries should not be under the jurisdiction of society
incurred in head-on automobile collisions. However, (D) interference with the actions of individuals does not
passengers wearing lap seat belts alone frequently suffer enhance the general welfare
internal injuries caused by the seat belt itself. Such injuries (E) in general, the interests of persons are mutually
do not occur when shoulder-and-lap seat belts are worn. exclusive
Which of the following conclusions about passengers
involved in head-on automobile conclusions is best supported 10. X melts at a higher temperature than P melts.
by the statements above? Y melts at a lower temperature than P melts, but at a
(A) No type of seat belt provides passengers with higher temperature than Q melts.
adequate protection from injury. If the statements above are true, it can be concluded with
(B) The injuries that passengers most frequently incur are certainty that S melts at a higher temperature than Y melts if
internal injuries. one knows in addition that
(C) Head-on automobile collisions cause more injuries to (A) Q and P melt at a higher temperature than S melts
passengers than any other kind of automobile accident does. (B) X melts at a higher temperature than S melts
(D) It is safer for passengers to wear a shoulder-and-lap (C) P melts at a lower temperature than S melts
seat belt than to wear a lap seat belt alone. (D) Q melts at the same temperature that S melts
(E) It is safer for passengers to wear no seat belt than to (E) S melts at a higher temperature than Q melts

SET 4
1. Nursing homes range from three to five stories in height. 3. High levels of fertilizer and pesticides, needed when
If a nursing-home room is above the first floor, it has a fire farmers try to produce high yields of the same crop year after
escape. year, pollute water supplies. Experts therefore urge farmers
If the above statements are true, which of the following must to diversify their crops and to rotate their plantings yearly.
also be true? To produce governmental price-support benefits for a crop,
(A) First-floor rooms in nursing homes do not have fire farmers must have produced that same crop for the past
escapes. several years.
(B) Fire escapes are allowed but not required for first floor The statements above, if true, best support which of the
rooms in nursing homes. following conclusions?
(C) Some nursing homes have no fire escapes. (A) The rules for governmental support of farm prices work
(D) Not all nursing homes have proper fire escapes. against efforts to reduce water pollution
(E) Second-floor rooms in nursing homes have fire escapes. (B) The only solution to the problem of water pollution
from fertilizers and pesticides is to take farmland out of
2. Masterpieces of literature are “intertextual”, that is, they production.
tend to be written in response not to reality but to other works (C) Farmers can continue to make a profit by rotating
of literature. To the extent that a writing is intertextual, it diverse crops, thus reducing costs for chemicals, but not by
becomes clouded as a mirror of social reality. planting the same crop each year.
The statements above provide the most support for which (D) New farming techniques will be developed to make it
of the following conclusions? possible for farmers to reduce the application of fertilizers
(A) To the extent that a writing fails to mirror social reality, and pesticides.
the writing is intertextual. (E) Governmental price supports for farm products are set
(B) The author who wishes to write a masterpiece should at levels that are not high enough to allow farmers to get out
avoid being influenced by other works of literature. of debt
(C) A writing that is not intertextual can have no significant
relationship to any other writings. 4. “When and only when the apples are green, the peppers
(D) Literary masterpieces of the past are suspect as sources are red, but the berries are not blue. When and only when
of information about the social reality of the past. the berries are blue, the cherries are not ripe. When and
(E) A work of literature is not intertextual if it is only when the cherries are not ripe, either the grass is brown
written in response to a writing that accurately mirrors or the leaves are small or both.”
social reality. If the grass is brown, which of the following must be true?

15
American Education Aids - LSAT - Logical Reasoning
A. The apples are not green. 8. Dense snow cover can cause unusually harsh weather
B. The berries are not blue. patterns to persist. If a severe winter storm blankets the
C. The peppers are red. Great Plains, the snow cover reflects the sun’s radiation back
D. The cherries are ripe. into space and thus keeps the temperature of the ground
E. The leaves are small. low. Consequently, cold air moving down from Canada
remains cold enough to cause more snowstorms.
5. “If George was born in New York State, then he is a Which of the following is a conclusion that can be properly
citizen of the United States”. drawn from the information above?
The statement above can be deduced logically from which (A) Winter weather on the Great Plains is the product of
of the given statements? unusual movements of air masses.
(A) Everyone born in New York State is a citizen of the (B) The Great Plains are more likely than other areas to
United States. suffer unusually harsh weather patterns.
(B) Every citizen of the United States is a resident either (C) If the Great Plains get more snow than usual early in
of one of the States or of one of the territories. the winter and the snow remains until the springthaw, the
(C) Some people born in New York State are citizens of winter is likely to be colder than usual.
the United States. (D) Even if the temperatures on the Great Plains are not
(D) George was born either in New York or in Florida. extremely cold but are just below freezing, a moderate snow-
(E) George is a citizen either of the United States or of storm will probably turn into a blizzard.
the Dominican Republic. (E) The temperature of the ground depends primarily on
the thickness of the snow cover.
6. Advertisement: Today’s customers expect high quality.
Every advance in the quality of manufactured products 9. (This passage was written before 1997 when, according
raises customer expectations. The company that is satisfied to a treaty between China and Great Britain, Hong Kong,
with the current quality of its products will soon find that which was then a Crown Colony of Great Britain, was to
its customers are not. At Megacorp, meeting or exceeding become a part of China.) One analyst predicts that Hong
customer expectation is our goal. Kong can retain its capitalist ways after it becomes part of
Which of the following must be true on the basis of the mainland China in 1997 as long as a capitalist Hong Kong
statements in the advertisement above? is useful to China; that a capitalist Hong Kong will be useful
(A) Megacorp’s competitors will succeed in attracting to China as long as Hong Kong is prosperous; and that Hong
customers only if those competitors adopt MegaCorp’s goal Kong will remain prosperous as long as it retains its
as their own. capitalist ways.
(B) A company that does not correctly anticipate the If the predictions above are correct, which of the following
expectations of its customers is certain to fail in advancing further predictions can logically be derived from them?
the quality of its products. (A) If Hong Kong fails to stay prosperous, it will no longer
(C) MegaCorp’s goal is possible to meet only if continuing remain part of mainland China.
advances in product quality are possible. (B) If Hong Kong retains its capitalist ways until 1997, it
(D) If a company becomes satisfied with the quality of its
will be allowed to do so afterward.
products, then the quality of its products is sure to decline.
(C) If there is a world economic crisis after 1997, it will
(E) MegaCorp’s customers are currently satisfied with the
not adversely affect the economy of Hong Kong.
quality of its products.
(D) Hong Kong will be prosperous after 1997.
7. The pattern of scientific grants awarded by foundations (E) The citizens of Hong Kong will have no restrictions
is changing as the number of worthy requests grows in the placed on them by the government of mainland China.
face of cuts in federal spending. Many foundations, formerly
willing to fund innovative but risky projects, have begun to 10. Meteorite explosions in the Earth’s atmosphere as
support relatively conservative projects only. large as the one that destroyed forests in Siberia, with
From the passage above it can be most reasonably approximately the force of a twelve-megaton nuclear blast,
concluded that which of the following was true when the occur about once a century. The response of highly automated
passage was written, as compared to preceding years? systems controlled by complex computer programs to
(A) Foundations were being forced to recognize that they unexpected circumstances is unpredictable.
could be more effective by granting smaller amounts of Which of the following conclusions can most properly be
money to a greater number of projects.
drawn, if the statements above are true, about a highly
(B) The decision-makers at foundations tended to be more
automated nuclear-missile defense system controlled by a
politically conservative.
complex computer program?
(C) Foundations had less money available to support
worthwhile projects because of the rising costs of other (A) Within a century after its construction, the system
activities. would react inappropriately and might accidentally start a
(D) Foundations were increasingly allowing decisions nuclear war.
about which projects to fund to be based on a reluctance to (B) The system would be destroyed if an explosion of a
see projects they supported fail. large meteorite occurred in the Earth’s atmosphere.
(E) Those programs that were more innovative than the (C) It would be impossible for the system to distinguish
ones funded by foundations were supported by the limited the explosion of a large meteorite from the explosion of a
federal funds that were still available. nuclear weapon.

16
American Education Aids - LSAT - Logical Reasoning
(D) Whether the system would respond inappropriately (E) It is not certain what the system’s response to the
to the explosion of a large meteorite would depend on the explosion of a large meteorite would be if its designers did
location of the blast. not plan for such a contingency.

SET 5
1. In the Southern United States in the early twentieth of their training. Today, with electronic instruments, pilots
century, the incidence of pellagra, a debilitating disease never need to look to the rear. Their ability to detect slight
caused by a deficiency of niacin and certain amino acids, was changes on electronic dials and gauges is more significant
inversely related to the price of cotton. The poverty of the than their keenness of long-distance vision or their developed
area and its dependence on a one-crop economy allowed most musculature.
of the population to buy only the cheapest food, which in the The information above best supports which of the following
worst years meant cornmeal and little else. conclusions?
Which of the following is a conclusion that can logically be (A) The reliance on increasingly sophisticated electronic
drawn from the passage above? instruments in air combat situations will soon make human
(A) The most important crop in the Southern United States pilots superfluous.
in the early twentieth century was corn (B) Visual acuity is of little help in air combat today because
(B) A diet consisting largely of cornmeal is deficient in some of the terrific speeds at which modern aircraft approach each
essential nutrients other.
(C) As the price of cotton went down, the price of most (C) Fitness with regard to military service must always be
foodstuffs went up defined in terms of the demands combat situation place on
(D) As the price of cotton went up, the incidence of pellagra combatants.
went up (D) The performance of pilots will necessarily decline if
(E) The Southern United States in the early twentieth the strenuous physical conditioning programs employed in
century was a poverty area because of its dependence on a the past are not continued.
one-crop economy (E) Revisions of military training programs at frequent
intervals can guarantee adaptability to the demands of future
2. In 1980, Japan’s income from exported goods totalled combat situations.
$92 billion, three times more than the United States’ income
from exported goods. However, the 1980 United States 5. Whenever the sun is shining and the windchill factor is
foreign-trade account, which balances a nation’s payments below zero, Susan wears her parka. Whenever it is raining
for imported goods against the moneys received for its and the windchill factor is above zero, Susan wears her
exported goods, showed a $4 billion surplus while that of raincoat. Sometimes it rains when the sun is shining.
Japan showed a deficit. If the above statements are true, which of the following must
Which of the following is the most logical conclusion to be also be true?
drawn from the information above? (A) If it is not raining and Susan is wearing her parka, the
(A) The United States imports few goods from Japan sun is shining.
(B) The United States exports more goods than does Japan (B) If the windchill factor is below zero and Susan is not
(C) Japan imported more goods manufactured in the wearing her parka, the sun is not shining.
United States in 1980 than ever before (C) If the windchill factor is below zero and it is not raining,
(D) Japan’s payments for imported goods in 1980 far Susan is wearing her parka.
exceeded United States' payments for imported goods in 1980 (D) If the windchill factor is below zero and it is raining,
(E) The price of goods manufactured in the United States Susan is wearing her raincoat.
is competitive with the price of those manufactured in Japan (E) If it rains while the sun is shining and the windchill
factor is zero, Susan wears her parka.
3. When the chemical vianzone is applied to any clear
solution containing sodium chlorate, the solution becomes 6. Many people argue that tobacco advertising plays a
cloudy, and when vianzone is applied to a clear solution crucial role in causing teen-agers to start or continue smoking.
containing potassium nitrate, the solution becomes cloudy. In Norway, however, where there has been a ban on tobacco
Vianzone does not alter the appearance of a clear solution advertising since 1975, smoking is at least as prevalent among
containing benzene. In a certain experiment, vianzone was teen-agers as it is in countries that do not ban such
applied to a clear solution, and the solution remained clear. advertising.
It can be validly concluded that the clear solution used in Which of the following statements draws the most reliable
the experiment contained conclusion from the information above?
(A) potassium nitrate (A) Tobacco advertising cannot be the only factor that
(B) both sodium chlorate and benzene affects the prevalence of smoking among teen-agers.
(C) no benzene (B) Advertising does not play a role in causing teen-agers
(D) no sodium chlorate to start or continue smoking.
(E) neither potassium nitrate nor benzene (C) Banning tobacco advertising does not reduce the
consumption of tobacco.
4. During the Second World War, fighter pilots watched (D) More teen-agers smoke if they are not exposed to
for enemies in the sky by direct visual perception. The pilots tobacco advertising than if they are.
had to turn their heads frequently in order to look to their (E) Most teen-agers who smoked in 1975 did not stop when
rear, and so calisthenics to develop neck muscles were part the ban on tobacco advertising was implemented.

17
American Education Aids - LSAT - Logical Reasoning
7. Laws requiring the use of headlights during daylight 9. A study of the use of the polygraph, or lie detector, found
hours can prevent automobile collisions. However, since that when a trained examiner using approved questioning
daylight visibility is worse in countries farther from the techniques gave the test, information from the lie detector
equator, any such laws would obviously be more effective in was accurate for 70 to 90 percent of the responses in
preventing collisions in those countries. In fact, the only determining whether the responses were truthful.
countries that actually have such laws are farther from the It can be reliably concluded from the results of the study
equator than is the continental United States. cited above that
Which of the following conclusions could be most properly (A) the lie detector gives accurate results only when
drawn from the information given above? employed by a trained examiner using approved questioning
(A) Drivers in the continental United States who used their techniques
headlights during the day would be just as likely to become (B) with a trained examiner using approved questioning
involved in a collision as would drivers who did not use their techniques, the lie detector gave incorrect results in at least
headlights. one out of ten instances
(B) In many countries that are farther from the equator (C) in at least 70 percent of the cases, a trained examiner
than is the continental United States poor daylight visibility using approved questioning techniques can discriminate
is the single most important factor in automobile collisions. between the questions for which the lie detector gives correct
(C) The proportion of automobile collisions that occur in results and those for which it does not
the daytime is greater in the continental Untied States than (D) the lie detector measures truth-telling objectively, and
in the countries that have daytime headlight laws. the validity of its use does not depend on subjective
(D) Fewer automobile collisions probably occur each year interpretation on the part of the examiner
in countries that have daytime headlight laws than occur (E) if a trained examiner using approved questioning
within the continental United States. techniques asks a specific question and the lie detector
(E) Daytime headlight laws would probably do less to indicates that the answer was false, the respondent definitely
prevent automobile collisions in the continental United States lied when giving that answer
than they do in the countries that have the laws.
10. Mass transit authorities in large cities are struggling
8. The number of patents granted to inventors by the with deficits. Riders complain about delays and breakdown,
United States Patent Office dropped from 56,000 in 1971 to cuts in service, and fares higher than they are accustomed to
45,000 in 1978. Spending on research and development, which paying. For all these reasons and because the price of gasoline
peaked at 3% of the gross national product (GNP) in 1964, is still not prohibitive, the number of passengers using public
was only 2.2% of the GNP in 1978. During this period, when transportation has fallen, adding to the deficits.
the United States percentage was steadily decreasing, West Which of the following statements about the relationship
Germany and Japan increased the percentage of their GNPs between the number of riders using public transportation and
spent on research and development to 3.2% and 1.6% the price of gasoline is best supported by the passage above?
respectively. (A) As the price of gasoline rises, the number of riders using
Which of the following conclusions is best supported by the public transportation rises.
information above? (B) Even if the price of gasoline rises, the number of riders
(A) There is a direct relationship between the size of a using public transportation will continue to decline.
nation’s GNP and the number of inventions it produces. (C) If the price of gasoline rises to a prohibitive level, the
(B) Japan and West Germany spent more money on number of riders using public transportation will rise.
research and development in 1978 than did the United States. (D) The majority of riders using public transportation do
(C) The amount of money a nation spends on research and not use gasoline; hence, fluctuations in gasoline prices are
development is directly related to the number of inventions unlikely to affect the number of riders using public
patented by that nation. transportation.
(D) Between 1964 and 1978, the United States consistently (E) The price of gasoline is always low enough to make
spent a larger percentage of its GNP on research and private transportation cheaper than public transportation;
development than did Japan. hence, fluctuations in gasoline prices are unlikely to affect
(E) Both West Germany and Japan will soon surpass the the number of riders using public transportation.
United States in the number of patents granted to inventors.

SET 6
1. Even those buildings made of the best of materials are (C) Engineers who were not responsible for the original
being sloppily designed and constructed today. But many construction of the building supervise any structural
flaws that are already part of the construction are made into improvements in the building
bigger and more costly problems by maintenance personnel (D) The design engineer oversees the maintenance of his
or other engineers because they do not take the time to study or her creation after construction is ended and the building
the buildings for which they become responsible. is occupied
Which of the following, if it could be carried out, is most (E) The plans of a design engineer are made available for
likely to alleviate the difficulties stated in the passage? the scrutiny of the tenants of the building the engineer
(A) Buildings are made only of more costly materials that designed
have passed all safety requirements
(B) Maintenance personnel inform the tenants in their 2. The following proposal to amend the bylaws of an
buildings about any repairs that are needed organization was circulated to its members for comment.

18
American Education Aids - LSAT - Logical Reasoning
“When more than one nominee is to be named for an office, light is not flashing and the tracks are not clear, the school
prospective nominees must consent to nomination and before bus waits until they are clear and then proceeds immediately
giving such consent must be told who the other nominees across them.
will be”. If the statements above are true and it is true that the school
Which of the following comments concerning the logic of bus stops at the tracks and then does not proceed to cross them,
the proposal is accurate if it cannot be known who the actual which of the following must also be true?
nominees are until prospective nominees have given their (A) The warning light is flashing and the tracks are clear.
consent to be nominated? (B) The warning light is flashing and the tracks are not
(A) The proposal would make it possible for each of several clear.
nominees for an office to be aware of who all of the other (C) The warning light is not flashing and the tracks are
nominees are not clear.
(B) The proposal would widen the choice available to those (D) The warning light is flashing, or the tracks are not
choosing among the nominees clear, or both.
(C) If there are several prospective nominees, the proposal (E) The warning light is not flashing, or the tracks are not
would deny the last nominee equal treatment with the first clear, or both.
(D) The proposal would enable a prospective nominee to
withdraw from competition in favour of a specific person 6. If carbon dioxide gas is generated in large enough
without others knowing about such preference quantities, it will collect in the atmosphere and cause an
(E) If there is more than one prospective nominee, the unwelcome warming effect on climate. The decay of plants,
proposal would make it impossible for anyone to become a including trees, generates carbon dioxide, but in forests such
nominee carbon dioxide is balanced by the carbon dioxide taken in by
living plants, which give off oxygen.
3. A violin constructed to have improved sound would sound The use in industry of fuels derived from plants generates
different from the best-sounding existing violins. To a large amount of carbon dioxide; such fuel include wood,
professional violinists, a violin that sounds different from the coal, and oil.
best-sounding existing violins sounds less like a violin and If the statements above are true, which of the following can
therefore worse than the best-sounding existing violins. properly be concluded from them?
Professional violinists are the only accepted judges of the (A) All of the carbon dioxide that can be generated in an
sound quality of violins. industrial society is attributable to plants, directly or
If all of the statements above were true, which of the indirectly.
following would be best supported by those statements? (B) An unwelcome warming effect on climate cannot be
(A) Only amateur violinists should be asked to judge the avoided, since carbon dioxide is given off by the natural
sound quality of newly constructed violins. processes of decay in plants.
(B) Professional violinists supervise the construction of (C) Forests contribute as much carbon dioxide to the
violins. atmosphere as does the industrial use of fuel derived from
(C) The best-sounding existing violins have been in plants.
existence for several centuries. (D) A society that uses plant-based fuels in industry will
(D) It is currently impossible to construct a violin that the contribute a net surplus of carbon dioxide to the atmosphere,
only accepted judges will evaluate as having improved sound. unless the gas is reabsorbed in some way.
(E) It is possible to construct a violin that sounds better (E) No matter which fuels are used by industry, there is
than the best-sounding existing violins to everyone but bound to be an increase in the total amount of carbon dioxide
professional violinists. in the atmosphere and a consequent unwelcome warming
effect on climate.
4. When a polygraph test is judged inconclusive, this is no
reflection on the examinee. Rather, such a judgment means 7. Forty-five percent of all blood donated in the Unites
that the test has failed to show whether the examinee was States is type O. Type O blood is essential for emergencies
untruthful or truthful. Nevertheless, employers will where there is no time for determining the blood type of
sometimes refuse to hire a job applicant because of an victims because type O blood can be used for everyone. Type
inconclusive polygraph test result. O blood is unique in that it is compatible with blood of all
Which of the following conclusions can most properly drawn types; any recipient, regardless of blood type, can be given it.
from the information above? But precisely because of this special usefulness, type O blood
(A) Most examinees with inconclusive polygraph test is chronically in short supply.
results are in fact untruthful. If the statements in the passage above are true, which of
(B) Polygraph tests should not be used by employers in the following must also be true?
the consideration of job applicants. (A) The special usefulness of type O blood lies in the fact
(C) An inconclusive polygraph test result is sometimes that it matches the blood type of most people.
unfairly held against the examinee. (B) Supplies of type O blood are continuously so low that
(D) A polygraph test indicating that an examinee is type O blood is unavailale for emergencies, where its
untruthful can sometimes be mistaken. usefulness would be greatest.
(E) Some employers have refused to consider the results (C) Forty-five percent of the total population of the United
of polygraph tests when evaluating job applicants. States has type O blood, which makes type O the most
common blood type.
5. The school bus always stops at the railroad tracks. When (D) Any decision to give blood of any type other than O
the warning light is not flashing, it then proceeds directly needs to be based on knowledge of the recipient’s blood type.
across if the tracks are clear. However, when the warning (E) Type O blood is the only blood that cannot be typed as
19
American Education Aids - LSAT - Logical Reasoning
fast as needed in emergencies. from the statements above?
(A) Most animals have sensory capacities superior to those
8. Most television viewers estimate how frequently a demonstrated by humans.
particular type of accident or crime occurs by how extensively (B) Some animals have sensory capacities that are
it is discussed on television news shows. Television news different from those of humans.
shows report more on stories that include dramatic pictures (C) During evolution the eyes and ears of human beings
such as fires and motor vehicle accidents than they do on
were modified to make human sense perception less acute.
more common stories that have little visual drama such as
(D) Researchers should not be surprised to find that all
bookkeeping fraud.
the sensory capacities of platypuses are greater than any of
If the statements above are true, it can be properly concluded
that which of the following is also true? those demonstrated by humans.
(A) The time that television news reporters spend (E) Any human who can see in dim light does so less well
researching news stories is directly related to the number of than any cat.
viewers who will be affected by events like those in the news
stories. 10. One theory of the Moon’s origin is that the Earth,
(B) It is easier for crimes such as bookkeeping fraud to go early in its development, was a rapidly rotating body of
unprosecuted than it is for crimes such as arson. molten rock in which most of the iron had settled to the
(C) The number of fires and motor vehicle accidents greatly core; some of this fluid was cast off from the surface of the
increases after each television news show that includes spinning mass and later solidified to form the Moon.
dramatic pictures of a fire or motor vehicle accident. Which of the following conclusions can best be supported
(D) Viewers of television news shows tend to overestimate by the theory above of the Moon’s origin, if that theory is
the number of fires and motor vehicle accidents that occur correct?
relative to the number of crimes of bookkeeping fraud. (A) The moon is the only sizable heavenly body in orbit
(E) The usual selection of news stories for television news around the earth.
shows is determined by the number of news reporters (B) The Moon has proportionately less iron at its core than
available for assignment. the Earth does.
(C) The surface of the Moon solidified after the surface of
9. A dog hears higher pitches than a human hears; a cat the earth did.
has a greater capacity to see in dim light than a human (D) Most of the fluid cast off from the Earth was dispersed
normally has; a platypus picks up weak electric signals to into outer space.
which a human is normally insensitive. (E) The Moon, like the Earth, has a solid surface and a
Which of the following conclusions can be properly drawn molten core.

SET 7 (Questions involving some arithmetic)


(Questions in Logical Reasoning / Critical Reasoning do not usually involve the use of arithmetic, but can be
answered using an argumentative process. But, occasionally, questions are asked which involve the use of basic
mathemetical principles. This set contains questions of this type.)
1. Between 1975 and 1985, nursing-home occupancy rates If the statements above are true, which of the following
averaged 87 percent of capacity, while admission rates conclusions can be properly drawn?
remained constant at an average of 95 admissions per 1000 (A) Husbands of wage-earning women report a higher rate
beds per year. Between 1985 and 1988, however, occupancy of participation in household tasks than is actually the case,
rates rose to an average of 92 percent of capacity, while because they overestimate the number of times they do
admission rates declined to 81 per 1000 beds per year. housework
If the statements above are true, which of the following (B) On the average, husbands of wage-earning women
conclusions can be most properly drawn? allocate a little more time to housework per month than do
(A) The average length of time nursing-home residents husbands of women who do not earn wages
stayed in nursing homes increased between 1985 and 1988. (C) The average time spent per occasion of performing
(B) The proportion of old people living in nursing homes individual household tasks is less for husbands of wage-
was greater in 1988 than in 1975. earning women than it is for husbands of women who do not
(C) Nursing home admission rates rend to decline earn wages
whenever occupancy rates rise. (D) There is no difference in the patterns of time spent on
(D) Nursing homes built prior to 1985 generally had fewer household tasks by husbands of wage-earning women and
beds than nursing homes built between 1985 and 1988. by husbands of women who do not earn wages
(E) The more beds a nursing home has, the higher its (E) Husbands of wage-earning women participate more
occupancy rate is likely to be. often in tasks that are completed in a short time than they
do in tasks that take a long time to complete
2. Husbands of wage-earning women spend, on the
average, the same amount of time on household work as do 3. Recent estimates predict that between 1982 and 1995
husbands of women who do not earn wages: eleven hours per the greatest increase in the number of people employed will
week. Husbands of wage-earning women perform household be in the category of low-paying service occupations. This
tasks on more occasions than do husbands of women who do category, however, will not increase its share of total
not earn wages. employment, whereas the category of high-paying service
occupations will increase its share.

20
American Education Aids - LSAT - Logical Reasoning
If the estimates above are accurate, which of the following (D) California’s proportion of non-orange citrus crops was
conclusions can be drawn? higher than Florida’s.
(A) In 1982 more people were working in low-paying service (E) California had more acreage that could be devoted to
occupations than were working in high-paying service agriculture than did Florida.
occupations.
(B) In 1995 more people will be working in high-paying 7. “Each year’s increase or decrease in productivity – that
service occupations than will be working in low-paying service is, in the amount of goods and services produced in an hour’s
occupations. paid working time – is calculated in relation to that of the
(C) Nonservice occupations will account for the same share previous year. In 1982, the productivity of workers in nonfarm
of total employment in 1995 as in 1982. private business was 5 percent higher than in 1981, while
(D) Many of the people who were working in low-paying overall private business productivity, when farming was
service occupations in 1982 will be working in high-paying included, was 4.8 percent higher than in 1981. That 4.8
service occupations by 1995. percent increase in 1982 was double the increase recorded in
(E) The rate of growth for low-paying service occupations 1981.”
will be greater than the overall rate of employment growth If the information above is accurate, which of the following
between 1982 and 1995. must be true?
(A) More workers left farming for work in nonfarm private
4. During the 1960’s only 15 percent of the growth in the business in 1982 than in 1981.
United States population was due to immigration. The other (B) Average hourly wages of workers in nonfarm private
85 percent was due to the amount by which the birthrate business in 1982 declined from 1981.
was higher than the death rate. The population has grown (C) In 1982, the increase in the productivity of nonfarm
continuously since then, but today immigration accounts for private business was more than double the increase recorded
30 percent of population growth. Nonetheless, the number of in 1981.
people immigrating each year has not changed significantly. (D) In 1982, the increase, if any, in the profits of nonfarm
From the information above it can be properly concluded private business was smaller than the increase in the profits
that, as compared to the 1960’s, in the United States today of farming.
(A) the birthrate is lower (E) In 1982, the increase, if any, in the productivity of
(B) population growth is roughly the same farming was smaller than the increase in nonfarm private
(C) the population is growing more slowly business productivity.
(D) the death rate is higher
(E) fewer people are emigrating 8. A three-hour movie replaced three regularly scheduled
television programs. The movie had only two equally long
5. The 38 corporations that filed United States income tax breaks for advertisements instead of the five equal breaks
returns showing a net income of more than $100 million that normally interrupted the regular programs. However,
accounted for 53 percent of the total taxable income from the total time used for advertisements during the movie
foreign sources reported on all tax returns. Sixty percent of equaled the time usually used for advertisements during the
the total taxable income from foreign sources came from the regular programs.
200 returns reporting income from 10 or more countries. If the above statements are true, which of the following must
If the statements above are true, which of the following must also be true?
also be true? (A) The average length of the advertisements shown during
(A) Most of the total taxable income earned by corporations the movie was shorter than that of the advertisements usually
with net income above $100 million was earned from foreign shown during the regular programs.
sources. (B) Only one sponsor’s advertisements were shown during
(B) Wealthy individuals with large personal incomes the movie, whereas more than one sponsor’s advertisements
reported 47 percent of the total taxable income from foreign were shown during the regular programs.
sources (C) The sponsors who advertised during the movie were
(C) Income from foreign sources amounted to between 53 not the same as the sponsors who usually advertised during
and 60 percent of all reported taxable income. the regular programs.
(D) Some of the corporations with net income above $100 (D) Each of the interruptions for advertisements in the
million reported income from 10 or more countries. movie was longer than each of the interruptions for
(E) Most of the tax returns showing income from 10 or advertisements that usually occurred in the regular
more countries reported net income of more than $100 million programs.
(E) The advertisements during the movie focused on only
6. In a recent year California produced an orange crop equal one kind of product, whereas the advertisements during the
to only seventy-six percent of Florida’s orange crop. However, regular programs focused on a variety of products.
when citrus crops as a group, including oranges, were
compared, the California crop was twentythree percent 9. Two hundred corporations with net incomes of more than
greater than Florida’s crop for the same year. $122 million each accounted for 77 percent of total corporate
If the information above is true, which of the following can gifts to United States higher education in 1985. That year,
properly be concluded about the Florida and California citrus 26 percent of total corporate gifts to United States higher
crops in the year mentioned? education came from 14 Japanese corporations, each of which
(A) Florida’s climate was suited only to growing oranges. received income from 27 or more countries.
(B) Florida produced larger oranges than California did. If the statements above are true, which of the following must
(C) California produced more oranges than it did non- also be true?
orange citrus.
21
American Education Aids - LSAT - Logical Reasoning
(A) Most of the net income earned by the 14 Japanese for about 40 percent of all births in this country annually,
corporations was earned outside of Japan. more than 25% have no health-care insurance to pay
(B) Individuals contributed 23 percent of total gifts to maternity costs.
United States higher education in 1985. If the statements above are true, which of the following must
(C) Gifts from corporations accounted for more than half also be true?
of the total contributions to United States higher education (A) Each year, about 75 percent of all births in this country
in 1985. are to women who have health-care coverage of maternity
(D) One or more of the 200 corporations with more than costs.
$122 million in net income received income from 27 or more (B) Each year, about 60 percent of all births in this country
countries. are to women who are younger than eighteen or older than
(E) Most of the 14 Japanese corporations earned more than twenty-four.
$122 million in net income in 1985. (C) For an average birth, health-care insurance pays about
75% of $3,200.
10. For some women the cost of giving birth can be an (D) In this country, about 75 percent of the women who do
unexpectedly large burden. The average normal birth now not have health-care coverage of maternity costs are younger
costs $3,200, and a birth with complications can cost than eighteen or older than twentyfour.
thousands of dollars more. Of women in the primary (E) In this country, nearly 75 percent of the women in the
childbearing age range of eighteen to twentyfour, who account primary childbearing age give birth with no complications.

22
American Education Aids - LSAT - Logical Reasoning

Chapter 3
'INFERENCE' QUESTIONS
Another type of questions under Logical Reasoning in LSAT passage; or being irrelevant to what is stated in the passage.
asks you to spot the choice which can be a logical inference
from the information contained in the given narration. We give below a number of examples of such questions of
In ordinary English parlance, the nouns ‘conclusion’ and the same types and levels of difficulty as those asked in LSAT
‘inference’ are synonyms of each other as are the corresponding of previous years.
verbs ‘conclude’ and ‘infer’.
Both ‘conclusion’ and ‘inference’ mean ‘something that can Example 1
be logically arrived at or deduced’. Currently people in the United States eat, on the average,
The Logical Reasoning questions in LSAT use the word 1,431 pounds of food per year, 35 pounds more than in 1980.
‘inference’ in a slightly more restricted sense, to refer to a This increase is, at least in part, because people between the
statement which is not the principal conclusion from the given ages of 15 and 64 have accounted for an increasing share of
information, but is one which is secondary, ancillary or the population.
tangential to it. Which of the following can be properly inferred from the
passage above?
Whereas only one ‘conclusion’ is normally possible from a (A) More than half of the current population of the United
given set of information (and no conclusion at all in some States is between the ages of 15 and 64.
cases), many different ‘inferences’ are possible from the same (B) The population has risen since 1980.
set of statements. (C) Children below the age of 15 require, on the average,
In some cases, the ‘conclusion’ itself will be found in the more food than do people over the age of 64.
given narration. (D) Before 1980, children below the age of 15 outnumbered
people between the age of 15 and 64.
For example, read the statement below: (E) Individuals between the ages of 15 and 64 consume,
“The time is 5.30 now and the movie starts at 6.30. Unless on the average, more food than do those younger or older.
we leave home within the next 15 minutes, we shall be missing
its beginning” Analysis
The above statement is a simple one that we often hear in The author attributes the increase in the average per-capita
day-to-day life. This statement itself contains the conclusion consumption of food in US during the last decade partly to
that ‘unless we leave home within the next 15 minutes, we the fact that the proportion of people in the age group 15 to
shall be missing its beginning”. 64 to the total population has increased during this period.
One inference that we can make from this statement is The obvious inference from this conclusion is that people
that the travel time between the speaker’s home and the movie in the age group 15 to 64 consume relatively more food per
theater is about 45 minutes. capita than people in other age groups. So, (E) is the answer.
This inference is not a main conclusion by itself, but is one What the given passage implies is that the proportion of
that is secondary to the main conclusion that “unless we leave people in the age group 15 to 64 is greater now than it was in
home within the next 15 minutes, we shall be missing its 1980. By stating that this proportion is more than 50%, (A)
beginning”. extends the scope of the given statement beyond what is
Another inference that is possible from the given sentence immediately implied by it, and is wrong.
(because of the use of the plural pronoun ‘we’) is that the If the given narration had said that the total consumption
speaker is not going alone to the movie and will have at least of food by people in the United States had increased since
one companion. 1980, an inference could be that the population had risen
The following statements, while they may be factually since that year. But what the narration says is that the per
correct, cannot be considered to be logical inferences from the capita consumption of food has increased by 30%. This
above statement, because they all extend the scope of that increase would have been possible even if the population had
statement beyond what immediately follows from it : decreased. Therefore, (B) is not a necessary inference from
(A) The speaker is fond of going to movies. the given information.
(B) The movie has the speaker’s favourite star as the hero. The given narration combines those below 15 and those
(C) The speaker never goes to a movie alone. above 64 as a single group. Choices (C) and (D), which make
(D) The speaker will prefer to return home without seeing distinctions between them, extend the scope of the given
the movie if he misses its beginning. narration beyond its immediate implication, and are not
(E) The speaker prefers movies to plays. necessary inferences from it.

The reasoning required for answering an ‘inference’ Example 2


question is similar to the reasoning required for answering a Increases in the level of high-density lipoprotein (HDL) in
‘conclusion question’. the human bloodstream lower blood-stream-cholesterol levels
The errors in the wrong choices under ‘inference’ questions by increasing the body’s capacity to rid itself of excess
will be similar to those under the ‘conclusion’ questions - cholesterol. Levels of HDL in the bloodstream of some
namely, contradicting a specific information in the given individuals are significantly increased by a program of regular
passage; extending the scope of what is stated in the passage exercise and weight reduction.
beyond what immediately follows from it; making a sweeping Which of the following can be correctly inferred from the
generalization from a single instance referred to in the statements above?
23
American Education Aids - LSAT - Logical Reasoning
(A) Individuals who are underweight do not run any risk anniversary of Aristarchus’ birth.
of developing high levels of cholesterol in the blood-stream. If the exact year of birth of this astronomer had been known
(B) Individuals who do not exercise regularly have a high as 320 BC, there would have been no ‘arbitrariness’ in the
risk of developing high levels of cholesterol in the bloodstream choice of the year 1980 for the celebration.
late in life. Therefore, it is (A) (‘the exact date of Aristarchus’ birth is
(C) Exercise and weight reduction are the most effective uncertain’) that we can infer from the given narration, and is
methods of lowering bloodstream cholesterol levels in the answer.
humans. The narration talks generally of Greek officials. By
(D) A program of regular exercise and weight reduction speculating that the officials belonged to the island of Samos,
lowers cholesterol levels in the bloodstream of some choice (B) extends the scope of the given information beyond
individuals. what immediately follows from it, and is wrong.
(E) Only regular exercise is necessary to decrease (C) contradicts the specific information in the first sentence
cholesterol levels in the bloodstream of individuals of average that Aristarchus was born in Samos, and is wrong.
weight. (D) and (E) also extend the scope of the given information
beyond its immediate implications, and are wrong.
Analysis
The second sentence says that, in the case of some Example 4
individuals, regular exercise and weight reduction result in Popular culture in the United States has become
higher HDL levels. The first sentence says that higher HDL Europeanized to an extent unimaginable twentyfive years
level in the bloodstream reduces cholesterol level in the body. ago. Not many people then drank wine with meals, and no
Reading the second statement first and the first statement one drank imported mineral water. No idea would have been
next as we have done above, we can infer that, in the case of more astonishing than that Americans would pay to watch
some individuals, regular exercise and weight reduction leads soccer games. Such thoughts arise because of a report that
to higher IDL levels and, therefore, a consequent reduction the American Association of State Highway and
in the cholesterol level in their bloodstream. It is (D) which Transportation Officials has just adopted a proposal to develop
says this, and is the answer. the country’s first comprehensive interstate system of routes
The given passage says that weight reduction can lead to for bicycles.
a reduction in blood cholesterol in the case of some individuals Which of the following inferences is best supported by the
only. Choice (A) makes a sweeping generalisation that any passage?
individual who is underweight does not run the risk of (A) Long-distance bicycle routes are used in Europe.
developing high levels of cholestrol in his blood stream, and (B) Drinking imported mineral water is a greater luxury
is not a necessary inference. than drinking imported wine.
Again, the statement that some individuals can reduce (C) United States culture has benefited from exposure to
blood cholesterol levels by undertaking regular exercises foreign ideas.
cannot lead to the sweeping generalisation that all individuals (D) Most Europeans make regular use of bicycles.
who do not exercise regularly will inevitably suffer from high (E) The influence of the United States on European culture
cholesterol levels. So, (B) is wrong. has assumed unprecedented proportions in the last twenty-
The passage merely says that regular exercise and weight five years.
reduction can significantly increase the level of HDL in some
individuals. The passage does not rule out other methods Analysis
(such as medication) for increasing the HDL levels. By stating Drinking wine, drinking imported mineral water, and
that these two are the most effective methods of lowering HDL watching soccer games on payment are mentioned as
levels in all humans, (C) extends the scope of the given instances of Europeanization of the United States’ popular
information beyond what follows immediately, and is not the culture during the last twentyfive years, and these are
answer. remembered by the author in the context of a proposal to
(E) also suffers from the same infirmity, and is wrong. develop compehensive inter-state bicycle routes in the United
States.
Example 3 We can therefore infer that the author considers long-
The astronomer Aristarchus was born on the Greek island distance cycling as the most recent influence of European
of Samos. Somewhat arbitrarily, Greek officials decided that culture on the United States population. This, in turn, implies
1980 was as good a year as any to celebrate the twenty-three that long-distance cycling is popular in Europe and that there
hundredth anniversary of Aristarchus’ birth. are exclusive routes for long-distance cycling there.
It can be inferred from the statements above that (A) is what states this, and is the answer.
(A) the exact date of Aristarchus’ birth is uncertain As instances of Europeanization, the author gives equal
(B) the officials were representatives from the island of weight to drinking wine and drinking imported mineral water.
Samos The statement ‘drinking imported mineral water is a greater
(C) Samos is not necessarily Aristarchus’ birthplace luxury than drinking wine’ extends the scope of the author’s
(D) the Greek government had learned of Aristarchus’ statement beyond its immediate implication. So, (B) is wrong.
astronomy only recently The given narration is restricted to just four instances of
(E) the celebration took place on the island of Samos European influence on American popular culture, and the
author does not imply that even this influence has been
Analysis beneficial to America. The statement in (C) that United States
The clue to the answer is in the phrase ‘somewhat culture has benefited from exposure to all foreign ideas is a
arbitrarily’, which means that there was no scientific reason sweeping generalization not warranted by the given
why 1980 was chosen to mark the twenty-three hundredth information.
24
American Education Aids - LSAT - Logical Reasoning
The inference that long-distance cycling is popular in Example 6
Europe does not mean that most Europeans make regular At Midwest University, there is no collusion between the
use of bicycles. So, (D) extends the scope of the given narration athletic office and the admissions office. Athletes must be
beyond what is immediately implied by it, and is not a accepted for admission to the university on the basis of their
necessary inference. academic records before they can be offered financial
The narration is about European influence on American assistance on the basis of athletic ability. The university thus
culture, and not about American influence on European hopes to avoid the kind of recruiting violations that have been
culture. So, (E) is irrelevant to what is stated by the author, a problem at some other universities.
and is not the answer. It can be inferred from the passage above that
(A) the admission policy described above is unique to
Example 5 Midwest University
If an investment has produced no profit, tax relief (B) some universities offer financial assistance to athletes
predicated on having made the investment is no help; any who would not be admitted to the university on the basis of
corporate manager who fears that a new asset will not make their academic records
money is scarcely comforted by promises of reductions in taxes (C) most student athletes in universities are admitted
the corporation will not owe. solely on the basis of athletic ability
Which of the following is the most reliable inference to draw (D) admissions offices do not modify academic admission
from the passage above? requirements for any students except student athletes
(A) An effective way to discourage unprofitable corporate (E) in many universities, athletic offices are often at odds
investment is to predicate tax relief on the making of with admission offices
profitable investments.
(B) Corporate managers are likely to ignore tax Analysis
considerations in deciding to invest in assets they believe will (Many American Universities offer financial assistance for
be profitable. higher studies to proven sportsmen.) The given passage says
(C) The promise of tax benefits for making new investments that, by stipulating that athletes must first be accepted for
will not in and of itself stimulate new investment. admission based on their academic records before they are
(D) The less importance a corporate manager attaches to offered financial aid by the athletic office, the Midwest
tax considerations, the more likely it is that the manager University hopes to avoid problems that have arisen in some
will accurately predict the profitability of an investment. other universities.
(E) The critical factor in a corporate investment decision Obviously, the author implies that a practice is prevalent
is likely to be a corporate manager’s emotional response to in some other universities in which the athletic office offers
perceived business conditions. financial assistance to athletes who are not subsequently
found fit for admission by the admissions office on the basis
Analysis of their academic records. It is (B) which states this, and is
In order to increase employment opportunities, the answer.
Governments periodically come out with incentive schemes The passage says that the problem of offering financial
for fresh investments in business. These schemes generally assistance to athletes even before they are offered admission
stipulate (or predicate) that there will be reduced rates of is prevalent only in some other universities, meaning that
taxes on the profits resulting from such fresh investments. many other universities are already adopting the procedure
The author of the passage first says that this concession that Midwest University proposes to adopt now. By stating
will be illusory in the cases of investments which produce no that the new admission policy will be unique to Midwest
profits at all, and concludes that a corporate manager, who University, (A) contradicts what is specifically stated in the
fears that a new asset will not make any profit at all, will not narration, and is wrong.
be swayed by such tax incentive while taking the decision on The statements in the narration are restricted to Midwest
whether to invest in that asset or not. University and some other universities. (C) is a sweeping
We can therefore infer that the promise of tax benefit will generalization on the state of affairs in all universities, and
not, by itself, stimulate new investments, unless the corporate is not warranted by the information in the narration.
managers are independently convinced that the investments (D) also is a sweeping generalization relating to all
will result in additional profits. It is (C) which states this, universities, and is not warranted by the information in the
and is the answer. narration.
The purpose of the tax relief is to encourage profitable By converting the given information about ‘some’
investments, and not to discourage unprofitable investments. universities into information about ‘many’ universities, (E)
(A) extends the scope of the given information, and is wrong. extends the scope of the given narration without sufficient
What the author says is that corporate managers are likely justification, and is wrong.
to ignore tax incentives when the believe that a particular
investment will not be profitable. He implies that the same Example 7
managers will be tempted by the tax incentives while deciding John Cleland, best known for his novel Fanny Hill, is
to invest on assets which they believe will be profitable. (B) becoming the Vivaldi of eighteenth-century literature in that
contradicts this implication, and is not the answer. his cult is promoted by people who do not much care for the
(D) is a sweeping generalization not warranted by the given period as a whole and who cheerfully ignore the works of its
information, and is wrong. greatest masters.
The narration talks of rational investment decisions taken It can be inferred that the author of the passage above
by corporate managers. (E), which refers to the ‘emotional’ believes which of the following?
responses of corporate managers is irrelevant to what is stated (A) Cleland was more esteemed as a writer in his own time
in the narration. than he is in ours.
25
American Education Aids - LSAT - Logical Reasoning
(B) Few people know much about the period in which Example 8
Cleland wrote. Rural households have more purchasing power than do
(C) Most eighteenth-century novels deserve more attention urban or suburban households at the same income level, since
than Fanny Hill. some of the income that urban and suburban households use
(D) The cult of Vivaldi has developed more recently than for food and shelter can be used by rural households for other
that of Cleland. needs.
(E) Vivaldi was not one of the greatest masters of his time. Which of the following inferences is best supported by the
statement made above?
Analysis (A) The average rural household includes more people than
(Cult means an excessive admiration for a person.) The does the average urban or suburban household.
author of the passage first says that John Cleland is becoming (B) Rural households have lower food and housing costs
the Vivaldi of eighteenth-century literature, and then explains than do either urban or suburban households.
why he says so - that Cleland’s cult is being promoted by (C) Suburban households generally have more purchasing
people who do not much care for the period as a whole, and power than do either rural or urban households.
who cheerfully ignore the works of its greatest masters. (D) The median income of urban and suburban households
We have been asked to spot the choice which can be inferred is generally higher than that of rural households.
as the author’s belief in making this statement. (E) All three types of households spend more of their income
The author compares John Cleland to Vivaldi only for the on food and housing than on all other purchases combined.
reason (as is indicated by his use of the phrase ‘in that’) that
Cleland’s cult is being promoted by people ‘who do not much Analysis
care for his period as a whole and cheerfully ignore the work The passage says that rural households at the same income
of the greatest masters of his time’. So, we can infer that the level as urban or suburban households have more purchasing
author believes that Vivaldi was not one of the greatest masters power because they can use for other needs some of the income
of his time. It is (E) which states this, and is the answer. used by the urban or suburban households for food and shelter.
Since the author’s statements “John Cleland is becoming This obviously implies that the rural households spend
the Vivaldi of eighteenth century literature”, and “his cult is less money on food and shelter than urban or suburban
promoted by people who do not much care....” are in the present households.
tense, we can conclude that Cleland is more esteemed now It is (B) which says this, and is the answer.
than he was during his own time. (A) contradicts this, and is If (A) is true, rural households will have to spend more on
therefore wrong. food than do the urban and suburban households. Thus, (A)
The author himself says that Cleland is becoming the contradicts what is stated in the passage, and is wrong.
Vivaldi of eighteenth century. So, it is clear that the period in The given passage combines urban and suburban
which Cleland wrote was the eighteen century, and that there households into a single group and contrasts it with the rural
is no mystery about it. So, (B) also contradicts a specific households. (C), which combines urban and rural households
statement in the given narration, and is wrong. into a single group and contrasts it with suburban households,
The last line complains that those who are promoting the is irrelevant to what is stated in the passage.
cult of John Cleland are ignoring “the works of its (eighteenth While (D) may be a factual statement, it does not follow
century’s) greatest masters”. This only implies that some immediately from the given information.
works of eighteenth century’s greatest masters deserve more (E) is an uncalled-for generalization not warranted by the
attention than Fanny Hill, and not that most eighteenth information in the passage.
century novels deserve more attention than Fanny Hill. So,
(C) extends the scope of what is stated in the passage, and is In the following pages, we are giving four sets of 10
wrong.
‘inference questions’ each.
If the cult of Cleland is presently developing like the cult of
Try to answer each set within 12 minutes by yourself before
Vivaldi, it means that the cult of Vivaldi had developed earlier.
referring to its analysis.
(D) states just the opposite of this, and is not the answer.

26
American Education Aids - LSAT - Logical Reasoning
SET 1
1. A company’s two divisions performed with remarkable homicides fall to a level below average. The yearly murder
consistency over the past three years: in each of those years, rate, however, is not affected by the number of well-publicized
the pharmaceuticals division has accounted for roughly 20 executions”
percent of dollar sales and 40% percent of profits, and the If the above passage is true, which of the following can be
chemicals division of the balance. validly inferred from it?
Which of the following can properly be inferred regarding (A) In the period studied, the number of well-publicized
the past three years from the statement above? executions remained virtually the same from year to year.
(A) Total dollar sales for each of the company’s divisions (B) For at least one week during any years in which there
have remained roughly constant. were well-publicized executions, murder rates were above
(B) The pharmaceuticals division has faced stiffer average.
competition in its markets than has the chemicals division. (C) During some weeks of each year of the study, an
(C) The chemicals division has realized lower profit per extraordinary number of public executions took place.
dollar of sales than has the pharmaceuticals division. (D) In the past, newspapers considered capital punishment
(D) the product mix offered by each of the company’s important news and devoted extensive coverage to all
divisions has remained unchanged. executions.
(E) Highly unprofitable products accounted for a higher (E) Many executions that took place during the period
percentage of the chemical division’s sales than those of the studied were not reported objectively by the media.
pharmaceuticals division.
6. “Weeds and trees illustrate extreme opposites of
2. Large national budget deficits do not cause large trade ecological adaptation. Weeds put most of their energy into
deficits. If they did, countries with the largest budget deficits production and dispersal of the season’s seeds and then die.
would also have the largest trade deficits. In fact, when deficit Trees put most of their energy into sustaining their own
figures are adjusted so that different countries are reliably growth and competitive ability over a long life span, while
comparable to each other, there is no such correlation. each season’s seed production is relatively moderate”.
If the statements above are all true, which of the following It can be inferred from the passage above that the author
can properly be inferred on the basis of them? uses which of the following criteria in judging weeds and trees
(A) Countries with large national budget deficits tend to to be opposites?
restrict foreign trade. (A) The time it takes weed seeds and tree seeds,
(B) Reliable comparisons of the deficit figures of one respectively, to germinate
country with those of another are impossible. (B) The total number of seeds that weeds and trees scatter
(C) Reducing a country’s national budget deficit will not over a given area in their respective habitats
necessarily result in a lowering of any trade deficit that (C) The differential dependence of weeds and trees on
country may have. climatic stability from growing season to growing season
(D) When countries are ordered from largest to smallest (D) The ratio between a season’s seed production and
in terms of population, the smallest countries generally have overall life span for weeds and trees respectively
the smallest budget and trade deficits. (E) The ratio between seeds that are produced and seeds
(E) Countries with the largest trade deficits never have that develop into mature specimens
similarly large national budget deficits.
7. “If Wanda visits Albuquerque on a trip, she will also
3. Jack: Jim has been arrested for stealing. visit Santa Fe, Phoenix and Tucson on that trip”
Bob: He and I are good friends, so I know he’s innocent. We can infer from the above statement that
From the conversation above, it can be inferred that (A) if Wanda visits Santa Fe, Phoenix and Tucson on a
(A) Bob knows Jim is innocent. trip, she will also visit Albuquerque on that trip.
(B) Bob thinks Jack is lying. (B) if Wanda visits Phoenix and Tucson on a trip, she will
(C) Jack saw Jim being arrested. also visit Santa Fe on that trip.
(D) Jack thinks Jim is guilty. (C) if Wanda visits Tucson on a trip, she will also visit
(E) Bob thinks Jim is innocent. Phoenix on that trip
(D) If Wanda does not visit Tucson on a trip, she will not
4. Antonio: Modern Art is a hoax. A child could scribble a visit Santa Fe on that trip.
better picture than most modern artists. (E) If Wanda does not visit Phoenix on a trip, she will not
Belinda: But I’m sure that they’ve all spent many years visit Albuquerque on that trip.
studying and training.
We can infer from the above conversation that Belinda has 8. Butterfat gets its yellowish color from carotene, the fat-
misunderstood Antonio to mean that modern artists soluble vitamin, a precursor that is also responsible for the
(A) have no talent. color of carrots. Not all butter-fat, however, is equally yellow.
(B) refuse to consider the public’s taste in art Some breeds of cows are more efficient than others at
(C) are not sufficiently qualified for their work converting carotene to the colorless vitamin A.
(D) deceive the public with hastily executed designs. If butter made from the milk of holstein cows is much paler
(E) are concerned only with commercial success. than that made from the milk of jersey or guernsey cows, then
the facts above suggest that
5. “According to a recent historical study, capital (A) there is less butterfat in the milk of holsteins than in
punishment deters murder only during weeks when well- the milk of jerseys or guernseys
publicized executions take place. During such weeks, (B) there is more vitamin A in the butterfat of holsteins
27
American Education Aids - LSAT - Logical Reasoning
than in the butterfat of jerseys or guernseys (E) Corporations that have vigorous affirmative-action
(C) there is more carotene in the butterfat of holsteins than programs also tend to follow stringent environmental-
in the butterfat of jerseys of guernseys protection policies.
(D) holsteins are less efficient converters of carotene than
are jerseys or guernseys 10. Sales taxes tend to be regressive, affecting poor people
(E) the carotene in the milk of holsteins is less fat-soluble more severely than wealthy people. When all purchases of
than the carotene in the milk of jerseys or guernseys consumer goods are taxed at a fixed percentage of the
purchase price, poor people pay a larger proportion of their
9. According to a recent study, fifteen corporations in the income in sales taxes than wealthy people do.
United States that follow a credo of social responsibility are It can be correctly inferred on the basis of the statements
also very profitable. Because of their credos, these fifteen above that which of the following is true?
corporations give generously to charity, follow stringent (A) Poor people constitute a larger proportion of the tax-
environmental-protection policies, and have vigorous
paying population than wealthy people do
affirmative-action programs.
(B) Poor people spend a larger proportion of their income
Which of the following can be correctly inferred from the
on purchases of consumer goods than wealthy people do
statements above?
(A) Following a credo of social responsibility helps to make (C) Wealthy people pay, on average, a larger amount of
a corporation very profitable. sales taxes than poor people do
(B) It is possible for a corporation that follows a credo of (D) The total amount spent by all poor people on purchases
social responsibility to be very profitable. of consumer goods exceeds the total amount spent by all
(C) A corporation that gives generously to charity must be wealthy people on consumer goods
doing so because of its credo of social responsibility. (E) The average purchase price of consumer goods bought
(D) Corporations that are very profitable tend to give by wealthy people is higher than that of consumer goods
generously to charity. bought by poor people

SET 2
1. Experienced pilots often have more trouble than novice each letter always represents the same digit. If the sum
pilots in learning to fly the newly developed ultralight ALMA
airplanes. Being accustomed to heavier aircraft, experienced + T I A
pilots, when flying ultralight craft, seem not to respect the = JU AN
wind as much as they should. holds when expressed in digits, all of the following can
The passage implies that the heavier aircraft mentioned properly be inferred EXCEPT:
above were (A) A cannot be 0.
(A) harder to land than ultralight aircraft (B) A must be less than 5.
(B) not as popular with pilots as ultralight aircraft (C) N must be even.
(C) not as safe as ultralight aircraft (D) L + T must be greater than 8.
(D) more fuel-efficient than ultralight aircraft (E) J must be greater than A by 1.
(E) easier to handle in wind than ultralight aircraft
4. “Although historical research and anthropological
2. A placebo is a chemically inert substance prescribed more research often complement one another, the two disciplines
for the mental relief of a patient than for its effect on the retain important distinctions because of the fundamental
patient’s physical disorder. It is prescribed in the hope of questions that each pose. Historians are mainly interested
instilling in the patient a positive attitude toward prospects in why change occurred, whereas anthropologists are
for his or her recovery. In some cases, the placebo actually concerned with the meanings of actions and systems”
produces improvement in the patient’s condition. In Which of the following is implied in the passage above?
discussing the use and effect of placebos, a well-known (A) The meanings of actions and systems do not fully
medical researcher recently paid physicians the somewhat explain why change occurred.
offbeat compliment of saying that physicians were the (B) the distinctions between life and art are diminishing.
ultimate placebo. (C) Actions and systems are usually subject to historical
By comparing a physician to a placebo, the researcher change.
sought to imply that (D) Anthropological research is not concerned with change.
(A) physicians should always maintain and communicate (E) The questions posed by the two disciplines share certain
an optimistic attitude toward their patients, regardless of assumptions about human behavior.
the prognosis
(B) the health of some patients can improve simply from 5. “Ken and Lou will eat together at a restaurant only if
their knowledge that they are under a physician’s care Jan comes along with them. Jan will go only to restaurants
(C) many patients actually suffer from imagined illnesses in her neighbourhood, which is several miles from the
that are best treated by placebos downtown area. Manuel will go to a restaurant only if Ken
(D) physicians could prescribe less medication and achieve also goes.”
the same effect From the statements above, we can reasonably infer
(E) it is difficult to determine what, if any, effect a (A) Manuel does not eat at a restaurant with Jan.
physician’s behavior has on a patient’s condition (B) Lou does not eat at a restaurant with Manuel and Jan.
3. In a certain code, the digits from 0 to 9 inclusive are (C) Ken, Manuel, and Lou do not eat together at a
each represented by a different letter of the alphabet and restaurant.
28
American Education Aids - LSAT - Logical Reasoning
(D) Manuel does not eat at a restaurant in the downtown Which of the following can be properly inferred from the
area. statements above?
(E) Ken and Lou do not eat together at a restaurant in the (A) The idea behind some computer programs can be
downtown area. distinguished from the expression of that idea.
(B) Anyone who writes a computer program is the inventor
6. “In a political system with only two major parties, the of the idea of that program.
entrance of a third-party candidate into an election race (C) Most products that are copyrighted are expressions of
damages the chances of only one of the two major candidates. ideas that are patented.
The third-party candidate always attracts some of the voters (D) Few inventors are owners of both patents and
who might otherwise have voted for one of the two major copyrights.
candidates, but not the voters who support the other (E) A patent for a computer program is no more difficult to
candidate. Since a third-party candidacy affects the two major win than a copyright.
candidates unequally, for reasons neither of them has any
control over, the practice is unfair and should not be allowed”. 9. The disparity between the location of jobs and the
If the factual information in the above passage is true, which location of affordable housing has been growing throughout
of the following can be most reliably inferred it? the country in recent years. Housing availability has not kept
(A) If the political platform of the third party is a up with the growth of jobs in suburban regions; the
compromise position between that of the two major parties, consequences are long commutes from rural and urban areas
the third party will draw its voters equally from the two major for middle-income workers and inaccessibility of job
parties. opportunities for lower-income workers.
(B) If, before the emergence of the third party, voters were Which of the following is the most reliable inference that
divided equally between the two major parties, neither of the can be drawn from the passage above?
major parties is likely to capture much more than one-half of (A) Lower-income workers cannot afford the expense of
the vote. long commutes to jobs
(C) A third-party candidate will not capture the votes of (B) Suburban housing is becoming less expensive
new voters who have never voted for candidates of either of (C) Middle-income workers tend to live closer to their places
the two major parties. of work than do lower-income workers
(D) The political stance of a third party will be more radical (D) Housing availability for workers has been an important
than that of either of the two major parties. factor in decisions to relocate businesses
(E) The founders of a third party are likely to be a coalition (E) Affordable housing in urban areas has become less
consisting of former leaders of the two major parties. available in recent years

7. Biochemists select for experimental study those types 10. Radio interferometry is a technique for studying details
of organisms that are most suitable for solving the scientific of celestial objects that combines signals intercepted by
problems that are of current interest to them. For this reason, widely-spaced radio telescopes. This technique requires
it was not until recently that biochemists began to give ultraprecise timing, exact knowledge of the locations of the
considerable attention to the biochemistry of insects. telescopes, and sophisticated computer programs. The
In the passage above, the author implies that successful interferometric linking of an Earth-based radio
(A) there was a recent shift in the scientific problems of telescope with a radio telescope on an orbiting satellite was
interest to biochemists therefore a significant technological accomplishment.
(B) scientists have recently made new contributions to Which of the following can be correctly inferred from the
knowledge by studying the biochemistry of insects statements above?
(C) biochemists recently changed their primary criterion (A) Special care was taken in the launching of the satellite
for selecting the type of organism to be studied so that the calculations of its orbit would be facilitated.
(D) there was a recent increase in the number of individual (B) The signals received on the satellite are stronger than
organisms that biochemists use in their experiments those received by a terrestrial telescope.
(E) biochemists would today have less interest in studying (C) The resolution of detail achieved by the satellite-Earth
insects if they had given more attention in the past to the interferometer system is inferior to that achieved by
biochemistry of insects exclusively terrestrial systems.
(D) The computer programs required for making use of
8. Computer programs are unusual in that they are the signals received by the satellite required a long time for
virtually the only products that have been protected both by development.
patent and by copyright. Patents protect the idea behind an (E) The location of an orbiting satellite relative to locations
innovation, whereas copyrights protect the expression of that on Earth can be well enough known for interferometric
idea. However, in order to win either protection, the idea must purposes.
be clearly distinguished from its expression.

29
American Education Aids - LSAT - Logical Reasoning
SET 3
1. For large farms to be as productive as they can be, their 4. A ten-year comparison between the United States and
owners need to invest heavily in expensive machinery. This the Soviet Union in terms of crop yields per acre revealed
typically requires them to go into considerable debt, and that when only planted acreage is compared, Soviet yields
interest on this debt is then a significant fixed cost. This high are equal to 68 percent of United States yields. When total
fixed cost makes those farmers vulnerable to operating losses agricultural acreage (planted acreage plus fallow acreage) is
if the prices of their products drop. compared, however, Soviet yield is 114 percent of United
The information above best supports which of the following States yield.
inferences about large farms? From the information above, which of the following can be
(A) They can be highly productive without being profitable most reliably inferred about United States and Soviet
(B) They tend to be so highly productive that they drive agriculture during the ten-year period?
down market prices (A) A higher percentage of total agricultural acreage was
(C) They tend to be consistently profitable if their owners fallow in the United States than in the Soviet Union.
borrow at low interest rates (B) The United States had more fallow acreage than
(D) They respond to operating losses by increasing their planted acreage.
productivity (C) Fewer total acres of available agricultural land were
(E) They cannot be profitable if their owners depend on fallow in the Soviet Union than in the United States.
credit (D) The Soviet Union had more planted acreage than fallow
acreage.
2. The nuclear polyhedrosis virus helps control gypsy moth (E) The Soviet Union produced a greater volume of crops
populations by killing the moth’s larvae. The virus is always than the United States produced.
present in the larvae, but only every sixth or seventh year
does the virus seriously decimate the number of larvae, 5. “Scientists do not live or work in isolation from the rest
thereby drastically setting back the gypsy moth population. of the world. They toil in government laboratories, universities
Scientists believe that the virus, ordinarily latent, is triggered and industry. Yet they espouse only what scientific evidence
only when the larvae experience biological stress. convinces them to be true, not what the public wants to hear.
If the scientists mentioned above are correct, it can be In fact, nothing ruins scientific research faster than mixing
inferred that the decimation of gypsy moth larvae populations it with political considerations”
by the nuclear polyhedrosis virus would be most likely to be It can be inferred from the passage that the author believes
triggered by which of the following conditions? that
(A) A shift from drought conditions to normal precipitation (A) the public has no faith in the discoveries of scientists.
in areas infested by gypsy moths (B) scientists cannot be effective politicians.
(B) The escalating stress of defoliation sustained by trees (C) scientific conclusions have few political implications.
attacked by gypsy moths for the second consecutive year (D) political aims are likely to be irrelevant to scientific
(C) Predation on larvae of all kinds by parasitic wasps and research.
flies (E) scientific research should be apolitical.
(D) Starvation of the gypsy moth larvae as a result of
overpopulation 6. By analyzing the garbage of a large number of average-
(E) Spraying of areas infested by gypsy moths with sized households, a group of modern urban anthropologists
laboratory-raised nuclear polyhedrosis virus have found that a household discards less food the more
standardized – made up of canned and prepackaged foods –
3. Consider three fish swimming together in a school. The its diet is. The more standardized a household’s diet is,
space within which each fish can be seen by predator Y is however, the greater the quantities of fresh produce the
defined by a sphere centered on the fish and having a radius household throws away.
that is the maximum distance Y can see. The school is Which of the following can be properly inferred from the
vulnerable to attack when Y is within one of the three spheres. passage?
The spheres overlap to a great extent, since the fish are in a (A) An increasing number of households rely on a highly
compact group. standardized diet
Which of the following is a reliable inference to be drawn (B) The less standardized a household’s diet is, the more
from the passage above? nonfood waste the household discards
(A) The vulnerability to attack of the school as a whole is (C) The less standardized a household’s diet is, the smaller
not much greater than the vulnerability to attack of any one is the proportion of fresh produce in the household’s food waste
fish in the school. (D) The less standardized a household’s diet is, the more
(B) There is less chance that predator Y will attack a school canned and prepackaged foods the household discards as
of four fish than that it will attack a school of three fish. waste
(C) Fish who swim in schools are less likely to be devoured (E) The more fresh produce a household buys, the more
by predators than are fish who do not swim in schools. fresh produce it throws away
(D) The maximum distance at which an individual fish is
visible depends less on the size of the fish than on whether 7. The rush to use distilled grains as petroleum substitutes
the fish is swimming in a school. poses potential market problems. By 1965, the value of corn
(E) The maximum distance at which predator Y can see as alcohol will exceed the value of corn as food. Alcohol
its prey is increased if Y is itself swimming in a school of Y’s. produced from grain will displace some imported oil, and the
price of oil will begin to dictate the price of corn.

30
American Education Aids - LSAT - Logical Reasoning
If the claims made in the passage above are true, which of advertisement in order to boost sales. Instead, one only needs
the following draws the most reliable inference about the effect to attract the potential customer’s attention; memory does
of a reduction in the price of imported oil after 1985? the rest, for it is more important for sales that people know
(A) Some corn would be diverted from energy markets into of a product than that they know something about it.
food markets Which of the following is implied by the passage above?
(B) A downward pressure would be exerted on the price of (A) People can remember a product without having much
corn information about it
(C) An upward pressure would be exerted on the world- (B) Advertisements, in their own way, function to improve
wide demand for corn people’s memories
(D) Farmers would have an incentive to grow more corn (C) Attracting a potential customer’s attention is a simple
(E) Energy companies would have an incentive to produce matter
more domestic oil (D) The advertising industry knows little of substance
about the products it promotes
8. As a tool for public health policy, the epidemiological (E) Advertisements seldom tell the truth about a product
study of disease has one major advantage over experiments
studying the effects of toxic substances on animals: there is 10. Rule 1 of Game X provides that anyone who refuses to
never any call for extrapolating inferences across species in become a player in Game X shall at the moment of refusal be
epidemiology. assessed a ten-point penalty in the game.
Which of the following can be inferred from the passage Which of the following claims is implicit in Rule 1?
above? (A) All those who agree to play Game X will achieve scores
(A) In epidemiology, extrapolating inferences across species higher than the scores of those who were assessed a penalty
is only an auxiliary tool under Rule 1.
(B) Epidemiology is a very precise science
(B) A person can avoid a ten-point penalty by initially
(C) In epidemiology, human populations are studied
agreeing to become a player and then withdrawing after the
directly
game is under way.
(D) Toxicological experimentation on animals is more
expensive than are epidemiological studies (C) The rules of Game X supply a procedure for determining
(E) Toxicological experimentation on animals yields results when the game is over.
less quickly than do epidemiological studies (D) A person who refuses to play Game X cannot be declared
a loser in the game.
9. One of the truisms of the advertising industry is that it (E) A person can at the same time decline to play Game X
is rarely necessary to say something of substance in an and yet be a part of the game.

SET 4
1. A man charged with theft of cable television services 3. At the end of the Second World War the number of women
said, “They even want restitution of $662 they claim I owe in their childbearing years was at a record low. Yet for almost
them, which is ridiculous, because I didn’t enjoy some of those twenty years they produced a record high number of children.
shows at all.” In 1957 there was an average of 3.72 children per family.
From the man’s assertion, we can infer that he Now the postwar babies are producing a record low number
(A) owes the money the cable service claims he owes of babies. In 1983 the average number of children per family
(B) did watch programs that were shown on the cable was about 1.79 – two children fewer than the 1957 rate and
service lower even than the 2.11 rate that a population needs to
(C) does owe the cable service an amount of money, but replace itself.
less than $662 It can properly be inferred from the passage that
(D) was aware that his hookup to the cable service was (A) for the birth rate to be high, there must be a relatively
unauthorized large number of women in their childbearing years
(E) is willing to pay a moderate price for the cable service (B) the most significant factor influencing the birth rate is
whether the country is engaged in a war
2. When a high-pressure front meets a low-pressure front, (C) unless there are extraordinary circumstances, the birth
some form of precipitation usually occurs. Meteorologists are rate will not dip below the level at which a population replaces
able to predict precipitation by measuring the speed at which itself
two fronts are traveling toward one another and determining (D) for the birth rate to be low, there must be a relatively
when and where they will meet. small number of women in their childbearing years
It can be inferred from the passage above that (E) the birth rate is not directly proportional to the number
(A) high-pressure fronts are a sign of rain of women in their childbearing years
(B) the chief function of meteorologists is to predict
precipitation 4. The expression ‘the doctrine of unshakable foundations’
(C) some predictions of precipitation are based on typical was once used by a critic in an effort to illuminate the
reactions of pressure fronts dogmatic nature of certain economic and political philosophies
(D) precipitation can occur without the presence of some whose adherents, when confronted with the failure of a policy
kind of low-pressure front designed to put their philosophy into practice, can conceive
(E) low-pressure fronts always travel in the direction of of only one reaction: to design another, different policy for
high-pressure fronts putting it into practice.

31
American Education Aids - LSAT - Logical Reasoning
It can be inferred from the passage above that the critic varieties of corn unaffected in the areas over which the disease
would approve if the adherents had spread.
(A) had the courage to try a failed policy again without Which of the following can be inferred from the passage
any changes. above?
(B) had refrained from trying to put any of their (A) Susceptibility to certain plant disease is genetically
philosophies into practice determined.
(C) allowed failure of a policy to lead them to question the (B) Eighty percent of the corn grown in the United States
underpinnings of their philosophies is resistant to southern leaf blight.
(D) concluded from the failure of a policy of theirs that the (C) The extinction of wild food plants can be almost every
policy must not have reflected their philosophy adequately case be traced to destructive plant diseases.
(E) carefully analyzed those traits of a failed policy that (D) Plant breeders focus on developing plants that are
appear promising despite the overall failure. resistant to plant disease.
(E) Corn is the only food crop threatened by southern leaf
5. Speakers of a language rely on their general knowledge blight.
of the world in order to make sense of statements they hear
or read. Computers can easily be ‘taught’ the formal rules of 8. A population study based on a ‘microdate sample’, a
a human language, but supplying them with this general sample just large enough to allow reasonably accurate
knowledge is another matter. It is extremely difficult to projections, found that in 1984 there had been a dramatic
develop computers that can extract the meaning of ordinary reversal of a twenty-year tendency toward migration of United
sentences. States residents older than sixty to small towns and rural
Which of the following can be validly inferred from the regions, both within and across state boundaries. This finding
passage above? turns out to have been in error. The problem was that people
(A) The interpretation of the meaning of ordinary sentences who had moved to the United States from abroad were
is dependent on a general knowledge of the world. accidentally counted as part of interstate migration that year.
(B) Computers that can be ‘taught’ the rules of a human Which of the following can be inferred from the passage
language can understand statements in that language. above?
(C) It is futile to try to ‘teach’ computers the formal rules (A) In 1984 the majority of people coming to the United
of any language. States from abroad were over the age of sixty.
(D) Speakers of a language often must reject the formal (B) In 1984 fewer United States residents over the age of
rules of that language in order to understand simple sixty changed residence than in any of the twenty years prior
sentences. to 1984.
(E) Computers that can understand ordinary sentences (C) People over the age of sixty who in 1984 came to the
have all the basic capabilities necessary to become ‘speakers’ United States from abroad did not settle predominantly in
of the language. small towns and rural regions.
(D) The microdata sample for 1984 that was used in the
6. The chief executive officer of one of the country’s most demographic study cited was too small to allow any
successful steel manufacturing firms is amused at the meaningful projections.
publicity her management practices have attracted. She (E) The twenty-year trend mentioned in the passage was
thinks managers get too much of the credit for upswings in due chiefly to moves over relatively short distances, mostly
their businesses. “A fickle public assumes too easily that within the same state.
managers are brilliant when demand is strong and idiotic
when it is weak,” she asserts. 9. Since the recent takeover of publishing concerns by
It can be inferred from the statement made by the chief communications-entertainment firms, management’s new
executive officer that she believes which of the following? methods have increased the financial profits of commercial
(A) A successful manager should attempt to control public publishing, at the price of narrowing the range of books made
opinion. available to the public and by catering to the vulgar tastes of
(B) There are no managers who can justifiably be said to the new buyers of books. Business has boomed; but the losers
be brilliant. are the majority of authors or aspiring writers, and all
(C) Her own management practices are likely to be of little discriminating readers.
value to other businesses. If the statement above is true, which of the following can
(D) The role of managers becomes less important during also be inferred to be true?
economic downturns than it is in relatively prosperous times. (A) Profitable business practices are relatively new in the
(E) The level of demand for the products of manufacturing publishing industry.
firms cannot be completely controlled by the managers of the (B) Commercial publishing is now catering to a different
firms. community of book readers than the book business has served
in the past.
7. To many environmentalists, the extinction of plants - (C) The new profits from methods introduced by
accompanied by the increasing genetic uniformity of species communications-entertainment management will encourage
of food crops - is the single most serious environmental writers of literary talent to persevere against the odds.
problem. Something must be done to prevent the loss of wild (D) The narrowed range of books coming to the public is
food plants or no longer-cultivated food plants. Otherwise, directed toward a more discriminating audience.
the lack of genetic diversity could allow a significant portion (E) The public is unaware of the trend in the publishing
of a major crop to be destroyed overnight. In 1970, for industry to specialize in books that produce blockbuster sales.
example, southern leaf blight destroyed approximately 20
percent of the United States corn crop, leaving very few
32
American Education Aids - LSAT - Logical Reasoning
10. The average after-tax income for a household was 2.4 in 1983 than in 1982.
percent higher in 1983 than in 1982. At the same time, (C) Total after-tax income for all households at the lower-
average after-tax income declined for households at the lower- income and middle-income levels was higher in 1983 than in
income and middle-income levels. 1982.
Which of the following can be reasonably inferred from the (D) Average after-tax income for households at the upper-
information above? income level rose by more than 2.4 percent between 1982
(A) There were more households overall in 1983 than in and 1983.
1982. (E) Average after-tax income for households at the lower-
(B) There were fewer households at the upper-income level income and middle-income levels was declining prior to 1982.

33
Chapter 4
'ASSUMPTION' QUESTIONS
Even in day-to-day life, when you come to a particular is: If what is contained in the statement is not true, the
conclusion on any matter, you may mention the overt reasons stated conclusion will not be valid.
for it to a third person to justify its soundness.
But, if you examine the issue carefully, you will find that, To be considered a valid assumption, the statement must
in addition to the stated reasons, there were other unstated have a direct, and not an indirect, nexus to the given
reasons which played an equally important role in your conclusion.
arriving at that conclusion. For example, the assumption “The tickets for the movie
show have not been sold out” has a direct nexus to the
As an example, let us recall the statement we had used to conclusion that, if the speaker leaves home within 15 minutes,
explain the ‘inference’ questions. he will be able to see the movie from the beginning.
“The time is 5.30 now and the movie starts at 6.30. Unless Because, if the above assumption is not valid, he will not
we leave home within the next 15 minutes, we shall be missing be able to see the movie at all, and his conclusion would be
its beginning” wrong.
The conclusion of the speaker in this case is that, unless
he starts before 5.45 for the movie which commences at 6.30, Now, consider the statement, “There have been
he would miss its beginning. unfavourable reviews of the movie in film magazines”.
This statement also implies that, if he starts before 5.45, You may be tempted to infer from this statement that,
he will not miss its beginning. therefore, there is not likely to be heavy demand for tickets
A relevant reason for his conclusion but which is not for the movie and, therefore, the tickets for the show will be
specifically stated by the speaker, is that he either thinks or available, and, therefore, the speaker’s conclusion that he will
knows that the travelling time between his house and the movie be able to see the movie from the beginning if he reaches the
house is 45 minutes. theater before 6.30 is valid.
If the travelling time is, in fact, 75 minutes, his conclusion This statement requires two inferences in a sequence to
that leaving home before 5.45 will enable him to watch the render the conclusion valid and, therefore, does not have a
movie from the beginning would be unsound. direct nexus to the given conclusion.
On the other hand, if the travelling time is only 15 minutes, Therefore, if this statement is given as one of the choices,
his conclusion that he would miss the movie’s beginning if he you should not identify it as a valid assumption.
does not start before 5.45 would again be wrong - because Let us now apply the touchstone mentioned earlier: Even
even if he leaves by 6.15 he will not miss the beginning. if this statement (‘There have been unfavourable reviews of
Therefore, the soundness of the speaker’s conclusion the movie in the film magazines”) is not true, the speaker’s
depends on the assumption that the travelling time from his conclusion that he will miss the beginning of the movie if he
house to the movie house is 45 minutes. does not start within 15 minutes will still be valid.
This confirms that this statement is not a necessary
In the parlance of LSAT, such an unstated reason which is, assumption for the speaker’s conclusion.
nevertheless, relevant to the soundness of a given conclusion
is known as an assumption. The narration in an ‘assumption’ question in Logical
Reasoning / Critical Reasoning will give (i) one or two stated
Every day-to-day decision of ours depends upon a number reasons (or premises) and (ii) a conclusion that the author
of such unstated assumptions. has drawn therefrom, and will ask you to spot, from among
In the above illustration, the speaker makes the following the given five choices, a further assumption that has been
further assumptions while stating his conclusion: made by him for reaching that conclusion.
(i) There will be no traffic jam on the way that will lengthen Among the five choices, only one will constitute such an
the travelling time beyond 45 minutes today. assumption while the other four will be irrelevant to the
(ii) The movie will start at 6.30, and neither earlier nor conclusion, and you have select it as the answer.
later.
(iii) The tickets for the movie show have not been sold out. In another variant of the ‘assumption’ question, four of
(iv) His companion is equally interested in seeing the movie the five given choices will be necessary assumptions, and you
from the beginning. will be asked to spot the only choice that is not a necessary
assumption.
The following statements cannot be considered to be
relevant assumptions made by him while coming to his The synonyms of the word ‘assumption’ are ‘presumption’,
conclusion: ‘presupposition’, ‘supposition’ and ‘belief’.
(i) There will be no rain on the way. (He may be travelling Their verb forms are ‘assume’, ‘presume’, ‘presuppose’,
by car, or may have an umbrella.) ‘suppose’ and ‘believe’.
(ii) The movie will last exactly two hours. The ‘assumption’ questions therefore may take the
(iii) There have been adverse reviews of the movie in film following forms:
magazines. 1. The conclusion above depends on which of the following
assumptions?
The touchstone for a particular statement to be 2. Which of the following is an assumption made in drawing
considered a valid assumption for a stated conclusion the conclusion above?
34
American Education Aids - LSAT - Logical Reasoning
3. In coming to the above conclusion the author presumes and observe their behavior with the new packaging.
that ..... The marketing director’s conclusion depends on which of
4. Which of the following is an assumption that will render the following assumptions?
the above conclusion logically correct? (A) If children playing with the new packaging spill juice
5. Which of the following is a presupposition of the argument on their parents’ furniture and carpets, the juice company
above? will be legally liable
6. The conclusion above would be more reasonably drawn (B) If children’s behavior with the new packaging is
if which of the following were inserted into the argument? observed, company personnel will be able to predict the sales
7. A plausible explanation of the researcher’s strategy is volume of the new packaging
that he believes that ..... (C) The children’s behavior with the new packaging in the
8. The company’s plan as described above assumes each of company’s laboratories will be a good indicator of children’s
the following EXCEPT behavior with the packaging at home
(D) Children playing in small groups will be less likely to
We shall now give you a number of typical examples of use the new packaging as a toy than will a child playing alone
‘assumption’ questions and analyse them for your better (E) The new packaging will be an improvement over
understanding of how they should be answered. previous packaging of grape juice if children do not use it as
a toy.
Example 1
“There is no reason to rule out the possibility of life on Analysis
Uranus. We must, then, undertake the exploration of that The Marketing Director first says that there is a possibility
planet”. that children will use the new grape juice packaging proposed
The argument above assumes that to be introduced by the Company as a toy, and will spill juice
(A) life exists on Uranus on their parents’ furniture and carpets.
(B) Uranus is the only other planet in the solar system (His concern, obviously, is that, if this happens, no parent
capable of supporting life will buy the juice in the new package a second time, and it
(C) Uranian life would be readily recognizable as life will affect the company’s sales.)
(D) the search for life is a sufficient motive for space He then says that the best way to find out (whether this
exploration possibility is likely to happen) is to assemble small groups of
(E) no one has previously proposed the exploration of children in the Company’s laboratories and observe their
Uranus behavior with the new packaging.
Obviously, what the Marketing Director assumes (while
Analysis talking of the ‘best way’ to find out about the possibility) is
From the only stated premise “There is no reason to rule that, if small groups of children who are assembled at the
out the possibility of life in Uranus”, the speaker comes to company’s laboratories use the new packaging as a toy and
the conclusion that “we must undertake the exploration of spill the juice on the furniture and the carpets, they are likely
that planet”. to do the same in their own houses also.
So, he assumes that the search for life is a sufficient reason In other words, he assumes that the children’s behaviour
for the exploration of other planets. with the new packaging in the company’s laboratories will be
It is (D) which is closest to this statement, and is the a good indicator of their behaviour with the packaging at home
answer. also.
The speaker only guesses that there may be life on Uranus. It is (C) which states this, and is the answer.
(A) raises the scope of his belief to the level of a certainty, and The Marketing Director’s immediate aim is to find out,
is wrong. through an experiment, whether children are likely to use
The speaker’s statement is confined to Uranus, and there the new packaging as a toy and spill juice on their parents’
is no reference in it to the possibility or impossibility of life in furniture and carpets. The legal liability, if any, will arise
any other planet in the solar system. So, (B) also extends the only as a subsequent consequence of the new packaging being
scope of his statement beyond what immediately follows from sold in the market and the children using it as a toy. So, (A)
it, and is wrong. does not have an immediate and direct nexus to the Marketing
Apart from speculating that there may be life on Uranus, Director’s proposal to conduct an experimental study, and is
the speaker does not speculate on how it will look like. So, not a necessary assumption by him.
(C) also extends the scope of the speaker’s statement, and is The prediction of the sales volume of the new packaging
wrong. (If there is indeed life on Uranus, it may have will also arise as a subsequent consequence of children using
thousands of different forms as it does on earth, and not just it as a toy and spilling juice on their parents’ furniture and
a single form!) carpets, and does not have a direct and immediate nexus to
While it may be true that noone has previously explored the proposal to conduct an experimental study. So, (B) is not
Uranus, the statement that noone has earlier proposed the the answer.
exploration of Uranus also extends the scope of what the (D) implies that the behaviour of the children playing in
speaker says, and is not a necessary assumption for his small groups is not representative of their behaviour while
conclusion. playing alone. This assumption would nullify the very purpose
of conducting the proposed experiment with small groups of
Example 2 children. So, (D) contradicts the thinking of the Marketing
Marketing director: There is a possibility that children will Director, is not an assumption by him.
use our new grape juice packaging as a toy and will spill juice (E) is in the nature of another conclusion by the Marketing
on their parents’ furniture and carpets. The best way to find Director, which will be drawn after the result of the
out is to assemble small groups of children in our laboratories experiment is known. It is not his assumption while proposing
35
American Education Aids - LSAT - Logical Reasoning
the experiment itself. is sick much of the time.
So, the assumption underlying the conclusion that Harry
Example 3 is sick much of the time is that Harry writes letters frequently,
If the laws of biology are ultimately based on the laws of and, in each of these, he mentions that he is sick.
physics and chemistry, then the laws of biology hold for living So, (D) is the answer.
organisms wherever they may be found in the galaxy and The statement that ‘Harry’s work cannot be done at home’
the universe. is an unwarranted extension of the scope of the given
The conclusion above logically depends on which of the information, and is not a necessary assumption in coming to
following assumptions? the speaker’s conclusion. So, (A) is wrong.
(A) The science of biology is not as precise as the sciences The statement ‘Harry is not sick when he is at the office’
of physics and chemistry could be another conclusion from the given information, and
(B) The sciences of biology, physics, and chemistry are is not an assumption for concluding that he is sick much of
ultimately a single science the time. So, (B) is wrong.
(C) The laws of physics and chemistry explain a narrower That Harry writes long letters is an unwarranted extension
range of phenomena than do the laws of biology of the scope of the statement that, whenever he writes a letter,
(D) The laws of physics and chemistry hold throughout he says that he is sick. This is not a necessary assumption in
the universe this case. So, (C) is wrong.
(E) A scientific law is necessarily universal (E) is a sweeping generalization not confined to Harry but
to everybody, and is wrong for that reason alone.
Analysis
The premise in this case is that the laws of biology are Example 5
ultimately based on the laws of physics and chemistry. Comparison of the growth rings of ancient trees enables
The conclusion drawn is that laws of biology should, scientists to determine from a piece of timber the year in
therefore, hold good for living organisms wherever they are which the tree used for the timber was felled. Hence, by
found in the universe. analyzing the growth rings in timber surviving from ancient
Obviously, the author of this sentence assumes that the buildings, archaeologists can determine precisely when those
laws of physics and chemistry hold good throughout the galaxy buildings were constructed.
and the universe. (D) says just this, and is the answer. Which of the following is an assumption on which the
The statement that the biology is not as precise as physics argument depends?
and chemistry is a sweeping statement that is not borne out (A) The timber used for construction purposes in ancient
by the information in the given sentence. So, (A) is wrong. times was made from very old trees.
The statement that ‘biology, physics and chemistry are (B) The timber that was used in ancient building
ultimately a single science’ extends the scope of the given construction had not, prior to being used in the construction
statement that ‘biology is ultimately based on physics and of the buildings, lain unused for an indeterminable amount
chemistry’ beyond its immediate implication. So, (B) is not a of time.
necessary assumption by the author. (C) The growth rings of any tree felled in a given year are
(C) is also an unwarranted extension of the scope of the identical to the growth rings of any other tree felled in that
statement that the laws of biology are based on the laws of year.
physics and chemistry. (D) The oldest of the ancient buildings that survive to the
If (E) were the assumption, the author could straight-away present day were constructed of the most durable woods.
have stated that the laws of biology, being scientific laws, (E) Ancient builders did not use more than one type of
hold good throughout the universe. He need not have wood for the construction of a given building.
mentioned the dependence of laws of biology on laws of physics
and chemistry for coming to his conclusion. So, (E) is wrong. Analysis
The first sentence says that a study of the growth rings in
Example 4 timber can enable scientists to determine the year in which a
We know that Harry is sick much of the time because tree was felled.
whenever we get a letter from him he tells us that he is home From this information, the second sentence draws the
from the office with the flu, a cold, or an injury. conclusion that an analysis of the growth rings of the timber
The logical assumption made in the passage above is that used in an ancient building can enable scientists to precisely
(A) Harry’s work cannot be done at home determine the year in which the building was constructed.
(B) Harry is not sick when he is at the office Obviously, the author is making the assumption that there
(C) Harry writes long letters was no long lapse of time between the time the tree was felled
(D) Harry writes frequently and the time when it was used in the construction of the
(E) being home from the office is equivalent to being sick building. It is (B) which states this, and is the answer. (If the
timber had been used in the building, say, 20 years after it
Analysis was felled, equating the age of the building to the age of the
The second part of the statement says that whenever Harry timber would give an inaccurate result.)
writes a letter, he mentions that he has not gone to office but (A) is an obvious fact, but does not by itself lead to the
is at home because of flu, a cold or an injury. conclusion that the building’s age can be precisely determined
From this fact the speaker comes to the conclusion that from the timber used in it. So, it is not the answer.
Harry is sick much of the time. The conclusion is about determining the age of an ancient
If Harry writes letters only rarely, say, once in six months building in which a piece of timber has been used. (C), which
or once a year, and he mentions in them that he is at home does not even mention the building, is not an assumption that
because of illness, one cannot come to the conclusion that he can lead to that conclusion.
36
American Education Aids - LSAT - Logical Reasoning
The phrase ‘timber surviving from ancient buildings’ im- Example 7
plies that some of the timber used in these buildings has A famous singer recently won a lawsuit against an
already perished. So, (D) contradicts the information in the advertising firm for using another singer in a commercial to
given passage, and is not the answer. evoke the famous singer’s well-known rendition of a certain
Since the age of any timber can be determined by studying song. As a result of the lawsuit, advertising firms will stop
the growth rings in it, whether only one type or more than using imitators in commercials. Therefore, advertising costs
one type of timber was used in an ancient building is irrelevant will rise, since famous singers’ services cost more than those
to the given conclusion. So, (E) is not a necessary assumption of their imitators.
in this case. The conclusion above is based on which of the following
assumptions?
Example 6 (A) Most people are unable to distinguish a famous singer’s
The federal government expects hospitals to perform rendition of a song from a good imitator’s rendition of the
10,000 organ transplants next year. But it is doubtful that same song.
this many donor organs will be available, since the number (B) Commercials using famous singers are usually more
of fatalities resulting from car and motorcycle accidents has effective than commercials using imitators of famous singers.
been dropping steadily over the past decade. (C) The original versions of some well-known songs are
The argument above makes which of the following unavailable for use in commercials.
assumptions? (D) Advertising firms will continue to use imitators to
(A) The number of car and motorcycle accidents will mimic the physical mannerisms of famous singers.
increase significantly during the next year. (E) The advertising industry will continue to use well-
(B) No more than 10,000 people will be in need of organ known renditions of songs in commercials.
transplants during the next year.
(C) A significant number of the organs used in transplants Analysis
come from people who die in car and motorcycle accidents. The narration concludes that, because of the court’s order
(D) In the past the federal government’s estimates of the that imitators cannot be used in advertisement commercials
number of organ transplants needed during a given year have to sing well-known renditions of songs by famous singers,
been very unreliable. advertising costs will go up, because famous singers will
(E) For any given fatality resulting from a car or motorcycle charge more for singing these songs than the imitator.
accident, there is a hospital in the vicinity in need of an organ Obviously, this conclusion will be valid only if an
for a transplant. assumption is made that the advertising industry will
continue to use well-known renditions of songs in their
Analysis commercials, and have to necessarily commission the famous
(Fatalities means deaths.) The only reason why the author singers to sing them, thereby incurring a higher cost.
doubts whether sufficient number of donor organs will be (E) is what states this, and is the answer.
available to meet the estimated 10,000 organ transplants in (If, on the other hand, the industry discontinues the use of
hospitals next year is that the number of fatalities resulting such songs and takes to some other cheaper method of
from car and motorcycle accidents has been dropping steadily advertising, there will be no additional advertising cost
over the past decade. arising out of the result of the lawsuit.)
Obviously, the author assumes that donor organs used in None of the other choices can logically lead to the conclusion
transplants mostly come from persons who die in car and that the advertising costs will go up because of the decision in
motorcycle accidents. Choice (C) says just this, and is the the lawsuit referred to.
answer.
The author’s argument is based on a continuing decrease Example 8
in the number of fatalities in car and motorcycle accidents. In March, 300 college students turned out in Washington
An assumption about an increase in the number of car and to protest against proposed cuts in student loan funds.
motorcycle accidents is not relevant to his argument. So, (A) Another 350,000 collegians flocked to Florida’s sun-drenched
is not the answer. beaches during March for “spring break”. Since the Florida
The number 10,000 is mentioned to be an estimate of the sun-seekers were more numerous, they were more
government. Therefore, according to the passage, the actual representative of today’s students than those who protested
figure could be a few more or a few less than this number. in Washington, and therefore Congress need not heed the
The assertion that it will not be more than 10,000 contradicts appeals of the protesting students.
this information, and is not a necessary assumption for the The argument above makes which of the following
stated conclusion. assumptions?
The argument of the author is not based on the assumption (A) The students who vacationed in Florida did not oppose
that the estimate of the need for 10,000 transplants is wrong. the cutting of student loan funds by Congress.
In fact, he presumes that this estimate is correct, but argues (B) The students who vacationed in Florida were not in
that this many number of organs are not likely to be available agreement with the opinion of the majority of United States
for transplant. (D) thus contradicts a specific information in citizens about the proposed cut in loan funds.
the narration, and is not the answer. (C) The students who protested in Washington were more
The author does not state or even imply that proximity of seriously concerned about their education than the students
the site of a fatal accident to a hospital is a prerequisite for who vacationed in Florida.
the successful transplant of an organ from a deceased person (D) The students who neither protested in Washington in
to a patient needing it. March nor vacationed in Florida in March are indifferent to
So, (E) is not relevant to the argument of the author, and governmental policies on education.
is not the answer. (E) The best way to influence congressional opinion about
37
American Education Aids - LSAT - Logical Reasoning
a political issue is to communicate with one’s elected repre- his conclusion is that at least some among the 1850
sentative in Washington. unsuccessful applicants had previous experience in weaving
but were still not selected. It is (A) which states this, and is
Analysis the answer.
From the fact that the number of college students who If the author’s assertion is true, (B) will constitute a
vacationed in Florida beaches (350,000) was much larger than probable reason for the industrialists’ preference. But (B) by
the number of college students who protested in Washington itself is not a necessary assumption that relates the evidence
against proposed cuts in student loan funds (300), the author quoted by the author to his conclusion. So, (B) is not the
comes to the conclusion that the Congress need not listen to answer.
the appeal of the protesting students. The author talks of “weaving experience” in general terms
We have been asked to locate the choice which forms an (including experience in one’s own household handloom) and
assumption in the above argument. does not limit it to weaving experience in a textile mill.
If the 300 students who participated in the protest Therefore, (C) does not form a necessary nexus between the
demonstration had, in fact, acted as the representatives of evidence cited by him and his conclusion. So, (C) is not the
the 350,000 students who had gone to the Florida beach, and answer.
had in fact reflected the views of such a large number of From the evidence cited by him, the author merely draws
students, the conclusion of the author would be wrong. the conclusion that industrialists preferred inexperienced
So, what the author is assuming while coming to his candidates, without trying to speculate on the reasons for
conclusion is that the 350,000 students who were vacationing such preference by them. Since there could be many reasons
in Florida beach did not oppose the cutting of loan funds, and other than economic for such preference (such as (i) the
the 300 demonstrators did not reflect their views. It is (A) difference in the type of work between household looms and
which states this, and is the answer. mill-looms, (ii) the inexperienced candidates being younger
(B) says that not only the 300 students, but a majority of and healthier, (iii) the inexperienced persons being more
the United States citizens, were against the proposed cut in obedient etc), (D) is not a necessary assumption for the author
student loan funds. Since the author’s conclusion in the in coming to his conclusion. So, (D) is not the answer.
passage is based on her assertion that Congress should heed (E) may be a historical fact, and may constitute one reason
majority opinion, (B) would mean that Congress should why most of the applicants for the job had no previous weaving
withdraw the proposed cut in student loan funds. This would experience. But there were household looms prior to the
contradict, and will not form the basis for, the author’s nineteenth century and these produced weavers with
conclusion. So, (B) is wrong. experience, though not in large numbers. But (E) does not
(C), (D) and (E) neither corroborate, nor contradict, the form a nexus between the author’s evidence and his conclusion
author’s conclusion, and are not relevant to it. that industrialists ‘preferred’ inexperienced candidates to
experienced weavers So, (E) is not the answer.
Example 9
In the early nineteenth century, industrialists preferred Example 10
to hire people with no weaving experience to work in their An electric-power company gained greater profits and
new textile mills. Evidence of this is that, when more than provided electricity to consumers at lower rates per unit of
2,000 people applied to work at one New England mill, only electricity by building larger-capacity, more efficient plants
30 of the 150 who were hired had previous experience in and by stimulating greater use of electricity within its area.
weaving. To continue these financial trends, the company planned to
The statistics cited above provide no evidence supporting replace an old plant by a plant with triple the capacity of its
the author’s contention unless it is assumed that largest plant.
(A) some of the unsuccessful applicants to the textile mill The company’s plan as describe above assumed each of the
had experience in weaving following EXCEPT:
(B) the nature of the work at the new textile mills made (A) Demand for electricity within the company’s area of
previous experience in weaving of no advantage in a worker service would increase in the future
(C) the 30 successful applicants with weaving experience (B) Expenses would not rise beyond the level that could be
had no experience of weaving in textile mills compensated for by efficiency or volume of operation, or both
(D) the industrialists’ preference for workers without (C) The planned plant would be sufficiently reliable in
previous experience was economically motivated service to contribute a net financial benefit to the company
(E) prior to the nineteenth century, there was no weaving as a whole
done on a large industrial scale (D) Safety measures to be instituted for the new plant
would be the same as those for the plant it would replace
Analysis (E) The tripling of capacity would not result in insuperable
The conclusion that industrialists preferred to hire persons technological obstacles to efficiency
with no weaving experience to work in their factories will be
valid only if persons experienced in weaving were available Analysis
for employment but were not selected. This question is of a type different from the previous ones.
The statistics cited by the author is that 2000 people had In this question, four of the five given choices represent
applied for the jobs; 150 of them were selected, and only 30 assumptions necessarily made by the company while
persons among these 150 had experience in weaving. formulating its plan, and only one of them does NOT represent
If, among all the 2000 applicants, only these 30 had such a necessary assumption.
experience in weaving and all of them had been selected for We have been asked to spot this particular choice.
employment, the author’s conclusion will be wrong. On the basis of its past experience that its profitability
So, what the author necessarily assumes while asserting and the standard of its service improved when it increased
38
American Education Aids - LSAT - Logical Reasoning
its capacity, efficiency and the market, the electric-power com- operation (stated in E).
pany wishes to triple the capacity of one of its plants. (D) is not a necessary assumption in this context, since
Obviously the company assumes that it will be able to sell the company may even think of improving the safety
the additional power generated by the new plant (stated in standards. Additionally, while the other choices have a direct
A); that it will be able to control its expenditure so as to nexus to the continuance of the financial trends mentioned
continue to maintain and increase its profits (stated in B); in the last sentence, ‘safety measures’, in spite of their
that the new plant will reliably continue to produce the importance, do not have a direct nexus to the financial trends.
estimated additional power (stated in C); and that its three- So, it is (D) which is not a necessary assumption for the
fold capacity increase will not result in inefficiency in investment decision, and is the answer.

In the following pages, we give you a number of sets of 'Assumption' questions


which you can answer by yourself!

39
American Education Aids - LSAT - Logical Reasoning
SET 1
1. To prevent some conflicts of interest, Congress could has caused severe health problems are mistaken. From 1940-
prohibit high-level government officials from accepting 1979, the average life expectancy for the entire United States
positions as lobbyists for three years after such officials leave population increased from 63.6 years to 73.6 years, and the
government service. One such official concluded, however, total age-adjusted death rate declined by fortyfive percent,
that such a prohibition would be unfortunate because it would including declines in every age, sex and racial group. These
prevent high-level government officials from earning a were also the years in which there was a dramatic increase
livelihood for three years. in the use of new chemicals in this country. Clearly, then,
The official’s conclusion logically depends on which of the these modern aids to industry have no adverse effects on the
following assumptions? general health of the society.
(A) Laws should not restrict the behavior of former Which of the following is an assumption underlying the
government officials. conclusion of the passage above?
(B) Lobbyists are typically people who have previously been (A) The increase in life expectancy would not have been
high-level government officials. greater if there had not been an increase in the use of
(C) Low-level government officials do not often become chemicals.
lobbyists when they leave government service. (B) The increase in life expectancy in the United States
(D) High-level government officials who leave government was paralleled by corresponding increases in other countries.
service are capable of earning a livelihood only as lobbyists. (C) The growth in the average life expectancy in the United
(E) High-level government officials who leave government States cannot be attributed to any single factor, but is a
service are currently permitted to act as lobbyists for only general phenomenon.
three years. (D) The increase in the use of new chemicals did not begin
until 1940
2. Two groups of laboratory mice were injected with (E) Growth in the use of chemicals was a crucial factor in
cancerous cells. One group’s cages were rotated in a the development of industry from 1940 to 1979.
disorienting manner. Two-thirds of these mice developed
cancers. One-tenth of the mice in stationary cages developed 5. Spending on research and development by United States
cancers. The researchers concluded that stress enhances the businesses for 1984 showed an increase of about 8 percent
development of cancer in laboratory mice. over the 1983 level. This increase actually continued a
The researchers’ conclusion logically depends on which of downward trend evident since 1981 - when outlays for
the following assumptions? research and development increased 16.4 percent over 1980
(A) Rotating the cages in a disorienting manner produced spending. Clearly, the 25 percent tax credit enacted by
stress in the mice in those cages. Congress in 1981, which was intended to promote spending
(B) The injections given to the two groups of mice were not on research and development, did little or nothing to stimulate
of equal strength. such spending.
(C) Injecting the mice with cancerous cells caused stress The conclusion of the argument above cannot be true unless
in the mice. which of the following is true?
(D) Even without the injections the mice in the rotated (A) Business spending on research and development is
cages would have developed cancers. usually directly proportional to business profits.
(E) Even the rotation of cages in a manner that is not (B) Business spending for research and development in
disorienting is likely to produce stress in mice in those cages. 1985 could not increase by more than 8.3%.
(C) Had the 1981 tax credit been set higher than 25%,
3. Even though most universities retain the royalties from business spending for research and development after 1981
faculty members’ inventions, the faculty members retain the would have increased more than it did.
royalties from books and articles they write. Therefore, faculty (D) In the absence of the 25% tax credit, business spending
members should retain the royalties from the educational for research and development after 1981 would not have been
computer software they develop. substantially lower than it was.
The conclusion above would be more reasonably drawn if (E) Tax credits marked for specific investments are rarely
which of the following were inserted into the argument as an effective in inducing businesses to make those investments.
additional premise?
(A) Royalties from inventions are higher than royalties 6. In an attempt to reverse the deterioration of the postal
from educational software programs. service in at least some respects, the government has raised
(B) Faculty members are more likely to produce the prices of postage stamps. This effort will be effective since
educational software programs than inventions. reducing the number of persons using the postal system
(C) Inventions bring more prestige to universities than do should result in a corresponding reduction in the percentage
books and articles. of letters lost.
(D) In the experience of most universities, educational The author of the passage above makes which of the
software programs are more marketable than are books and following assumptions?
articles. I. Fewer people will buy postage stamps if they are more
(E) In terms of the criteria used to award royalties, expensive
educational software programs are more nearly comparable II. The percentage of letters lost by the postal service has
to books and articles than to inventions. been evidence of the deterioration of the postal system
III. The government will improve the efficiency of the
4. The sharp decline in the mortality rate since 1941 shows
postal service with revenues from higher priced stamps
that those who argue that the introduction of new chemicals
40
American Education Aids - LSAT - Logical Reasoning
(A) I only 9. A study of illusionistic painting inevitably begins with
(B) II only the Greek painter Zeuxis. In an early work, which is the basis
(C) III only for his fame, he painted a bowl of grapes that was so lifelike
(D) I and II only that birds pecked at the fruit. In an attempt to expand his
(E) I and III only achievement to encompass human figures, he painted a boy
carrying a bunch of grapes. When birds immediately came to
7. The marketing researcher was interested only in the peck at the fruit, Zeuxis judged that he had failed.
attitude that those interviewed had toward vacationing in Zeuxis’ judgment that he had failed in his later work was
Florida. Yet, in an effort to obtain valid information, she did based on an assumption. Which of the following can have
not reveal that Florida’s Bureau of Tourism was sponsoring served as that assumption?
her research; she asked participants in the interview to give (A) People are more easily fooled by illusionistic techniques
opinions on and express preferences among many places for than are birds.
vacationing, a technique that yielded as much information (B) The use of illusionistic techniques in painting had
on Europe and California as on Florida. become commonplace by the time Zeuxis completed his later
A plausible explanation for the researcher’s strategy is that work.
she assumed that the participants (C) The grapes in later painting were even more realistic
(A) were from a variety of backgrounds and hence had a than the ones in the earlier work.
variety of opinions about vacationing (D) Birds are less likely to peck at fruit when they see that
(B) had attitudes toward Florida that were more easily a human being is present.
predicted than were their opinions on their places for (E) After the success of his early work, Zeuxis was unable
vacationing to live up to the expectations of the general public.
(C) would tend to express favourable opinions when they
thought such responses would please the sponsor of the 10. From a certain farming region, trucks can carry
research vegetables to markets in New Mexico in two days for a total
(D) would speak honestly about places for vacation only cost of $300. A train will carry the vegetables there in four
during direct questioning days for $200. If reducing time in transit is more important
(E) were more knowledgeable and candid about Europe to the owner of the vegetables than is reducing the shipping
and California than about Florida bill, he or she will send the vegetables by truck.
Which of the following is an assumption made in the
8. Buying credit may induce many consumers to buy things passage above?
they do not really want, but it also allows them to take (A) Vegetables can be sold more profitably when shipped
advantage of bargains when they are short of cash. Anyone by train than by truck.
who takes advantage of this latter possibility saves money (B) Other than speed and cost, there are no significant
by buying on credit. differences between truck and train transportation from the
Which of the following is an assumption made in the farming region to New Mexico.
passage above? (C) The time required to ship vegetables by train from the
(A) Credit is available to everyone who wants it farming region to New Mexico could be reduced to two days
(B) The cost of credit does not exceed the savings on if the price for this service were raised.
purchases that credit made possible (D) Most owners of vegetables in the region are more
(C) Consumers who use credit wisely buy only bargains concerned with shipping costs than with the time involved in
with it shipping vegetables to market.
(D) Most unnecessary purchases are made on credit (E) Transportation of vegetables by truck is worth at least
(E) It is difficult to use credit with restraint if it is available $200 per day to owners of the vegetables in the farming
to consumers at bargain rates region.

SET 2
1. Studies show that treatment of stress-related disorders effectively with psychotherapy
with psychotherapy can dramatically cut a person’s need for (E) few people who have medical insurance are treated for
medical treatment for those disorders. Insurance companies stress-related disorders
could, therefore, lower their costs by subsidizing
psychotherapy and thereby encouraging people who have 2. Many of those who advocate trimming hospital costs
stress-related disorders and are not now using psychotherapy argue that the federal money saved by such cuts could be
to do so. utilized for important social purposes: rebuilding the cities,
The plan above depends on the assumption that reducing safety hazards in the workplace, preserving the
(A) people who have stress-related disorders tend to have environment, improving schooling, or developing better
little faith in psychotherapy sources of energy. Their enthusiasm would be dampened if
(B) any person with a stress-related disorder can be the savings were diverted to other objectives: expanding the
effectively treated with psychotherapy military, balancing the budget, revitalizing the space program,
(C) the projected cost to insurance companies of medical or cutting capital gains taxes. One cannot be confident that
treatment for stress-related disorders exceeds the cost of the any useful result would be obtained by diverting funds from
proposed subsidies for psychotherapy for those disorders the hospital system.
(D) any stress-related disorder that cannot be treated The author of the passage above assumes that
effectively with conventional medical treatment can be treated (A) those who favor government expenditure for some social
41
American Education Aids - LSAT - Logical Reasoning
purposes oppose government expenditure in other areas able to understand what they see on television.
(B) federal money should not be used for rebuilding the (E) A child’s reaction to different forms of sensory stimuli
cities and other social purposes cannot be predicted, since every child is different.
(C) hospital costs are not excessive
(D) hospital costs should be trimmed after the projected 6. I. Neither Peter nor Sarah has any common sense.
savings have been allocated for specific purposes II. Neither Peter nor Sarah is able to run the factory.
(E) trimming hospital costs will decrease the quality of Statement II will follow from Statement I if which of the
hospital care following assumptions is additionally made?
(A) Neither Peter nor Sarah has any experience in running
3. While a quota on imported cars might protect jobs in factories.
the domestic automobile industry, it also means fewer (B) Even a person who lacks common sense would be able
inexpensive imported cars and less money for those buying to run a factory if he or she had taken courses in factory
cars to spend on other things. Therefore, such a quota will management.
ultimately result in fewer jobs in domestic industries (C) To be able to run a factory, a person must have common
unrelated to the automobile industry than there would be sense.
without a quota. (D) Peter and Sarah, working with someone who has good
Which of the following is an assumption made in drawing common sense, would be able to run the factory.
the conclusion above? (E) If Sarah had any common sense, she and Peter would
(A) The number of car purchases will decrease if a quota be able to run the factory.
is imposed on imported cars
(B) Most car buyers prefer inexpensive imported cars to 7. One of the world’s most celebrated paintings, The Man
inexpensive domestic cars with the Golden Helmet, long attributed to Rembrandt, is not
(C) Without a quota, money that car buyers would save by a Rembrandt after all. So say several art experts who base
buying inexpensive imported cars would be spent on goods their conclusion on an analysis of stylistic features, especially
produced by unrelated domestic industries details both of shading and of brushwork. In order to ascertain
(D) With a quota, the domestic automobile industry will who really painted the well-known masterpiece, the experts
employ more people than it currently does have begun a series of sophisticated new tests, including one
(E) A quota on imported cars would cause the price of that involves the activation of neutrons. These tests yield
particular models of imported cars to rise patterns for any painter that are as distinctive as a good set
of fingerprints.
4. Some geologists argue that if oil is as common in Which of the following is an assumption on which the
unsampled areas of the world as it is in those already sampled, conclusion of the art experts depends?
our current estimate of reserves that exist underground must (A) The Man with the Golden Helmet was not painted
be multiplied by a factor of 10,000. From this we can conclude during Rembrandt’s lifetime.
that we can meet the oil needs of the entire world for at least (B) If even The Man with the Golden Helmet is of
five centuries, even assuming that future consumption grows questionable attribution, then any supposedly authentic
at an accelerating rate. Rembrandt has now become suspect.
To reach the stated conclusion, the author must assume (C) The painting known as The Man with the Golden
which of the following? Helmet is a copy of a Rembrandt original.
(A) It is possible to recover the oil contained in unexplored (D) The original ascription of The Man with the Golden
areas of the world. Helmet to Rembrandt was a deliberate fraud.
(B) The consumption rate for oil will not grow rapidly. (E) There are significant consistencies among authentic
(C) Oil will remain an important energy source for at least Rembrandts in certain matters of style.
500 years.
(D) The world will achieve and maintain zero population 8. Some insects are able to feed on the leaves of milkweed,
growth. a toxic plant, by first cutting and draining the vein that
(E) New technology will make oil discovery and drilling secretes the toxin. This method of detoxification guarantees
more feasible than ever before. that some insects will always be able to eat milkweed, because
the plant could never evolve to produce a toxin that is lethal
5. A child watching television experiences a procession of in the trace amounts left after the vein is cut.
sights and sounds that flash from the screen just long enough The conclusion drawn in the passage above depends on
for the eyes and ears to take them in. Unlike the pages of a which of the following assumptions?
book, which can be read as slowly or as quickly as the child (A) The insects that successfully detoxify milkweed are
wishes, television images appear with a relentless velocity not able to undergo the evolutionary changes necessary to
that stunts rather than enhances the child’s powers of allow them to detoxify other plants.
imagination. (B) Unlike milkweed, other kinds of toxic plants would be
The view expressed above is based on an assumption. Of able to overcome their vulnerabilities to predators through
the following, which can best serve as that assumption? evolutionary changes.
(A) When allowed to choose a form of entertainment, (C) The toxin-carrying veins of the milkweed plant can
children will prefer reading to watching television. never evolve in such a way that insects cannot cut through.
(B) A child’s imagination cannot be properly stimulated (D) The method of detoxification used by insect predators
unless the child has access both to television and to books. of milkweed would not successfully detoxify other kinds of
(C) A child’s imagination can develop more fully when the toxic plants.
child is able to control the pace of its entertainment. (E) There are insects that use means other than draining
(D) Children should be taught to read as soon as they are the toxin in order to feed on toxic plants.
42
American Education Aids - LSAT - Logical Reasoning
9. A researcher discovered that people who have low levels Therefore, innovation in bicycle technology is limited by what
of immune-system activity tend to score much lower on tests authorities will accept as standard for purposes of competition
of mental health than do people with normal or high immune- in bicycle races.
system activity. The researcher concluded from this Which of the following is an assumption made in drawing
experiment that the immune system protects against mental the conclusion above?
illness as well as physical disease. (A) The market for cheap, traditional bicycles cannot
The researcher’s conclusion depends on which of the expand unless the market for high-performance competition
following assumptions? bicycle expands.
(A) High immune-system activity protects against mental (B) High-performance bicycles are likely to be improved
illness better than normal immune-system activity does. more as a result of technological innovations developed in
(B) Mental illness is similar to physical disease in its effects small workshops than as a result of technological innovations
on body systems. developed in major manufacturing concerns.
(C) People with high immune-system activity cannot (C) Bicycle racers do not generate a strong demand for
develop mental illness. innovations that fall outside what is officially recognized as
(D) Mental illness does not cause people’s immune-system standard for purposes of competition.
activity to decrease. (D) The technological conservatism of bicycle
(E) Psychological treatment of mental illness is not as manufacturers results primarily from their desire to
effective as medical treatment. manufacture a product that can be sold without being altered
to suit different national markets.
10. The technological conservatism of bicycle
(E) The authorities who set standards for high-performance
manufacturers is a reflection of the kinds of demand they
bicycle racing do not keep informed about innovative bicycle
are trying to meet. The only cyclists seriously interested in
design.
innovation and willing to pay for it are bicycle racers.

SET 3
1. Child’s World, a chain of toy stores, has relied on a breaking them is rare. As surveys show, millions of people
“supermarket concept” of computerized inventory control and have dropped the habit of smoking, and many people have
customer self-service to eliminate the category of sales clerks successfully managed a substantial weight loss.
from its force of employees. It now plans to employ the same If all of the statements above are correct, an explanation of
concept in selling children’s clothes. their apparent contradiction is provided by the hypothesis that
The plan of Child’s World assumes that (A) there have been some successes in therapy, and those
(A) supermarkets will not also be selling children’s clothes successes were counted in the surveys
in the same manner (B) it is easier to stop smoking than it is to stop overeating
(B) personal service by sales personnel is not required for (C) it is easy to break the habits of smoking and overeating
selling children’s clothes successfully by exercising will power
(C) the same kind of computers will be used in inventory (D) the group of people selected for the survey did not
control for both clothes and toys at Child’s World include those who failed to break their habits even after
(D) a self-service plan cannot be employed without therapy
computerized inventory control (E) those who succeed in curing themselves do not go for
(E) sales clerks are the only employees of Child’s World treatment and so are not included in the therapists’ data
who could be assigned tasks related to inventory control
4. Until 1984 only aspirin and acetaminophen shared the
2. A weapons-smuggling incident recently took place in lucrative nonprescription pain-reliever market. In 1984,
country Y. We all know that Y is closed society. So Y’s however, ibuprofen was expected to account for fifteen percent
government must have known about the weapons.
of all nonprescription pain-reliever sales. On that basis
Which of the following is an assumption that would make
business experts predicted for 1984 a corresponding fifteen
the conclusion above logically correct?
percent decrease in the combined sales of aspirin and
(A) If a government knows about a particular weapons
acetaminophen.
smuggling incident, it must have intended to use the weapons
The prediction mentioned in the last sentence above was
for its own purposes
based on which of the following assumptions?
(B) If a government claims that it knew nothing about a
(A) Most consumers would prefer ibuprofen to both aspirin
particular weapons-smuggling incident, it must have known
and acetaminophen.
everything about it
(B) Aspirin, acetaminophen, and ibuprofen all relieve
(C) If a government does not permit weapons to enter a
country, it is a closed society headache pain and muscular aches, but aspirin and ibuprofen
(D) If a country is a closed society, its government has a can also cause stomach irritation.
large contingent of armed guards patrolling its borders (C) Before 1984 ibuprofen was available only as a
(E) If a country is a closed society, its government has prescription medicine.
knowledge about everything that occurs in the country (D) The companies that manufacture and sell aspirin and
acetaminophen would not also manufacture and sell
3. Therapists find that treatment of those people who seek ibuprofen.
help because they are unable to stop smoking or overeating (E) The introduction of ibuprofen would not increase total
is rarely successful. From these experiences, therapists have sales of nonprescription pain relievers.
concluded that such habits are intractable, and success in
43
American Education Aids - LSAT - Logical Reasoning
5. Some United States psychologists have concluded that destructive to grain production than are long-range warming
one specific set of parental behaviors toward children always trends.
signifies acceptance and a second set always signifies
rejection, for there is remarkable agreement among 8. In any negotiations between a party with limited aims
investigators about the maternal behaviors designated as and an opposing party with unlimited aims, the party with
indicative of these parental attitudes. limited aims is bound to lose. This is so because the scope of
The conclusion of the psychologists mentioned above a negotiating party’s aims determines the energy and the
logically depends on the assumption that perseverance that will be brought to the negotiations by that
(A) most maternal behaviors have been interpreted as party.
conveying either acceptance or rejection. Which of the following is an assumption implicit in the
(B) the maternal behaviors indicating acceptance or passage above?
rejection are exhibited by fathers as well (A) The intensity with which parties conduct negotiations
(C) the behaviors of fathers toward children have been affects the outcome of those negotiations.
studied as carefully as have the behaviors of mothers (B) Negotiations almost always pit against one another
(D) acceptance and rejection are the easiest to recognize parties whose aims differ in scope.
of all parental behaviors (C) The outcome of negotiations cannot be correctly
(E) parental attitudes are best conveyed through behaviors predicted in advance.
that the parents have consciously agreed on (D) A negotiator who has exceptionally high aims needs
exceptional perseverance in order to avoid losing.
6. Last year, support for the social and behavioral sciences (E) Negotiating parties are typically not aware of the scope
represented only about three percent of the government’s total of each other’s aims until the outcome is no longer in doubt.
budget for research funds in the United States. Thus, the
particularly sharp reductions imposed on such programs this 9. The overall operating costs borne by many small farmers
year seem dictated not by financial constraints but by social are reduced when the farmers eliminate expensive
philosophy. commercial chemical fertilizers and pesticides in favor of crop
Which of the following is an assumption on which the rotation and the twice-yearly use of manure as fertilizer.
conclusion of the passage above is based? Therefore, large farmers should adopt the same measures.
(A) The government funds allocated for research in the They will then realize even greater total savings than do the
social and behavioral sciences are not sufficient for the work small farmers.
that needs to be done. The argument above assumes that
(B) The social and behavioral sciences are as valuable as (A) it is more cost-effective for small farmers to eliminate
the physical and biological sciences. the use of commercial fertilizers and pesticides than it is for
(C) The current reductions will stop research in the social large farmers to do so
and behavioral sciences. (B) a sufficient amount of manure will be available for the
(D) Government funding is the primary source of research fields of large farmers
money in the United States. (C) large farmers would not realize similar cost benefits
(E) Three percent is an insignificant portion of the by using treated sewage sludge instead of commercial
government’s total budget for research funds. chemical fertilizers
(D) large farmers generally look to small farmers for
7. Some doomsayers are warning that long-range warming innovative ways of increasing crop yields or reducing
or cooling trends in weather patterns will drastically reduce operating costs
grain production. More optimistic reports, however, point (E) the smaller the farm, the more control the farmer has
out that, even if such drifts in average temperature do occur, over operating costs
we should expect little change in grain production because
there is little evidence that changes in rainfall patterns will 10. A compelling optical illusion called illusion of velocity
occur. Moreover, for most crops, climate-induced yield trends and size makes objects appear to be moving more slowly the
will be masked by both the year-to-year fluctuation of yields larger the objects are. Therefore, a motorist’s estimate of the
and by the enhancement of yields because of technological time available for crossing a highway with a small car
factors. approaching is bound to be lower than it would be with a
Which of the following is an assumption on which the more large truck approaching.
optimistic reports mentioned in the passage are based? The conclusion above would be more properly drawn if it
(A) Long-range changes in weather patterns cannot be were made clear that the
accurately predicted. (A) truck’s speed is assumed to be lower than the car’s
(B) The growing of grain is so highly dependent on (B) truck’s speed is assumed to be the same as the car’s
technological factors that improvements in yield are unlikely, (C) truck’s speed is assumed to be higher than the car’s
regardless of climatic conditions. (D) motorist’s estimate of time available is assumed to be
(C) Trends in rainfall patterns are more difficult to isolate more accurate with cars approaching than with trucks
than are trends in temperature. approaching
(D) Long-range warming or cooling trends are more (E) motorist’s estimate of time available is assumed to be
damaging to grain production if they are accompanied by more accurate with trucks approaching than with cars
changes in rainfall patterns than if they are not. approaching
(E) Long-range cooling trends are potentially more

44
American Education Aids - LSAT - Logical Reasoning
SET 4
1. Of those persons who became teachers in 1968 and who drawing the conclusion above from the reason given for that
later left the profession, 30 percent today earn salaries above conclusion?
$35,000 a year; of those who became teachers in 1968 and (A) Some furniture is made to be placed in museums, where
have remained in the profession, only 15 percent today earn it will not be used by anyone.
salaries above $35000 a year. These figures indicate how (B) Some cabinetmakers are more concerned than others
underpaid teachers are today. with the practical utility of the products they produce.
The argument above depends on which of the following (C) Cabinetmakers should be more concerned with the
assumptions about the persons for whom statistics are cited? practical utility of their products than they currently are.
(A) At least one-third of the group of persons who have (D) An object is not an art object if its maker pays attention
remained in teaching would today be earning more than to the object’s practical utility.
$35,000 a year if they had left teaching. (E) Artists are not concerned with the monetary value of
(B) The group of persons who left teaching and the group their products.
who did not are comparable in terms of factors that determine
how much people outside the teaching profession are paid. 5. Traditionally, decision-making by managers that is
(C) Most of those persons who left teaching did so entirely reasoned step-by-step has been considered preferable to
because of the low salaries teachers earn. intuitive decision-making. However, a recent study found
(D) As a group, those persons who have remained in that top managers used intuition significantly more than did
teaching are abler and more dedicated than the group of most middle-or lower-level managers. This confirms the
persons who left teaching. alternative view that intuition is actually more effective than
(E) The group of persons who left teaching and who today careful, methodical reasoning.
earn more than $35,000 a year were more capable teachers The conclusion above is based on which of the following
than the group who remained in the profession. assumptions?
(A) Methodical, step-by-step reasoning is inappropriate for
2. Instead of blaming an airline accident on pilot error, making many real-life management decisions.
investigators should find out why the error was made by (B) Top managers have the ability to use either intuitive
analyzing airplane design, airline management, and pilot- reasoning or methodical, step-by-step reasoning in making
training programs. For only then can changes be made to decisions.
ensure that the same type of error does not recur and cause (C) The decisions made by middle-and lower-level
another accident. managers can be made as easily by using methodical
Which of the following is a presupposition of the argument reasoning as by using intuitive reasoning.
above? (D) Top managers use intuitive reasoning in making the
(A) Pilot error is not a contributing factor in most airline majority of their decisions.
accidents. (E) Top managers are more effective at decision-making
(B) Airline companies themselves should be the agents who than middle-or lower-level managers.
investigate airline accidents.
(C) Stricter government regulation of airline companies 6. The financial value of any particular painting is at best
will make air travel significantly safer. only indirectly connected with its own aesthetic merit. This
(D) Investigators of airline accidents should contribute to is demonstrated by the enormous difference in financial value
the prevention of future accidents that always exists between two paintings that are
(E) Most pilots who make errors in flying will repeat their indistinguishable by the naked eye when one of them is a
errors unless they are retrained. forgery of the other.
The conclusion above can most reliably be inferred when
3. The president of a consulting firm analyzed the decisions which of the following assumptions is made?
made about marketing by her clients and concluded that the (A) The authenticity of a painting does not contribute to
decisions were correct only about half of the time. its aesthetic merit
The conclusion above depends on the presupposition that (B) Accurate forgeries of valuable paintings are not
(A) companies can be successful even when about half of particularly rare
the decisions they make about marketing prove to be wrong (C) Paintings are forged because of the originals’ financial
(B) companies hiring her consulting firm make no more value
incorrect marketing decisions than do companies in general (D) Even experts are frequently incapable of determining
(C) executives consistently making correct marketing the authenticity of certain paintings
decisions rarely enlist the aid of a consulting firm (E) It is as easy to determine a painting’s aesthetic merit
(D) marketing decisions are just as likely to be correct as as it is to determine its financial value.
they are to be incorrect
(E) it is possible to classify a marketing decision properly 7. In elections in the United States, the proper role of the
as being either right or wrong. press is to cover only those factors in the campaign which
bear on the eventual outcome. Since the outcome is invariably
4. In recent years many cabinetmakers have been winning a victory for the candidate of one of the two major parties,
acclaim as artists. But since furniture must be useful, the press should not cover the campaigns of candidates of
cabinetmakers must exercise their craft with an eye to the minor parties.
practical utility of their product. For this reason, cabinet The argument above relies on which of the following
making is not art. assumptions?
Which of the following is an assumption that supports (A) The press has an obligation to cover the candidates of
45
American Education Aids - LSAT - Logical Reasoning
the two major parties because these candidates are likely is surprising, because it follows a period in which the formerly
to be better known to the public than are candidates of minor obscure Q received a great deal of publicity in the news media.
parties Which of the following, if assumed by the author to be true,
(B) It is unlikely that there will be more than three would provide the most logical explanation of the author’s
candidates in any given race surprise at the reported decline?
(C) Many eligible voters do not bother to vote, and of those (A) Increased public awareness of a disease usually
who do not vote, some would probably have supported a stimulates efforts to eradicate the disease
candidate of a minor party (B) Increased public awareness of a disease usually leads
(D) The number of votes cast for a candidate of a minor to an increase in reported instances of the disease
party is not likely to affect the outcome of the contest between (C) An obscure disease may sometimes receive a great deal
the candidates of the two major parties of publicity even though doctors have begun to suspect that
(E) Supporters of candidates of minor parties are less likely the disease no longer exists
to be influenced by the press than are supporters of candidates (D) The Center for Disease Control typically concerns itself
of the two major parties. only with diseases that are of public importance
(E) It is unusual for the reported instances of a disease to
8. Hittite tablets corroborate many of the descriptions of decline sharply after a long period of stability
ancient life that appear in the Iliad and even list Greek cities
that reportedly sent ships to Troy. What this means is that 10. The theoretical principle of democracy is majority rule,
the Iliad is not creative literature, as is commonly believed, but at least in presidential politics, majority rule has never
and hence not a matter for literary discussion. It is history triumphed. In every presidential election, more people of
and should be examined by historical science. voting age failed to vote than voted for the eventual winner.
The author of the passage above makes which of the “None of the above” is evidently the perennially favorite
following assumptions? candidate of most citizens.
I. A work cannot be classified as creative literature if that The conclusion of the passage is based on which of the
work records historical fact following assumptions?
II. The Hittite tablets record actual events rather than (A) Non-Presidential elections are less worthy of study than
legends are presidential elections
III. Cities and events mentioned in the Iliad but not in the (B) People of voting age who fail to vote in presidential
tablets are fictitious elections do so to express their dissatisfaction with the
(A) I only candidates
(B) II only (C) A presidential candidate need not receive votes from a
(C) III majority of those who actually vote in order to win an election
(D) I and II only (D) The ratio of voters to non-voters among those of voting
(E) II and III only age indicates the relative popularity of the competing
candidates
9. Recent data from the Center for Disease Control indicate (E) The principle of majority rule has never been correctly
a decline in the reported instances of disease Q. This decline defined

SET 5
1. When limitations were in effect on nuclear-arms testing, (A) When Harry is absorbed in reading a good book, nobody
people tended to save more of their money, but when nuclear- is calling him
arms testing increased, people tended to spend more of their (B) Harry cannot know someone has called him unless he
money. The perceived threat of nuclear catastrophe, therefore, heard that call
decreases the willingness of people to postpone consumption (C) When Harry is not absorbed in reading a good book,
for the sake of saving money. someone often calls him
The argument above assumes that (D) Harry is even more absorbed than he claims to be
(A) the perceived threat of nuclear catastrophe has (E) Harry reads good books and he is called, but not at the
increased over the years same time
(B) most people supported the development of nuclear arms
(C) people’s perception of the threat of nuclear catastrophe 3. A study of twentieth-century data on volcanic eruptions
depends on the amount of nuclear-arms testing being done shows a slow sustained rise in reported eruptions, with sharp
(D) the people who saved the most money when nuclear- drops that coincide with the two world wars. The most
arms testing was limited were the ones who supported such reasonable interpretation of these data is that volcanic
limitations activity in this century has held at a steady level throughout.
(E) there are more consumer goods available when nuclear- Which of the following is most likely to be an assumption
arms testing increases underlying the interpretation of the data described?
(A) The slow rise in reported volcanic eruptions in the
2. Harry: If I am reading a good book, I become so absorbed twentieth century can reasonably be expected to be sustained
that if someone calls me, I am not aware of it and hear nothing. for the remainder of this century
James: If you are not able to hear anything, you could not (B) Volcanic eruptions constitute only a small percentage
know that anyone was calling you! of the sum total of volcanic activity, but they tend to get
James’s response shows that he makes which of the recorded more faithfully than other volcanic activity
following assumptions? (C) The fact that the sharp drops in reports of volcanic
46
American Education Aids - LSAT - Logical Reasoning
eruptions occurred during the two wars is attributable to of curtailing crime.
random fluctuations in the data (E) Less severe punishment of crime would be likely to
(D) The variations in the frequency of reports of volcanic result in more crime.
eruptions can reasonably be attributed to factors impinging
on the reporting activity itself 7. The sense of delayed gratification, of working now for
(E) Pre-twentieth-century data are too unsystematic to later pleasure, has helped shape the economic behavior of
support any reasonably firm conclusions about long-term our society. However, that sense is no longer nurtured as
levels of volcanic activity consistently in our children as it once was. For example, it
used to take a bit of patience to put together the toys that
4. Much of what passes for political dialogue in this country children got in cereal boxes; now the toys come from the boxes
is really a veiled discussion of whether the distribution of whole.
wealth in our society is equitable. If it is not equitable, we Which of the following is an assumption of the passage
should consider how much and how quickly the situation can above?
be changed. If the issue were phrased thus, complicated (A) The toys in cereal boxes have changed partly because
questions of economic policy, which are now beyond the the economic conditions of our society have improved.
attention span of most voters, could be addressed (B) The influence of promotion grimmicks on the economic
The major assumption made in the passage above is that behavior of our society has increased over the years.
(A) political dialogue in this country is a simplistic attempt (C) The toys that used to come in cereal boxes were put
to address complex issues together by the same children who played with them.
(B) the economic policy of this country is not fair to all its (D) Part of the pleasure of any toy lies in putting the toy
citizens together before playing with it.
(C) economic policy is too complex to be understood by most (E) Today’s children do not expect a single toy to provide
voters pleasure for a long period of time.
(D) economic policy has implications for the distribution
of wealth 8. The best argument for the tenure system that protects
(E) political dialogue is a result of the short attention span professional employment in universities is that it allows
of most voters veteran faculty to hire people smarter than they are and yet
remain secure in the knowledge that unless they themselves
5. Although historical research and anthropological are caught in an act of moral turpitude – a concept that in
research often complement one another, the two disciplines the present climate almost defies definition – the younger
retain important distinctions because of the fundamental faculty cannot turn around and fire them. This is not true in
questions that each poses. Historians are ultimately industry.
interested in why change occurred, whereas anthropologists Which of the following assumptions is most likely to have
are concerned with the meanings of actions and systems. been made by the author of the argument above?
Which of the following is implied in the passage above? (A) Industry should follow the example of universities and
(A) The meanings of actions and systems do not fully protect the jobs of managers by instituting a tenure system.
explain why change occurred (B) If no tenure system existed, veteran faculty would be
(B) The distinctions between life and art are diminishing reluctant to hire new faculty who might threaten the veteran
(C) Actions and systems are not usually subject to historical faculty’s own jobs.
change (C) The traditional argument that the tenure system
(D) anthropological research is not concerned with change protects scholars in universities from being dismissed for
(E) The questions posed by the two disciplines share certain holding unconventional or unpopular beliefs is no longer
assumptions about human behavior persuasive.
(D) If a stronger consensus concerning what constitutes
6. That social institutions influence the formation of moral turpitude existed, the tenure system in universities
character has become a generally accepted proposition. This would be expendable.
doctrine views individuals as but compliant recipients of social (E) Veteran faculty will usually hire and promote new
influence: personalities are entirely the products of society, faculty whose scholarship is more up-to-date than their own.
and at any point in life an individual’s personality can be
changed by management of the social world. Crime is said to 9. Some people assert that prosecutors should be allowed
exist only because society has in some ways failed in its to introduce illegally obtained evidence in criminal trials if
responsibility to give every person the resources to lead a the judge and jury can be persuaded that the arresting officer
productive life. However, whereas it is true that extreme was not aware of violating or did not intend to violate the
poverty forces some people to steal, it is obvious that some law while seizing the evidence. This proposed “good-faith
persons will commit crimes no matter how well society treats exception” would weaken everyone’s constitutional protection,
them. lead to less careful police practices, and promote lying by law
Which of the following is implied by the “doctrine” (line 3) enforcement officers in court.
described in the passage above? The argument above for maintaining the prohibition
(A) Social institutions may reflect personality as much as against illegally obtained evidence assumes that
they shape it. (A) defendants in criminal cases should enjoy greater
(B) Social influence on personality is most strongly felt by protection from the law than other citizens do
the affluent. (B) law enforcement authorities need to be encouraged to
(C) The concentration of wealth in the hands of a privileged pursue criminals assiduously
few accounts for the existence of crime. (C) the legal system will usually find ways to ensure that
(D) Bringing about social reform is the most likely means real crimes do not go unprosecuted
47
American Education Aids - LSAT - Logical Reasoning
(D) the prohibition now deters some unlawful searches and attitudes of its time.
seizures The author of the passage above assumes that
(E) courts should consider the motives of law enforcement (A) plays written for Restoration audiences do not appeal
officers in deciding whether evidence brought forward by the to modern audiences
officers is admissible in a trial
(B) plays are superior to novels as a form of narrative art
10. Unlike other forms of narrative art, a play, to be (C) Restoration audiences were representative of the whole
successful, must give pleasure to its immediate audience by population of their time
reflecting the concerns and values of that audience. A novel (D) playgoers and novel readers are typically distinct and
can achieve success over months or even years, but a play exclusive groups.
must be a hit or perish. Successful drama of the Restoration (E) Restoration drama achieved popular success at the
period, therefore, is a good index to the typical tastes and expense of critical success

SET 6
1. After a rebellion in a certain country was put down, the be improved in the future.
country’s parliament debated how to deal with the defeated (E) The world hunger crisis will not exist in the future.
rebels.. One side proposed that all the rebels be imprisoned
in order to deter those who might be strongly tempted to rebel 4. Konstantin Stanislavski’s justly praised method for
in the future. The other side argued against imprisonment training actors arose from Stanislavski’s own awkwardness
because it would only discourage future insurrectionists from and susceptibility to theatrical clichés as a young actor. The
surrendering. “method” must be understood in terms of Stanislavski’s
Both positions logically depend on the assumption that personal search for release from the temptations of stock
(A) imprisonment is a harsh penalty gestures, well-tried vocal intonations, and standard emotional
(B) a rebel will prefer a sentence of imprisonment to death formulas. Despite the pretensions of certain of his disciples
(C) there will be no future rebellion in the country in the United States, the Russian director never intended to
(D) it is unlikely that future rebels will surrender formulate a textbook of rigid solutions to acting problems.
(E) resistance to authority is weakened by harsh threats It can be inferred that the author of the preceding statements
about Stanislavski’s method holds which of the following
2. Although many brands of gasoline are sold on Haibei opinions about acting?
Islands, gasoline companies there get all of the refined (A) Acting is essentially spontaneous emotional expression,
gasoline they sell from Haibei seaport’s only storage tank, with which systematic training usually interferes.
which is always refilled with the same quality of gasoline. (B) The Stanislavski method has lost some of its flexibility
Therefore, the brands of gasoline for sale on Haibei may be and exploratory qualities as it has been used by some followers
different in name and price, but they are identical in quality. of Stanislavski in the United States.
The conclusion drawn above depends on which of the (C) The Stanislavski method has misled those actors in
following assumptions? the United States who have adopted it.
(A) Consumers are usually unaware of variations in the (D) Virtually the only advice young actors need be given is
quality of the gasoline they buy unless those variations are that they must systematically suppress theatrical clichés in
announced by the gasoline companies their performances.
(B) When tankers make gasoline deliveries at Heibei’s (E) The Stanislavski method is useful primarily for young
seaport, the storage tank on Haibei always receives the same actors who must overcome artificiality and immaturity in
quality of gasoline as that in the preceding delivery their performances.
(C) There is wide variation in the prices at which the
different brands of gasoline on Haibei are sold 5. It has been hypothesized that much of the matter in the
(D) If any gasoline company on Haibei alters the quality universe is “dark”: i.e., unseen. Studies have shown that
of its gasoline before sale, the other gasoline companies also galaxies in many galaxy clusters are moving faster with
alter the quality in the same manner respect to one another than they would if visible stars
(E) The gasoline storage tank on Haibei is large enough to constituted all their mass. The studies suggest that the
meet the needs of all of Haibei’s different gasoline companies galaxies are moving under the gravitational influence of
unseen mass in considerable quantity.
3. The world’s annual food production slightly exceeds the Which of the following is an assumption underlying the
amount of food required to provide a minimally adequate diet passage above?
for the world’s population. To predict that insufficient food (A) Measurements of the speed of moving galaxies are
production will cause a hunger crisis in the future is nonsense. extremely unreliable.
Any hunger crisis will result from a distribution problem (B) The workings of gravitational forces are not particularly
rather than a production problem. well understood.
The statement above assumes which of the following? (C) The aggregate mass of visible stars in the galaxies
(A) The world’s food requirements are greater than they mentioned above can be estimated with some confidence.
will be in the future. (D) The general composition of unseen matter in the
(B) A shortfall in the world’s food production can be universe has been determined.
prevented by a better distribution system. (E) Without exception, the galaxies mentioned above move
(C) The world’s food production will continue to be sufficient toward one another.
to meet or exceed needs.
(D) The distribution of the world’s existing food supply will 6. New Jersey has one of the lowest motor vehicle fatality
48
American Education Aids - LSAT - Logical Reasoning
rates in the country, and it is one of the few states that (A) Employees at the professional level who do not smoke
require extensive annual automobile safety inspections. should keep their enclosed offices.
Therefore, all states should adopt similar safety inspection (B) Employees with enclosed offices should not smoke at
procedures. their desks, even though the new regulations permit them to
The argument above is based on which of the following do so.
assumptions? (C) Employees at the secretarial level should be allowed
(A) Most states tend to impose few rules and regulations to smoke at their desks, even if they do not have enclosed
on the automobile drivers operating within those states. offices.
(B) Most states are skeptical that annual automobile safety (D) The smoking regulations should allow all employees
inspections can reduce fatality rates. who smoke an equal opportunity to do so, regardless of an
(C) Annual automobile safety inspections contribute employee’s job level.
significantly to holding down New Jersey’s fatality rate. (E) The smoking regulations should provide equal
(D) Drivers in New Jersey are more safety conscious than protection from any hazards associated with smoking to all
are drivers in other parts of the country. employees who do not smoke.
(E) A smaller number of people die on the roads in New
Jersey than in most other states. 9. The greatest chance for the existence of extra-terrestrial
life is on a planet beyond our solar system. The Milky Way
7. When people evade income taxes by not declaring taxable galaxy alone contains 100 billion other suns, many of which
income, a vicious cycle results. Tax evasion forces lawmakers could be accompanied by planets similar enough to Earth to
to raise income tax rates, which causes the tax burden on make them suitable abodes of life.
nonevading taxpayers to become heavier. This, in turn, The statement above assumes which of the following?
encourages even more taxpayers to evade income taxes by (A) Living creatures on another planet would probably have
hiding taxable income. the same appearance as those on Earth.
The vicious cycle described above could not result unless (B) Life cannot exist on other planets in our solar system.
which of the following is also assumed? (C) If the appropriate physical conditions exist, life is an
(A) An increase in tax rates tends to function as an inevitable consequence.
incentive for taxpayers to try to increase their pretax incomes. (D) More than one of the suns in the galaxy is accompanied
(B) Some methods of detecting tax evaders, and thus by an Earth-like planet.
recovering some tax revenue lost through evasion, bring in (E) It is likely that life on another planet would require
more than they cost, but their success rate varies from year conditions similar to those on Earth.
to year.
(C) When lawmakers establish income tax rates in order 10. A small dose of a widely used tranquilizer allows people
to generate a certain level of revenue, they do not allow to lie during lie detector tests without being discovered. The
adequately for revenue that will be lost through evasion. stress responses that lie detector tests measure can be
(D) Noone who routinely hides some taxable income can inhibited by the drug without noticeable side effects. One of
be induced by a lowering of tax rates to stop hiding such the implications of this fact is that the drug can also be
income unless fines for evaders are raised at the same time. effective in reducing stress in everyday situations.
(E) Taxpayers do not differ from each other with respect An assumption of the passage is that
to the rate of taxation that will cause them to evade taxes. (A) tranquilizers are always an effective treatment for
stress
8. According to the new office smoking regulations, only (B) the inhibition of stress responses increases subjective
employees who have enclosed offices may smoke at their stress
desks. Virtually all employees with enclosed offices are at (C) stress as measured by a lie detector is similar to
the professional level, and virtually all secretarial employees everyday stress
lack enclosed offices. Therefore, secretaries who smoke should (D) persons who lie during a lie detector test always display
be offered enclosed offices. signs of stress
Which of the following is an assumption that enables the (E) it is not desirable to reduce stress in everyday situations
conclusion above to be properly drawn?

49
American Education Aids - LSAT - Logical Reasoning

Chapter 5
'WEAKENS' QUESTIONS
Another, and most frequently asked, type of questions on different ways such as:
Critical Reasoning will mention (in the initial narration) a
set of premises and the conclusion drawn therefrom, and will Which of the following, if true, most seriously weakens the
then ask you to spot that statement from among the five given conclusion drawn above?
choices which, if true, will weaken the soundness of that Which of the following, if true, casts the most serious doubt
conclusion. on the conclusion drawn above?
In other words, this choice will contradict, directly or Which of the following statements identifies a flaw in the
indirectly, one of the premises on which the conclusion is logical coherence of the statement above?
based, or will contain some additional information that will Which of the following, if true, would be most damaging
nullify the force of one of the premises on which the conclusion to the argument above?
was based. Which of the following, if true, constitutes the most serious
objection to the analysis above?
To explain the above in simpler terms, let us go back to Which of the following, if it were discovered, would be
the example from day-to-day life which we had used in the pertinent evidence against the speculation above?
previous chapters. Which of the following, if true, would be most likely to
“The time is 5.30 now and the movie starts at 6.30. Unless cause the above prediction to be inaccurate?
we leave home within the next 15 minutes, we shall be missing Which of the following statements, if true, would help to
its beginning” refute the above claim?
Suppose that the speaker’s watch is running fast and it is Which of the following, if true, best counters the argument
showing 5.30 when the real time is only 5.15. If this is true, presented above?
he will be able to watch the beginning of the movie even if he Which of the following, if true, most seriously jeopardizes
leaves home within the next 30 minutes. Therefore, the the validity of the above conclusion?
statement, “The speaker’s watch is running fast by 15 Which of the following, if true, most forcefully undermines
minutes” will weaken his conclusion that he will miss the the argument of the passage above?
beginning of the movie if he does not leave home within the
next 15 minutes. In a variant of the ‘weakens’ question that is occasionally
asked, four of the five choices, if true, will weaken the given
Now consider the statement: “There was a projector conclusion, while the fifth will either strengthen it or will be
breakdown during the afternoon show in the movie theater, irrelevant to it, and you will be asked to select this choice as
because of which that show will end only at 7.00”. your answer.
If this is true, the next show will not begin at least until Such questions will take the form:
7.00, and the speaker will not miss the beginning even if he Each of the following, if true, would weaken the conclusion
leaves home within the next 45 minutes. drawn above EXCEPT
So this statement will also weaken the conclusion of the
speaker that he will miss the beginning of the movie if he The largest percentage of Critical Reasoning questions
does not leave home within the next 15 minutes. are under the ‘weakens’ category.

Consider the statement: “The speaker’s watch is running We are giving below a number of solved examples of this
slow by 10 minutes”. type, following by 12 sets of 10 questions each for your
This means that the real time just now is 5.55 and not practice.
5.45. This will not weaken, but will strengthen, the speaker’s
conclusion that he will miss the beginning of the movie if he Example 1
does not leave within 15 minutes. Neighboring landholders: Air pollution from the giant
aluminium refinery that has been built next to our land is
Consider the statement: “The local mayor is expected to killing our plants.
see the movie during the evening show”. Company spokesperson: The refinery is not to blame, since
Obviously, this statement neither weakens nor strengthens our study shows that the damage is due to insects and fungi.
the speaker’s conclusion that he will miss the beginning of Which of the following, if true, most seriously weakens the
the movie if he does not leave home within the next 15 minutes, conclusion drawn by the company spokesperson?
and is irrelevant to it. (A) The study did not measure the quality of pollutants
emitted into the surrounding air by the aluminium refinery
The various choices in the ‘weakens’ question will be of (B) The neighboring landholders have made no change in
the above types. the way they take care of their plants
(C) Air pollution from the refinery has changed the
Among the choices given under a ‘weakens’ question, only chemical balance in the plants’ environment, allowing the
one will weaken the given conclusion (and will be the answer), harmful insects and fungi to thrive
while each of the others will either strengthen it or will be (D) Pollutants that are invisible and odorless are emitted
irrelevant to it. into the surrounding air by the refinery
(E) The various species of insects and fungi mentioned in
The ‘weakens’ questions may be worded in a number of
50
American Education Aids - LSAT - Logical Reasoning
the study have been occasionally found in the locality during of $50 or more has increased by 30% since the beginning of
the past hundred years the scheme.
This claim regarding the success of the scheme (the basic
Analysis aim of which is to get new customers) would be weakened if it
The accusation of the landholders is that air pollution from is proved that these purchases have been made only by
the aluminium refinery is responsible for the damage to their traditional customers of Red Label who have now combined
plants. their many purchases into a single one just in order to benefit
The refinery’s spokesperson admits that the plants are from the discount, and not by new customers.
getting damaged, but says that the studies undertaken by (E) is what states this, and is the answer.
the refinery found that the damage to the plants was, in fact, (A) would strengthen and not weaken the claim of the
being caused by insects and fungi. He therefore concludes executives, because it would mean that Red Label had indeed
that air pollution from the refinery is not responsible for the attracted new customers, and these new customers were
damage to the plants. making big purchases just to get the benefit of the discount.
We have been asked to spot the argument which would If (B) is true, then there would have been no spurt in the
weaken the spokesperson’s statement. sales of items above $50, and this contradicts the information
Obviously, his conclusion would be most weakened if it is given in the passage.
found that the growth of insects and fungi in the area was (C) and (D) would both strengthen, and not weaken, the
itself facilitated by the air pollution caused by the refinery. executives’ claim that the scheme has been a grand success.
In this case, it is the refinery which would be indirectly, (In this example three of the wrong choices strengthen the
even if not directly, responsible for the damage to the plants. given conclusion, while one contradicts the information given
This is what is stated in (C), which is the answer. in the passage.)
It is easy to see that none of the other choices logically
connects (a) air pollution by the refinery, (b) growth of insects Example 3
and fungi, and (c) damage to the plants. This year the New Hampshire Division of Company X
(In this example, all the four wrong choices are irrelevant set a new record for annual sales by that division. This record
to the conclusion in the passage.) is especially surprising since the New Hampshire Division
has the smallest potential market and the lowest sales of
Example 2 any of Company X’s divisions.
To entice customers away from competitors, Red Label Which of the following identifies a flaw in the logical
supermarkets have begun offering discounts on home coherence of the statement above?
appliances to customers who spend $50 or more on any (A) If overall sales for Company X were sharply reduced,
shopping trip to Red Label. Red Label executives claim that the New Hampshire Division’s new sales record is irrelevant
the discount program has been a huge success, since cash to the company’s prosperity
register receipts of $50 or more are up thirty percent since (B) Since the division is competing against its own record,
the beginning of the program. the comparison of its sales record with that of other divisions
Which of the following, if true, most seriously weakens the is irrelevant
claim of the Red Label executives? (C) If this is the first year that the New Hampshire
(A) Most people who switched to Red Label after the Division has been last in sales among Company X’s divisions,
program began to spend more than $50 each time they shop the new record is not surprising at all
at Red Label (D) If overall sales for Company X were greater than usual,
(B) Most people whose average grocery bill is less than it is not surprising that the New Hampshire Division was
$50 would not be persuaded to spend more by any discount last in sales
program (E) Since the New Hampshire Division has the smallest
(C) Most people who received discounts on home potential market, it is not surprising that it had the lowest
appliances through Red Label’s program will shop at Red sales
Label after the program ends Analysis
(D) Since the beginning of the discount program, most of The first sentence merely means that, this year, the New
the people who spend $50 or more at Red Label are people Hampshire Division of the Company has beaten its own
who have never before shopped there and whose average previous record for the highest annual sales. This does not
grocery bill has always been higher than $50 mean that it has exceeded the sales figure of any other division
(E) Almost all of the people who have begun spending $50 of the Company having a higher market potential.
or more at Red Label since the discount program began are So, the ‘special surprise’ expressed in the second sentence
longtime customers who have increased the average amount of the passage (comparing it with the sales in other divisions
of their shopping bills by making fewer trips with higher market potentials) is unwarranted.
This flaw in the logical coherence of the given statement
Analysis is best expressed by (B), which is the answer.
(In this question, ‘Red Label’ is the name of a Supermarket The other choices do not bring out the lack of coherence in
chain. ‘Cash register receipts’ mean the amounts received for the line of argument in the given statement.
purchases by the billing staff in the supermarket) The purpose
of the discount scheme is ‘to entice customers away from Example 4
competitors’. This means that, for the scheme to be considered Lark Manufacturing Company initiated a voluntary
successful, the supermarket chain must have attracted more Quality Circles program for machine operators. Independent
new customers after the discount started. surveys of employee attitudes indicated that the machine
The Red Label executives’ claim that the scheme has been operators participating in the program were less satisfied with
a huge success is based only on the fact that single purchases their work situations after two years of the program’s
51
American Education Aids - LSAT - Logical Reasoning
existence than they were at the program’s start. Obviously, high-consumption group than of the low-consumption group
any workers who participate in a Quality Circles program demonstrated a high level of anxiety. The investigator
will, as a result, become less satisfied with their jobs. concluded that it was greater caffeine consumption that
Each of the following, if true, would weaken the conclusion resulted in higher anxiety.
drawn above EXCEPT: Which of the following, if true, most seriously weakens the
(A) The second survey occurred during a period of conclusion above?
recession when rumors of cutbacks and layoffs at Lark (A) Some subjects in the high-consumption group
Manufacturing were plentiful experienced lower levels of anxiety than did other subjects in
(B) The surveys also showed that those Lark machine the high-consumption group
operators who neither participated in Quality Circles nor (B) Some subjects in the low-consumption group did not
knew anyone who did so reported the same degree of lessened experience any anxiety
satisfaction with their work situations as did the Lark (C) High levels of anxiety when consuming large amounts
machine operators who participated in Quality Circles of caffeine were the reason some subjects restricted their
(C) While participating in Quality Circles at Lark intake of caffeine
Manufacturing, machine operators exhibited two of the (D) Some subjects consumed caffeine by drinking coffee
primary indicators of improved job satisfaction: increased and tea, but others consumed caffeine by taking medication
productivity and decreased absenteeism that contained caffeine
(D) Several workers at Lark Manufacturing who had (E) High levels of anxiety caused some subjects to consume
participated in Quality Circles while employed at other large amounts of caffeine
companies reported that, while participating in Quality
Circles in their previous companies, their work satisfaction Analysis
had increased (Caffeine is the substance in coffee which makes it a mild
(E) The machine operators who participated in Quality stimulant. A ‘subject’ means a person whose behaviour has
Circles reported that, when the program started, they felt been studied during the experiment.) From the fact that a
that participation might improve their work situations larger percentage of those in the high-caffeine consumption
group also showed a high level of anxiety, the investigator
Analysis has drawn the conclusion that greater caffeine consumption
(Note the question carefully because attempting to answer results in higher anxiety.
it. The question implies that four of the five given choices will We have been asked to identify that choice which, if true,
weaken the conclusion drawn in the passage, while one will would weaken the investigator’s conclusion.
either strengthen it or will have no relevance to it. We have Given that higher level of caffeine consumption and higher
been asked to spot this particular choice.) The conclusion level of anxiety occur together, the investigator has jumped
reached in the illustration, as a result of observations in a to the conclusion that the former is the cause and the latter is
specific case, is that any worker who participates in a Quality the effect. He has not considered the possibility that the higher
Circles program will get less satisfied with his job as time level of anxiety may be the cause and the higher level of
goes by. caffeine consumption may be its effect. This is therefore the
(A) implies that the unhappiness of the workers brought weakness in his conclusion. (E), which points out such a
out by the survey was attributable to extraneous causes, and possibility, is the answer.
not to their participation in the Quality Circles. It will thus (Watch out for this type of question in which two related
weaken the given conclusion, and is not the answer. phenomena are mentioned, and one is straightaway presumed
(B) also implies that the unhappiness of the workers was to the cause and the other its effect, without considering the
not caused by Quality Circles, but by some other causes. This possibility that the latter could be the cause and the former
will also weaken the stated conclusion that their its effect. We have come across similar cases under ‘conclusion’,
dissatisfaction was caused by their participation in Quality ‘inference’ and ‘assumption’ questions also.)
Circles, and is not the answer. The second sentence of the narration, “A significantly
(C) and (D) mean that the workers who participated in larger percentage of the high-consumption group than of the
Quality Circles actually benefited by it, and were not low-consumption group demonstrated a high level of anxiety”
dissatisfied with it. These not merely weaken the stated means that, in each group, some people felt a higher level of
conclusion, but contradict it. anxiety while some others felt a lower level of anxiety. So, (A)
The primary purpose of Quality Circles is to increase the corroborates what is stated in the narration, and does not
efficiency of operation, and not an improvement in the working weaken it.
conditions of the workers (though the latter may be an (B) strengthens, and does not weaken, the conclusion of
incidental benefit in some cases). If, as stated in (E), the the researcher relating higher consumption of caffeine to
workers had presumed that participation in Quality Circles higher level of anxiety.
would improve their working conditions, but this did not (C) also strengthens the researcher’s finding that higher
happen, then they would be disappointed, and therefore would consumption of caffeine leads to higher levels of anxiety, and
be less satisfied with their present working conditions than does not weaken it.
they were at the outset. The study was to find the correlation between the
So it is (E), if true, that will not weaken the conclusion in consumption levels of caffeine by different individuals to their
the passage, but will strengthen it, and is the answer. anxiety levels. The method of intake of the caffeine was
irrelevant to the study. (D) neither weakens nor strengthens
Example 5 the researcher’s conclusion, but is irrelevant to it.
An investigator divided 84 adults into two distinct groups (In this instance, three of the wrong choices strengthen
(low and high consumption) based on each person’s daily the conclusion, while the fourth is irrelevant to it.
caffeine consumption. A significantly larger percentage of the
52
American Education Aids - LSAT - Logical Reasoning
Example 6 share of the global export trade stable over the years.
The cost of flood damage since 1980 has been greater in Which of the following, if true, casts the most serious doubt
areas that have adopted flood-control measures than in those on the suitability of those four countries as models in the sense
that have not. Thus, flood-control projects are expensive described?
engineering mistakes and a substantial waste of resources. (A) Many countries wish to increase their share of world
Which of the following, if true, would be most damaging export trade, not just keep it stable
to the argument above? (B) Many countries are less concerned with exports alone
(A) Those areas that had not suffered flood damage before than with the balance between exports and imports
1980 are the only ones that have not adopted flood-control (C) With respect to the mix of products each exports, the
measures four countries are very different from each other
(B) Since 1980, those areas that have taken flood-control (D) Of the four countries, two had a much larger, and two
measures have suffered greater flood damage than they did had a much smaller, share of total world exports in 1970 than
in the previous decade in 1984
(C) The cost of flood damage had increased every year (E) The exports of the four countries range from 15 percent
since 1980 to 75 percent of the total national output
(D) Faulty engineering has not been the only cause of the
failure of flood-control projects Analysis
(E) The amount of rainfall since 1980 has been From the fact that the four countries had the same share
substantially greater than normal of total world exports in 1984 as they had in 1953, the author
comes to the conclusion that they can be adopted as models
Analysis by other countries which wish to keep their share of the global
The given argument is that, since areas that had adopted export trade stable over the years.
flood-control measures have suffered more flood damage since The author’s recommendation applies only to countries
1980 than those areas which had not taken such measures, which wish to have a stable share in the world export trade,
the expenditure on flood control in the former has been a and not to those countries which either wish to increase their
waste. share of the trade, or those whose primary aim is to have a
We have been asked to identify the choice which will be balance between exports and imports. So, (A) and (B) neither
most damaging to this argument. strengthen nor weaken the conclusion of the author, and are
The only reason given for the author’s conclusion that flood irrelevant to it.
control projects are a substantial waste of resources is that (C) and (E) will both strengthen the author’s
the cost of flood damage since 1980 has been greater in areas recommendation, because these models themselves cover
that have adopted flood-control measures than in those that different situations, and can apply to countries with different
have not. mixes of products and different percentages of exports.
If it is a fact, as stated in (A), that it is only those areas By citing these four countries as models for countries
which had not suffered flood damages before 1980 that have which wish to keep their share of the global export trade stable
not adopted flood-control measures since then, it implies that over the years, the author obviously presumes that the share
these areas are not prone to flood damage at all. Comparing of these four countries in the global export trade had remained
the absence of damage in non-flood-prone areas to the damage almost the same in each year between 1953 and 1984. If this
in flood-prone areas where flood-control measures were needed is not true, and if there had been wide fluctuations in these
is therefore illogical, and is most damaging to the argument shares year by year (and that it was merely accidental that
of the author. the share in 1984 was the same as the share in 1953), the
So, (A) is the answer. suitability of these four countries as models by countries
(B) will strengthen, and not weaken, the author’s which wish to have stable export trade share would be belied.
conclusion, and is not the answer. (D) says that, in one of the intermediate years 1970, two of
(C) talks generally of the ‘cost of flood damage’, without these four countries had a much larger share of the world
comparing the relative figures in areas where flood control exports than they had in 1984, and the other two had a much
measured had been implemented and where they had not smaller share of the world exports than they had in 1984. If
been implemented. So, it neither strengthens nor weakens the this is true, these four countries obviously cannot be models
given conclusion, and is irrelevant to it. for other countries which wish to keep their share of the global
By stating that faulty engineering was not the only cause export trade stable over the years.
for the failure of the flood control projects, (D) concedes that Thus, (D) casts the most serious doubt about the validity
it was one of the causes, and thereby strengthens the given of the given conclusion, and is the answer.
argument. (Note that, in this example, two of the four wrong choices
(E) also talks generally of rainfall, and does not distinguish strengthen the conclusion, while two are irrelevant to it.)
between the amount of rain that fell in areas where flood
control measures had been implemented and the areas where Example 8
such measures had not been implemented. Thus (E) neither Reviewer: The book Art’s Decline argues that European
strengthens nor weakens the given conclusion, and is painters today lack skills that were common among European
irrelevant to it. painters of preceding centuries. In this the book must be
right, since its analysis of 100 paintings, 50 old and 50
Example 7 contemporary, demonstrates convincingly that none of the
Of the countries that were the world’s twenty largest contemporary paintings are executed as skillfully as the older
exporters in 1953, four had the same share of total world paintings.
exports in 1984 as in 1953. These countries can therefore Which of the following points to the most serious logical
serve as models for those countries that wish to keep their flaw in the reviewer’s argument?
53
American Education Aids - LSAT - Logical Reasoning
(A) The paintings chosen by the book’s author for analysis that neither subscribers nor advertisers will be lost if the
could be those that most support the book’s thesis magazine’s plan is instituted.
(B) There could be criteria other than the technical skill Which of the following, if true, provides the strongest
of the artist by which to evaluate a painting evidence that the magazine’s profits are likely to decline if the
(C) The title of the book could cause readers to accept the plan is instituted?
book’s thesis even before they read the analysis of the (A) With the new postage rates, a typical issue under the
paintings that supports it proposed plan would cost about one-third more to mail than
(D) The particular methods currently used by European a typical current issue would
painters could require less artistic skill than do methods used (B) The majority of the magazine’s subscribers are less
by painters in other parts of the world concerned about a possible reduction in the quantity of the
(E) A reader who was not familiar with the language of magazine’s articles than about a possible loss of the current
art criticism might not be convinced by the book’s analysis of high quality of its articles
the 100 paintings (C) Many of the magazine’s long-time subscribers would
continue their subscriptions even if the subscription price
Analysis were increased
There are two arguments mentioned in the narrative. The (D) Most of the advertisers that purchase advertising
first is the argument of the author of the book ‘Art’s Decline’ space in the magazine will continue to spend the same amount
that European painters of today lack the skills that were on advertising per issue as they have in the past
common among European painters of preceding centuries. (E) Production costs for the magazine are expected to
The second is the argument of the reviewer that what is remain stable
stated in the book must be right, because its analysis of 100
paintings (50 old and 50 contemporary) convincingly Analysis
demonstrates that none of the contemporary paintings are The problem faced by the magazine is the rise in the
executed as skillfully as the older paintings. postage rates. In order to maximize its profits, the magazine
The question is about the reviewer’s argument, and we wants to reduce its expenses on postage. To achieve this end,
have been asked to locate the choice which points out the it wants to reduce by one half the number of issues it produces
most serious logical flaw in it. every year.
The 50 old paintings and the 50 contemporary paintings For example, if the magazine is a monthly now and postage
have been chosen by the author of the book himself. It is quite has to be incurred 12 times a year, the new proposal will make
likely that, as the representatives of old paintings, he had it a bi-monthly so that postage has to be incurred only 6 times
deliberately chosen those which showed high skills and, as a year.
the representatives of contemporary paintings, he had The decision of the management that the number of
deliberately chosen those which showed only minimum skills. articles or their quality or the annual subscription rate will
If this were the case, the author had not arrived at his not change implies that each of the new issues will have nearly
conclusion on the basis of objective evidence, but had come to double the number of pages that the present issues have, and
his conclusion first and tailored the evidence to suit that each new issue will be priced at twice the present price.
conclusion. If so, his conclusion certainly cannot be considered The narrative also says that the market research has
to be right as is asserted by the reviewer. shown that neither subscribers nor advertisers will be lost if
(A) points out this flaw, and is the answer. the new plan is implemented.
The argument in both the book and the review is about Having been given all this information, we are asked to
the skills of the artists, and not about general evaluation of spot that choice which implies that the magazine’s profits
paintings based on any other criteria. The question is also are likely to decline if the plan is implemented in spite of the
about the skills only. So, even if (B) were true, it does not projected saving on postage.
constitute a logical flaw in the argument about their skills. (A) implies that, if the cost of posting each current monthly
Thus (B) neither strengthens nor weakens the reviewer’s issue is 10 cents, the cost of posting each new bi-monthly
conclusion, and irrelevant to it. issue will be 13 cents. So, the annual postage expense for
The question is about the argument of the reviewer. The each subscriber will be only 78 cents (6 x13) hereafter, as
view of other readers of the same book are irrelevant to the against 120 cents at present. So, the profits will increase and
question. So, (C) is not the answer, not decline. So, (A) is not the answer.
(D) compares today’s European painters with today’s If (B) is true, each new issue need not have twice the
painters in other parts of the world, whereas the book and number of articles that each old issue had, and each new
the review compare today’s European painters with the issue can therefore have less number of pages than two old
European painters of the preceding centuries. So, (D) is issues together, thereby reducing the expenses on paper,
irrelevant to the argument. printing and postage. So, in this case, the profits will increase,
(E) also talks about the views of a reader, and is not relevant and not decline. So, (B) is not the answer.
to the issue why the reviewer’s argument is flawed. If subscription rate is increased and all old subscribers
(Note that all the four wrong choices in this example are continue to buy the magazine, the profits will increase and
irrelevant to the given conclusion.) not decline. So, (C) is not the answer.
(D) implies that, if the advertising revenue from each old
Example 9 issue was $x, the advertising revenue from each new issue
Because postage rates are rising, Home Decorator will also be $x. This means that, as against the earlier annual
magazine plans to maximize its profits by reducing by one advertisement revenue of $12x, the new annual
half the number of issues it publishes each year. The quality advertisement revenue will be only $6x. So, if (D) is true, and
of articles, the number of articles published per year, and the the saving in postage is less than $6x, the magazine’s profits
subscription price will not change. Market research shows
54
American Education Aids - LSAT - Logical Reasoning
will decline. So, (D) is the answer. that period showing symptoms of the disease. Recently,
If production costs remain stable, but the postage expenses however, chemical analyses of skeletons have led scientists
come down, the profit will increase and not decline. So, (E) is to propose that the joint damage was actually caused by
not the answer. chemicals used by Egyptian embalmers.
Which of the following, if true, would additionally weaken
Example 10 the traditional view that alkaptonuria afflicted many Egyptian
Some analysts maintain that an embargo by country 2,500 years ago?
Litora on the export of a strategic metal to country Zenda, if (A) X-rays of the mummies showed shadows that clearly
imposed, would drive up the price of the metal in Zenda at suggested joint damage, and recent inspection of the skeletons
least tenfold. They note that few other countries export the has confirmed that hypothesis.
metal and that, with an embargo, Zenda might have to depend (B) Although alkaptonuria is a disease that can be
on as-yet-unexploited domestic sources of the metal. inherited, it did not appear in the descendants of the Egyptian
Which of the following, if true, constitutes the most serious population in which the symptoms were found.
objection to the analysis above? (C) Egyptian embalming methods were highly secret, and
(A) Litora’s economy depends heavily on foreign currency scientist are still not certain of the nature of some of the
earned by the export of the strategic metal to other countries. chemicals that were used.
(B) There are foreign-policy steps that Zenda could take (D) Possible evidence of alkaptonuria has been pointed
to appease Litora and avoid being subjected to an embargo out in pictures representing the human figure found on
on the metal. artifacts left by other Middle Eastern cultures of that period.
(C) Geologists believe that additional deposits of the metal (E) Some mummies of that period show no evidence of
could possibly be found within the territory of Litora. joint damage at all.
(D) Only a small proportion of Zenda’s import expenditures
is devoted to the import of the metal from Litora. Analysis
(E) In case of an embargo, Zenda could buy the metal (“Embalmer” is a person who chemically treats dead bodies
indirectly from Litora on the world market at a less than- in order to preserve them from decomposing. Egyptian
one-third increase in cost. mummies are dead bodies which have been so preserved for
over 2500 years). The recent evidence, according to scientists,
Analysis has shown that the Egyptians whose bodies have been
The facts that we can note from the passage are: (i) preserved as mummies did not have the disease alkaptonuria
Country Zenda needs a particular strategic metal; (ii) Though when they were alive, and the damage to the joints in the
it has some deposits of this metal, it has not so far exploited mummies was actually caused by the chemicals used by the
them; (iii) it has so far been depending mainly on imports of embalmers after the death of the persons.
this metal from another country Litora, because few other We have been asked to identify the argument among the
countries export this metal; (iv) Litora is presently planning choices which further weakens the view that many Egyptians
to impose an embargo on the export of this metal to Zenda. had suffered from the disease when they were alive.
The passage then says that some analysts point out that (B) says that the disease is one that can be inherited. So,
such embargo, if imposed, is likely to push up the price of if this disease had been an epidemic among Egyptians 2500
this metal in Zenda at least tenfold. years back, then some at least of their descendants in the
We have been asked to spot that choice which, if true, will present generation must have this disease now. So, (B) which
weaken the view of these analysts. says that this disease did not appear in any of the descendants,
(A) is not the answer because the proposed embargo is will weaken the view that this was an epidemic in Egypt at
not against all exports of this metal from Litora, but against one time. So (B) is the answer.
the export of the metal to Zenda alone. So, Litora can continue (A) merely confirms that mummies have joint damage,
to earn most of its needed foreign exchange even after the and neither strengthens nor weakens the earlier assumption
proposed embargo comes into operation. that these persons had joint damage when they were alive.
(B) is not the answer, because the analysts’ views relate (D) will strengthen and not weaken the original hypothesis
to the consequences on Zenda if the proposed embargo comes that the disease was prevalent among ancient Egyptians.
into effect, and will not be not relevant to a situation in which (C) will neither strengthen nor weaken the earlier or the
the embargo is avoided completely through diplomatic means. new hypothesis, and is irrelevant to the case.
(C) is not the answer because, if the embargo comes into (E) is already implied in the second sentence in the passage
effect, the discovery of additional deposits of the metal in that “a high proportion of mummies” and not all of them,
Litora will be of no use to Zenda. showed the symptoms of the disease, and is not a new
(D) is not the answer, because the speculation of the information. It neither strengthens nor weakens the new or
analysts is not on the consequences of the embargo on Zenda’s the old hypothesis.
overall economy, but is limited to the possible steep spurt in (Note that, in this example, one of the wrong choices
the price of this metal alone. strengthens the earlier hypothesis, while the three others are
If (E) is true, the spurt in the import price of this metal by irrelevant to that hypothesis.)
Zenda will be only about 33%, and will not be tenfold or 1000%
as speculated by the analysts. So, it is (E) which will constitute Example 12
the most serious objection to their analysis, and is the answer. Researchers studying sets of identical twins who were
raised apart in dissimilar environments found that in each
Example 11 case the twins were similar in character, medical history, and
It was long thought that a now-rare disease of the joints, life experiences. The researchers saw these results as
alkaptonuria, was epidemic in Egypt 2,500 years ago. confirmation of the hypothesis that heredity is more
Evidence came from the high proportion of mummies from important than environment in determining human
55
American Education Aids - LSAT - Logical Reasoning
personalities and life histories. most seriously weaken the hypothesis stated above.
The existence of which of the following would tend to We should therefore look for that choice which states that,
weaken the support for the hypothesis above most seriously? in another experiment, identical twins raised in dissimilar
(A) A set of identical twins raised together who are shown environments showed markedly different personality traits.
by appropriate tests to have very similar value systems It is (B) which says this, and is the answer.
(B) A pair of identical twins raised apart who differ (A) will strengthen, and not weaken the hypothesis, and is
markedly with respect to aggressiveness and other not the answer.
personality traits (C) mentions a case in which both the heredity and the
(C) A younger brother and older sister raised together who environment of growing were the same. It can therefore
have similar personalities and life experience
neither strengthen nor weaken the stated hypothesis.
(D) A mother and daughter who have the same profession
A mother could have grown up in an environment that
even though they have very different temperaments
was totally different from the one in which her daughter grows
(E) A pair of twins raised together who have similar
up. Moreover, some of the hereditary traits of the daughter
personality traits but different value systems
could have come from her father. So, the fact that a mother
Analysis and daughter exhibited different temperaments would neither
The first sentence says that, in a study of identical twins strengthen nor weaken the hypothesis that heredity plays a
raised separately in dissimilar environments, the twins more important role than environment in shaping one’s
showed similarities in character, medical history and life personality traits. So, (D) is not the answer.
experiences. The researchers felt that this study confirmed The hypothesis is about personality traits and not about
the hypothesis that heredity is more important than value systems. So, (E) would also neither strengthen nor
environment in determining human personalities and life weaken the stated hypothesis.
histories. (In this example, one of the choices strengthens the
We have been asked to spot the choice which, if true, would hypothesis, while three others are irrelevant to it.)

We shall now give a large number of ‘weakens’ questions for you practice on.

56
American Education Aids - LSAT - Logical Reasoning
SET 1
1. Corporations exist to make money for their shareholders. catlike animals of the period.
Corporations have no money to give away because any excess (E) The bones in the cave did not include bones of an
funds belong to the shareholders. Therefore, corporations elephantlike animal that existed in the area at the time of
should not contribute to charities. the australopithecines.
Which of the following most seriously weakens the reasoning
in the argument above? 4. A research study reports that a particular educational
(A) Managers are being trained to consider the ethical, program has improved the prospects for success in later
moral, and social effects of their decisions in addition to the schooling for those children aged three to five who were
economic factors subjects of the study. It follows, then, that introducing similar
(B) Some of the largest corporations in the country are programs for all children aged three to five will improve their
known to be major contributors to philanthropic organizations chances for success in later schooling.
(C) Shareholders tend to make larger individual donations Which of the following, if true, would most weaken the
to charities than do people who hold no shares in corporations argument above?
(D) Most corporations contribute solely to large, well- (A) The parents of pre-school children in the United States
known, and highly organized charitable organizations rather are attracted to educational fads and do not have a clear idea
than to individuals in need of what sorts of early education programs might benefit their
(E) The favourable publicity and tax advantages gained children.
by corporate charity often result in an increase in profits (B) The cognitive abilities of children are constantly
greater than the actual costs of making the donations changing between ages three and five.
(C) The researchers unwittingly included a substantial
2. Tocqueville, a nineteenth-century writer known for his number of children who had been previously exposed to
study of democracy in the United States, believed that a another educational enrichment program.
government that centralizes power in one individual or (D) Many parents erroneously presume that early formal
institution is dangerous to its citizens. Biographers claim that instruction takes up time that children could better spend
Tocqueville disliked centralized government because he exploring their worlds independently.
blamed Napoleon’s rule for the poverty of his childhood in (E) It would require extraordinary public expense to
Normandy. establish such educational enrichment programs on a national
Which of the following, if true, would cast the most serious basis.
doubt on the biographers’ claim?
(A) Although Napoleon was popularly blamed at the time 5. Most archaeologists have held that people first reached
for the terrible living conditions in Normandy, historians now the Americas less than 20,000 years ago by crossing a land
know that bad harvests were really to blame for the poor bridge to North America. But recent discoveries of human
economic conditions. shelters in South America dating from 32,000 years ago have
(B) Napoleon was notorious for refusing to share power led researchers to speculate that people arrived in South
with any of his political associates. America first after voyaging across the Pacific, and then
(C) Tocqueville said he knew that, if his father had not spread northward.
suffered ill health, his family would have had a steady income Which of the following, if it were discovered, would be
and a comfortable standard of living. pertinent evidence against the speculation above?
(D) Although Tocqueville asserted that United States’ (A) A rock shelter near Pittsburg, Pennsylvania, contains
political life was democratic, the United States of the evidence of use by human beings 19,000 years ago.
nineteenth century allowed political power to be concentrated (B) Some North American sites of human habitation
in a few institutions. predate any sites found in South America.
(E) Tocqueville once wrote in a letter that, although his (C) The climate is warmer at the 32,000-year-old South
childhood was terribly impoverished, it was not different from American site than at the oldest known North American site.
the experience of his friends and neighbors in Normandy. (D) The site in South America that was occupied 32,000
years ago was continuously occupied until 6,000 years ago.
3. Found in caves with the bones of australopithecines, (E) The last Ice Age, between 11,500 and 20,000 years ago,
which are thought by some to be ancient ancestors of the considerably lowered worldwide sea levels.
human species, were great collections of animal bones. From
the frequencies of types of bones, it can be seen that many 6. Some who favour putting governmental enterprises into
bones represented only parts of animals that must have died private hands suggest that conservation objectives would in
elsewhere. The australopithecines thus must have been general be better served if private environmental groups were
mighty hunters, to have brought home so much meat. put in charge of operating and financing the national park
Which of the following, if true, most seriously weakens the system, which is now run by the government.
conclusion drawn above? Which of the following, assuming that it is a realistic
(A) The australopithecines sometimes moved from cave to possibility, argues most strongly against the suggestion above?
cave for shelter and did not remain in one cave for a lifetime. (A) Those seeking to abolish all restrictions on exploiting
(B) The australopithecine bones found in the caves were the natural resources of the parks might join the private
those of adult males, adult females, and juveniles. environmental groups as members and eventually take over
(C) Evidence of the use of fire was absent from the caves their leadership.
in which the collections of bones were found. (B) Private environmental groups might not always agree
(D) Marks on the bones, including the bones of the on the best ways to achieve conservation objectives.
australopithecines, are consistent with teeth marks of large (C) If they wished to extend the park system, the private
57
American Education Aids - LSAT - Logical Reasoning
environmental groups might have to seek contributions from (C) No provision was made to ensure that a dead bird would
major donors and the general public. not be reported by more than one observer
(D) There might be competition among private (D) Initial increases in bird deaths had been noticed by
environmental groups for control of certain park areas. agricultural workers long before any publicity had been given
(E) Some endangered species, such as the California condor, to the matter
might die out despite the best efforts of the private (E) Dead birds of the same species as those found in
environmental groups, even if those groups are not hampered agricultural areas had been found along coastal areas where
by insufficient resources. no farming took place

7. The Commerce Department recently put limits on 9. Politician: The governor claims that the state’s economy
machine-tool imports from two countries whose exports of is prospering. Yet during her term of office the state has lost
machine tools into the United States have been substantial. 26,000 manufacturing jobs. Clearly, the state’s economy is
As a result of these restrictions, analysts predict that domestic not doing very well.
sales of machine tools manufactured in the United States Which of the following, if it occurred during the four years
are bound to rise considerably, starting in the very near future. of the governor’s term of office, most seriously weakens the
Which of the following, if true, would be most likely to cause politician’s conclusion?
the analysts’ prediction to be inaccurate? (A) An influx of non-manufacturing industries led to a net
(A) A new tax bill that, if passed, would discourage gain of 200,000 new jobs in the state
investment in capital equipment such as machine tools is (B) The average hourly wage paid for jobs in the state kept
being studied and debated seriously in the United States up with inflation
Congress. (C) The average level of training required for entry level
(B) United States companies’ orders for metal-cutting jobs in the state increased
machines, which account for 75 percent of sales by the (D) The governor’s budgets increased state spending on
machine-tool industry, rose faster than orders for other types services for health, education, poor people, and senior citizens
of machine tools during the past year. (E) Statewide opinion polls indicated that a majority of
(C) Worldwide orders for machine tools made in the United the state’s employed workers were reasonably well-satisfied
States dropped by more than 10 percent during the past year. with the governor’s performance.
(D) Substantial inventories of foreign made machine tools
were stockpiled in the United States during the past year. 10. Some soil scientists have asserted that decaying matter
(E) Companies in the industrial sectors of many countries on the forest floor is a far greater source of the acidity in
showed a significantly expanded demand for machine tools mountain lakes than is the acid rain that falls on these lakes.
during the past year. Therefore, they contend, reducing acid rain will not
significantly reduce the acidity levels of mountain lakes.
8. Throughout the 1950’s, there were increases in the Which of the following statements, if true, most seriously
numbers of dead birds found in agricultural areas after weakens the argument above?
pesticide sprayings. Pesticides manufacturers claimed that (A) It is natural for mountain lakes to have acidity levels
the publicity given to bird deaths stimulated volunteers to higher than those of other lakes.
look for dead birds, and that the increase in numbers reported (B) The harmful effects of increased acidity levels in lakes
was attributable to the increase in the number of people have been greatly underestimated.
looking. (C) Acid rain is found in urban and heavily industrialized
Which of the following statements, if true, would help to regions of the country.
refute the claim of the pesticide manufacturers? (D) There is much disagreement among soil scientists about
(A) The publicity given to bird deaths was largely regional the causes of acid rain.
and never reached national proportions (E) While plant life remains, acid rain significantly
(B) Pesticide sprayings were timed to coincide with various increases the amount of decaying organic matter in natural
phases of the life cycles of the insects they destroyed environments.

SET 2
1. The average normal infant born in the United States 2. The increased concentration of salt in the bay, which is
weighs between twelve and fourteen pounds at the age of the result of recent drought and high temperatures, will cause
three months. Therefore, if a three-month-old child weighs many fish to die. Shrimp, however, can tolerate high salt
only ten pounds, its weight gain has been below the United levels; the shrimp industry will not, therefore, be hurt by the
States average. increased concentration of salt.
What is the flaw in this argument? Which of the following statements, if true, would weaken
(A) Weight is only one measure of normal infant the argument above?
development (A) Some fish will migrate to areas that have lower
(B) Some three-month-old children weigh as much as concentrations of salt.
seventeen pounds (B) Lack of rainfall for extended periods of time lowers the
(C) It is possible for a normal child to weigh ten pounds at water level of bays.
birth (C) The organisms on which young shrimp feed cannot
(D) The phrase “below average” does not necessarily mean survive in such salty waters.
insufficient (D) Increased water temperature often causes shrimp to
(E) Average weight gain is not the same as average weight multiply more quickly.
58
American Education Aids - LSAT - Logical Reasoning
(E) Shrimp are more abundant in areas of the bay that 6. Nonprescription sunglasses shield the wearer’s eyes from
are sparsely populated by fish. damaging ultraviolet sunlight. Squinting, however, provides
protection from ultraviolet rays that is at least as good as the
3. Teen-agers are often priced out of the labor market by protection from nonprescription sun-glasses. There is,
the government-mandated minimum-wage level because therefore no health advantage to be gained by wearing
employers cannot afford to pay that much for extra help. nonprescription sunglasses rather than squinting.
Therefore, if Congress institutes a sub-minimum wage, a new Which of the following, if true, most seriously weakens the
lower legal wage for teenagers, the teen-age unemployment support for the conclusion above?
rate, which has been rising since 1960, will no longer increase. (A) Many opticians offer prescription sunglasses that not
Which of the following, if true, would most weaken the only screen out ultraviolet sunlight but also provide corrective
argument above? vision.
(A) Since 1960, the teen-age unemployment rate has risen (B) Some nonprescription sunglasses provide less
when the minimum wage has risen protection from ultraviolet sunlight than does squinting.
(B) Since 1960, the teen-age unemployment rate has risen (C) Squinting strains facial muscles and causes headaches
even when the minimum wage remained constant and fatigue.
(C) Employers often hire extra help during holidays and (D) Many people buy sunglasses because they feel that
warm-weather seasons sunglasses are fashionable.
(D) The teen-age unemployment rate rose more quickly in (E) Some people squint even when they are wearing
the 1970’s than it did in the 1960’s sunglasses.
(E) The teen-age unemployment rate has occasionally
declined in the years since 1960 7. A common misconception is that university hospitals
are better than community or private hospitals. In fact,
4. Industries can and will build new plants in a world of university hospitals have a lower survival rate for patients
high environmental standards. But in a world of uncertainty, than do other hospitals. From this it seems clear that the
where the environmental standards that must be met can quality of care at university hospitals is lower than that at
change and construction that was once acceptable can be other hospitals.
halted because it conflicts with newly passed regulations, new Which of the following, if true, most forcefully undermines
factories simply will not be built. the argument of the passage above?
Which of the following proposals, if accepted, would work (A) Many doctors divide their working hours between a
most to reduce the threat to the construction of new factories university and a community or private hospital.
described in the passage above? (B) Doctors at university hospitals often earn less than
(A) Environmental cases involving construction should be doctors at private hospitals.
heard and decided by the courts as quickly as possible (C) University and community hospitals often cannot afford
(B) Environmental regulations should be published widely the elaborate facilities of private hospitals.
as soon as they are established (D) The emphasis at many university hospitals is on pure
(C) The government should pay for all recently completed research rather than on the treatment and care of patients.
factories to be brought up to new environmental standards (E) The patients who seek help at university hospitals are
(D) Industries should conform to strict environmental usually more seriously ill than those at private or community
standards without delay hospitals.
(E) Environmental standards for any factory should not
be altered once construction is underway 8. The Occupational Safety and Health Administration
(OSHA) was established to protect workers from accidents
5. Public education suffers from what can be diagnosed as and unsafe conditions on the job. There has actually been an
the sickness of an overgoverned society. This sickness denies increase in the number of job-related accidents under OSHA.
many parents control over the kind of education their children This demonstrates the agency’s ineffectiveness.
receive. The power once held by parents has gravitated to Which of the following, if true concerning the period during
professional educators. The sickness has been aggravated by which the increase occurred, most seriously weakens the
increasing centralization and bureaucratization of schools. argument above?
Which of the following, if true, would weaken the claim (A) A number of job categories, excluded from the
that there is continuing erosion of parents’ control over their jurisdiction of OSHA in the legislation originally establishing
children’s education? the agency, have continued to be outside OSHA’s jurisdiction.
(A) As a result of community pressure, growing numbers (B) OSHA has been assigned a greater number of kinds of
of school administrators follow recommendations made by workplace activities to monitor.
parents. (C) There has been an increase in the total number of
(B) The number of professional educators has risen sharply people at work, and the ratio of work-related deaths and
over the last decade even though the number of students has injuries to the size of the work force has fallen in OSHA-
declined. supervised occupation.
(C) Parents’ organizations that lobby for changes in school (D) Regulations issued by OSHA have met with political
curriculums are generally ineffectual. criticism from elected officials and the mass media.
(D) More members of school boards are appointed by school (E) The increase in job-related accidents has occurred
administrators than are elected by the public. mainly in a single job category, whereas the number of job-
(E) The use of state-wide curriculum programs increased related accidents has remained approximately constant in
in the United States during the past two decades. other categories.

59
American Education Aids - LSAT - Logical Reasoning
9. Treatment for hypertension forestalls certain medical 10. A study of the medical records of people below the age
expenses by preventing strokes and heart disease. Yet any of forty found that those whose spouses have died have a
money so saved amounts to only one-fourth of the significantly lower chance of surviving to forty than do those
expenditures required to treat the hypertensive population. with living spouses. This suggests that the emotional stress
Therefore, there is no economic justification for preventive of being widowed severely affects widows’ and widowers’
treatment for hypertension. health.
Which of the following, if true, is most damaging to the Which of the following, if true, casts most doubt on this
conclusion above? study’s support for the conclusion suggested above?
(A) The many fatal strokes and heart attacks resulting (A) Even before the study was made, it was generally
from untreated hypertension cause insignificant medical acknowledged that the emotional trauma of being widowed
expenditures but large economic losses of other sorts. is associated with deterioration in health
(B) The cost, per patient, of preventive treatment for (B) The study did not exclude deaths caused by accidents
hypertension would remain constant even if such treatment in which both members of a married couple received fatal
were instituted on a large scale. injuries
(C) In matters of health care, economic considerations (C) A similar, though less severe, effect on health is
should ideally not be dominant. observed in the immediate aftermath of divorce or separation
(D) Effective prevention presupposes early diagnosis, and (D) Many of the widows and widowers whose records were
programs to ensure early diagnosis are costly. studied showed little susceptibility to stress before their
(E) The net savings in medical resources achieved by some spouses’ deaths
preventive health measures are smaller than the net losses (E) Medical examinations of newly widowed people
attributable to certain other measures of this kind. generally show them to be no less healthy than the population
at large.

SET 3
1. The recent decline in the value of the dollar was triggered (E) At altitudes above 5,500 feet, middle-distance runners
by a prediction of slower economic growth in the coming year. often better their sea-level running times by several seconds.
But that prediction would not have adversely affected the
dollar had it not been for the government’s huge budget deficit, 3. During the War of 1812, the United States Congress
which must therefore be decreased to prevent future currency licensed privateers (armed pirates) who were empowered to
declines. plunder enemy ships. Those privateers financed their
Which of the following, if true, would most seriously weaken ventures through the sale of the seized cargo. A Florida man
the conclusion about how to prevent future currency declines? has petitioned Congress to license modern privateers to mount
(A) The government has made little attempt to reduce the a private “war-for-profit” against seagoing smugglers of illegal
budget deficit. drugs.
(B) The budget deficit has not caused a slowdown in Which of the following, if true, is a drawback to the Florida
economic growth. man’s proposal?
(C) The value of the dollar declined several times in the (A) Modern ships are much faster than those of the
year prior to the recent prediction of slower economic growth. nineteenth century.
(D) Before there was a large budget deficit, predictions of (B) Although the United States Constitution still
slower economic growth frequently caused declines in the authorizes the licensing of privateers, no licenses have been
dollar’s value. issued for over 150 years.
(E) When there is a large budget deficit, other events in (C) Modern privateers would be unable to finance their
addition to predictions of slower economic growth sometimes operations by selling the seized cargo without being in
trigger declines in currency value. violation of the law.
(D) The 1812 privateers were plundering ships that
2. If athletes want better performances, they should train belonged to citizens of another country.
at high altitudes. At higher altitudes, the body has more red (E) Most of the ships used by drug smugglers are modified
blood cells per unit volume of blood than at sea level. The red fishing boats.
blood cells transport oxygen, which will improve performance
if available in greater amounts. The blood of an athlete who 4. The difficulty with the proposed high-speed train line is
trains at high altitudes will transport more oxygen per unit that a used plane can be bought for one-third the price of the
volume of blood, improving the athlete’s performance. train line, and the plane, which is just as fast, can fly
Which of the following, if true, would be most damaging to anywhere. The train would be a fixed linear system, and we
the argument above, provided that the athlete’s heart rate is live in a world that is spreading out in all directions and in
the same at high and low altitudes? which consumers choose the free-wheel system (cars, buses,
(A) Scientists have found that an athlete’s heart requires aircraft) which do not have fixed routes. Thus a sufficient
a period of time to adjust to working at high altitudes. market for the train will not exist.
(B) Scientists have found that the body’s total volume of Which of the following, if true, most severely weakens the
blood declines by as much as 25 percent at high altitudes. argument presented above?
(C) Middle-distance runners who train at high altitudes (A) Cars, buses, and planes require the efforts of drivers
sometimes lose races to middle-distance runners who train and pilots to guide them, whereas the train will be guided
at sea level. mechanically.
(D) The performances of athletes in competitions at all (B) Cars and buses are not nearly as fast as the high-speed
altitudes have improved markedly during the past 20 years. train will be.
60
American Education Aids - LSAT - Logical Reasoning
(C) Planes are not a free-wheel system because they can (B) International relief efforts for victims of natural
fly only between airports, which are less convenient for disasters have been better organized in the past decade than
consumers than the high-speed train’s stations would be. in previous decades.
(D) The high-speed train line cannot use currently (C) There are records for major earthquakes, volcanic
underutilized train stations in large cities. eruptions, droughts, landslides, and floods occurring in the
(E) For long trips, most people prefer to fly rather than to distant past, as well as in the recent past.
take ground-level transportation. (D) Population pressures and poverty have forced
increasing numbers of people to live in areas prone to natural
5. Opponents of laws that require automobile drivers and disasters.
passengers to wear seat belts argue that, in a free society, (E) There have been no changes in the past decade in
people have the right to take risks as long as the people do people’s land-use practices that could have affected the
not harm others as a result of taking the risks. As a result, climate.
they conclude that it would be each person’s decision whether
or not to wear a seat belt. 8. Red blood cells in which the malarial-fever parasite
Which of the following, if true, most seriously weakens the resides are eliminated from a person’s body after 120 days.
conclusion drawn above? Because the parasite cannot travel to a new generation of
(A) Many new cars are built with seat belts that red blood cells, any fever that develops in a person more than
automatically fasten when someone sits in the front seat. 120 days after that person has moved to a malaria-free region
(B) Automobile insurance rates for all automobile owners is not due to the malarial parasite.
are higher because of the need to pay for the increased injuries Which of the following, if true, most seriously weakens the
or deaths of people not wearing seat belts. conclusion above?
(C) Passengers in airplanes are required to wear seat belts (A) The fever caused by the malarial parasite may resemble
during takeoffs and landings. the fever caused by flu viruses.
(D) The rate of automobile fatalities in states that do not (B) The anopheles mosquito, which is the principal insect
have mandatory seat belt laws is greater than the rate of carrier of the malarial parasite, has been eradicated in many
fatalities in states that do have such laws. parts of the world.
(E) In automobile accidents, a greater number of (C) Many malarial symptoms other than the fever, which
passengers who do not wear seat belts are injured than are can be suppressed with anti-malarial medication, can
passengers who do wear seat belts. reappear within 120 days after the medication is discontinued.
(D) In some cases, the parasite that causes malarial fever
6. A proposed ordinance requires the installation in new travels to cells of the spleen, which are less frequently
homes of sprinklers automatically triggered by the presence eliminated from a person’s body than are red blood cells.
of a fire. However, a home builder argued that because more (E) In any region infested with malaria-carrying
than ninety percent of residential fires are extinguished by a mosquitoes, there are individuals who appear to be immune
household member, residential sprinklers would only to malaria.
marginally decrease property damage caused by residential
fires. 9. Traditionally, the Supreme Court has published only
Which of the following, if true, would most seriously weaken one majority and one minority opinion with each ruling. Some
the home builder’s argument? judicial experts are disturbed by the Court’s recent tendency
(A) Most individuals have no formal training in how to to publish multiple opinions with its rulings on controversial
extinguish fires. cases. This new “splintering” of opinions, they argue, creates
(B) Since new homes are only a tiny percentage of available serious problems for lower courts and other branches of
housing in the city, the new ordinance would be extremely government that need guidance.
narrow in scope. Statements that if true would weaken the claim of the
(C) The installation of smoke detectors in new residences judicial experts include which of the following?
costs significantly less than the installation of sprinklers. I. The Court’s ruling on a case constitutes a sufficient guide,
(D) In the city where the ordinance was proposed, the regardless of the opinions that accompany it
average time required by the fire department to respond to a II. The Court is no more divided than the nation as a whole
fire was less than the national average. on controversial issues
(E) The largest proportion of property damage that results III. When the justices offer multiple opinions, the
from residential fires is caused by fires that start when no differences among them are usually complex and highly
household member is present technical
(A) I only
7. Earthquakes, volcanic eruptions, and unusual weather (B) II only
have caused many more natural disasters adversely affecting (C) III only
people in the past decade than in previous decades. We can (D) I and II only
conclude that the planet Earth as a natural environment has (E) I and III only
become more inhospitable and dangerous, and we should
employ the weather and earth sciences to look for causes for 10. Noting that the number of crimes committed in a
this trend. certain city had decreased in 1982 by 5.2 percent in
The conclusion drawn above is most seriously weakened if comparison with 1981, the police chief of the city said, “We
which of the following is true? see here the result of the innovative police program put into
(A) The weather and earth sciences have provided better effect in the city at the beginning of 1982.”
early warning systems for natural disasters in the past decade Which of the following, if true, most seriously weakens the
than in previous decades. conclusion drawn by the police chief?
61
American Education Aids - LSAT - Logical Reasoning
(A) Several cities that have recently increased spending figure for 1981 and were nearly equal in number to those in
for police programs experienced no decrease in crime in 1982, the city in 1982.
as compared with 1981. (D) The number of crimes committed in the city in 1982
(B) The number of crimes committed in the city is was 10 percent higher than the number committed in 1972.
estimated, by the same method each year, from the number (E) The size of the age-group most likely to commit crimes
of crimes reported. decreased considerably in the city in 1982, as against 1981,
(C) The number of crimes committed in the suburban areas because of a declining birth rate.
surrounding the city rose by about 5 percent in 1982 over the

SET 4
1. The popular notion that teachers are generally apathetic being employed in a particular occupation.
about microcomputer technology is false, or at least dated: a (C) Some jobs might involve health risks other than the
recently published survey indicates that 86 percent of 5,000 risk of heart attack.
teachers who responded to survey questionnaires expressed (D) Employees who have a 90 percent chance of suffering
a high level of interest in microcomputers. a heart attack may be unaware that their risk is so great.
Which of the following, if true, would be most damaging to (E) The number of people applying for jobs at a company
the argument above? might decline if the company, by screening applicants for risk
(A) No attempt was made in the survey to determine of heart attack, seemed to suggest that the job entailed high
whether the teachers who received questionnaires had any risk of heart attack.
previous experience with microcomputers.
(B) Teachers who are interested in microcomputer 4. Most consumers do not get much use out of the sports
technology were more likely than others to complete and equipment they purchase. For example, seventeen percent
return their questionnaires. of the adults in the United States own jogging shoes, but only
(C) Questionnaires were sent to teachers without regard forty-five percent of the owners jog more than once a year,
to their areas of subject-matter expertise or teaching and only seventeen percent jog more than once a week.
experience. Which of the following, if true, casts most doubt on the claim
(D) There have been several important developments in that most consumers get little use out of the sports equipment
the classroom applications of microcomputer technology since they purchase?
the survey results were tabulated. (A) Joggers are most susceptible to sports injuries during
(E) The survey was conducted as part of a marketing study the first six months in which they jog.
by a company that manufactures and sells microcomputers. (B) Joggers often exaggerate the frequency with which they
jog in surveys designed to elicit such information.
2. Private ownership of services traditionally considered (C) Many consumers purchase jogging shoes for use in
to be the responsibility of the government will typically activities other than jogging.
improve those services. The turnpike system in the United (D) Consumers who take up jogging often purchase an
States of the nineteenth century demonstrates the truth of athletic shoe that can be used in other sports.
this principle. The system, which had previously been (E) Joggers who jog more than once a week are often active
controlled by the government, became a more reliable system participants in other sports as well.
when taken over by private organizations.
Which of the following describes a significant flaw in the 5. Lists of hospitals have been compiled showing which
author’s argument above? hospitals have patient death rates exceeding the national
(A) The author defends the conclusion by appealing to a average. The data have been adjusted to allow for differences
person of authority. in the ages of patients.
(B) The author distorts an opposing view in trying to show Each of the following, if true, provides a good logical ground
its weaknesses. for hospitals to object to interpreting rank on these lists as
(C) The author defends what the author perceives as a one of the indices of the quality of hospital care EXCEPT:
wrong action by pointing out another perceived wrong action. (A) Rank order might indicate insignificant differences,
(D) The author generalizes from a sample not rather than large differences, in numbers of patient deaths
representative enough to establish the conclusion. (B) Hospitals that keep patients longer are likely to have
(E) The author attributes two very different meanings to higher death rates than those that discharge patients earlier
the same word. but do not record deaths of patients at home after discharge
(C) Patients who are very old on admission to a hospital
3. Recently a court ruled that current law allows companies are less likely than younger patients to survive the same types
to reject a job applicant if working in the job would entail a of illnesses or surgical procedures
90 percent chance that the applicant would suffer a heart (D) Some hospitals serve a larger proportion of low-income
attack. The presiding judge justified the ruling, saying that patients, who tend to be more seriously ill when admitted to
it protected both employees and employers. a hospital
The use of the court ruling as part of the law could not be (E) For-profit hospitals sometimes do not provide intensive-
effective in regulating employment practices if which of the care units and other expensive services for very sick patients
following were true? but refer or transfer such patients to other hospitals
(A) The best interests of employers often conflict with the
interests of employees. 6. Unlike the wholesale price of raw wool, the wholesale
(B) No legally accepted methods exist for calculating the price of raw cotton has fallen considerably in the last year.
risk of a job applicant’s having a heart attack as a result of Thus, although the retail price of cotton clothing at retail
62
American Education Aids - LSAT - Logical Reasoning
clothing stores has not yet fallen, it will inevitably fall. season
Which of the following, if true, most seriously weakens the (B) athletes can be on teams in more than one season
argument above? (C) some of the team sports require a larger number of
(A) The cost of processing raw cotton for cloth has increased athletes on the team than no others
during the last year (D) more athletes participate in team sports during one
(B) The wholesale price of raw wool is typically higher than season than during another
that of the same volume of raw cotton (E) an athlete might not participate in every one of the
(C) The operating costs of the average retail clothing store practice sessions and athletic contests in his or her sport
have remained constant during the last year
(D) Changes in retail prices always lag behind changes in 9. It is important to teach students to use computers
wholesale prices effectively. Therefore, students should be taught computer
(E) The cost of harvesting raw cotton has increased in the programming in school.
last year Which of the following, if true, most weakens the argument
above?
7. Researchers have recently argued that ancient Greek (A) Only people who use computers effectively are skilled
coins, seals, and gems with elaborate miniature engravings at computer programming.
were produced by artisans whose nearsightedness enabled (B) Only people skilled at computer programming use
them to do such intricate work. They also suggest that since computers effectively.
nearsightedness is hereditary, guilds and families specializing (C) Some people who use computers effectively cannot write
in this work may have favored the recurrence of computer programs.
nearsightedness. (D) Some schools teach computer programming more
Which of the following, if true, would weaken the effectively than others.
researchers’ argument? (E) Most people who are able to program the computers
(A) Roman artisans were influenced by their Greek use computers effectively.
predecessors and often translated designs of Greek miniature
engravings into Roman designs 10. A year after the start of an experiment to decrease crime
(B) Nearsighted individuals are less able than those with in two high-crime subway stations by the installation of
ordinary vision to identify objects that are at a great distance closed-circuit televisions, the experiment is being
from them discontinued. City officials say the program has led to an
(C) Magnifying glass was not used as a visual aid until increase in crime, citing the fact that following the
the thirteenth century, when Roger Bacon invented eyeglasses installation, both stations showed increases in the number
(D) Artisans today are unable to reproduce the miniature of crimes reported.
engravings of ancient Greek artisans without the aid of high- Which of the following, if true, most seriously weakens the
powered magnifying lenses, first produced in the thirteenth claim of the city officials that ‘the program has led to an
century increase in crime”?
(E) The eyesight of nearsighted individuals and those with (A) The two subway stations had been chosen on the basis
ordinary vision is not significantly different if the objects of high frequency of crime and were not typical subway
viewed are less than a few feet away stations.
(B) The rate of increase in crimes reported for the two
8. Athletic director: “Members of our sports teams included, subway stations was higher than that of other high-crime
for the fall season, 80 football players and 40 cross-country subway stations not equipped with closed-circuit television.
runners; for the winter season, 20 wrestlers and 40 swimmers; (C) The percentage of all crimes committed at the two
for the spring season, 50 track-team members and 20 lacrosse subway stations that were reported rose as a result of
players. Each team athlete participates in his or her sport increased instances of observations of crime on the closed-
five days a week for the whole three-month season, and no circuit televisions.
athlete is on two teams during any one season. Therefore, (D) The year in which the experiment was conducted was
adding these figures, we find that our team sports program a year in which the total number of crime reported in the city
serves 250 different individual athletes.” fell.
In drawing the conclusion above, the athletic director fails (E) Closed-circuit televisions installed in shops and stores
to consider the relevant possibility that throughout the city have proved to be useful in the prevention
(A) athletes can be on more than one team in a single of shoplifting and burglaries.

SET 5
1. Six months or so after getting a video recorder, many saturated until there was one in 80 percent of homes
early buyers apparently lost interest in obtaining videos to (B) Among the items handled by video distributors are
watch on it. The trade of businesses selling and renting videos many films specifically produced as video features
is still buoyant, because the number of homes with video (C) Few of early buyers of video recorders raised any
recorders is still growing. But, clearly, once the market for complaints about performance aspects of the new product
video recorders is saturated, businesses distributing videos (D) The early buyers of a novel product are always people
face hard times. who are quick to acquire novelties, but also often as quick to
Which of the following, if true, would most seriously weaken tire of them
the conclusion above? (E) In a shrinking market, competition always intensifies
(A) The market for video recorders would not be considered and marginal businesses fail
63
American Education Aids - LSAT - Logical Reasoning
2. Identical twins tend to have similar personalities; if (D) Domestic “mini-mills” consistently produce better
environment outweighs heredity in personality development, grades of steel than do the big American mills.
twins raised together should presumably have more similar (E) Domestic “mini-mills” produce low-volume, specialized
personalities than those raised apart. A recent study of types of steels that are not produced by the big American
identical twins in both situations measured 11 key traits steel mills.
through a questionnaire, and concluded that 7 of the 11 are
primarily products of heredity. 5. Tropical forests produce a large percentage of the Earth’s
Which of the following, if established, would cast the most supply of oxygen. Continued destruction of these forests is
doubt on the study’s results? likely to reduce the Earth’s oxygen supply to levels that will
(A) Fewer than half of the pairs of twins studied were raised threaten the existence of all oxygen-dependent life on Earth.
separately. Which of the following, if true, most seriously weakens the
(B) The ages of all of the twins studied fell within a 10- prediction above?
year range. (A) All of the oxygen produced by the Earth’s tropical forests
(C) Some of the traits that the study attributed to heredity is consumed by bacteria and animals living in those forests
developed in the separately raised twins because those pairs (B) Some of the oxygen produced by the Earth’s tropical
all grew up in similar families. forests escapes through the upper atmosphere without being
(D) Although over half the traits measured were consumed
determined to be linked to heredity, the nature of those traits (C) The oxygen produced by tropical forests is consumed
varied widely. by animals all over the Earth
(E) The 11 traits that were measured constitute a (D) The Earth’s tropical forests are being destroyed at a
representative sample of a larger, generally accepted pool of rate of hundreds of acres every day
key personality traits. (E) Tropical forests contain the most diverse selection of
oxygen-dependent animals on the earth
3. The number of people diagnosed as having a certain
intestinal disease has dropped significantly in a rural county 6. The link between cigarette advertising and increased
this year, as compared to last year. Health officials attribute cigarette consumption is tenuous at best. In Italy, for instance,
this decrease entirely to improved sanitary conditions at where cigarette advertising is prohibited, cigarette
water-treatment plants, which made for cleaner water this consumption among teen-agers has continued to rise.
year and thus reduced the incidence of the disease. Which of the following, if true, would most seriously weaken
Which of the following, if true, would most seriously weaken the argument above?
the health officials’ explanation for the lower incidence of the (A) Most Italian teen-agers are aware of the debilitating
disease? effect that cigarette smoking has on a person’s health
(A) Many new water-treatment plants have been built in (B) Most Italian teen-agers watch television channels that
the last five years in the rural county. broadcast from countries where cigarette advertising is legal
(B) Bottled spring water has not been consumed in (C) Most Italian teen-agers who smoke cigarettes have at
significantly different quantities by people diagnosed as least one parent who also smokes cigarettes
having the intestinal disease, as compared to people who did (D) Italian teen-agers who smoke cigarettes regularly tend
not contract the disease. to miss more days of school than those who do not smoke
(C) Because of a new diagnostic technique, many people (E) Italian teen-agers tend to respond more strongly to
who until this year would have been diagnosed as having the advertising than do teen-agers in other European countries
intestinal disease are now correctly diagnosed as suffering
from intestinal ulcers. 7. Corporate management’s increasing neglect of long-term
(D) Because of medical advances this year, far fewer people growth is a result of pressures brought by investors holding
who contract the intestinal disease will develop severe cases large blocks of stock in a corporation who want to see
of the disease. concentration on long-term growth sacrificed in order to
(E) The water in the rural county was brought up to the increase stock value over the short term.
sanitary standards of the water in neighboring counties ten Which of the following, if true and known to all investors,
years ago. would most seriously weaken the explanation above?
(A) Investors who hold large blocks of stock tend to sell
4. The imposition of quotas limiting imported steel will whenever the stock’s value drops
not help the big American steel mills. In fact, the quotas will (B) Investors who hold large blocks of stock in a corporation
help “mini-mills” flourish in the United States. Those small often have direct access to that corporation’s top management
domestic mills will take more business from the big American (C) A corporation’s stock decreases in value when the
steel mills than would have been taken by the foreign steel corporation’s short-term prospects are poor
mills in the absence of quotas. (D) A corporation’s stock increases in value when investors
Which of the following, if true, would cast the most serious buy large amounts of it
doubt on the claim made in the last sentence above? (E) A corporation’s stock increases in value over the short
(A) Quality rather than price is a major factor in term only when the corporation’s long-term growth prospects
determining the type of steel to be used for a particular are good
application.
(B) Foreign steel mills have long produced grades of steel 8. Injections of bovine growth hormone, which is normally
comparable in quality to the steel produced by the big produced in small amounts by a cow’s pituitary gland, have
American mills. been shown to increase milk production in cows from ten to
(C) American quotas on imported goods have often induced forty percent with only a modest increase of feed. The United
other countries to impose similar quotas on American goods. States Food and Drug Administration (FDA) plans to consider
64
American Education Aids - LSAT - Logical Reasoning
licensing commercial producers of the hormone. (E) The average newsstand price of newspapers sold in
Which of the following, if true, would provide the strongest Town S is lower than the average price of newspapers sold in
criticism of the FDA’s plan? Town T.
(A) Increased milk production increases the susceptibility
of cows to a variety of diseases 10. In the United States, the financing of industrial
(B) The FDA intends to employ stringent requirements research by private industrial firms remained steady as a
when issuing permits for producing bovine growth hormone percentage of sales during the period between 1968 and 1978
(C) The chemical companies that will be producing the (after correcting for inflation). But slowdowns in the growth
hormone will obtain large short-term profits of industrial productivity also occurred during that period, a
(D) Bovine growth hormone will not hurt the chances for fact that refutes the notion that the growth of industrial
survival of the family dairy farm, which typically has about productivity is directly proportional to the amount invested
fifty cows in industrial research.
(E) Many small dairy farms are going out of business Which of the following, if true for the United States, most
because they cannot charge prices high enough to cover their weakens the argument above?
costs (A) Federal funds, which constituted a significant portion
of the support for industrial research from 1968 to 1978, fell
9. A greater number of newspapers are sold in Town S than annually and substantially during that period
in Town T. Therefore, the citizens of Town S are better (B) The inflation that occurred between 1968 and 1978
informed about major world events than are the citizens of was more severe than leading economists had expected
Town T. (C) Industrial executives generally favor investing an
Each of the following, if true, weakens the conclusion above appreciably larger portion of corporate funds in short-term
EXCEPT: product development than in basic research
(A) Town S has a larger population than Town T. (D) The scientists and engineers who worked in industry
(B) Most citizens of Town T work in Town S and buy their from 1968 to 1978 were, as a group, more experienced in their
newspapers there. jobs than were those who worked in industry during the
(C) The average citizen of Town S spends less time reading previous ten-year period
newspapers than does the average citizen of Town T. (E) Corporate financing of industrial research increased
(D) A weekly newspaper restricted to the coverage of local in several of the years immediately following 1978 (after
events is published in Town S. correcting for inflation)

SET 6
1. In Diersville the new Environmental Action party won the state the child will attend.
two seats on the seven-member town council in 1988. It lost (B) The amount of money accumulated by putting the
both of those seats in the 1992 election, even though the prepayment funds in an interest-bearing account today will
party’s pro-environment platform had essentially remained be greater than the total cost of tuition for any of the public
unchanged. This decline in the party’s fortunes clearly colleges when the child enrolls.
demonstrates that, in Diersville, environmental concerns (C) The annual cost of tuition at the state’s public colleges
faded in significance between 1988 and 1992. is expected to increase at a faster rate than the annual
Which of the following, if true, most seriously weakens the increase in the cost of living.
argument? (D) Some of the state’s public colleges are contemplating
(A) Between 1998 and 1992, the number of eligible voters large increases in tuition next year.
in Diersville rose, but not the percentage who actually voted. (E) The prepayment plan would not cover the cost of room
(B) Between 1988 and 1992, Diersville’s leading political and board at any of the state’s public colleges.
party revised its platform, adopting the policies of the
Environmental Action party. 3. To produce a pound of protein, herbivorous sea creatures
(C) The parties that ran candidates in the 1992 election in such as abalones require relatively little plant food, whereas
Diersville were the same as those that had done so in the carnivores such as tuna feed on herbivores and thus require
1988 election. the conversion of thousands of pounds of plant food. Hence it
(D) In 1992 the Environmental Action party won fewer would be better for ecological balance if people ate abalone
votes in Diersville than it had won in 1988. instead of tuna.
(E) Between 1988 and 1992, some measures intended to Which of the following, if true, is the best criticism of the
benefit the environment had been adopted by the town argument above?
council, but with inconclusive results. (A) Abalone costs much more per pound than tuna does
(B) Tuna is much easier to harvest, than abalone is
2. A program instituted in a particular state allows parents (C) Abalones mature so slowly that meeting market
to prepay their children’s future college tuition at current demands would exhaust the stock
rates. The program then pays the tuition annually for the (D) Human beings would require the same amount of
child at any of the state’s public colleges in which the child protein from either abalone or tuna
enrolls. Parents should participate in the program as a means (E) The average specimen of abalone is much smaller than
of decreasing the cost of their children’s college education. the average specimen of tuna
Which of the following, if true, is the most appropriate
reason for parents not to participate in the program? 4. Last year, health insurance paid $111 billion of the $162
(A) The parents are unsure about which public college in billion spent on health care. It is thus indispensable for a

65
American Education Aids - LSAT - Logical Reasoning
health-care provider to be eligible for reimbursement by a deregulation of the airlines.
patient’s insurer. As a result, it is the insurers who decide The argument above would be most seriously weakened if
who is a healer in modern society. it were true that
Which of the following, if true, weakens the argument above? (A) a poll of people in the Unites States expressed
(A) The kinds of health services not covered by health exceptionally strong support for airline deregulation
insurance are relatively inexpensive (B) fewer passengers now travel on commercial airlines
(B) Health insurance companies must reimburse any than traveled on them in 1978, with the consequence that
provider licensed by state agencies that regulate health care fewer employees are needed to operate the airlines than were
(C) Patients sometimes choose to receive a particular needed in 1978
treatment from a familiar practitioner even if that treatment (C) airlines now fly a more restricted regular schedule of
is not eligible for reimbursement routes than they did in 1978, with the consequence that the
(D) Many of the health-care services demanded by insured industry is more highly concentrated and competitive than
patients are unnecessary it was before 1978
(E) A significant portion of uninsured patients forgo needed (D) several major airlines now enjoy significantly higher
treatment because they are unable to afford the high cost profits and levels of employment than they did in the years
preceding the Deregulation Act
5. In a recent study, sedentary middle-aged men who drink (E) smaller carriers of passengers have thrived as a result
more than two cups of coffee a day were found more likely of deregulation and now provide more new jobs than the major
than other sedentary middle-aged men to have a high blood airlines have eliminated since 1978
level of cholesterol, which is a factor increasing the risk of
heart disease. Cholesterol can reach the blood from food and 8. In mammals it is the secondary palate that permits
drink but is not contained in coffee. breathing while eating. Clearly, breathing while eating is
Which of the following, if true, most seriously weakens a necessary to maintain the high rate of metabolism of
conclusion from the study that for sedentary middle-aged men mammals.
coffee increases the risk of heart attack? The author’s assertions would be most weakened by the
(A) A sedentary style of life increases levels of cholesterol discovery of a mammalian species that had a
in the blood. (A) high rate of metabolism and the ability to breathe while
(B) Coffee contains caffeine, which acts as a stimulant that eating
increases heart rate. (B) low rate of metabolism and the ability to breathe while
(C) The men studied drank their coffee without milk or eating
cream, which contain cholesterol. (C) low rate of metabolism and no ability to breathe while
(D) In both groups, the men were likely to be over-weight, eating
and excess weight is a factor that increases the risk of heart (D) high rate of metabolism and no secondary palate
disease. (E) low rate of metabolism and a secondary palate
(E) The men who drank more than two cups of coffee a day
also ate more foods high in cholesterol. 9. The average age of chief executive officers (CEO’s) in a
large sample of companies is 57. The average age of CEO’s in
6. Biological functions of many plants and animals vary in those same companies 20 years ago was approximately eight
cycles that are repeated every 24 hours. It is tempting to years younger. On the basis of those data, it can be concluded
suppose that alteration in the intensity of incident light is that CEO’s in general tend to be older now.
the stimulus that controls these daily biological rhythms. But Which of the following casts the most doubt on the
there is much evidence to contradict this hypothesis. conclusion drawn above?
Which of the following, if known, is evidence that (A) The dates when the CEO’s assumed their current
contradicts the hypothesis stated in lines 2-5 above? positions have not been specified.
(A) Human body temperature varies throughout the day, (B) No information is given concerning the average number
with the maximum occurring in the late afternoon and the of years that CEO’s remain in office.
minimum in the morning. (C) The information is based only on companies that have
(B) While some animals, such as the robin, are more active been operating for at least 20 years.
during the day, others, such as mice, show greater activity at (D) Only approximate information is given concerning the
night. average age of the CEO’s 20 years ago.
(C) When people move from one time zone to another, their (E) Information concerning the exact number of companies
daily biological rhythms adjust in a matter of days to the in the sample has not been given.
periods of sunlight and darkness in the new zone.
(D) Certain single-cell plants display daily biological 10. A government agency that reimburses its clients for
rhythms even when the part of the cell containing the nucleus bills they have paid for medical care has had this year’s budget
is removed. cut. To save money without cutting reimbursements or
(E) Even when exposed to constant light intensity around otherwise harming clients financially, it plans to delay
the clock, some algae display rates of photosynthesis that reimbursements to clients for forty days, thereby earning $180
are much greater during daylight hours than at night. million per year in interest on the reimbursement money.
Which of the following, if true, is the best criticism of the
7. Since the Airline Deregulation Act of 1978, major airline agency’s plan?
companies in the United States have cut their employee ranks (A) Hospitals and physicians typically hold patients
by more than 3,000 persons. Thus, although deregulated responsible for the ultimate payment of their bills.
competition has afforded consumers dramatically lower fares, (B) The agency cannot save money by cutting staff because
the economy of the United States has been harmed by the it is already understaffed.
66
American Education Aids - LSAT - Logical Reasoning
(C) Some clients borrow money to pay their medical bills; for reimbursement.
they will pay forty extra days of interest on these loans. (E) The agency’s budget was cut by more than $180 million
(D) Some clients pay their medical bills immediately, but last year.
they often take more than forty days to file with the agency

SET 7
1. W: The rapid rise in the rate of unemployment is a (B) Researchers do not yet know what makes one person’s
serious economic problem; it will undoubtedly cost the messenger molecules more easily activated than another’s.
majority party votes in the coming election (C) Such a medication would not become available for
Z: I disagree. Statistics for the past 40 years show that several years, because of long lead times in both development
there is no significant relationship between the prevailing and manufacture.
level of unemployment and political gains or losses by the (D) Such a medication would be unable to distinguish
party that happens to be in the majority at election time. between messages triggered by pollen and house-hold dust
Which of the following most seriously weakens the force of and messages triggered by noxious air.
Z’s counter argument? (E) Such a medication would only be a preventive and
(A) Z fails to cite any actual figures would be unable to alleviate an asthma attack once it had
(B) Z’s decision to consider a time span of 40 years seems started.
arbitrary
(C) Z relies on conventional criteria for the notion of a 4. Ms. Friedman noticed that average productivity fell for
significant relationship day-shift workers at about 3:00 p.m. Since that is the time
(D) Z’s statistics focus on level of unemployment, rather many children leave school, she hypothesized that the cause
than direction of change of the problem was employees’ minds turning to their children
(E) Z assumes that there is no connection between economic instead of being focused on the job.
and political trends Which of the following, if true, casts the most doubt on the
truth of Ms. Friedman’s hypothesis?
2. Corporate Officer: Last year was an unusually poor one (A) About one-third of all children in the United States
for our chemical division, which has traditionally contributed between the ages of 6 and 13 are home alone for some period
about 60 percent of the corporation’s profits. It is therefore after school
encouraging that there is the following evidence that the (B) After-school and day-care facilities are inadequate to
pharmaceutical division is growing stronger: it contributed serve all of the children in need of such facilities
45 percent of the corporation’s profits, up from 20 percent (C) Employers have found that the provision of after-school
the previous year. and day-care facilities is an excellent incentive in recruiting
On the basis of the facts stated, which of the following is and retaining workers
the best critique of the evidence presented above? (D) Some differences in average daily productivity have
(A) The increase in the pharmaceutical division’s been found between employees with one child and employees
contribution to corporation profits could have resulted largely with several children
from the introduction of a single, important new product. (E) No average differences in productivity after 3:00 p.m.
(B) In multi-divisional corporations that have have been found between employees with children and
pharmaceutical divisions, over half of the corporation’s profits employees without children.
usually comes from pharmaceuticals.
(C) The percentage of the corporation’s profits attributable 5. People rarely recognize the fact that being underweight
to the pharmaceutical division could have increased even if is as dangerous to health as being overweight. But the death
that division’s performance had not improved. rate for individuals who are ten percent underweight, like
(D) The information cited does not make it possible to that for those thirty percent overweight, is almost twice what
determine whether the 20 percent share of profits cited was it is for individuals of normal weight.
itself an improvement over the year before. The dangers of being underweight supported by the
(E) The information cited does not make it possible to statistics above appear less serious if which of the following is
compare the performance of the chemical and pharmaceutical true?
divisions in terms of the percent of total profits attributable (A) Many of the underweight persons studied had lost
to each. weight as a result of illness or disease
(B) Conceptions of underweight and overweight varied
3. Certain messenger molecules fight damage to the lungs widely among those studied
from noxious air by telling the muscle cells encircling the (C) The sample of persons studied represented a variety
lungs’ airways to contract. This partially seals off the lungs. of age groups
An asthma attack occurs when the messenger molecules are (D) The demands of jobs and life-styles were comparable
activated unnecessarily, in response to harmless things like for persons in both groups studied
pollen or household dust. (E) The definition of normal weight in the study was
Which of the following, if true, points to the most serious narrower than definitions generally accepted
flaw of a plan to develop a medication that would prevent
asthma attacks by blocking receipt of any messages sent by 6. Researchers conducting a long-term study of drug
the messenger molecules referred to above? abusers, who initially demonstrated no serious psychiatric
(A) Researchers do not yet know how the body produces illnesses, observed that after six years a significant number
the messenger molecules that trigger asthma attacks. of amphetamine abusers had become schizophrenic, whereas
67
American Education Aids - LSAT - Logical Reasoning
many of those using depressants suffered from severe in which the last digit is one greater than the second-to-last
depression. The researchers concluded that particular drugs, digit.
if abused, cause specific psychiatric illnesses. (D) The example cannot be shown to be correct unless the
Which of the following, if true, weakens the researchers’ truth of the generalization is already presupposed.
conclusion? (E) The generalization is valid only for those birth years
(A) Preexisting undetected personality disorders lead drug in which the last digit is greater than five.
abusers to choose particular drugs
(B) Regular use of drugs that are typically abused hastens 9. The excessive number of safety regulations that the
the development of psychiatric disorders federal government has placed on industry poses more serious
(C) Constant tampering with the brain’s chemistry causes hardships for big businesses than for small ones. Since large
permanent as well as temporary changes companies do everything on a more massive scale, they must
(D) The use of drugs other than amphetamines and alter more complex operations and spend much more money
depressants may mask a number of underlying psychiatric to meet governmental requirements.
problems Which of the following, if true, would most weaken the
(E) Persons used to taking a particular kind of drug argument above?
involuntarily change their behavior to mimic the effect of the (A) Small companies are less likely than large companies
drug to have the capital reserves for improvements.
(B) The operations of small companies frequently rely on
7. Only a member of the Regionalist Party would oppose the same technologies as the operations of large companies.
the bill for a new recycling law that would protect the (C) Safety regulation codes are uniform, established
environment from industrial interests. Ellen cannot be a without reference to size of company.
member of the Regionalist Party because she supports the (D) Large companies typically have more of their profits
bill. invested in other businesses than do small companies.
Which of the following statements points out why the (E) Large companies are in general more likely than small
conclusion above is invalidly drawn? companies to diversify their markets and products.
(A) Regionalist party members have organized to oppose
industrial interests on several other issues. 10. New regulations in Mullentown require manufacturers
(B) Industrial interests need not oppose the protection of there to develop five-year pollution-reduction plans. The
the environment. regulations require that each manufacturer develop a detailed
(C) Past attempts to protect the environment through plan for reducing its released pollutants by at least 50 percent.
recycling laws have failed. Clearly, the regulations will not result in significant pollution
(D) It is possible that some Regionalist Party members reduction, however, since the regulations do not force
may not oppose the bill for a new recycling law. manufacturers to implement their plans.
(E) Ellen has attended programs and distributed literature Which of the following, if true, most weakens the above
prepared by the Regionalist Party. argument?
(A) Mullentown’s manufacturing plants are not the only
8. Roberta was born in 1967 and so, in 1976, she was nine source of pollution there
years old. It is clear from this example that the last two digits (B) Detailed plans would reveal that measures to reduce
of a person’s birth year will be the same as the last two digits released pollutants would also reduce manufacturers’ costs
of the year of that person’s ninth birthday, except that the for materials, waste disposal, and legal services
position of the digits will be reversed. (C) Pollutants that manufacturing processes create but
Which of the following is the best criticism of the assertions that are not released directly into the environment must
made above? nonetheless be collected and prepared for disposal
(A) The generalization is valid only for those birth years (D) Any reductions in pollutants released from
that do not end in two zeroes. Mullentown’s manufacturing plants would not be noticeable
(B) The example does not exhibit the same principle as is for at least five years
expressed in the generalization based on it. (E) Each manufacturer will be required to submit its plan
(C) The generalization is valid only for those birth years to a committee appointed by Mullentown’s officials

SET 8
1. As a practical matter, the copper available for industrial (B) The production of copper from other metals in industrial
use should not be thought of as limited by the quantity of quantities would be prohibitively expensive in energy and
copper deposits, known or unknown. The transmutation of materials.
one chemical element into another is a modern reality, (C) Synthetic materials have been discovered that can
through the methods of nuclear physics. Therefore, the serve as practical substitutes for copper in most of its uses.
quantity of a natural resource such as copper cannot be (D) It will be impractical, in the foreseeable future, to mine
calculated even in principle, because copper can be made from any deposits of metal that may exist on the Moon or on other
other metals. planets.
Which of the following, if true, is the strongest argument (E) Methods for estimating the amount of copper available
against the conclusion above? in currently known deposits have become very sophisticated
(A) Although it is possible that additional deposits of copper and have proved quite accurate.
will be found, geological considerations strongly indicate that
they will not amount to more than a fifty-year supply.

68
American Education Aids - LSAT - Logical Reasoning
2. The ratio of divorces to marriages has increased since 5. According to an independent computer-industry analyst,
1940. Therefore, there must be a greater proportion of the new Regent microcomputer is of high quality, is fast, and
children living with only one natural parent than there was costs less than any currently existing competing model. It is
in 1940. reasonable to conclude, therefore, as the manufacturer’s
Which of the following, if true, most strongly weakens the prospectus does, that the Regent will quickly establish itself
inference drawn above? as a fast-selling, low-priced alternative to currently available
(A) The number of marriages entered into by women microcomputers.
twenty-five to thirty-five years old has decreased since 1940. Which of the following, if true, would LEAST weaken the
(B) When there is a divorce, children are often given the argument above?
option of deciding which parent they will live with. (A) Many retailers already carry one or more low-priced
(C) Since 1940 the average number of children in a family microcomputer models and are disinclined to carry another.
has remained approximately steady and has not been subject (B) Several faster and lower-period models of
to wide fluctuations. microcomputers will soon be introduced by other computer
(D) Before 1940 relatively few children whose parents had manufacturers.
both died were adopted into single-parent families. (C) The Regent Corporation’s microcomputer can be used
(E) The proportion of children who must be raised by one in conjunction with higher-priced microcomputers
parent because the other has died has decreased since 1940 manufactured by other companies.
as a result of medical advances. (D) Most of those individuals and companies that could be
expected to make up the potential market for the Regent
3. Our words are meaningless and cannot be distinguished microcomputer have already filled their microcomputer needs.
from their opposites, as can be proved by an example. People (E) The independent computer-industry analyst whose
think that they know the difference between the meanings of assessment was incorporated in the prospectus has used
“bald” and “having hair”. Suppose an average person twenty- measures of quality that are not universally accepted by the
one years of age has N hairs on his or her head. We say that computer-buying public.
person is not bald but has hair. But surely one hair less would
make no difference, and a person with N–1 hairs on his or 6. It is true that unionized women earn, on average, more
her head would be said to have hair. Suppose we kept on, than a third more than non-unionized women do. But the
with one hair less each time. The result would be the same. unionized women work in industries where wages happen to
But what would be the difference between someone who had be high; their non-unionized counterparts in these industries
one hair and someone who had none? We call them both earn about as much as they do. Therefore unionization does
bald. Nowhere can we make a distinction between “bald” not raise women’s wages.
and “having hair”. Which of the following, if true, most seriously weakens the
Which of the following statements best counters the argument above?
argument above? (A) Besides wage increases, unions bargain for benefits
(A) The word “bald” can be translated into other languages. such as medical insurance and workplace safety.
(B) A word can have more than one meaning. (B) The most highly paid women are in executive positions,
(C) A word such as “cat” can be applied to several animals which are not unionized.
that differ in some respects. (C) Wages in many industries vary from one part of the
(D) Words can lack precision without being meaningless. country to another, regardless of whether workers are
(E) People cannot think clearly without using words. unionized or not.
(D) Non-unionized women in an industry often receive
4. In respectable periodicals, books are given reviewing income increases as a result of increases won by unions
space in inverse proportion to the likely size of their sales. representing women who work for other employers in the
Airport and supermarket bookstalls stock only books that same industry.
are expected to sell in large numbers. Consequently, those (E) The unionization of women who work for one employer
who buy books at such bookstalls have to do so without any in a given industry frequently prompts the unionization of
guidance whatever from the book reviewers whose work is women who work for other employers in the same industry.
published in respectable periodicals.
Which of the following is a valid criticism of the argument 7. The price the government pays for standard weapons
above? purchased from military contractors is determined by a
(A) Bookstalls like those found at airports and in super- pricing method called “historical costing”. Historical costing
markets are designed to induce people to buy books on allows contractors to protect their profits by adding a
impulse. percentage increase, based on the current rate of inflation,
(B) The assortment of books available at airport book stalls to the previous year’s contractual price.
is different from the assortment of books available at Which of the following statements, if true, is the best basis
supermarket bookstalls. for a criticism of historical costing as an economically sound
(C) The fact that a book is expected to sell well does not pricing method for military contracts?
guarantee that actual sales will be large. (A) The government might continue to pay for past
(D) Many who later come to be respected as book reviewers inefficient use of funds.
start their careers by writing for trashy magazines. (B) The rate of inflation has varied considerably over the
(E) The conclusion that respectable periodicals never past twenty years.
publish reviews of projected bestsellers is unwarranted. (C) The contractual price will be greatly affected by the
cost of materials used for the products.
(D) Many taxpayers question the amount of money the
government spends on military contracts.
69
American Education Aids - LSAT - Logical Reasoning
(E) The pricing method based on historical costing might (A) The group of people who never marry includes people
not encourage the development of innovative weapons. who died before they were old enough to get married
(B) The group of people who never marry includes men
8. Agrostis tenuis, a wild grass, grows abundantly in and women in roughly equal proportions
unpolluted pastures near coal mines. It grows sparsely in (C) The group of people who marry includes people who
areas covered by mine waste where the soil is contaminated have been married more than once.
by heavy metals. A geneticist determined that the plants (D) The group of people who marry includes people who
growing in the contaminated areas are “tolerant” - they are have since become divorced
exceptional individuals that have a tolerance for the heavy (E) The group of people who marry is larger than the groups
metals. A mining company therefore decided to plant Agrostis of people who never marry.
on contaminated areas and turn them into cow pastures,
planning to sell dairy products from the cows. 10. Although the surge in consumer spending over the last
Which of the following, if true, most strongly suggests that two years has mostly benefited foreign companies because
the mining company’s plan will fail? United States consumers purchased imported products, some
(A) When tolerant Agrostis grows in soil contaminated by United States service industries have benefited as well.
heavy metals, it does not grow as fast as when it grows in Payments to medical doctors, for example, have increased
unpolluted pastures significantly in the past year after a lengthy fall.
(B) Most species of wild grass have no tolerance for heavy Which of the following, if true, would most weaken the
metals and therefore will not grow at all in contaminated author’s assertion that the medical doctors have benefited from
areas the recent surge in consumer spending?
(C) Dairy products from cows that eat Agrostis grown in (A) Consumers of medical care in the United States tend
contaminated areas contain levels of heavy metals so high to postpone treatment for minor medical problems during
that the products cannot be sold periods of economic recession
(D) In normal soil nontolerant Agrostis grows at the same (B) Increases in the cost of malpractice insurance have
rate as does tolerant Agrostis, but cows prefer to eat required doctors to increase their fees accordingly
nontolerant Agrostis (C) Preventive measures against catastrophic illnesses
(E) Established colonies of Agrostis are of significant value such as heart disease have generally improved the health of
in preventing erosion and therefore help prevent the spread United States consumers
of contaminants into ground water. (D) The number of nonprofit, publicly owned hospitals in
the United States has increased over the last year.
9. People who marry have a longer average life span than (E) The average proportion of each medical bill paid by
do people who never marry. Therefore, marry and live longer. individuals rather than by medical insurance has decreased
Which of the following, if true, would cast the most serious over the last year.
doubt on the advice given above?

SET 9
1. Under a proposed program to conserve the genetic to increase its already burdensome hours. The most
diversity of soybean seeds, all known varieties of soybean reasonable long-range solution to this problem is to allow
seeds would be collected and centrally stored. Then the stored the Court to decide many cases without hearing oral
seeds, which eventually become infertile, would be replanted argument; in this way the Court might eventually increase
at the storage center, as needed, to produce new seeds. dramatically the number of cases it decides each year.
Which of the following statements, if true, would most Which of the following, if true, could best be used to argue
strongly indicate that the program, even if carried out as against the feasibility of the solution suggested above?
planned, would not achieve its intended result? (A) The time the Court spends hearing oral argument is
(A) A single variety of high-yield soybean seed is only a small part of the total time it spends deciding a case.
increasingly replacing the wide variety of soybean seeds (B) The Court cannot legitimately avoid hearing oral
traditionally planted by farmers argument in any case left over from last year.
(B) The cost of collecting all known varieties of soybean (C) Most authorities agree that 160 hours of oral argument
seeds will exceed the cost of storing and replanting the seeds is the maximum number that the Court can handle per year.
(C) Although some varieties of soybean seeds can be stored (D) Even now the Court decides a small number of cases
for a long period of time without becoming infertile, other without hearing oral argument.
varieties can be stored for only a short period of time (E) In many cases, the delay of a hearing for a full year
(D) An extended program of storing and then replanting can be extremely expensive to the parties involved.
soybean seeds in a single environment can eventually lead to
the loss of genetic traits 3. Fossilized animal bones marked with scratches other
(E) Most other programs to conserve the genetic diversity than tooth marks were discovered in the sand near some stone
of seeds have focused on seeds that are less genetically diverse tools. When bones were scratched using similar tools, the
than are soybean seeds. resulting scratches resembled the scratches found on the
fossils. Therefore, stone tools had probably been used on the
2. The Supreme Court is no longer able to keep pace with animal bones that became fossilized.
the tremendous number of cases it agrees to decide. The Court Which of the following, if true, most seriously undermines
schedules and hears 160 hours of oral argument each year, the argument?
and 108 hours of next year’s term will be taken up by cases (A) The fossilized bones with scratches are not as old as
left over form this year. Certainly the Court cannot be asked nearby fossilized bones found to have tooth marks.
70
American Education Aids - LSAT - Logical Reasoning
(B) Trampling on sand in which a bone is buried produces (A) The groups being compared did not contain the same
scratches on that bone that are similar to those on the fossils. number of children.
(C) Stone tools have been found in areas where nearby (B) More time was spent observing the interactions of
fossilized bones were not scratched. children with their mothers than with their fathers.
(D) The stone tools were too hard to be scratched by animal (C) Most of the researchers involved in the study were
bones. persons who had no brothers or sisters.
(E) Modern microscopes can clearly reveal the differences (D) The first-born children were, on the average, nearly
between tooth marks and other kinds of scratches. three when their parents has second children.
(E) The “other adults” described in the study consisted
4. Manufacturers of household appliances in the United mainly of members of the research team.
States are introducing an array of computerized technologies
in the work of many of their factories in an effort to regain a 7. Geographers and historians have traditionally held the
lead eroded by international competition. On the basis of view that Antarctica was first sighted around 1820, but some
changes that have already taken place, experts predict a sixteenth-century European maps show a body that resembles
golden age for the consumers of better-designed and better- the polar landmass, even though explorers of the period never
built products. saw it. Some scholars, therefore, argue that the continent
Which of the following, if true, would LEAST support the must have been discovered and mapped by the ancients,
experts’ claim that appliances produced by computerized whose maps are known to have served as models for the
technologies will be better built? European cartographers.
(A) Computerized inventory procedures ensure that parts Which of the following, if true, is most damaging to the
are ordered in sufficient quantities and that production moves inference drawn by the scholars?
smoothly and consistently. (A) The question of who first sighted Antarctica in modern
(B) Computer-directed machines carry out repetitive tasks times is still much debated, and no one has been able to
with the result that errors due to human fatigue are present conclusive evidence.
eliminated. (B) Between 3,000 and 9,000 years ago, the world was
(C) Computer-controlled ultrasound devices are better able warmer than it is now, and the polar landmass was
to detect hidden flaws and defects that require repair than presumably smaller.
are human inspectors. (C) There are only a few sixteenth-century global maps
(D) The flow of heat used to weld parts together is more that show a continental landmass at the South Pole.
consistent when directed by computer programs and results (D) Most attributions of surprising accomplishments to
in a more accurate and uniform weld. ancient civilizations or even extraterrestrials are eventually
(E) Computer-driven screwdrivers ensure that screws used discredited or rejected as preposterous.
in appliances will be consistently tight. (E) Ancient philosophers believed that there had to be a
large landmass at the South Pole to balance the northern
5. According to a newspaper article, a customs inspector continents and make the world symmetrical.
believed that he could always tell if people were trying to
deceive him. He based this belief on the fact that, in ten 8. A group of children of various ages was read stories in
years of experience, in cases where he suspected that a tourist which people caused harm, some of those people doing so
was carrying contraband, he was always right. Careful intentionally, and some accidentally. When asked about the
inspections of the luggage and persons of the suspects always appropriate punishments for those who had caused harm,
turned up the contraband goods. the younger children, unlike the older ones, assigned
Which of the following points up a logical flaw in the punishments that did not vary according to whether the harm
inspector’s reasoning? was done intentionally or accidentally. Younger children, then,
(A) The tourists who passed through the inspector’s do not regard people’s intentions as relevant to punishment.
checkpoint could have known that they might be searched. Which of the following, if true, would most seriously weaken
(B) Only a small percentage of the tourists passing through the conclusion above?
the inspector’s checkpoint could be expected to be (A) In interpreting these stories, the listeners had to draw
intentionally carrying contraband. on a relatively mature sense of human psychology in order to
(C) Other customs inspectors at the same checkpoint tell whether harm was produced intentionally or accidentally.
caught tourists carrying contraband just as often as this (B) In these stories, the severity of the harm produced was
inspector did. clearly stated.
(D) Some tourists whom the inspector decided not to search (C) Younger children are as likely to produce harm
could have knowingly carried contraband. unintentionally as are older children.
(E) Some tourists who passed through the checkpoint (D) The older children assigned punishment in a way that
without being searched could have been carrying contraband closely resembled the way adults had assigned punishment
unwittingly. in a similar experiment.
(E) The younger children assigned punishments that
6. Being an only child has little to do with a child’s social varied according to the severity of the harm done by the agents
development. A recent study that followed thirty only children in the stories.
and thirty-five first-born children to the age of three found
that the two groups of children behaved very similarly to each 9. The cities with densest population have the highest ratio
other toward their peers, their parents, and other adults. of police officers to citizens. Such cities also have the lowest
Which of the following, if true, most weakens the conclusion rates of property crime without contact between perpetrator
drawn above? and victim. Thus, maintaining a high ratio of police officers

71
American Education Aids - LSAT - Logical Reasoning
to citizens can serve as an effective deterrent to at least certain interviewing the top candidates for a position all together in
kinds of property crime. a single group. This technique is supposed to afford a direct
Which of the following, if true, most seriously weakens the comparison of the candidates with respect to some personal
argument above? qualities that cannot be gleaned from a resume.
(A) The quality of training that police receive varies from Which of the following, if true, casts the most serious doubt
city to city. on the value of the simultaneous interview technique?
(B) High population density itself makes it difficult to (A) Resumes do sometimes allow reliable inferences to be
commit a property crime that involves no contact between made about a candidate’s personal qualities.
perpetrator and victim. (B) The simultaneous interview could become cumbersome
(C) Many nonviolent crimes in large cities are drug-related. if there were a great many candidates for a position.
(D) A majority of the perpetrators of property crimes in (C) The more perceptive the interviewer, the more revealing
densely populated cities are not apprehended by the police. the simultaneous interview is apt to be.
(E) Property crimes without contact between perpetrator (D) There are certain personal qualities that only an
and victim represent only a small proportion of overall crime. extended simultaneous interview can bring out.
(E) The simultaneous interview distorts each candidate’s
10. In an effort to go beyond resumes as tools in its search response style by inducing stresses unlike any an executive
for executives, one leading company has resorted to position induces.

SET 10
1. It often happens that, as a recession deepens, the stock and often explode soon after being released. In order to
market soars. Later, after the economy has become stronger, prevent passenger deaths from gas inhalation, safety officials
the market often appears to become shaky. For these reasons, recommend that passengers be provided with smoke hoods
many have concluded that the market is a poor indicator of that prevent inhalation of the gases.
economic trends. Which of the following, if true, constitutes the strongest
Which of the following, if true, best counters the argument reason not to require implementation of the safety officials’
presented above? recommendation?
(A) Stock prices do not reflect the current state of the (A) Test evaluations showed that putting on the smoke
economy, but rather the expert judgment of investors about hoods added considerably to the overall time it took
the future strength of the economy passengers to leave the cabin
(B) Brokers and analysts say that stock prices are (B) Some airlines are unwilling to buy the smoke hoods
sometimes inconsistent when business is beginning to emerge because they consider them to be prohibitively expensive
from a slump (C) Although the smoke hoods protect passengers from the
(C) The crash of the stock market in 1929 preceded a toxic gases, they can do nothing to prevent the gases from
prolonged and severe depression igniting
(D) Economists note that sharp gains in the stock market (D) Some experienced flyers fail to pay attention to the
have sometimes been recorded in the worst months of a safety instructions given on every commercial flight before
recession takeoff
(E) Some brokers and analysts believe that the stock (E) In many airplane accidents, passengers who were able
market is a useful index of economic trends because it is to reach emergency exits were overcome by toxic gases before
updated more frequently than other indicators they could exit the airplane

2. Banning cigarette advertisements in the mass media 4. A study of marital relationships in which one partner’s
will not reduce the number of young people who smoke. They sleeping and waking cycles differ from those of the other
know that cigarettes exist and they know how to get them. partner reveals that such couples share fewer activities with
They do not need the advertisements to supply that each other and have more violent argument than do couples
information. in a relationship in which both partners follow the same
The above argument would be most weakened if which of sleeping and waking patterns. Thus, mismatched sleeping
the following were true? and waking cycles can seriously jeopardize a marriage.
(A) Seeing or hearing an advertisement for a product tends Which of the following, if true, most seriously weakens the
to increase people’s desire for that product argument above?
(B) Banning cigarette advertisements in the mass media (A) Married couples of whom both spouses follow the same
will cause an increase in advertisements in places where sleeping and waking patterns also occasionally have
cigarettes are sold arguments that can jeopardize the couple’s marriage
(C) Advertisements in the mass media have been an (B) The sleeping and waking cycles of individuals tend to
exceedingly large part of the expenditure of the tobacco vary from season to season
companies (C) The individuals who have sleeping and waking cycles
(D) Those who oppose cigarette use have advertised against that differ significantly from those of their spouses tend to
it in the mass media ever since cigarettes were found to be argue little with colleagues at work
harmful (D) People in unhappy marriages have been found to
(E) Older people tend to be less influenced by mass media express hostility by adopting a different sleeping and waking
advertisements than younger people tend to be cycle from that of their spouses
(E) According to a recent study, most people’s sleeping and
3. Passengers must exit airplanes swiftly after accidents, waking cycles can be controlled and modified easily
since gases released following accidents are toxic to humans
72
American Education Aids - LSAT - Logical Reasoning
5. The ice on the front windshield of the car had formed 8. A diet that contains polyunsaturated fats and is low in
when moisture condensed during the night. The ice melted saturated fats reduces the risk of heart disease. Fish are an
quickly after the car was warmed up the next morning excellent source of omega-3, a polyunsaturated fat they obtain
because the defrosting vent, which blows only on the front by eating the green plant cells of plankton. But cows and
windshield, was turned on full force. other ruminants obtain omega-3 polyunsaturates when they
Which of the following, if true, most seriously jeopardizes eat grass. Therefore, a diet rich in beef and a diet rich in fish
the validity of the explanation for the speed with which the should be equally effective in reducing the risk of heart
ice melted? disease.
(A) The side windows had no ice condensation on them Which of the following, if true, would weaken the conclusion
(B) Even though no attempt was made to defrost the back above?
window, the ice there melted at the same rate as did the ice (A) The diet of beef cattle can be monitored far more
on the front windshield. accurately than can the diet of fish.
(C) The speed at which ice on a window melts increases as (B) Consuming polyunsaturates reduces the amount of
the temperature of the air blown on the window increases. cholesterol produced by the liver and reduces the chance of
(D) The warm air from the defrosting vent for the front blockage in coronary blood vessels.
windshield cools rapidly as it dissipates throughout the rest (C) Studies have shown that people who eat fish only once
of the car. a week have a lower risk of heart disease than do people who
(E) The defrosting vent operates efficiently even when the never eat fish.
heater, which blows warm air toward the feet or faces of the (D) Medical researchers are expected soon to make a
driver and passengers, is on. synthetic form of omega-3 that can be injected into beef.
(E) Cows and other ruminants convert most of the omega-
6. A conservation group in the United States is trying to 3 polyunsaturates they consume into saturated fats.
change the long-standing image of bats as frightening
creatures. The group contends that bats are feared and 9. Studies of workplace safety in construction and
persecuted solely because they are shy animals that are active manufacturing firms have found that the rate of injuries tends
only at night. to rise when the firms’ work loads increase. Since
Which of the following, if true, would cast the most serious inexperienced workers are often hired by these firms when
doubt on the accuracy of the group’s contention? work loads increase, the higher rate of injuries is undoubtedly
(A) Bats are steadily losing natural roosting places such due to a higher accident rate for inexperienced workers.
as caves and hollow trees and are thus turning to more Which of the following statements, if true, would most
developed areas for roosting. weaken the conclusion drawn above?
(B) Bats are the chief consumers of nocturnal insects and (A) Many of the inexperienced workers hired when the
thus can help make their hunting territory more pleasant firms’ work loads increase are hired only for temporary
for humans. positions.
(C) Bats are regarded as frightening creatures not only in (B) The studies of workplace safety were focused only on
the United States but also in Europe, Africa and South injuries that resulted in lost workdays.
America. (C) There is a much higher rate of injury in construction
(D) Raccoons and owls are shy and active only at night; firms than in manufacturing firms.
yet they are not generally feared and persecuted. (D) The accident rate for experienced workers tends to
(E) People know more about the behavior of other greatly- increase whenever the firms’ work loads increase.
feared animal species, such as lions, alligators, and snakes, (E) Firms that hire inexperienced workers for potentially
than they do about the behavior of bats. dangerous jobs are required to provide them with training.

7. Spiritualism, the doctrine that it is possible to 10. Although the human population around the forest land
communicate with the spirits of the deceased through in Middlesex County has increased, the amount of forest land
specially talented persons called mediums, is fraudulent. As has not been reduced. Therefore, the decrease in the county’s
long ago as the 1870’s Professor Edwin Lankester showed songbird population cannot be attributed to the growth in
that the purported “spirit writing” of the famed medium the county’s human population.
Henry Slade was present on a slate before the “spirits” were Which of the following, if true, most seriously weakens the
supposed to have begun writing on it. This example conclusion above?
demonstrates that the doctrine of spiritualism is worthless. (A) As the human population of Middlesex County has
If the example above is correctly reported, which of the grown, there has been an increase in the number of shopping
following is the best argument against the conclusion drawn malls built.
above? (B) The presence of more garbage cans resulting from the
(A) There cannot be proof that the spirits of the deceased increase in the county’s human population ensures the
do not exist survival of more raccoons, which prey on songbird eggs
(B) The conclusion depends on a historical report, and such whenever available.
reports of past events do not recount all of the circumstances (C) There has recently been a decrease in the amount of
(C) The cited evidence presupposes what is to be proved rain-forest land in Central and South America, where
(D) A single instance of fraud cannot show that the doctrine songbirds spend the winter months.
is false in general (D) Although several species of songbirds are disappearing
(E) The correctness of the report depends on the veracity from Middlesex County, these species are far from being
of anti-spiritualists, who may be expected to be biased endangered.
(E) The disappearance of songbirds, which eat insects, often
results in increased destruction of trees by insects.
73
American Education Aids - LSAT - Logical Reasoning
SET 11
1. Since the mayor’s publicity campaign for Greenville’s 4. A certain mayor has proposed a fee of five dollars per
bus service began six months ago, morning automobile traffic day on private vehicles entering the city, claiming that the
into the midtown area of the city has decreased seven percent. fee will alleviate the city’s traffic congestion. The mayor
During the same period, there has been an equivalent rise in reasons that, since the fee will exceed the cost of round-trip
the number of persons riding buses into the midtown area. bus fare from many nearby points, many people will switch
Obviously, the mayor’s publicity campaign has convinced from using their cars to using the bus.
many people to leave their cars at home and ride the bus to Which of the following statements, if true, provides the best
work. evidence that the mayor’s reasoning is flawed?
Which of the following, if true, casts the most serious doubt (A) Projected increases in the price of gasoline will increase
on the conclusion drawn above? the cost of taking a private vehicle into the city.
(A) Fares for all bus routes in Greenville have risen an (B) The cost of parking fees already makes it considerably
average of five percent during the past six months. more expensive for most people to take a private vehicle into
(B) The mayor of Greenville rides the bus to City Hall in the city than to take a bus.
the city’s midtown area. (C) Most of the people currently riding the bus do not own
(C) Road reconstruction has greatly reduced the number private vehicles.
of lanes available to commuters in major streets leading to (D) Many commuters opposing the mayor’s plan have
the midtown area during the past six months. indicated that they would rather endure traffic congestion
(D) The number of buses entering the midtown area of than pay a five-dollar-per-day fee.
Greenville during the morning hours is exactly the same now (E) During the average workday, private vehicles owned
as it was one year ago. and operated by people living within the city account for
(E) Surveys show that longtime bus riders are no more twenty percent of the city’s traffic congestion.
satisfied with the Greenville bus service than they were before
the mayor’s publicity campaign began. 5. Sales of telephones have increased dramatically over
the last year. In order to take advantage of this increase,
2. On the basis of a decrease in the college-age population, Mammoth Industries plan to expand production of its own
many colleges now anticipate increasingly smaller freshman model of telephone, while continuing its already very
classes each year. Surprised by a 40% increase in qualified extensive advertising of this product.
applicants over the previous year, however, administrators Which of the following, if true, provides most support for
at Nice College now plan to hire more faculty for courses taken the view that Mammoth Industries cannot increase its sales
by all freshmen. of telephones by adopting the plan outlined above?
Which of the following statements about Nice College’s (A) Although it sells all of the telephones that it produces,
current qualified applicants, if true, would strongly suggest Mammoth Industries’ share of all telephone sales has declined
that the administrators’ plan is flawed? over the last year.
(A) A substantially higher percentage than usual plan to (B) Mammoth Industries’ average inventory of telephones
study for advanced degrees after graduation from college. awaiting shipment to retailers has declined slightly over the
(B) According to their applications, their level of last year.
participation in extracurricular activities and varsity sports (C) Advertising has made the brand name of Mammoth
is unusually high. Industries’ telephone widely known, but few consumers know
(C) According to their applications, none of them lives in a that Mammoth Industries owns this brand.
foreign country. (D) Mammoth Industries’ telephone is one of three brands
(D) A substantially lower percentage than usual rate Nice of telephone that have together accounted for the bulk of the
College as their first choice among the colleges to which they last year’s increase in sales.
are applying. (E) Despite a slight decline in the retail price, sales of
(E) A substantially lower percentage than usual list Mammoth Industries’ telephones have fallen in the last year.
mathematics as their intended major.
6. Dental researchers recently discovered that
3. A drug that is highly effective in treating many types of toothbrushes can become contaminated with bacteria that
infection can, at present, be obtained only from the bark of cause pneumonia and strep throat. They found that
the ibora, a tree that is quite rare in the wild. It takes the contamination usually occurs after toothbrushes have been
bark of 5,000 trees to make one kilogram of the drug. It used for four weeks. For that reason, people should replace
follows, therefore, that continued production of the drug must their toothbrushes at least once a month.
inevitably lead to the ibora’s extinction. Which of the following, if true, would most weaken the
Which of the following, if true, most seriously weakens the conclusion above?
argument above? (A) The dental researchers could not discover why
(A) The drug made from ibora bark is dispensed to doctors toothbrush contamination usually occurred only after
from a central authority. toothbrushes had been used for four weeks.
(B) The drug made from ibora bark is expensive to produce. (B) The dental researchers failed to investigate
(C) The leaves of the ibora are used in a number of medical contamination of toothbrushes by viruses, yeasts, and other
products. pathogenic microorganisms.
(D) The ibora can be propagated from cuttings and grown (C) The dental researchers found that, among people who
under cultivation. used toothbrushes contaminated with bacteria that cause
(E) The ibora generally grows in largely inaccessible areas. pneumonia and strep throat, the incidence of these diseases

74
American Education Aids - LSAT - Logical Reasoning
was no higher than among people who used uncontaminated (D) Repair of satellites requires heavy equipment, which
toothbrushes. adds to the amount of fuel needed to lift a spaceship carrying
(D) The dental researchers found that people who rinsed astronauts into orbit.
their toothbrushes thoroughly in hot water after each use (E) Technical obsolescence of robot satellites makes
were as likely to have contaminated toothbrushes as were repairing them more costly and less practical than sending
people who only rinsed their toothbrushes hurriedly in cold new, improved satellites into orbit.
water after each use.
(E) The dental researchers found that, after six weeks of 9. In an attempt to promote the widespread use of paper
use, greater length of use of a toothbrush did not correlate rather than plastic, and thus reduce nonbiodegradable waste,
with a higher number of bacteria being present. the council of a small town plans to ban the sale of disposable
plastic goods for which substitutes made of paper exist. The
7. Although migraine headaches are believed to be caused council argues that since most paper is entirely biodegradable,
by food allergies, putting patients on diets that eliminate paper goods are environmentally preferable.
those foods to which the patients have been demonstrated to Which of the following, if true, indicates that the plan to
have allergic migraine reactions frequently does not stop ban the sale of disposable plastic goods is ill-suited to the
headaches. Obviously, some other cause of migraine town council’s environmental goals?
headaches besides food allergies must exist. (A) Although biodegradable plastic goods are now available,
Which of the following, if true, would most weaken the members of the town council believe biodegradable paper
conclusion above? goods to be safer for the environment.
(A) Many common foods elicit an allergic response only (B) The paper factory at which most of the townspeople
after several days, making it very difficult to observe links are employed plans to increase production of biodegradable
between specific foods patients eat and headaches they paper goods.
develop. (C) After other towns enacted similar bans on the sale of
(B) Food allergies affect many people who never develop plastic goods, the environmental benefits were not discernible
the symptom of migraine headaches. for several years.
(C) Many patients report that the foods that cause them (D) Since most townspeople prefer plastic goods to paper
migraine headaches are among the foods that they most enjoy goods in many instances, they are likely to purchase them in
eating. neighbouring towns where plastic goods are available for sale.
(D) Very few patients have allergic migraine reactions as (E) Products other than those derived from wood pulp are
children and then live migraine-free adult lives once they
often used in the manufacture of paper goods that are entirely
have eliminated from their diets foods to which they have
biodegradable.
been demonstrated to be allergic.
(E) Very rarely do food allergies cause patients to suffer a
10. Since the deregulation of airlines, delays at the nation’s
symptom more severe than that of migraine headaches.
increasingly busy airports have increased by 25 percent. To
combat this problem, more of the takeoff and landing slots at
8. Robot satellites relay important communications and
the busiest airports must be allocated to commercial airlines.
identify weather patterns. Because the satellites can be
Which of the following, if true, casts the most doubt on the
repaired only in orbit, astronauts are needed to repair them.
effectiveness of the solution proposed above?
Without repairs, the satellites would eventually malfunction.
Therefore, space flights carrying astronauts must continue. (A) The major causes of delays at the nation’s busiest
Which of the following, if true, would most seriously weaken airports are bad weather and overtaxed air traffic control
the argument above? equipment.
(A) Satellites falling from orbit because of malfunctions (B) Since airline deregulation began, the number of
burn up in the atmosphere. airplanes in operation has increased by 25 percent.
(B) Although satellites are indispensable in the (C) Over 60 percent of the takeoff and landing slots at the
identification of weather patterns, weather forecasters also nation’s busiest airports are reserved for commercial airlines.
make some use of computer projections to identify weather (D) After a small midwestern airport doubled its allocation
patterns. of takeoff and landing slots, the number of delays that were
(C) The government, responding to public pressure, has reported decreased by 50 percent.
decided to cut the budget for space flights and put more money (E) Since deregulation the average length of delay at the
into social welfare programs. nation’s busiest airports has doubled.

75
Chapter 6
'STRENGTHENS' QUESTIONS
The ‘strengthens’ questions in Logical Reasoning in LSAT The ‘strengthens’ questions may be worded in a number of
have a form which is just the converse of that of the ‘weakens’ different ways such as:
questions. Which of the following, if true, most significantly strength-
Questions of this type will mention (in the initial narra- ens the conclusion above?
tion) a set of premises and a conclusion drawn therefrom, Which of the following, if true, most strongly supports the
and will then ask you to spot that statement from among the above argument / claim / conclusion / hypothesis?
five given choices which, if true, will strengthen the sound- Which of the following, if true, most helps to explain the
ness of that conclusion. above hypothesis / preference?
The correct choice will either contain some additional Which of the following, if true, provides or contributes to
information which will support, directly or indirectly, one of an explanation for the above result?
the premises on which the conclusion is based, or will give The above conclusion would be most strengthened if which
one more relevant premise that will support the conclusion of the following were true?
itself. The argument above would be more persuasive if it were
true that .....
Let us now revert to the example from day-to-day life which Which of the following may be cited as a piece of additional
we had used in the previous chapters: evidence for the soundness of the above plan?
“The time is 5.30 now and the movie starts at 6.30. Unless All the following are valid objections to the above plan
we leave home within the next 15 minutes, we shall be missing EXCEPT
its beginning”.
In one variant of the ‘strengthens’ question that is
Suppose that the speaker’s watch is running slow by 10 occasionally asked, four of the five choices will strengthen
minutes, and that it is showing 5.30 when the time is really the given conclusion, while the fifth will either weaken it or
5.40. If this is true, he must leave home even within the next will be irrelevant to it. You will be asked to select this odd-
five minutes if he wishes to see the movie from the beginning. one-out as your answer.
So, the statement, “The speaker’s watch is running slow Such questions will take the form:
by 10 minutes” will strengthen the given conclusion that he Each of the following, if true, will strengthen the conclusion
will miss the beginning of the movie if he doesn’t leave home drawn above EXCEPT
within the next 15 minutes. Which of the following, if true, least supports the above
conclusion?
Consider the statement, “Very often there is a traffic jam
at a road junction on the way from the speaker’s home to the In another variation of this type, the conclusion will not
movie theater which could hold up traffic for upto 10 minutes be found in the narration, but will be incorporated as part of
at a time”. the question itself, as in Examples 3 and 5 below.
This statement also strengthens the speaker’s conclusion
that he will miss the beginning of the movie if he does not We are giving below a number of solved examples of this
leave home within 15 minutes. type, followed by 6 sets of 10 questions each for your practice.
These are of the same types and levels of difficulty as those
Consider the statement, “The speaker’s watch is running asked in LSAT.
fast by 15 minutes”.
This means that the real time now is only 5.30 and not Example 1
5.45. So, the speaker will not miss the beginning of the movie Many companies now have employee assistance programs
even if he leaves home within the next 30 minutes. So, this that enable employees, free of charge, to improve their
statement weakens the speaker’s conclusion that he will miss physical fitness, reduce stress, and learn ways to stop
the beginning of the movie if he does not leave within the smoking. These programs increase worker productivity,
next 15 minutes. reduce absenteeism, and lessen insurance costs for employee
health care. Therefore, these programs benefit the company
Consider the statement, “The movie that the speaker is as well as the employee.
planning to see has been nominated for an Oscar Award”. Which of the following, if true, most significantly
Obviously, this statement by itself neither strengthens nor strengthens the conclusion above?
weakens the speaker’s conclusion that he will miss the (A) Physical fitness programs are often the most popular
beginning of the movie if he does not leave home within the services offered to employees
next 15 minutes. This statement is therefore irrelevant to (B) Studies have shown that training in stress
the speaker’s conclusion. management is not effective for many people
(C) Regular exercise reduces people’s risk of heart disease
The various choices under the ‘strengthens’ questions will and provides them with increased energy
be of the above types. (D) Physical injuries sometimes result from entering a
Among the choices given under the question, only one will strenuous physical fitness program too quickly
strengthen the given conclusion (and will be the answer), while (E) Employee assistance programs require companies to
each of the others will either weaken that conclusion or be hire people to supervise the various programs offered
irrelevant to it.
76
American Education Aids - LSAT - Logical Reasoning
Analysis new drugs through their subsequent sales, the period of
The conclusion in the passage is that the employee assist- patent protection for newly developed drugs should be
ance program (which enables employees to improve their extended beyond 20 years. The industry points out that,
physical fitness, reduce stress and learn ways to stop smok- otherwise, new drugs will not be developed at all.
ing) benefit the company as well as the employee. Having mentioned the drug industry’s demand, the author
We have been asked to locate the choice which strengthens of the narrative challenges it. (A last sentence starting with
this conclusion. ‘however’ usually gives the opinion of the author contradicting
We should therefore look for that choice which implies that what is stated earlier.) He argues that industries other than
the program is beneficial both to the company and to the pharmaceuticals, for whom the period of protection is the
employee. same twenty years, also continue to incur high development
(C) says that regular exercise reduces people’s risk of heart costs on new inventions, and that there is therefore no need
disease (beneficial to the employee) and provides them with for extension of this period in the case of pharmaceutical
increased energy (which will benefit the company in which industry alone.
they are working). So, it is (C) which says that the program is We have been asked to spot that choice which supports the
beneficial both to the company and the employee, and is the pharmaceutical industry’s plea for the extension of the patent
answer. period beyond twenty years.
(A) mentions that such programs are popular with (A) merely repeats what the author says, and does not
employees, but does not indicate how they are beneficial to strengthen the drug industries’ plea. So, (A) is not the answer.
the company as well as the employee, and is not the answer. (B) points out that, after a patent is granted for a new drug,
(B), in fact, contradicts the conclusion that the program it has to undergo clinical trials for as long as ten years before
benefits all employees, and is not the answer. it can be marketed. This implies that, even though there is
(D) also contradicts the conclusion that the program patent protection for twenty years for a drug, the effective
benefits employees, and is not the answer. period of monopolistic sale of the product available to its
(E) does not mention any benefit that accrues to the inventor is only ten years, and the pharmaceutical industry
employees because of the program. On the contrary, it says has to recoup the cost of development within this shorter
that it may cause additional expenditure to the company, period. This is a special situation applicable to only the
meaning that it does not benefit the company either. So, (E) pharmaceutical industry and not to any of the other
contradicts, and does not strengthen, the conclusion, and is industries, all of which can enjoy full twenty years of sale for
not the answer. recouping their costs. So, (B) gives a valid argument why the
pharmaceutical industry should be treated on a special
Example 2 footing, and is the answer.
The pharmaceutical industry argues that because new (C) only strengthens the author’s challenge, and weakens
drugs will not be developed unless heavy development costs the pharmaceutical industry’s claim.
can be recouped in later sales, the current 20 years of (D) neither strengthens nor weakens the pharmaceutical
protection provided by patents should be extended in the case industry’s plea, and is not the answer.
of newly developed drugs. However, in other industries, new- (E) refers to industries other than drugs, and neither
product development continues despite high development strengthens nor weakens the plea of the drug industry.
costs, a fact that indicates that the extension is unnecessary.
Which of the following, if true, most strongly supports the Example 3
pharmaceutical industry’s argument against the challenge Caterpillars of all species produce an identical hormone
made above? called “juvenile hormone” that maintains feeding behavior.
(A) No industries other than the pharmaceutical industry Only when a caterpillar has grown to the right size for
have asked for an extension of the 20-year limit on patent pupation to take place does a special enzyme halt the
protection production of juvenile hormone. This enzyme can be
(B) Clinical trials of new drugs, which occur after the patent synthesized and will, on being ingested by immature
is granted and before the new drug can be marketed, often caterpillars, kill them by stopping them from feeding.
now take as long as 10 years to complete Which of the following, if true, most strongly supports the
(C) There are several industries in which the ratio of view that it would not be advisable to try to eradicate
research and development costs to revenues is higher than it agricultural pests that go through a caterpillar stage by
is in the pharmaceutical industry spraying crop lands with the enzyme mentioned above?
(D) An existing patent for a drug does not legally prevent (A) Most species of caterpillar are subject to some natural
pharmaceutical companies from bringing to market predation
alternative drugs, provided they are sufficiently dissimilar (B) Many agricultural pests do not go through a caterpillar
to the patented drug stage
(E) Much recent industrial innovation has occurred in (C) Many agriculturally beneficial insects go through a
products - for example, in the computer and electronics caterpillar stage
industries - for which patent protection is often very (D) Since caterpillars of different species emerge at
ineffective different times, several sprayings would be necessary
(E) Although the enzyme has been synthesized in the
Analysis laboratory, no large-scale production facilities exist as yet
(The grant of patent to its inventor for a product implies
that no other manufacturer can make the same product Analysis
commercially during the period of the patent’s validity.) The (Pupation means the process of a caterpillar evolving into
pharmaceutical industry’s argument is that, in order to enable a pupa.) The narrative first says that caterpillars of all species
it to recoup the huge expenditure incurred in developing the produce an identical hormone called ‘juvenile hormone’ which
77
American Education Aids - LSAT - Logical Reasoning
maintains their feeding behaviour. When the caterpillar has (D) Customers who never previously purchased products
grown old enough to evolve into a pupa, a special enzyme is from Exco purchased, on the average, fewer products in 1984
produced in its body which stops the production of the ‘juvenile than did previous customers.
hormone’. The caterpillar will then stop eating, go into (E) The increase between 1983 and 1984 in the cost of
hibernation and develop into a pupa. mailing the catalogs was greater than the increase in sales
The narrative then says that it is possible to produce this from 1983 to 1984.
special enzyme artificially and expose the immature
caterpillars to them. When they eat this enzyme, their bodies Analysis
will stop producing the juvenile hormone, and the immature What the passage states is that there was an increase in
caterpillars will stop eating even before they attain the pupa the volume of sales by Exco between 1983 and 1984, but the
stage, and will die of starvation, and their further propagation profit during 1984 was less than the profit in 1983.
will thereby by arrested. We have been asked to identify the choice which would
One may therefore be inclined to think that, if agricultural explain such a reduction in profit despite the increased sales
crops are sprayed with this enzyme, agricultural pests that volume.
go through caterpillar stage can be eradicated, and the Normally, if the profit margin remains the same, increase
spraying will therefore be beneficial to the crops. in sales must result in simultaneous increase in profits. So, if
The question, however, asks us to spot that choice which there has been a reduction in profit in spite of increase in
would support the view that it is not advisable to use this sales, it means that the profit margin had come down. Now,
method for eradicating agricultural pests. the profit margin in selling an article is equal to “(sale price)
If, as stated by (C), many agriculturally beneficial insects - (cost of the article + cost of marketing)”. So, the reduction in
(such as butterflies which help in pollination) also go through profit margin could have resulted from either an increase in
a caterpillar stage, the spraying of the enzyme on the the basic cost of the article sold, or an increase in the marketing
agricultural crop will destroy these beneficial insects also along cost.
with harmful pests. This will be detrimental to the growth of We must therefore look for that choice which mentions
the crops. So, it is (C) which implies that the spraying of the either that the cost of the articles had increased or that the
enzyme is not an advisable practice from the overall point of cost of marketing had increased between 1983 and 1984.
view of crop growth, and is the answer. (A) states that the sales tax had to be paid by the consumers,
(‘Predation’ means killing of a living being by a stronger and not by Exco. So, an increase in the rate of sales tax could
being) (A) does not say that all agricultural pests are not have affected the profits of Exco. So, (A) is not the answer.
destroyed through natural predation. It only says that ‘most (B) implies that it was the previous customers of Exco who
species’ of caterpillar are subject to ‘some’ natural predation. had contributed more to the increased sales in 1984, but it
So, (A) is not a valid argument for not using the enzymes for does not give any explanation as to why the profits had come
destroying the pests. down.
The question is whether the enzyme should be used to If (C) is true, it implies that the profit margin per article
eradicate agricultural pests which go through the caterpillar had increased. In this case, the overall profits must have
stage. The fact that many pests do not go through this stage is increased between 1983 and 1984, and not decreased. So, (C),
not an argument against using the enzyme against those pests in fact, contradicts, and does not explain, why the profits had
which go through that stage. So, (B) is not the answer. come down in 1984.
The fact that several sprayings may be necessary does not (D) only implies that a greater proportion of the increased
make such spraying inadvisable. So, (D) is not the answer. sales in 1984 was accounted for by old customers, and throws
The fact that the enzyme is not being produced on a no light on the reason for the reduction in the overall profits.
commercial scale now is not a reason why its use is (E) contains information which makes us conclude that
inadvisable. If its use is found advisable, it can be produced the marketing cost during 1984 was more than the increase
on a commercial basis in future. So, (E) is not a valid reason in sales registered during that year. If this is true, it perfectly
for arguing that the use of enzymes is not advisable. explains why the overall profits had come down during 1984.
So, (E) is the answer.
Example 4
In 1984, Exco, which sells its products only through mail- Example 5
order catalogs, began distributing its catalog to people who In order to reverse a downward trend in occupancy rates,
had never purchased Exco’s products, while it continued to many hospitals are now turning to advertising and are also
distribute the catalog to previous customers. Total dollar sales offering, for an extra fee, new amenities such as gourmet
increased in 1984, but Exco’s profits that year were smaller meals, luxury rooms, and valet parking.
than in 1983. Which of the following, if it were the reason that the beds
Which of the following, if true, contributes most to an in the hospitals involved were being left empty, would increase
explanation of Exco’s smaller profits in 1984, as compared to the likelihood that the marketing technique mentioned above
1983? would be effective?
(A) There was a two percent increase in 1984 in the sales (A) The population of the areas around the hospitals is
tax that consumers had to pay on all purchases from the generally healthier than in the past
catalog. (B) Hospital costs are prohibitively high for many people
(B) A greater number of catalogs were sent to previous (C) Patients have been choosing other hospitals offering
customers than to people who never purchased products from better amenities than those previously available in the
Exco. hospitals involved
(C) In 1984 Exco’s product-manufacturing costs increased (D) Many medical procedures that once required
by a smaller amount than the products’ selling prices hospitalization are now done in doctors’ offices or in clinics
increased. with limited facilities
78
American Education Aids - LSAT - Logical Reasoning
(E) The hospitals involved do not offer important medical the answer.
services that are available elsewhere (C) may be the result of less number of students staying in
the university premises during vacation time, and may have
Analysis nothing to do with stress or lack of stress. So, (C) is not
The passage implies that the problem faced by many relevant to the given conclusion, and is not the answer.
hospitals is the downward trend in the rate of occupancy of The given conclusion is that a stressful situation causes
the beds in them. These hospitals are trying to reverse this illness that leads to hospitalization. (E) reverses this
trend through advertising, and by offering new amenities such conclusion by stating that it is hospitalization that causes a
as good food, luxury rooms and parking by chauffeurs. condition of stress! So, (E) is not the answer.
We have been asked to identify a possible reason for the
downward trend in the occupancy of these hospitals, which Example 7
can be set right through the measures proposed to be adopted Scientists have found that inserting genes from a plant
by them. virus into the genes of a plant susceptible to that virus
No healthy person would like to spend time and money increases the plant’s resistance to the virus. Because viral
occupying hospital rooms, merely because good food, diseases account for a significant proportion of crop losses,
comfortable rooms and chauffeurs are available there. such genetic alterations, even if carried out on only a modest
Therefore, while formulating the new schemes, the scale, will significantly reduce crop losses.
presumption that has been made is that there are people Each of the following, if true, strengthens the conclusion
living in the neighbourhood who require hospitalization, but above EXCEPT
who have been preferring other hospitals where the facilities, (A) In areas where two successive crops are raised per year,
now proposed by these hospitals, are already available. protecting the first crop from a virus will generally protect
So, if these hospitals now introduce equal or better the second crop from that virus as well.
amenities than those available in those other hospitals and (B) By repeatedly attacking plants that are genetically
also give publicity to these amenities, these patients may altered to become virus-resistant, the virus often becomes
change their option. It is (C) which implies this, and is the less lethal to plants that are not genetically altered in that
answer. way.
You can easily see that none of the other choices mentions (C) Plants that are genetically altered to become virus-
a cause which can be rectified by the proposed amenities of resistant often pass virus-resistant genes on to their offspring.
these hospitals and the publicity given to them. (D) Plants that are made genetically resistant to one kind
virus often acquire resistance to related viral strains as well.
Example 6 (E) Plants that are made genetically resistant to one kind
Recent surveys show that many people who seek medical of virus are often more susceptible to infection by unrelated
help are under a great deal of stress. Medical research also viruses.
shows that stress can affect an individual’s immune system,
which is responsible for combating many infections. Thus Analysis
when a person is under stress, he or she is more likely to (Note that, in this example, four of the five given choices
become ill. strengthen the given conclusion, while only one does not. We
Which of the following, if true, would most strengthen the have been asked to spot this particular choice.)
conclusion above? Plants are often susceptible to a number of viruses and, if
(A) Many businesses that provide health insurance for their attacked by them, may die or become sick, thereby giving a
employees also provide seminars on stress management. much lower crop yield to the farmer. The given narration says
(B) Many businesses report a significant decrease in that inserting a gene of a virus in the gene of a plant
absenteeism during periods when employees feel pressured susceptible to it makes the plant immune to that virus, and
by management. recommends the adoption of this procedure for reducing crop
(C) There is a marked decrease in the number of complaints losses.
presented at college infirmaries during vacation time. (A) strengthens the recommendation because it will result
(D) There is a marked increase in the number of illnesses in reduced crop losses in the second season also.
treated at college infirmaries around the time of (B) says that even the plants which are not subjected to
examinations. gene manipulation would be benefited by the recommended
(E) Most people report that being in a hospital or an procedure. The crop loss will thereby diminished still further.
infirmary is a stressful situation. So, (B) also strengthens the recommendation.
(C) says that the gene manipulation will make the plant
Analysis immune to the virus for many generations. Obviously, this
The conclusion in the passage is that “when a person is also strengthens the recommendation.
under stress, he or she is more likely to become ill”. (D) says that the genetically manipulated plants will be
We should therefore look for that choice which gives an resistant not only to the specific virus strain but also to other
instance of stress leading to more instances of hospitalisation. related virus strains. This will reduce crop losses still further.
It is (D) which gives such an instance, and is the answer. So, (D) also strengthens the recommendation.
(Infirmary means hospital) (E) points out that plants which have been immunized
(A) neither strengthens nor weakens the given conclusion, against one virus through gene manipulation become more
and is irrelevant to it. susceptible to be attacked by other unrelated viruses. If this is
(B) says that employees attend office more regularly when true, there could be more loss of crop than there would have
they are under pressure from the management. This, been if there had been no immunization. So, (E) does not
obviously, contradicts the given conclusion that persons are strengthen, but actually weakens the given recommendation.
more likely to fall ill when they are under stress, and is not So, (E) is the answer.
79
American Education Aids - LSAT - Logical Reasoning
Example 8 Which of the following, if true, best supports the theory
Kale has more nutritional value than spinach. But since above?
collard greens have more nutritional value than lettuce, it (A) Modern volcanic explosions have sent enough dust into
follows that kale has more nutritional value than lettuce. the atmosphere to change the color of sunsets around the
Any of the following, if introduced into the argument as an world for several years
additional premises, makes the argument above logically (B) Meteorite craters are abundant in several areas of the
correct EXCEPT Southwest where numerous dinosaur fossils have also been
(A) Collard greens have more nutritional value than kale found
(B) Spinach has more nutritional value than lettuce (C) Recent studies of bone structure suggest that dinosaurs
(C) Spinach has more nutritional value than collard greens were quite agile beasts, rather than the ponderous, awkward
(D) Spinach and collard greens have the same nutritional animals they were once throughout to be
value (D) The Earth’s sedimentary rock strata that were formed
(E) Kale and collard greens have the same nutritional value from atmospheric dust at the time the dinosaurs vanished
contain minerals and other trace elements in proportions
Analysis characteristic of meteorites
Let us denote ‘more nutritional value’ by the symbol >. (E) Several meat-and plant-eating species of mammals that
From the premises that K > S, and C > L, it is concluded coexisted with the dinosaurs continue to exist today, although
that K > L. their appearance is considerably modified
Obviously, the first two inequalities do not lead to the given
conclusion, and there has to be one or more relevant premises Analysis
which will render the conclusion valid. The theory propounded in the given passage is that the
(A) says C > K. Now we have two pieces of information: C extinction of dinosaurs was consequent to a huge meteorite
> K and C > L. We cannot conclude therefrom that K > L. striking the earth.
(B) says S > L. We can therefore form the combined (A) talks of the change in the colour of sunsets because of
inequality K > S > L, from which it follows that K > L. So, (B) volcanic explosions, and this is not relevant to the extinction
strengthens the given conclusion. of dinosaurs due to a meteorite hit on the earth. So, (A) is not
(C) says S > C. From the resulting combined inequality K the answer.
> S > C > L, it follows that K > L. So, (C)also strengthens the The given passage refers to the impact of a single meteorite
given conclusion. on the earth which was huge enough to give out a debris that
(D) says S = C. The combined inequality now is K > S = C blocked sunlight for a long time. The existence of a large
> L, from which it follows that K > L. So, So, (D)also number of small meteorite craters in the Southwest is not
strengthens the given conclusion. relevant to the conclusion in the given passage. So, (B) is not
(E) says K = C. So, from the given information C > L, we the answerw.
can say that K > L. So, (E) also strengthens the given (C) contradicts the information in the passage because, if
conclusion. it were true, the dinosaurs could have moved out from the
Among the choices, it is only (A) which does not lead to the areas where plant life got extinguished to those areas where
conclusion K > L. So, (A) is the answer. they were still available.
(D) implies that the extinction of the dinosaurs was
Example 9 contemporaneous with the impact of a meteorite which was
The dinosaurs may have been destroyed when the Earth large enough to give out atmospheric dust. If this is true, it
was struck by a huge meteorite whose impact threw a mass strengthens the conclusion in the passage. So, (D) is the
of pulverized meteorite debris into the atmosphere. This dust answer.
cut off sunlight so depleting plant life that the plant-eating (E) contradicts the information in the passage that both
dinosaurs perished; the meat-eating dinosaurs, who preyed the plant-eating and meat-eating dinosaurs were destroyed
on the plant-eaters, then starved to death. when the meteorite struck the earth.

We shall now give you 6 sets of ‘strengthens’ questions for your practice.

80
American Education Aids - LSAT - Logical Reasoning
SET 1
1. “In a civilized society, personal merit will not serve you C. The use of fireworks deprives many people of their lives.
as well as money will. You may make the experiment. Go into D. Fireworks are not essential to the spirit of the
the street and give one person a lecture on morality and Declaration.
another a dollar, and see which will respect you more”. E. Few legislators profit financially from the sale of
The argument above would be more persuasive if it were fireworks.
true that
(A) Personal merit is a characteristic that is not readily 5. “In 1930 there were, on the average, 4.1 persons reported
apparent as living in each household. By 1950 there were 3.5, and by
(B) Lecturing on morality is a strong indication of personal 1970 there were 3.1. Today there are 2.8 persons reported
merit per household, and it appears that the trend toward smaller
(C) Most people profess to value money because they are households is continuing.”
expected to Each of the following, if true, could help to account for this
(D) Giving money to strangers is itself a sign of personal trend EXCEPT:
merit A. The divorce rate has increased
(E) Social behaviour can be investigated with scientific rigor B. The birth rate has decreased
C. Institutional geriatric care has become more widely
2. “Offshore blasting in oil exploration does not hurt fishing; practiced.
blasting started this year, and this year’s salmon catch has D. Tax advantages for claiming resident dependents have
been the largest in a long time.” decreased.
All of the following statements, if true, are valid objections E. The enrollment of children at day-care centers has
to the argument EXCEPT: increased.
(A) The salmon is only one of the many species of fish that
might be affected by the blasts. 6. A cost-effective solution to the problem of airport
(B) The rapid changes of water pressure caused by the congestion is to provide high-speed ground transportation
blasts make salmon mate more frequently. between major cities lying 200 to 500 miles apart. The
(C) The noise of the blasts interferes with the food chain successful implementation of this plan would cost far less
salmon depend on. than expanding existing airports and would also reduce the
(D) Factors that have nothing to do with the well-being of number of airplanes clogging both airports and airways.
salmon may significantly affect the size of one year’s catch. Which of the following, if true, could the proponents of the
(E) Vibrations from the blasts destroy fish eggs. plan above most appropriately cite as a piece of evidence for
the soundness of their plan?
3. A recent spate of launching and operating mishaps with (A) An effective high-speed ground transportation system
television satellites led to a corresponding surge in claims would require more repairs to many highways and mass-
against companies underwriting satellite insurance. As a transit improvements.
result, insurance premiums shot up, making satellites more (B) One-half of all departing flights in the nation’s busiest
expensive to launch and operate. This, in turn, has added to airport head for a destination in a major city 225 miles away.
the pressure to squeeze more performance out of currently (C) The majority of travelers departing from rural airports
operating satellites. are flying to destinations in cities over 600 miles away.
Which of the following, if true, taken together with the (D) Many new airports are being built in areas that are
information above, best supports the conclusion that the cost presently served by high-speed ground-transportation
of television satellites will continue to increase? systems.
(A) Since the risk to insurers of satellites is spread over (E) A large proportion of air travelers are vacationers who
relatively few units, insurance premiums are necessarily very are taking long-distance flights.
high.
(B) When satellites reach their orbit and then fail, the 7. Airline: Newly developed collision-avoidance systems,
causes of failure are generally impossible to pinpoint with although not fully tested to discover potential malfunctions,
confidence. must be installed immediately in passenger planes. Their
(C) The greater the performance demands placed on mechanical warnings enable pilots to avoid crashes.
satellites, the more frequently those satellites break down. Pilots: Pilots will not fly in planes with collision-avoidance
(D) Most satellites are produced in such small numbers systems that are not fully tested. Malfunctioning systems
that no economies of scale can be realized. could mislead pilots, causing crashes.
(E) Since many satellites are built by unwieldy The pilots’ objection is most strengthened if which of the
international consortia, inefficiencies are inevitable. following is true?
(A) It is always possible for mechanical devices to
4. “The Declaration of Independence says that everyone is malfunction.
entitled to life, liberty, and the pursuit of happiness. When (B) Jet engines, although not fully tested when first put
the state legislature forbade the use of fireworks, the entire into use, have achieved exemplary performance and safety
population was denied liberty and happiness. This is records.
dictatorship.” (C) Although collision-avoidance systems will enable pilots
All of the following, if true, are valid objections to the to avoid some crashes, the likely malfunctions of the not-fully-
argument above EXCEPT: tested systems will cause even more crashes.
A. Relatively few people derive happiness from fireworks. (D) Many airline collisions are caused in part by the
B. The legislature does, if indirectly, represent the people. exhaustion of overworked pilots.
81
American Education Aids - LSAT - Logical Reasoning
(E) Collision-avoidance systems, at this state of differences among the bowerbuilding styles of the local
development, appear to have worked better in passenger bowerbird population that has been studied most extensively.
planes than in cargo planes during experimental flights made (B) Young male bowerbirds are inept at bowerbuilding and
over a six-month period. apparently spend years watching their elders before becoming
accomplished in the local bower style.
8. Extinction is a process that can depend on a variety of (C) The bowers of one species of bowerbird lack the towers
ecological, geographical, and physiological variables. These and ornamentation characteristic of the bowers of most other
variables affect different species of organisms in different species of bowerbird.
ways, and should, therefore, yield a random pattern of (D) Bowerbirds are found only in New Guinea and
extinctions. However, the fossil record shows that extinction Australia, where local populations of the birds apparently
occurs in a surprisingly definite pattern, with many species seldom have contact with one another.
vanishing at the same time. (E) It is well known that the song dialects of some songbirds
Which of the following, if true, forms the best basis for at are learned rather than transmitted genetically.
least a partial explanation of the patterned extinctions revealed
by the fossil record? 10. Bats emit sounds and generally use the echoes of these
(A) Major episodes of extinction can result from widespread sounds highly efficiently to detect, locate, and catch their prey.
environmental disturbance that affect numerous different However, it is claimed that the characteristic efficiency of
species. this process is reduced by moths able to hear the sounds
(B) Certain extinction episodes selectively affect organisms emitted by insect-eating bats.
with particular sets of characteristics unique to their species. Which of the following statements, if true, best supports
(C) Some species become extinct because of accumulated the claim above?
gradual changes in their local environments. (A) Those moths that cannot hear the sounds emitted by
(D) In geologically recent times, for which there is no fossil
insect-eating bats live longer on the average than those that
record, human intervention has changed the pattern of
can hear such sounds when both kinds of moth are in an
extinctions.
environment continuously free of such bats.
(E) Species that are widely dispersed are the least likely
(B) Those moth species that hear the sounds emitted by
to become extinct.
insect-eating bats are among the species of insects that are
most likely to be caught by such bats.
9. Male bowerbirds construct elaborately decorated nests,
(C) When a moth changes its speed or direction of flight,
or bowers. Basing their judgment on the fact that different
there is a change in the sound pattern generated by the moth’s
local populations of bowerbirds of the same species build
wing movements.
bowers that exhibit different building and decorative styles,
researchers have concluded that the bowerbirds’ building (D) Moth species that can hear the sounds emitted by
styles are a culturally acquired, rather than a genetically insect-eating bats are less likely to be caught by such bats
transmitted, trait. than are moth species that cannot hear these sounds.
Which of the following, if true, would most strengthen the (E) Moths that are capable of hearing the sounds emitted
conclusion drawn by the researchers? by insect-eating bats differ in their abilities to use evasive
(A) There are more common characteristics than there are action to escape capture by such bats.

SET 2
1. Two decades after the Emerald River Dam was built, 2. Since the routine use of antibiotics can give rise to
none of the eight fish species native to the Emerald River resistant bacteria capable of surviving antibiotic
was still reproducing adequately in the river below the dam. environments, the presence of resistant bacteria in people
Since the dam reduced the annual range of water temperature could be due to the human use of prescription antibiotics.
in the river below the dam from 50 degrees to 6 degrees, Some scientists, however, believe that most resistant bacteria
scientists have hypothesized that sharply rising water in people derive from human consumption of bacterially
temperatures must be involved in signaling the native species infected meat.
to begin the reproductive cycle. Which of the following statements, if true, would most
Which of the following statements, if true, would most significantly strengthen the hypothesis of the scientists?
strengthen the scientists’ hypothesis? (A) Antibiotics are routinely included in livestock feed so
(A) The native fish species were still able to reproduce only that livestock producers can increase the rate of growth of
in side streams of the river below the dam where the annual their animals.
temperature range remains approximately 50 degrees. (B) Most people who develop food poisoning from
(B) Before the dam was built, the Emerald River annually bacterially infected meat are treated with prescription
overflowed its banks, creating backwaters that were critical antibiotics.
breeding areas for the native species of fish. (C) The incidence of resistant bacteria in people has tended
(C) The lowest recorded temperature of the Emerald River to be much higher in urban areas than in rural areas where
before the dam was built was 34 degrees, whereas the lowest meat is of comparable quality.
recorded temperature of the river after the dam was built (D) People who have never taken prescription antibiotics
has been 43 degrees. are those least likely to develop resistant bacteria.
(D) Non-native species of fish, introduced into the Emerald (E) Livestock producers claim that resistant bacteria in
River after the dam was built, have begun competing with animals cannot be transmitted to people through infected
the declining native fish species for food and space. meat.
(E) Five of the fish species native to the Emerald River
are not native to any other river in North America.
82
American Education Aids - LSAT - Logical Reasoning
3. Professor Panovich argues that, since dinosaur teeth (E) In late August, when most of the hummingbirds had
exhibit annual growth rings like those found in the teeth of migrated but hawkmoths were still plentiful, red scarlet gilia
modern crocodiles, dinosaurs, like crocodiles, were cold- plants produced fruit more frequently than they had earlier
blooded. But the teeth of desert bighorn sheep also exhibit in the season.
growth rings, and everyone knows bighorn sheep are warm-
blooded 6. In the 1980 Unites States census, marital status was
Which of the following, if true, strengthens Panovich’s described under one of five categories: single, now married
argument? (but not separated), separated, divorced, widowed. In the
(A) Structures in dinosaurs that are apparently similar to category “separated”, including both those who were legally
structures in modern species of reptiles do not always serve separated and those who were estranged and living apart
the same functions that such structures do in modern species from their spouses, one million more women than men were
(B) Dinosaurs were so diverse that some species could have counted.
been cold-blooded and others warm-blooded Which of the following, if true, provide(s) or contributes to
(C) Several modern species of animals, some cold- blooded, an explanation for this result?
have teeth with annual growth rings I. There are more women of marriageable age than men of
(D) Annual growth rings in warm-blooded animals are marriageable age in the United States.
caused by extreme seasonal variations in temperatures, but II. More of the separated men than separated women in
this could not have been the case for dinosaurs, who lived in the United States could not be found by the census takers
constant climates during the census.
(E) The small number of fossilized remains of dinosaurs III. Many more separated men than separated women left
cannot be assumed to represent the most typical species of the United States for residence in another country.
dinosaurs (A) I only
(B) II only
4. The chemical process of converting sugarcane to ethanol (C) III only
requires more energy than that of converting corn to ethanol. (D) I and II only
It is surprising, then, that most ethanol producers prefer to (E) II and III only
use sugarcane as their raw material.
Which of the following, if true, most helps to explain the 7. Luis has just seen two ravens; therefore, the next bird
preference of the producers? Luis sees will be a raven.
(A) The price of any source of ethanol fluctuates seasonally, Which of the following, if true, most strengthens the
whereas the cost of the conversion process is relatively argument above?
constant. (A) Ravens tend to move in flocks.
(B) The production of ethanol from corn requires less time (B) Ravens generally build their nests at a considerable
than the production of ethanol from sugarcane. distance from the nests of other ravens.
(C) The quality of ethanol produced varies less with the (C) Luis is in California, and ravens are occasionally seen
grade of corn converted than with the grade of sugarcane in California.
converted. (D) Luis has seen ravens in other places than the place
(D) Conversion of sugarcane into processed sugar or other where he is now.
foodstuffs is a lengthy procedure. (E) Luis is actively looking for birds.
(E) Energy for the process of conversion to ethanol can be
supplied by burning residue from the sugarcane but not from 8. The Wheat Farmers Alliance, a political action
corn. committee, attracts 70 percent of its contributors from an
advertisement requesting contributions placed only in the
5. The plant called the scarlet gilia can have either red or September, October, and November issues of the Grange
white flowers. It had long been thought that humming birds, Report, a monthly newsletter for wheat farmers. The
which forage by day, pollinate its red flowers and that president of the Wheat Farmers Alliance, to increase the
hawkmoths, which forage at night, pollinate its white flowers. number of contributors, decides to advertise in each of the
To try to show that this pattern of pollination by colors exists, monthly issues of the Grange Report. She expects that, as a
scientists recently covered some scarlet gilia flowers only at result of the additional Grange Report advertisements, the
night and others only by day: plants with red flowers covered number of contributors will be increased to at least double
at night became pollinated; plants with white flowers covered the present number.
by day became pollinated. Which of the following, if true, would most strongly support
Which of the following, if true, would be additional evidence the president’s expectation?
to suggest that hummingbirds are attracted to the red flowers (A) The September, October, and November advertisements
and hawkmoths to the white flowers of the scarlet gilia? were noticed by fewer than one-third of those readers of
(A) Uncovered scarlet gilia flowers, whether red or white, Grange Report who would be willing to contribute to the
became pollinated at approximately equal rates. Wheat Farmers Alliance.
(B) Some red flowers of the scarlet gilia that remained (B) Wheat farmers traditionally repay their bank loans in
uncovered at all times never became pollinated. late summer after the winter wheat crop has been harvested
(C) White flowers of the scarlet gilia that were covered at and sold.
night became pollinated with greater frequency than white (C) The majority of the readers of the Grange Report with
flowers of the scarlet gilia that were left uncovered. a great enough interest in the Wheat Farmers Alliance to
(D) Scarlet gilia plants with red flowers covered by day contribute have already responded to the advertisements.
and scarlet gilia plants with white flowers covered at night (D) Most of those who contribute to the Wheat Farmers
remained unpollinated. Alliance in the course of a year do so in response to
83
American Education Aids - LSAT - Logical Reasoning
advertisements in the Grange Report. (E) More retired people than working people belong to
(E) The total number of readers of the Grange Report is associations that mobilize their members to take action
stable from year to year. concerning political issues that may effect them

9. The numbers of retired people will grow as the 10. Millions of identical copies of a plant can be produced
population becomes older, and their political power will using new tissue-culture and cloning techniques. If plant
expand even faster than their numbers. propagation by such methods in laboratories proves
Which of the following, if true, LEAST supports the economical, each of the following, if true, represents a benefit
statement about the growth of the political power of retirees? of the new techniques to farmers EXCEPT:
(A) Those retired people who had supported political (A) The techniques allow the development of superior
parties while they were in the work force are more likely to strains to take place more rapidly, requiring fewer generations
support parties after retirement than are those retired people of plants grown to maturity
who had not done no previously (B) It is less difficult to care for plants that will grow at
(B) Retired people devote more time to political activities rates that do not vary widely
than do members of the work force (C) Plant diseases and pests, once they take hold, spread
(C) The adult children of retired people tend to vote in more rapidly among genetically uniform plants than among
ways that support the interests of their parents even though those with genetic variations
such actions may not be in the children’s own economic (D) Mechanical harvesting of crops is less difficult if plants
interest are more uniform in size
(D) Retired people are more likely to vote than are members (E) Special genetic traits can more easily be introduced
of the work force into plant strains with the use of the new techniques

SET 3
1. Y has been believed to cause Z. A new report, noting high-speed racing drivers were living longer than their
that Y and Z are often observed to be preceded by X, suggests predecessors. Race-car safety features that reduced the
that X, not Y, may be the cause of Z. severity of crashes of the kind that formerly took drivers’ lives
Which of the following further observations would best were primarily responsible for the increase in the average
support the new report’s suggestion? age of the Indianapolis 500 competitors.
(A) In cases where X occurs but Y does not, X is usually Which of the following, if true, would be most likely to be
followed by Z part of the evidence used to show that safety features on the
(B) In cases where X occurs, followed by Y, Y is usually cars that protected drivers in major crashes were responsible
followed by Z for the increase in the average age of drivers at the Indianapolis
(C) In cases where Y occurs but X does not, Y is usually race?
followed by Z (A) Younger drivers at high-speed racetracks were involved
(D) In cases where Y occurs but Z does not, Y is usually in major accidents at a slightly higher rate than were older
preceded by X drivers between 1965 and 1980.
(E) In cases where Z occurs, it is usually preceded by X (B) Major accidents on high-speed racetracks occurred at
and Y about the same frequency in the years after 1965 as in the
years before 1965.
2. Mr. Lawson: We should adopt a national family policy (C) The average age of drivers attempting to qualify for
that includes legislation requiring employers to provide paid the Indianapolis 500 decreased slightly between 1965 and
parental leave and establishing government-sponsored day 1980.
care. Such laws would decrease the stress levels of employees (D) Accidents on highways in the United States occurred
who have responsibility for small children. Thus such laws at about the same frequency in the years after 1965 as in the
would lead to happier, better-adjusted families. years before 1965.
Which of the following, if true, would most strengthen the (E) Other safety features, involving the condition of the
conclusion above? racetrack and the uniforms worn by the drivers while driving,
(A) An employee’s high stress level can be a cause of were adopted at Indianapolis between 1965 and 1980.
unhappiness and poor adjustment for his or her family
(B) People who have responsibility for small children and 4. Pharmaceutical companies spend more than ever on
who work outside the home have higher stress levels than research and development; yet the number of new drugs
those who do not patented each year has dropped since 1963. At the same
(C) The goal of a national family policy is to lower the stress time, profits at constant 1963 dollars for the industry as a
levels of parents whole have been steadily increasing.
(D) Any national family policy that is adopted would Which of the following, if true, is the single factor most-
include legislation requiring employers to provide paid likely to explain, at least in part, the three trends mentioned
parental leave and establishing government-sponsored day above for money spent, drugs patented, and profits made?
care (A) Government regulations concerning testing
(E) Most children who have been cared for in day care requirements for novel drugs have become steadily more
centers are happy and well adjusted stringent
(B) Research competition among pharmaceutical
3. The average age and racing experience of the drivers at companies has steadily intensified as a result of a general
the Indianapolis 500 automobile race increased each year narrowing of research targets to drugs for which there is a
between 1965 and 1980. The reason for the increase is that large market
84
American Education Aids - LSAT - Logical Reasoning
(C) Many pharmaceutical companies have entered into information in ways that are irrational in this sense.
collaborative projects with leading universities, while others Any of the following, if true, would provide evidence in
have hired faculty members away from universities by support of the assertions above EXCEPT:
offering very generous salaries (A) People tend to act on new information, independent of
(D) The number of cases in which one company’s its perceived relative merit, rather than on information they
researchers duplicated work done by another company’s already have.
researchers has steadily grown (B) People prefer a major risk taken voluntarily to a minor
(E) The advertising budgets of the major pharmaceutical one that has been forced on them, even if they know that the
companies have grown at a higher rate than their profits have voluntarily taken risk is statistically more dangerous
(C) People tend to take up potentially damaging habits
5. Black Americans are, on the whole, about twice as likely even though they have clear evidence that their own peers
as White Americans to develop high blood pressure. This as well as experts disapprove of such behavior.
likelihood also holds for westernized Black Africans when (D) People avoid situations in which they could become
compared to White Africans. Researchers have hypothesized involved in accidents involving large numbers of people more
that this predisposition in westernized Blacks may reflect than they do situations where single-victim accidents are
an interaction between western high-salt diets and genes that possible, even though they realize that an accident is more
adapted to an environmental scarcity of salt. likely in the latter situations than in the former.
Which of the following statements about present day (E) People usually give more weight to a physician’s opinion
Westernized Black Africans, if true, would most tend to confirm about the best treatment for a disease than they do to the
the researchers’ hypothesis? opinion of a neighbor if they realize that the neighbor is not
(A) The blood pressures of those descended from peoples an expert in disease treatment.
situated throughout their history in Senegal and Gambia,
where salt was always available, are low 8. In 1846 about 80 percent of the towns in New York State
(B) The unusually high salt consumption in certain areas banned the sale of alcoholic beverages. A recent article about
of Africa represents a serious health problem the bans concludes that mid nineteenth-century supporters
(C) Because of their blood pressure levels, most White of the temperance movement were not residents of remote
Africans have markedly decreased their salt consumption rural areas, as has often been asserted; rather, they were
(D) Blood pressures are low among the Yoruba, who, concentrated in centers of economic opportunity.
throughout their history, have been situated far south of Which of the following, if true, best supports the conclusion
Saharan salt mines reached in the article?
(E) No significant differences in salt metabolism have been (A) After 1846 the temperance movement grew rapidly and
found between those peoples who have had salt available it flourished until the turn of the century.
throughout their history and those who have not. (B) Support for the ban on alcohol was strongest in New
York towns where the economy was based on new, growing
6. In 1985 a consumer agency concluded that Xylo brand industries.
bicycles are safer to ride than are Zenon brand bicycles. The (C) Many young New York State farmers supported the
agency based the conclusion on the ratio of the number of ban on alcohol.
rider injuries to the number of riding hours for each brand of (D) The most adamant opponents of the ban included
bicycle from 1981 through 1984. Yet, for identically designed several affluent factory and mill owners.
bicycles manufactured since 1985, the number of rider injuries (E) In New York City, which was a commercial center in
has been twice as great among riders of Xylos as among riders 1846, the sale of alcoholic beverages was not banned.
of Zenons. Therefore, the agency’s conclusion would have been
different for the period since 1985. 9. When people predict that a certain result will not take
Which of the following is an assumption that, if true, place unless a certain action is taken, they believe that they
supports the claim that the agency’s conclusion would have
have learned that the prediction is correct when the action is
been different for the period since 1985?
taken and the result occurs. On reflection, however, it often
(A) For the period since 1985, the number of riding hours
becomes clear that the result admits of more than one
for Zenons totaled at least half the number of riding hours
interpretation.
for Xylos.
Which of the following, if true, best supports the claims
(B) Of all the bicycles ridden in the period since 1985, the
above?
percentage of Xylos ridden was twice the percentage of Zenons
(A) Judging the success of an action requires specifying
ridden.
the goal of the action.
(C) Prior to 1985, Zenon owners were more likely than
(B) Judging which action to take after a prediction is made
Xylo owners to report the injuries they sustained while riding
their bicycles. requires knowing about other actions that have been
(D) In 1985 the agency had miscalculated the ratio for successful in similar past situations.
Xylos, or for Zenons, or for both. (C) Learning whether a certain predictive strategy is good
(E) Soon after the agency had issued its report, consumer requires knowing the result of using that strategy through
demand for Xylos increased more rapidly than did consumer several trials.
demand for Zenons. (D) Distinguishing a correct prediction and effective action
from an incorrect prediction and ineffective action is often
7. People often do not make decisions by using the basic impossible.
economic principle of rationally weighing all possibilities and (E) Making a successful prediction requires knowing the
then making the choice that can be expected to maximize facts about the context of that prediction.
benefits and minimize harm. Routinely, people process
85
American Education Aids - LSAT - Logical Reasoning
10. Mr. Primm: If hospitals were private enterprises Nakia’s attempt to refute Mr. Primm’s claim?
dependent on profits for their survival, there would be no (A) Doctors at teaching hospitals command high salaries
teaching hospitals, because of the intrinsically high cost of (B) Sophisticated, non-routine medical care commands a
running such hospitals. high price
Ms. Nakia: I disagree. The medical challenges provided (C) Existing teaching hospitals derive some revenue from
by teaching hospitals attract the very best physicians. This, public subsidies
in turn, enables those hospitals to concentrate on non-routine (D) The patient mortality rate at teaching hospitals is high
cases. (E) The modern trend among physicians is to become highly
Which of the following, if true, would most strengthen Ms. specialized

SET 4
1. Recent evidence appears to contradict earlier findings today’s world, physics classes attracted few high school
that suggested that those who are physically fit cope better students. So, to attract students to physics classes, the board
with stressful real-life events. Of a group of healthy women, proposed a curriculum that emphasizes principles of physics
those randomly assigned to a ten-week program of aerobic involved in producing and analyzing visual images.
exercises performed no better in laboratory tests simulating Which of the following, if true, provides the strongest reason
stressful situations than did the subgroup assigned to a to expect that the proposed curriculum will be successful in
program without exercise. attracting students?
Which of the following, if true, provides evidence for (A) Several of the fundamental principles of physics are
determining whether physical fitness makes one react better involved in producing and analyzing visual images
to stress? (B) Knowledge of physics is becoming increasingly
(A) Superior reaction to laboratory stress situations was important in understanding the technology used in today’s
found to be more prevalent among women than among men. world
(B) Healthy men, after training six months in weight (C) Equipment that a large producer of photographic
lifting, encountered fewer potentially stressful situations in equipment has donated to the high school could be used in
the subsequent six months. the proposed curriculum
(C) Subjects following a regimen during which they (D) The number of students interested in physics today is
perfected their skills in a variety of relaxation techniques much lower than the number of students interested in physics
found that their lives seemed calmer after they began the 50 years ago
regimen. (E) In today’s world the production and analysis of visual
(D) College students with previous high levels of stressful images is of major importance in communications, business,
life events showed a markedly reduced reaction to such events and recreation
after training in aerobics for six months.
(E) Subjects with a high level of self-esteem more often 4. Small-business groups are lobbying to defeat proposed
engaged in physical-fitness regimens than did a control group federal legislation that would substantially raise the federal
of subjects with average levels of self-esteem. minimum wage. This opposition is surprising since the
legislation they oppose would, for the first time, exempt all
2. Defense Department analysts worry that the ability of small businesses from paying any minimum wage.
the United States to wage a prolonged war would be seriously Which of the following, if true, would best explain the
endangered if the machine-tool manufacturing base shrinks opposition of small-business groups of the proposed
further. Before the Defense Department publicly connected legislation?
this security issue with the import quota issue, however, the (A) Under the current federal minimum-wage law, most
machine tool industry raised the national security issue in small businesses are required to pay no less than the
the petition for import quotas. minimum wage to their employees
Which of the following, if true, contributes most to an (B) In order to attract workers, small companies must
explanation of the machine-tool industry’s raising the issue match the wages offered by their larger competitors, and these
above regarding national security? competitors would not be exempt under the proposed laws
(A) When the aircraft industries retooled, they provided a (C) The exact number of companies that are currently
large amount of work for tool builders. required to pay no less than the minimum wage but that
(B) The Defense Department is only marginally concerned would be exempt under the proposed laws is unknown
with the effects of foreign competition on the machine-tool (D) Some states have set their own minimum wages - in
industry. some cases, quite a bit above the level of the minimum wage
(C) The machine-tool industry encountered difficulty in mandated by current federal law - for certain key industries
obtaining governmental protection against imports on (E) Service companies make up the majority of small
grounds other than defense. businesses and they generally employ more employees per
(D) A few weapons important for defense consist of parts dollar of revenues than do retail or manufacturing businesses
that do not require extensive machining.
(E) Several federal government programs have been 5. Fact 1: Television advertising is becoming less effective:
designed which will enable domestic machine-tool the proportion of brand names promoted on television that
manufacturing firms to compete successfully with foreign viewers of the advertising can recall is slowly decreasing.
toolmakers. Fact 2: Television viewers recall commercials aired first
or last in a cluster of consecutive commercials far better than
3. The local board of education found that, because the they recall commercials aired somewhere in the middle.
current physics curriculum has little direct relevance to Fact 2 would be most likely to contribute to an explanation
86
American Education Aids - LSAT - Logical Reasoning
of fact 1 if which of the following were also true? great deal of attention by the news media and because they
(A) The average television viewer currently recalls fewer enjoy such perquisites as free mail privileges and generous
than half the brand names promoted in commercials he or travel allowances, incumbents enjoy an overwhelming
she saw. advantage over their challengers in elections for the United
(B) The total time allowed to the average cluster of States Congress.
consecutive television commercials is decreasing. Which of the following, if true, best supports the claim
(C) The average number of hours per day that people spend above?
watching television is decreasing. (A) In the last congressional elections, incumbents met with
(D) The average number of clusters of consecutive a larger number of lobbyists than did challengers.
commercials per hour of television is increasing. (B) In the last congressional elections, 98 percent of the
(E) The average number of television commercials in a incumbents in the House of Representatives who were seeking
cluster of consecutive commercials is increasing. reelection won.
(C) Incumbent members of Congress are frequently critical
6. If a physician can be confident in his or her diagnosis of of the amount of attention given to them by the news media.
acute illness, especially when that diagnosis is supported by (D) The support that political action committees provide
repeated contact with the patient during treatment, the to challengers for congressional seats often compensates for
treatment prescribed will be conservative. This is one reason the perquisites enjoyed by incumbent members of Congress.
that treatment for a specific illness usually costs more in (E) Of all incumbent senators surveyed before the last
hospital emergency rooms than in the physicians’ private congressional elections. 78 percent said that their challengers
offices. did not pose a serious great to their chances for reelection.
All of the following, if true, explain why treatment in an
emergency room costs more than treatment in a physician’s 9. Instead of relying on general tax revenue, as it now does,
private office EXCEPT: the government should rely more heavily on passenger fares
(A) The medical equipment in hospital emergency rooms to finance public bus and train service. In order for public
is more sophisticated than in physicians’ private offices transportation to be maintained without cutting service, users
(B) When patients come to physicians’ private offices, the should pay all the operating costs even if these costs should
state of their illness is usually such that physicians have time increase. Such charges would be fair since only users benefit
to try one test or medication at a time in order to monitor the from public transportation.
results Which of the following is a principle on which the position
(C) The variety of illnesses treated by emergency room above could be based?
physicians is much smaller than the variety treated by (A) The number of users of a public service should
physicians in their offices determine the amount of governmental financial support for
(D) The illness of many patients who apply for emergency the service.
room treatment is so severe that physicians apply several (B) The amount of public transportation provided should
remedies at once in hope of obtaining immediate results be dependent on the operating cost of each transportation
(E) In emergency rooms a patient is often attended by service.
nurses, paramedics, and physicians, whereas visits to private (C) If necessary, general taxes should be raised to ensure
offices rarely involve more than two staff members that public transportation services are provided.
(D) The government should provide support from general
7. The garment industry is labor-intensive; the production tax revenue to any transportation industry that has passenger
of garments requires the employment of a relatively large service available to the public.
number of people. The auto industry is capital-intensive; a (E) General tax revenues should not be used to finance
large amount of money is invested in elaborate equipment public services that benefit a limited number of people.
run by a relatively small number of people. If fringe benefits
are not considered, a typical United States garment worker 10. When the manufacturer of Voltage, a major soft drink,
in 1979 earned 46 percent of a typical auto worker’s wages. changed its secret formula last year, the export earnings of
Which of the following, if true, is likely to be among the an island in the Indian Ocean began to fall. This island’s
factors that account for the disparity between auto workers’ only export comprises more than half of the world’s supply of
and garment workers’ wages? vanilla beans. Analysis concluded that the original formula
(A) There is generally less variation among the wages of of Voltage contained vanilla from beans, but the new formula
garment industry workers than among those of auto industry did not.
workers. Which of the following, if true, would most strengthen the
(B) Wage increases in the auto industry have a smaller conclusion drawn by the analysts?
effect on manufacturers’ total costs than do wage increases (A) The vanilla-bean plantings of nearby island were
in the garment industry. beginning to produce crops.
(C) The fringe benefits that auto makers provide for their (B) A new process for synthesizing vanilla was under
employees are more comprehensive than are those provided development in a laboratory in the United States.
for garment workers. (C) The island’s trade agreement, under which the vanilla
(D) The auto industry faces more competition from beans were exported to the country that manufactures
companies outside the United States paying low wages than Voltage, had lapsed.
does the garment industry. (D) Imports of vanilla beans dropped in countries where
(E) The auto industry employs a large total number of Voltage is made.
workers than does the garment industry. (E) There were decreases in sales of several widely sold
products that were known to contain vanilla.
8. Because incumbent members of Congress are given a
87
American Education Aids - LSAT - Logical Reasoning
SET 5
1. Toughened hiring standards have not been the primary television sets per capita have had the lowest incidence of a
cause of the present staffing shortage in public schools. The serious brain disease, mosquito-borne encephalitis. The
shortage of teachers is primarily caused by the fact that, in researchers have concluded that people in these counties stay
recent years, teachers have not experienced any improvement indoors more and thus avoid exposure to the disease.
in working conditions and their salaries have not kept pace The researchers’ conclusion would be most strengthened if
with salaries in other professions. which of the following were true?
Which of the following, if true, would most support the (A) Programs designed to control the size of disease-bearing
claims above? mosquito populations have not affected the incidence of
(A) Many teachers already in the profession would not have mosquito-borne encephalitis
been hired under the new hiring standards. (B) The occupations of county residents affects their risk
(B) Today more teachers are entering the profession with of exposure to mosquito-borne encephalitis more than does
a higher educational level than in the past. television watching
(C) Some teachers have cited higher standards for hiring (C) The incidence of mosquito-borne encephalitis in
as a reason for the current staffing shortage. counties with the largest number of television sets per capita
(D) Many teachers have cited low pay and lack of is likely to decrease even further
professional freedom as reasons for their leaving the (D) The more time people in a county spend outdoors, the
profession. greater their awareness of the dangers of mosquito-borne
(E) Many prospective teachers have cited the new hiring encephalitis
standards as a reason for not entering the profession. (E) The more television sets there are per capita in a county,
the more time the average county resident spends watching
2. “Newborn kittens or monkeys that wear a patch over television
one eye for two weeks after birth never develop normal vision,
even after the patch is removed. There thus appears to be a 5. Verbal patterns in four works known to be written by a
critical early period during which visual experience is certain author were compared to those in a work of uncertain
necessary for normal development of vision.” authorship sometimes attributed to that author. Many
Which of the following, if true, most strengths the argument patterns were studied, including frequency of specific words
above? and recurrence of certain phrases. The questioned work
A. Adult men and women who wear a patch over one eye displayed verbal patterns very similar to those in the other
for two weeks retain normal vision in the unpatched eye. four works, establishing that the same author wrote all five.
B. Newborn animals normally do not have good visual Which of the following, if true, most strengthens the
acuity. conclusion above?
C. Animals two months old are much less affected than (A) No two writers are likely to display similar verbal
are newborns by wearing a patch over one eye for two weeks. patterns in their works.
D. When young children wear dark glasses, they often have (B) Writers from different historical periods sometimes use
difficulty with depth perception. the same words and phrases, but the meanings of such words
E. Some aspects of vision have been proved to be learned, and phrases change over time.
rather than inherited, behavior. (C) Many writers consciously attempt to experiment with
innovative verbal patterns in each new work.
3. Residents of large cities are moving to small towns and (D) A relatively small number of words in any language
small cities in record numbers seeking not only intimacy and occur with great frequency, and those words make up the
peace, but a place where their actions count. They are fed up largest portion of all discourse.
with an ineffectual, unresponsive big government that (E) Word choice is generally considered an insignificant
interferes in their lives and is no longer able to provide the component of an author’s style.
services it has promised.
If one wished to argue that a move to a small town or city 6. For many people in the United States who are concerned
would not solve the problem of the interference of big about the cost of heating homes and businesses, wood has
government, which of the following, if true, would be most become an alternative energy source to coal, oil, and gas.
likely to support that claim? Nevertheless, wood will never supply more than a modest
(A) Most people born in big cities never move away, fraction of our continuing energy needs.
preferring the unique advantages a big city offers Which of the following, if true, does NOT support the claim
(B) The government agencies most often perceived as made in the last sentence in the passage above?
unresponsive are national agencies, whose policies are (A) There are many competing uses for a finite supply of
uniform across the nation wood, and suppliers give the lumber and paper industries a
(C) Over fifty percent of the new arrivals in small townshigher priority than they give individual consumers.
and small cities in the 1970’s were farmers or farm employees (B) Wood produces thick smoke in burning, and its
from rural areas extensive use in densely populated cities would violate federal
(D) Governmental structures in some large cities are moreantipollution guide-lines.
responsive to individual needs than one would imagine (C) There are relatively narrow limits to how far wood can
(E) Long-time residents of small towns believe that the be trucked before it becomes more economical to burn the
image of small-town life as intimate and peaceful is erroneous
gasoline used for transportation instead of the wood.
(D) Most apartment dwellers do not have adequate storage
4. Reviewing historical data, medical researchers in space for the amount of wood necessary to supply energy for
California found that counties with the largest number of heating.
88
American Education Aids - LSAT - Logical Reasoning
(E) Most commercial users of energy are located within spendable income in the 1970’s
range of a wood supply, and two-thirds of United States homes (C) The spendable income of workers declined in the 1950’s
are located outside of metropolitan areas. (D) Supervisors and owners maintained the level of their
spendable income in the 1970s
7. Car radiators have generally been aluminium, not (E) By 1964 there was already a sizable gap between
copper, since the early 1980’s, when many makers of radiators economic classes in the United States
retooled their plants to suit aluminum, then cheaper than
copper. This year aluminum prices rose sharply; yet radiators 9. Technological innovation is widely extolled as the way
are unlikely to revert to copper because carmakers have to guarantee future prosperity. The after-effects of industrial
exploited the much lighter aluminum radiators to design innovation, however, may be far from benign. Revolutionary
better-performing cars. new technologies such as robots may not only eliminate jobs
Which of the following, if true, best supports the view that but also diminish the quality of working life in factories where
the author’s prediction about continued use of aluminum in robots work closely with people.
radiators is correct, but correct for the wrong reason? Which of the following, if true, provides the strongest
(A) For a large segment of the car-buying public, the support for the author’s position?
purchase price is by far the most important criterion in (A) To be effective, robots require close supervision by
choosing a car skilled employees
(B) Leaving aside the cost of the metal itself, the cost of (B) Revolutionary technologies can reduce unemployment
making an aluminum radiator exceeds the cost of making a (C) Robots can do jobs that people find enjoyable
copper radiator (D) Robots relieve people of boring, repetitious work
(C) The great majority of car owners, having once gotten (E) The quality of the finished work depends on the quality
used to better-performing cars, come to value superior of the worker
performance highly
(D) This year’s high aluminum prices reflect a temporary 10. The manner in which an artist is paid profoundly affects
shortage, and prices are expected to collapse, stabilizing at the artwork produced. If the artist is supported by the state
levels below those of the early 1980’s or had potential losses underwritten, the temptation to self-
(E) When makers of radiators switched from copper to indulgence or waste is strong. If the artist has to please the
cheaper aluminum, radiator prices did not drop, because the public in order to earn an income, the pleasure of the public
manufacturers needed to recover the cost of retooling their will dictate artistic style to the artist.
plants. All of the following, if true, support the argument above
EXCEPT:
8. It is clear from trends in the 1970’s that economic classes (A) Lack of a steady income forces artists to become more
in the United States have been growing farther apart from concerned with financial success than with adherence to their
one another rather than becoming more nearly equal. The artistic principles
weekly spendable earnings of private-sector nonsupervisory (B) The availability of federal funds encourages artists to
workers have declined by ten percent since 1970, and by 1979 experiment with materials before completely working out an
had returned to their level of 1964. The claim of equality in idea
the United States is more and more difficult to sustain. (C) Once freed from financial worries, an artist evolves a
Which of the following, if true, would best support the style in response to concerns other than public approval
argument of the passage above? (D) Only an artist who has received public recognition and
(A) The rate of unemployment fluctuated greatly in the support can receive federal funds
1970’s (E) Artists are like other people because they become self-
(B) Public-sector workers also suffered a decline in their indulgent whenever economic circumstances permit

SET 6
1. Recently a number of writers have proposed that our (E) Any extraterrestrial civilization advanced enough to
ancestors were influenced by visits from extra-terrestrials have interplanetary space travel during the time of our
and that these visitors were the builders of such monuments ancestors could easily find means of communicating with
as the pyramids of Egypt. them
Which of the following statements, if true, would constitute
the strongest evidence in support of the proposal described 2. Forestry experts are beginning to question the policy of
above? attempting to extinguish all forest fires instead of allowing
(A) The level of technology achieved by our ancestors who them to run their course. Observations about the results of
designed and built the monuments in question was actually forest fires have indicated that their effects may be, in the
much higher than it was formerly believed to be long run, beneficial to the overall ecology of the forest.
(B) Monuments such as the pyramids of Egypt are very Which of the following could most logically be one of the
often positioned so that their prominent design features point “observations” referred to in the passage above?
to heavenly bodies (A) More than half of all forest fires are the result of natural
(C) Stories about beings from heaven appear in the causes such as lightning rather than the result of human
mythologies of cultures that could have had no interchange carelessness
with one another (B) More people are killed or seriously injured each year
(D) Modern builders cannot conceive of a way to duplicate attempting to fight forest fires than would be killed or injured
the monuments without resorting to the most recent if the fires were allowed to burn themselves out
technological innovations (C) Frequent small forest fires prevent the accumulation
89
American Education Aids - LSAT - Logical Reasoning
of highly flammable dead brushwood and timber, which the loss of a few members of the species is not significant.
can support massive, uncontrollable fires (B) Other species of mammals have increased their
(D) The animal and plant life destroyed by forest fires knowledge of dangers through the experience of generation
seldom represent endangered species after generation of animals.
(E) Study of tree tissues indicates that forest fires were a (C) In other species of mammals, hunters can readily
regular occurrence in the times preceding human occupation distinguish between males and females or between young
of forest areas animals and adults.
(D) In other species of mammals, some members of the
3. Japanese factory workers are guaranteed life-time jobs, species are genetically better equipped to withstand a disease
bonuses paid on the basis of productivity and corporate profits, or event that destroys other members of the species.
and a wage rate that is not attached to a particular job. (E) Other species of mammals have retained habits of
Paradoxically, these guarantees do not discourage factory caution and alertness because they have not been protected
owners from introducing labor-saving machinery. Such as endangered species.
innovations are to the factory owners’ advantage despite the
fact that the owners must protect the wages of their workers. 6. A group of paintings made approximately 15,000 years
Which of the following, if true, logically explains why the ago in a cave in the Loire River valley in what is now France
introduction of labor-saving machinery is advantageous to depicts a number of different animals. One of the animals
factory owners? depicted seems to resemble the chiru, a rare antelope of the
(A) Before a Japanese factory worker is hired, he or she Himalayas.
must present a record of his or her previous productivity. Which of the following if true, best supports the hypothesis
(B) Labor-saving machinery increases productivity, thus that in painting the animal that resembles a chiru, the cave
yielding profits that more than cover the cost of retraining artist painted a chiru with which she or he was familiar?
workers for other jobs. (A) There are numerous representations of imaginary
(C) The purchase and maintenance of new machinery adds animals in cave paintings of similar age
significantly to the final cost of the goods produced. (B) Fossilized remains of a chiru, approximately 16,000
(D) Factory workers demand a change of procedure in the years old, have been found at the northern end of the valley
routine tasks they perform. (C) The cave that contains the depiction of an animal that
(E) Limited competition exists among Japanese factories resembles a chiru contains stylized representations of plant
for consumer markets. life
(D) Older caves from the same region contain no
4. The attitude that it is all right to do what harms no one representations of animals that resemble a chiru
but oneself is usually accompanied by a disregard for the (E) The antlers of the animal in the painting are longer
actual interdependence of people. Destroying one’s own life than those of the mature Himalayan chiru
or health means not being available to help family members
or the community; it means, instead, absorbing the limited 7. Color-tinting of movies originally photographed in black-
resource of the community for food, health services, and and-white is inappropriate. Hundreds of the original artistic
education without contributing fully to the community. choices would have been made differently had these movies
Which of the following, if true, most strongly supports the been filmed in color. Lighting on the sets of these movies was
view expressed above? arranged to make highlights and shadows look right in black-
(A) The cost of avoidable accidents and illnesses raises and-white.
health insurance rates for everyone. Which of the following claims, if substituted for the claim
(B) Harm to one person can result in an indirect benefit, about the lighting of movie sets, would lend the same type of
such as the availability of work in health-related fields, to support to the argument above?
others. (A) An important part of what gives these movies their
(C) Life would be dull if it were necessary to abstain from identity is the result of the black-and-white format.
all of the minor pleasures that entail some risk of harm to a (B) Color film would have better captured the film-makers’
person who indulges in them. intentions.
(D) The contribution a person makes to the community (C) Color film is superior to black-and-white film for
cannot be measured by that person’s degree of health. making movies.
(E) The primary damage caused by the consumption of (D) Makeup for the actors was applied so as to look best in
alcohol, tobacco, and unauthorized drugs is done to the person black-and-white films.
who uses those substances. (E) The choice of black-and-white film over color film is
entirely a matter of taste.
5. The population of elephant seals, reduced by hunting to
perhaps a few dozen animals early in this century, has soared 8. In Borkland University, scholarship stipends worth $4.2
under federal protection during the last few decades. However, million were unclaimed last year because several scholarship
because the species repopulated itself through extensive programs attracted no applicants who met the programs’
inbreeding, it now exhibits a genetic uniformity that is almost qualifying criteria. This is an egregious waste of funds in a
unparalleled in other species of mammals, and thus it is in country where thousands of promising students each year
far greater danger of becoming extinct than are most other cannot afford tuition. Qualifying criteria for those
species. scholarships, therefore, should be revised.
Given the information in the passage above, which of the Which of the following, if true about Borkland, most
following is most likely the reason that other species of strongly supports the conclusion drawn above?
mammals are less likely than elephant seals to become extinct? (A) Many scholarships are unclaimed not because their
(A) Other species of mammals have large populations, so qualifying criteria are too restrictive but because of the
90
American Education Aids - LSAT - Logical Reasoning
scholarships’ availability. (B) An increase in the number of crimes committed by
(B) Because of inflation, the stipends offered by certain employees of the state
scholarship programs now appear less attractive to potential (C) A gradual decrease in the median age of judges in the
applicants than they did when they were first offered. state
(C) A significant number of scholarship programs awarded (D) The overcrowding of prisons in the state
all of their available scholarship stipends last year, yet most (E) The passage of drunken-driver legislation in other
of those programs have very restrictive qualifying criteria. states
(D) Certain scholarship programs accept applications only
from students enrolled in specific major fields of study, but 10. Many geologists theorize that the trail of volcanic
Borkland’s universities no longer offer courses in the fields craters and cinder cones along the Snake River plain of
specified by many of those programs. southern Idaho was produced as the North American
(E) Several scholarship programs have found it impossible continent slid westward over a stationary “plume,” a vertical
to revise their qualifying criteria without engaging in lengthy channel through which molten rock rose intermittently from
and costly legal proceedings. the Earth’s core to burst through its crust.
Which of the following, if true, tends to support the
9. Approximately 5,000 people who have been convicted of geologists’ theory of how the trail was produced?
nonviolent crimes in the state have been given community- (A) The largest craters and cinder cones are on the eastern
service sentences instead of prison sentences. These offenders margin of the trail.
perform services commensurate with their training and skills, (B) The most violent volcanic activity apparently occurred
from scrubbing floors to conducting research for the state. at the western margin of the trail.
The community service program, which began in 1979, has (C) The craters and cinder cones are evenly spaced
grown immensely as a result of drunken-driver legislation throughout the extent of the trail.
enacted a few months ago. (D) The newest craters and cinder cones are on the eastern
The introduction of the community-service program in 1979 margin of the trail.
was most probably prompted by which of the following, all of (E) The craters and cinder cones on the western margin
which occurred during the 1970’s? of the trail generally took longer to form than did those on
(A) A decrease in the number of violent crimes in the state the eastern margin.

91
Chapter 7
'COMPLETION' QUESTIONS
In another type of question that is occasionally asked un- (C) currently available models are user-friendly enough
der Logical Reasoning in LSAT, a phrase or clause at the end to suit them
of the last sentence in the given narration is omitted, and (D) the people promoting personal computers use them in
you are asked to spot the best alternative among the choices their own homes
that will logically complete that sentence. (E) they have enough sensible uses for a personal computer
to justify the expense of buying one
We are giving below three such questions along with their
analysis. Analysis
The clue to this question lies in the word ‘premature’ and
Example 1 the phrase ‘logically prior issue’.
Which of the following best completes the passage below? The emphasis in advertisements promoting the sale of
In a survey of job applicants, two-fifths admitted to being personal computers is their ‘user-friendliness’.
at least a little dishonest. However, the survey may What is the question that any potential purchaser of any
underestimate the proportion of job applicants who are item will ask himself before deciding to buy it? Whether the
dishonest, because ———. item will be of any sensible use to him so as to justify the
(A) some dishonest people taking the survey might have expenditure on its purchase, obviously.
claimed on the survey to be honest If personal computers cannot be of any sensible use to a
(B) some generally honest people taking the survey might particular person, the mere fact that they are user-friendly is
have claimed on the survey to be dishonest an irrelevant consideration for him for deciding to purchase
(C) some people who claimed on the survey to be at least a it. It is (E) which points this out, and is the answer.
little dishonest may be very dishonest (A) is not the answer, because ‘servicing’ the computer
(D) some people who claimed on the survey to be dishonest comes after purchasing it, and is not a prior issue before the
may have been answering honestly purchase.
(E) some people who are not job applicants are probably The given paragraph does not relate exclusively to ‘more
at least a little dishonest user-friendly’ personal computers (and hence the more costly
ones), but to all personal computers (including the less
Analysis expensive ones), which are also claimed to be more user-
The first sentence says that 2/5 of the applicants admitted friendly than other models of currently available computers.
that they were at least a little dishonest. So, (B) is not a logically prior issue.
We are asked to spot that choice which is in the nature of (C) is not the answer since, even if the currently available
a reason that could lead us to suspect that the proportion of models are user-friendly enough, the claim that personal
the dishonest applicants is, in fact, more than 2/5. computers are more user-friendly can be a logical and
This reason must obviously be one that implies that at legitimate sales-pitch.
least some of the remaining 3/5 of the applicants are also at (D) is irrelevant, since noone would decide to buy an item
least a little dishonest, though they have claimed to be honest. merely because the person who is marketing it claims that
(In their cases, claiming to be honest would itself be on act of he uses it himself!
dishonesty!)
It is (A) which states this, and is the answer. Example 3
(B) and (C) are wrong, because honest people would not It has been argued that the consumer has benefited from
claim to be dishonest, and even if they had wrongly claimed agricultural technology in the United States. Consumers are
so, it would reduce and not increase the proportion of really spending, on the average, a decreasing proportion of their
dishonest people. income on food. But the demand for food, especially in
(D) would only confirm the proportion of dishonest persons prosperous countries, is virtually insensitive to income, so
as 2/5, and would not result in its underestimation. that as real incomes rise, the amount spent on food becomes
(E) refers to persons who had not applied for jobs, and a smaller proportion of expenditure. Therefore, in order to
would therefore result in neither underestimation nor judge whether consumers have really benefited from
overestimation of the proportion of dishonest persons among agricultural technology, it is necessary to know whether —
the job-applicants. ——.
Which of the following is the most logical completion of the
Example 2 passage above?
Which of the following best completes the passage below? (A) incomes will continue to rise
Sales campaigns aimed at the faltering personal computer (B) the proportion of income spent on food has declined
market have strongly emphasized ease of use, called user- more sharply than rises in real income warrant
friendliness. This emphasis is oddly premature and irrelevant (C) technological costs constitute a substantial portion of
in the eyes of most potential buyers, who are trying to address the cost of food production
the logically prior issue of whether ———. (D) the quantity of food produced by American farmers
(A) user-friendliness also implies that owners can service has increased
their own computers (E) consumers will increase the proportion of their income
(B) personal computers cost more the more user-friendly spent on food as food prices rise
they are
92
AAmerican Education Aids - LSAT - Logical Reasoning
Analysis to 10% whether or not there has been technological improve-
This is a rather tricky question, which is easy to answer if ment in agriculture. In fact, even if the prices of food articles
you recognize that the first two sentences give a particular increase by 50% during this period, forcing this person to
argument, but the third sentence raises a doubt on the validity spend $7500 on food, the proportion of expenditure on food
of that argument. would be only 15% as against 20% earlier. Therefore, the
The first two sentences together mean that consumers are mere fact that the proportion of the expenditure on food to
today spending a lesser proportion of their earnings on food,
total income has come down does not necessarily mean that
and that this must be because of reduction in the prices of
the prices of food items have come down. Therefore, unless it
food articles due to advances of agricultural technology.
is shown that the proportion of expenditure on total income
The third sentence point out the fallacy in this argument.
It says that, at least in affluent countries, the consumption has come down even after adjusting for the growth rate in
of food by a person does not increase with the increase in his total income, we cannot say that the technological develop-
income. (If, for example, a person with an annual income of ment in agriculture has benefited the consumer. This is what
$25,000 spends $5,000 on food and his income increase to is stated in (B), which is the answer.
$50,000, his consumption of food articles does not show a None of the other choices is relevant to the doubt that has
corresponding increase. The proportion of expenditure on food been raised on the validity of the argument in the first two
to the total income, which was 20% initially, would come down sentences.

This type of questions is rather infrequent in LSAT. We are giving 10 questions of this type in the
following pages

93
AAmerican Education Aids - LSAT - Logical Reasoning
SET 1
1. Which of the following best completes the passage given? (B) provide less secure employment for their journalists
When a project is failing and should be terminated, plan than do government-owned new media
to bring in a new manager. New managers are more likely to (C) tend to be more prevalent in countries where many
terminate the project than are the original managers because other industries are also privately owned
———. (D) are likely to be less dependent on a single source of
(A) the project may have failed for reasons that the original revenue than are government-owned news media
manager could not have foreseen (E) generally rely on revenue derived from carrying
(B) organizations tend to reward managers who can advertising by large, politically influential corporations
overcome problems
(C) managerial decisions to terminate a project should 5. George Orwell said that the novelist Henry Miller wrote
depend on the likelihood of the project’s eventual success the way many ordinary people talked. Like them, Miller
(D) the original managers were not necessarily able to shrugged off the contradictions of life, but he did so with the
overcome problems caused by external events over which they piercing clarity that was revolutionary. He saw reality from
had no control the inside. By the time he died at the age of eighty-eight,
(E) the new managers have no need to justify the earlier however, Miller’s loathing of politics and his celebration of
decision to maintain the project private passions were widely shared by the middle class. In
the end ———.
2. Which of the following best completes the passage below? Which of the following provides the most logical completion
People buy prestige when they buy a premium product. for the blank above?
They want to be associated with something special. Mass- (A) Miller’s revolutionary vision became conventional
marketing techniques and price-reduction strategies should attitude.
not be used because ———. (B) Orwell himself become complaisant.
(A) affluent purchasers currently represent a shrinking (C) the contradictions of life had been all but reconciled.
portion of the population of all purchasers (D) Orwell was wrong.
(B) continued sales depend directly on the maintenance of (E) ordinary people chose to ignore Miller’s piercing vision
an aura of exclusivity of reality.
(C) purchasers of premium products are concerned with
the quality as well as with the price of the products 6. Choose the best completion of the paragraph below:
(D) expansion of the market niche to include a broader The Spanish painter El Greco is known for the elongated,
spectrum of consumers will increase profits stretched-out forms of the objects and figures he painted.
(E) manufacturing a premium brand is not necessarily Because the distortion of the subject matter is consistent
more costly than manufacturing a standard brand of the same throughout his works, art historians have suggested that El
product Greco suffered from astigmatism, a defect of the eye that
produces a distorted, vertically elongated image. This
3. Which of the following best completes the passage below? explanation cannot be valid, however, because if El Greco
At a recent conference on environmental threats to the had been an astigmatic painter who attempted to capture
North Sea, most participating countries favored uniform his subjects in the same proportions that his eye perceived,
controls on the quality of effluents, whether or not specific the resulting images on the canvas would appear to ———.
environmental damage could be attributed to a particular (A) overestimate width relative to height
source of effluent. What must, of course, be shown, in order (B) overestimate height relative to width
to avoid excessively restrictive controls, is that ———. (C) alter the shapes of subjects randomly
(A) any uniform controls that are adopted are likely to be (D) produce figures that are unrecognizable
implemented without delay (E) reproduce the actual proportions
(B) any substance to be made subject to controls can
actually cause environmental damage 7. Which of the following best completes the passage below?
(C) the countries favoring uniform controls are those The computer industry’s estimate that it loses millions of
generating the largest quantities of effluents dollars when users illegally copy programs without paying
(D) all of any given pollutant that is to be controlled for them is greatly exaggerated. Most of the illegal copying
actually reaches the North Sea at present is done by people with no serious interest in the programs.
(E) the environmental damage already inflicted on the Thus, the loss to the industry is much smaller than estimated
North Sea is reversible because ———.
(A) many users who illegally copy programs never find any
4. Which of the following, if true, best completes the use for them
argument below? (B) most of the illegally copied programs would not be
In questioning the objectivity of government-owned news purchased even if purchasing them were the only way to
media, critics often observe that such media are usually obtain them
financially dependent on the government. The point is that (C) even if the computer industry received all the revenue
one cannot take an objective view of one’s patron. However, it claims to be losing, it would still be experiencing financial
the same point applies analogously in the case of privately difficulties
owned news media, because privately owned news media — (D) the total market value of all illegal copies is low in
——. comparison to the total revenue of the computer industry
(A) are likely to have less direct access to sources of news (E) the number of programs frequently copied illegally is
in government than are government-owned news media low in comparison to the number available for sale
94
AAmerican Education Aids - LSAT - Logical Reasoning
8. Which of the following best completes the passage below? (C) in times of high inflation, the interest paid to depositors
Established companies concentrate on defending what they by banks can actually be below the rate of inflation.
already have. Consequently, they tend not to be innovative (D) at any one time, a commercial bank will have a single
themselves and tend to underestimate the effects of the rate of interest that it will expect all of its individual borrowers
innovations of others. The clearest example of this defensive to pay.
strategy is the fact that ———. (E) the potential return on investment in a new company
(A) ballpoint pens and soft-tip markers have eliminated is typically lower than the potential return on investment in
the traditional market for fountain pens, clearing the way a well-established company.
for the marketing of fountain pens as luxury or prestige items.
(B) a highly successful automobile was introduced by the 10. Which of the following best completes the passage below?
same company that had earlier introduced a model that had Cars fueled by methanol have a much lower level of
been a dismal failure. emissions of pollutants such as carbon monoxide and
(C) a once-successful manufacturer of slide rules reacted environmentally harmful hydrocarbons than gasoline-fueled
to the introduction of electronic calculators by trying to make cars do. Methanol fuel does produce somewhat higher
better slide rules. formaldehyde emissions than gasoline does. Nevertheless, a
(D) one of the first models of modern accounting machines, methanol-powered car actually produces less atmospheric
designed for use in the banking industry, was purchased by a formaldehyde pollution than a comparable gasoline-powered
public library as well as by banks. car because ———.
(E) the inventor of a commonly used anesthetic did not (A) compared to carbon monoxide and some hydrocarbons
intend the product to be used by dentists, who currently produced by gasoline-powered cars, formaldehyde pollution
account for almost the entire market for that drug. is not a serious threat to the environment.
(B) the technical difficulties involved in mass-producing
9. Which of the following best completes the passage below? methanol-powered cars will prevent them from seriously
The more worried investors are about losing their money, competing with gasoline-powered cars for several years.
the more they will demand a high potential return on their (C) gasoline-powered cars are required by United States
investment: great risks must be offset by the chance of great law to be equipped with catalytic converters that reduce
rewards. This principle is the fundamental one in determining emissions of many pollutants.
interest rates, and it is illustrated by the fact that ———. (D) measuring a car’s emissions is generally an accurate
(A) successful investors are distinguished by an ability to method of assessing that car’s contribution to atmospheric
make very risky investments without worrying about their pollution.
money. (E most formaldehyde pollution generated by gasoline-
(B) lenders receive higher interest rates on unsecured loans powered cars results from the photochemical conversion of
than on loans backed by collateral. hydrocarbon emissions into formaldehyde in the atmosphere.

95
Chapter 8
'EVIDENCE ANALYSIS' QUESTIONS
Another type of questions under Logical Reasoning expects (C) Subjects following a regimen during which they per-
you to carefully analyse the evidence presented in the narra- fected their skills in a variety of relaxation techniques found
tion and, on that basis, answer the question given below. that their lives seemed calmer after they began the regimen.
There are no standard forms for the 'evidence analysis' (D) College students with previous high levels of stressful
questions, and the wording will depend upon the evidence life events showed a markedly reduced reaction to such events
presented in the narration. after training in aerobics for six months.
(E) Subjects with a high level of self-esteem more often
Let us now see a few examples. engaged in physical-fitness regimens than did a control group
of subjects with average levels of self-esteem.
Example 1
The claim that learning computer programming is a sure Analysis
way to a bright future is analogous to the contention, popular What we have been asked to do is to spot that choice which
a few years ago, that if one wanted a successful career, one can provide evidence for determining whether physical fitness
should study law. Now, of course, there are more law students makes one react better to stress.
graduating than the market can absorb. We should therefore look for that statement among the
The point of the analogy above is that choices which cites an instance of a person’s capacity to face
(A) lawyers are making increasing use of computers in situations of stress having either increased or decreased after
their work a period of physical fitness training.
(B) computer programmers will increasingly need the It is (D) which gives such a specific instance, and is the
services of lawyers answer.
(C) there will soon be more jobs for lawyers than there are (B) also looks like a plausible answer, but is wrong because
now it says that the men who underwent a weight-lifting training
(D) there will soon be more programming students program for six-months encountered fewer stressful situations
graduating than there are jobs for them during a subsequent period. This does not help us to state
(E) graduating law students and programming students whether the weight lifting program improved their capacity
will soon be competing with each other for the same jobs to cope better with a stressful situation when they actually
encountered it.
Analysis
What the author of the passage says that, a few years ago, Example 3
the popular view was that, in order to have a successful career, Approximately two hundred brands of personal computers
one should study law. The result is that there are more law are being manufactured, but we currently limit our inventory
graduates now than the market can absorb. to only the eight most popular brands. We plan to increase
He compares the currently popular view that learning greatly the number of computers we sell by expanding our
computer programming will lead one to a bright career to inventory to include the ten best-selling brands.
this popular view regarding law education a few years ago. Which of the following, if true, points out a major weakness
Obviously, what the author implies by the analogy is that of the plan above?
this is likely to lead to the result that, quite soon, there will (A) The capabilities of the three most popular personal
be more programming graduates than the market can absorb. computers tend to be approximately equivalent, with no brand
It is (D) which states this, and is the answer. having consistent superiority in all respects.
You can easily notice that none of the other choices is (B) The seven most popular brands of personal computers
relevant to the analogy sought to be drawn by the author account for almost all computers sold.
between the craze for computer education now and a similar (C) As the users of personal computers become more
craze for law education a few years back. sophisticated, they are more willing to buy less well-known
brands of computers.
Example 2 (D) Less popular brands of computers often provide less
Recent evidence appears to contradict earlier findings that profit to the retailer because prices must be discounted to
suggested that those who are physically fit cope better with attract customers.
stressful real-life events. Of a group of healthy women, those (E) The leading brand of personal computer has been losing
randomly assigned to a ten-week program of aerobic exercises sales to less popular brands that offer similar capabilities for
performed no better in laboratory tests simulating stressful less money.
situations than did the subgroup assigned to a program
without exercise. Analysis
Which of the following, if true, provides evidence for The shop is now stocking and selling the eight most popular
determining whether physical fitness makes one react better brands of computers. The owner says that, by adding the next
to stress? two most popular brands also to his inventory, he hopes to
(A) Superior reaction to laboratory stress situations was greatly increase the number of computers sold by him.
found to be more prevalent among women than among men. We have been asked to identify the choice which would
(B) Healthy men, after training six months in weight point out a major weakness in this plan.
lifting, encountered fewer potentially stressful situations in We should therefore look for that choice which implies that
the subsequent six months. adding the additional two lower-ranking brands is not likely
96
American Education Aids - LSAT - Logical Reasoning
to make much difference to the overall sales of the shop. other choices.
It is (B) which is the answer because, if it were true, even
the 8th most popular brand already stocked is not contributing Example 5
much to the sales of the shop, and therefore the 9th and 10th A person who agrees to serve as mediator between two
most popular brands will add even less to the overall sales warring factions at the request of both abandons by so
figure of the shop. It will therefore belie the hopes of the owner agreeing the right later to take sides. To take sides at a later
that he can thereby ‘greatly increase’ the sales of computers point would be to suggest that the earlier presumptive
in his shop. So (B) is the answer. impartiality was a sham.
(A) talks of the first three most popular computers, and The passage above emphasizes which of the following points
has no nexus to the sale of the 9th and 10th most popular about mediators?
computers, and can be discarded. (A) They should try to form no opinions of their own about
(C) can be ruled out because, if it were true, the shop must any issue that is related to the dispute.
stock the least popular among the 200 brands of computers, (B) They should not agree to serve unless they are
and its adding the 9th and 10th most popular computers will committed to maintaining a stance of impartiality.
not greatly add to its sales. (C) They should not agree to serve unless they are equally
(D) is wrong because the owner’s stated aim is to increase acceptable to all parties to a dispute
the number of computers sold by him, and not to increase his (D) They should feel free to take sides in the dispute right
profits. from the start, provided that they make their biases publicly
(E) talks of the most popular brand of personal computer known.
losing its market to cheaper and less popular brands. If these (E) They should reserve the right to abandon their
preferred brands are already among the next 7 most popular impartiality so as not to be open to the charge of having been
brands, then the addition of the 9th and 10th most popular deceitful.
brands to the shop’s inventory will not greatly add to its sales. Analysis
So, (E) is not the answer. In the first sentence, the author says that, while agreeing
to act as a mediator between two warring parties, the
Example 4 mediator voluntarily gives up his right to take sides later. He
The major goal of physical education programs in schools buttresses his statement by pointing out that reserving the
is to help all children become physically fit. But only a small right to take sides at a later point would, in fact, mean that
proportion of children ever participate in team sports. the person was never impartial from the beginning.
Moreover, team sports usually do less to encourage fitness in We have been asked to spot that choice which can be
participants than do physical education programs that focus considered to be the author’s emphasis about the mediators.
directly on aerobic exercise. (A) is wrong because the very role of a mediator is to arrive
The considerations above, if true, could be used most at an opinion after hearing both sides dispassionately and
effectively to argue against weighing all the evidence put forth by them, and give that
(A) the use of in-school physical education programs to opinion as his judgment on the dispute.
encourage lifelong fitness habits in students (B) says that a person should not agree to serve as a
(B) the participation by young children in community mediator unless he is committed to maintain his stance of
sports teams impartiality (till the end). This is exactly what the author
(C) schools’ relying heavily on aerobic exercise programs says in the passage, and is the answer.
to help all children become physically fit The passage relates only to those persons who agree to
(D) the use of a large part of a school’s physical education serve as mediators between two warring factions at the request
curriculum for team sports of both parties. (C) refers to situation when a person is not
(E) the use of team sports in schools as an occasional acceptable as mediator to at least one of the parties, and is
activity for talented athletes irrelevant to the passage.
(D) is wrong because a person who makes known his bias
Analysis in favour of one of the parties to a dispute, will not be invited
Having first said that the major goal of physical education by the other party to be a mediator.
program in schools is to help all children become physically (E) is precisely what is objected to by the author of the
fit, the author points out that team sports usually do less to passage. In fact, he says that any person who reserves the
encourage fitness in participants than other physical right to take sides at a later date could not have been impartial
education programs that focus directly on aerobic exercise. even in the beginning, and any pretended impartiality by him
If this contention is valid, one can use it effectively against would have been deceitful (sham). So, (E) is wrong.
the plan of using a major part of a school’s physical education
curriculum for team sports (as against aerobic exercises).So, The next two pages contain 10 questions of this type whch
(D) is the answer. are similar in level of difficulty to those asked in LSAT in the
You can easily verify that the given information cannot be last few years.
used effectively to argue against the statements in any of the

97
American Education Aids - LSAT - Logical Reasoning
SET 1
1. The people who are most at risk of contracting a (A) Consumers would continue to discard even recyclable
particular type of influenza are those who have never containers
previously contracted that type of influenza and have never (B) Returning bottles is an inconvenience for consumers
been vaccinated for it. Cases of type A influenza have been (C) The extra expense of recycling would raise beverage
recently diagnosed again in the United States. Type A prices
influenza was last found in the United States in 1950. (D) Recycling would harm the glass and plastic industries
Which of the following groups of people have the LEAST that make containers
risk of contracting type A influenza? (E) Recycling containers is only a partial answer to the
(A) People who were not vaccinated for any type of problem of disposal of solid waste
influenza in 1950 and did not contract any type of influenza
(B) People who have previously contracted a different type 5. In recent years shrimp harvests of commercial fishermen
of influenza, but not type A in the South Atlantic have declined dramatically in total
(C) People who were vaccinated for a different type of weight. The decline is due primarily to competition from a
influenza in 1950, but not for type A growing number of recreational fishermen, who are able to
(D) People who had type A influenza in 1950, but were not net young shrimp in the estuaries where they mature.
vaccinated for it Which of the following regulatory actions will most likely
(E) People born after 1950 who were exposed to different help increase the shrimp harvests of commercial fishermen?
types of influenza, but who never contracted any type of (A) Requiring commercial fishermen to fish in estuaries
influenza (B) Limiting the total number of excursions per season for
commercial fishermen
2. Insurance Company X is considering issuing a new policy (C) Requiring recreational fishermen to use large-mesh
to cover services required by elderly people who suffer from nets in their fishing
diseases that afflict the elderly. Premiums for the policy must (D) Putting an upper limit on the size of the shrimp
be low enough to attract customers. Therefore, Company X recreational fishermen are allowed to catch
is concerned that the income from the policies would not be (E) Allowing recreational fishermen to move out of
sufficient to pay for the claims that would be made. estuaries into the South Atlantic
Which of the following strategies would be most likely to
minimize Company X’s losses on the policies? 6. Infection is the biggest threat to the life of a burn patient.
(A) Attracting middle-aged customers unlikely to submit The skin, the body’s natural barrier against bacteria, is
claims for benefits for many years damaged or gone in the burned areas. The bacteria that are
(B) Insuring only those individuals who did not suffer any a threat are unpredictable in both variety and number.
serious diseases as children Moreover, those found affecting any one patient may change
(C) Including a greater number of services in the policy completely from one day to the next. The standard treatment,
than are included in other policies of lower cost therefore, is the administration of broad-spectrum antibiotics.
(D) Insuring only those individuals who were rejected by Considering only the information given about burn patients
other companies for similar policies in the passage above, which of the following is most likely to
(E) Insuring only those individuals who were wealthy enhance the effectiveness of the standard treatment of a burn
enough to pay for the medical services. patient?
(A) Keeping the patient in an air-conditioned room until
3. Neither a rising standard of living nor balanced trade, recovery is assured
by itself, establishes a country’s ability to compete in the (B) Keeping the areas affected by burns as dry as possible
international marketplace. Both are required simultaneously (C) Continuously monitoring the patient’s vital signs with
since standards of living can rise because of growing trade electronic equipment
deficits and trade can be balanced by means of a decline in a (D) Feeding the patient a diet extra rich in calories
country’s standard of living. (E) Keeping the patient in a maximally sterile environment
If the facts stated in the passage above are true, a proper
test of a country’s ability to be competitive is its ability to 7. The cost of producing radios in Country Q is ten present
(A) balance its trade while its standard of living rises less than the cost of producing radios in Country Y. Even
(B) balance its trade while its standard of living falls after transportation fees and tariff charges are added, it is
(C) increase trade deficits while its standard of living rises still cheaper for a company to import radios from Country Q
(D) decrease trade deficits while its standard of living falls to Country Y than to produce radios in Country Y.
(E) keep its standard of living constant while trade deficits The statement above, if true, best supports which of the
rise following assertions?
(A) Labor costs in Country Q are ten percent below those
4. Under laws requiring the use of recyclable containers, in Country Y.
beverages are sold only in containers that can be reused. (B) Importing radios from Country Q to Country Y will
Those who object to such laws point out that collecting eliminate ten percent of the manufacturing jobs in Country
reusable bottles requires gasoline, and washing them requires Y.
water. However, it takes less water to wash them than it (C) The tariff on a radio imported from Country Q to
does to make throwaway bottles. This saving in water exceeds Country Y is less than ten percent of the cost of manufacturing
the cost of the gasoline needed to transport empty bottles. the radio in Country Y.
The passage above most directly answers which of the (D) The fee for transporting a radio from Country Q to
following objections to recyclable beverage containers? Country Y is more than ten percent of the cost of
98
American Education Aids - LSAT - Logical Reasoning
manufacturing the radio in Country Q. To discourage the buying and selling of free-travel coupons,
(E) It takes ten percent less time to manufacture a radio it would be best for Bravo Airlines to restrict the
in Country Q than it does in Country Y. (A) number of coupons that a person can be awarded in a
particular year
8. For over fifty years, the ocean-freight industry worked (B) use of the coupons to those who were awarded the
to make ocean freighters faster and lower their fuel coupons and members of their immediate families
consumption. Despite considerable success, the economics of (C) days that the coupons can be used to Monday through
the industry grew worse, until the industry was almost dead. Friday
What was wrong was an incongruity between assumptions (D) amount of time that the coupons can be used after
and realities. The real costs came, not from time spent at they are issued
sea, but from time spent in port during loading and unloading. (E) number of routes on which travels can use the coupons
Which of the following actions would be most likely to lead
to a solution of the problem faced by the ocean-freight industry, 10. State X’s income-averaging law allows a portion of one’s
as it is analyzed in the passage? income to be taxed at a lower rate than the rate based on
(A) Developing a ship’s engine that runs on a cheaper type one’s total taxable income. To use income averaging, the
of fuel than that traditionally used by ocean freighters taxpayer must have earned taxable income for a particular
(B) Developing a ship with accessible cargo compartments year that exceeds 140 percent of his or her average taxable
that can be mechanically loaded and unloaded very rapidly income for the previous three years. People using income
(C) Developing a ship whose freight capacity relative to averaging owe less tax for that year than they would without
the ship’s total volume is much larger than that of any existing income averaging.
ship Which of the following individuals would be most seriously
(D) Implementing a system to ensure that ocean freighters affected if income averaging were not permitted in computing
are loaded to capacity whenever they leave a port the taxes owed for the current year?
(E) Implementing a marketing plan that focuses on routes (A) Individuals whose income has steadily decreased for
that are known to be least threatened by unfavorable weather, the past three years
thus permitting rapid trips and reliable arrival times (B) Individuals whose income increased by 50 percent four
years ago and has remained the same since then
9. Company Alpha buys free-travel coupons from people (C) Individuals whose income has doubled this year after
who are awarded the coupons by Bravo Airlines for flying remaining about the same for five years.
frequently on Bravo airplanes. The coupons are sold to people (D) Individuals who had no income this year, but did in
who pay less for the coupons than they would pay by each of the previous three years
purchasing tickets from Bravo. This marketing of coupons (E) Individuals who are retired and whose income has
results in lost revenue for Bravo. remained about the same for the past ten years

99
Chapter 9
'RELEVANCE' QUESTIONS
Another type of questions you may encounter in Logical From a quick reading of the choices, we can notice that it
Reasoning will incorporate a conclusion or hypothesis in the is only (E) which mentions the smearing of foreign substances
given narration, and give you five statements beneath it. You on the guitar strings, and is the answer.
will be asked to spot that statement, the examination, study None of the other choices is relevant to the hypothesis of
or investigation of which would be most relevant for evaluat- the researcher that dirt and oil, rather than the material
ing whether the given conclusion or hypothesis is valid or not. properties of the string, are responsible for the guitar strings
In a variation of this type of question, the examination, going “dead”.
study or investigation of four of the five choices will be relevant
for such evaluation, and only one will not be so relevant. You Example 2
will be asked to spot this choice and choose it as your answer. According to a recent survey, marriage is fattening. Cited
Let us now see a few examples: as evidence is the survey’s finding that the average woman
gains 23 pounds and the average man gains 18 pounds during
Example 1 13 years of marriage.
Guitar strings often go “dead” - become less responsive The answer to which of the following questions would be
and less bright in tone - after a few weeks of intense use. A most relevant in evaluating the reasoning presented in the
researcher whose son is a classical guitarist hypothesized survey?
that dirt and oil, rather than changes in the material (A) Why was the period of time studied in the survey 13
properties of the string, were responsible. years, rather than 12 or 14?
Which of the following investigations is most likely to yield (B) Did some of the men surveyed gain less than 18 pounds
significant information that would help to evaluate the during the time they were married?
researcher’s hypothesis? (C) How much weight is gained or lost in 13 years by single
(A) Determining if a metal alloy is used to make the strings people of comparable age to those surveyed?
used by classical guitarists (D) Were the women surveyed as active as the men
(B) Determining whether classical guitarists make their surveyed, at the time the survey was made?
strings go dead faster than do folk guitarists (E) Will the reported gains be maintained over the lifetime
(C) Determining whether identical lengths of string, of the of the persons surveyed?
same gauge, go dead at different rates when strung on various
brands of guitars Analysis
(D) Determining whether a dead string and a new string From a survey result that the average woman gains 23
produce different qualities of sound pounds and the average man gains 18 pounds during 13 years
(E) Determining whether smearing various substances on of marriage, the author comes to the conclusion that marriage
new guitar strings causes them to go dead is fattening.
We have been asked to spot that question among the
Analysis choices the answer to which will be most relevant in
The first sentence of the given narration states it as an evaluating the above reasoning.
observed fact that guitar strings often become less responsive The author attributes the gain in weight of individuals
and less bright in tone after a few weeks of intense use. It solely to their getting and remaining married for 13 years.
also quotes one researcher as stating that this deterioration This conclusion will be valid only if it is proved in another
is due to dirt and oil, and not due to changes in the material survey that unmarried men and women in the same age group
properties of the string. do not gain weight during a similar period of 13 years. It is
We have been asked to identify the choice which can form (C) which states this, and is the answer.
an investigation that will help to evaluate the researcher’s (A) is wrong, because the conclusion is not based on the
hypothesis. specific time period of 13 years, and would have been drawn
We can easily guess that any relevant investigation will by the author even if the period covered by the survey was 12
have to involve the use of some foreign substances like dirt years or 14 years.
and oil on the strings. For example if, during an investigation, Since it is only the average figure that is mentioned, the
extreme care is taken to ensure that the strings are always given narrative itself implies that some men surveyed gained
kept free of any foreign substances, but it is still noticed that less than 18 pounds while some had gained more than 18
there is progressive deterioration in the quality of the music pounds, while the average gain remained as 18 pounds. So,
produced by the strings, one can say that the researcher’s (B) is not an additional information, and is not the answer.
hypothesis is wrong, and that the deterioration in quality is (D) is irrelevant to the argument.
attributable to the material properties of the string alone. The conclusion that marriage is fattening can be analysed
On the other hand, if the deliberate smearing of some dirt only on the basis of the weight-gain in the immediate post-
and oil on brand new guitar strings does not result in the marriage period. There could be many other causes for a
deterioration of the quality of music produced by them, then person gaining or losing weight during various other periods
also the researcher’s hypothesis can be considered to be wrong. in his entire lifetime. Therefore, whether an immediate post-
Conversely, if such deliberate smearing of foreign substances marriage gain in weight was maintained over the lifetimes of
on brand new guitar strings results in the immediate the persons surveyed is not relevant to the analysis of the
deterioration of the quality of the sound produced by them, given conclusion. So, (E) is also wrong.
then the researcher’s hypothesis would be strengthened.
100
American Education Aids - LSAT - Logical Reasoning
Example 3 Example 4
People living in the undeveloped wilderness area want jobs, In order to contain rising energy costs, the plant engineer
and commercial development of the area will create jobs. But of a large factory replaced its old insulation with a new kind
if the new commercial development plan is carried out, the of insulation. The total cost of heating the factory was 18
wilderness will suffer. Thus the Board considering the area’s percent lower the first full month after installation than the
future must choose between the preservation of the last full month before.
wilderness and the wishes of the local people. In evaluating the precise heating cost savings, if any, that
The answer to which of the following questions would be can be properly attributed to the new insulation, all of the
LEAST relevant to evaluating whether the Board indeed faces following would be important to know both about the last full
the choice the author says it faces? month before and about the first full month after installation
(A) Could commercial development be carried out under EXCEPT the
an alternative plan without damaging the wilderness? (A) average daily temperature outside the factory
(B) Would commercial development of the wilderness area (B) average daily temperature inside the factory
significantly benefit people living elsewhere? (C) average amount of fuel used per day
(C) Would the jobs created by the new development plan (D) chemical composition of the insulation in use
be filled by the local people? (E) cost, per gallon, of fuel used in heating the factory
(D) Do local people support or oppose commercial
development of the wilderness area? Analysis
(E) Can jobs be provided without commercial development The given narrative states that, in the first full month
of the wilderness area? after the old insulation was replaced by a new kind of
insulation, the total cost of heating the factory was 18% lower
Analysis than what had been incurred in the last full month before the
The author of the paragraph states that the Board that is replacement.
considering the future of the area has to choose between (a) The lower energy cost after the replacement could have
the preservation of its wilderness and (b) wishes of the local been due (i) entirely to the new insulation, or (ii) partly due
people who want jobs. to the efficiency of the new insulation and party due to other
We have been asked to identify that choice which is least factors.
relevant to the evaluation whether the Board really faces such The question states that, out of the five given choices, four
a choice. choices mention factors which are relevant to the evaluation
If it is possible to carry out commercial development under of how much of the difference in energy costs between the
an alternative plan without damaging the wilderness, the two months was attributable solely to the new insulation,
Board will not be faced with the problem of choosing between and how much was attributable partly to other factors also,
commercial development and the preservation of wilderness. and that only one of choice is not a relevant factor for such an
So, the answer to the question in (A) is very relevant in evaluation. We have been asked to spot that choice which is
evaluating the author’s statement. So, (A) is not the answer. not a relevant factor.
The demand for the commercial development of the (Obviously, the factory is located in a cold area and
wilderness has come from people living in that area who want therefore needs heating in order to maintain the inside
jobs. Therefore, the answer to the question whether such temperature within a range that is comfortable for the
development will be of benefit to people living elsewhere is workers. If there is no insulation, the factory will have to
least relevant to the author’s statement that the Board faces use more energy for maintaining the temperature in this
a choice between commercial development and preservation range, because the cold from outside the factory would
of the wilderness. So, (B) is the answer. continuously permeate inside.)
If the jobs created by the commercial development of the The energy costs for heating would obviously be higher
wilderness would require skills that the local people do not during the peak winter month than during any of the other
possess, then people from outside will have to be employed in months. If the previous full month had been the peak winter
those jobs, defeating the very purpose for which the month, there would have been a reduction in the energy cost
development is attempted. So, the answer to the question in in the subsequent month even if the new insulation had not
(C) is very relevant to the author’s statement that the Board been there. In this case, the entire 18% reduction in energy
has to choose between the two alternatives. So, (C) is not the cost cannot be attributed solely to the new installation.
answer. Suppose, on the other hand, the peak winter month had
If local people oppose the commercial development of the followed the installation of the new insulation system. The
wilderness area, then the Board can easily decide not to energy cost during this month should normally have been
develop it, and thus preserve the wilderness. In this case, the higher than the energy cost during the previous month; the
Board is not faced with the dilemma of having to choose fact that it was 18% lower would imply that the saving in
between two conflicting interests. So, the answer to the heat costs attributable to the new installation is even more
question in (D) is also very relevant to the evaluation of the than 18%.
author’s statement. So, (D) is not the answer. Therefore, the average daily temperature outside the
The primary demand of the local people is the creation of factory during each of the two months is a relevant factor in
jobs for them. If this could be achieved without the commercial evaluating the cost savings that can be attributable solely to
development of the wilderness area, then also the Board will the new system. So, (A) is not the answer.
not face the dilemma of having to choose between two For a similar reason, the average daily temperature inside
alternative demands. the factory during the two months is also a relevant factor in
So, the answer to the question in (E) is also very relevant such evaluation. So, (B) is not the answer.
to the evaluation of the author’s statement. So, (E) is not the (C) could be a consequence of (A) and (B), because the
answer. average amount of fuel used per day during these two months
101
American Education Aids - LSAT - Logical Reasoning
could be related to the average outside temperature and/or (E) Nature of the pollutants most frequently emitted in
the average inside temperature during these periods, and is chemical processes
therefore a relevant consideration for evaluating the cost
savings attributable solely to the new insulation system. So, Analysis
(C) is not the answer. The claim made is that ‘chemical industry is as deeply
(D) talks of the chemical composition of the insulations concerned with pollution control as the public is, and is taking
used in the old and the new systems. While this might have active steps to eliminate pollution’. The latter two sentences
been a factor in the capital cost of the two systems, it is an give particulars in justification of this claim.
irrelevant factor for calculating the reduction in the recurring The question implies that four out of the five choices are
energy cost. So, it is (D) which is an irrelevant factor in relevant considerations for evaluating the claim, and the
determining how much of the 18% reduction in energy cost remaining one is not relevant. We have been asked to identify
between the two months is attributable to the new insulation this irrelevant choice.
system. So, (D) is the answer. The claim will not be of much validity if, out of 10,000
If the cost of the fuel used in heating the factory had come persons said to be working on anti-pollution programs, the
down by, say, 5% between the two months, then a part of the majority are working in one or two industries only, while a
18% savings in energy cost must be attributable to this factor, large number of the remaining industries have no anti-
and the reduction attributable solely to the new insulation pollution staff. So, a knowledge of the distribution of the
system would be lower than 18%. If, on the other hand, the anti-pollution staff of 10,000 throughout the industry will help
cost of the fuel had increased by 5%, and there was still a us to evaluate the claim. So, (A) is a relevant consideration,
reduction of 18% in the energy cost between the two months, and is not the answer.
then the credit to be given to the new insulation system would If the total emission from the factory mentioned is, say,
be even higher than 18% that was actually achieved. So, (E) 100,000,000 tons, the fact that only 75,000 tons are eliminated
is also a relevant factor in the evaluation. So, (E) is not the cannot sustain the claim of the chemical industry that it has
answer. taken steps to eliminate pollution. Thus, the knowledge of
the total emission from the factory is also very relevant for
Example 5 evaluating the industry’s claim.
The chemical industry is as deeply concerned with So, (B) is not the answer.
pollution control as the public is, and has already made Depending on whether only 10% or 95% of the chemical
considerable progress toward meeting federal clean air facilities have instituted pollution control programs, we can
standards. The industry already employs over 10,000 people either debunk or confirm the claim. Thus, (C) is also a valid
whose sole job it is to operate, maintain and monitor pollution consideration for evaluating the claim, and is not the answer.
control programs. At one facility, such a program will If we know that the number of employees required for
eliminate 75,000 tons of air pollutants per year. antipollution measures throughout the industry is 500,000,
Knowledge of which of the following would be LEAST useful the fact that only 10,000 are presently employed would
in evaluating the claims made in the passage above? invalidate the claim of the industry. Thus (D) is also a relevant
(A) Distribution throughout the industry of the employees consideration in the evaluation of the industry’s claim.
who work in pollution control programs Therefore, by elimination, (E) must be the answer. Even
(B) Total emission of the facility from which 75,000 tons of logically it is so, since every pollutant has its own
pollutants are to be eliminated antipollutant device. In the light of the facts mentioned in
(C) Percentage of chemical facilities that have instituted the passage. the mere knowledge of the nature of the pollutant
pollution control programs cannot help us in evaluating whether the industry has taken
(D) Number of employees necessary to operate pollution steps to combat it or not.
control programs throughout the chemical industry

We shall now give you one set of 10 questions of this type for your practice.

102
American Education Aids - LSAT - Logical Reasoning
SET 1
1. Although custom prosthetic bone replacements produced devices
through a new computer-aided design process will cost more (E) The average number of logic devices and memory
than twice as much as ordinary replacements, custom devices needed for an average computer system
replacements should still be cost-effective. Not only will
surgery and recovery time be reduced, but custom 4. The results of a recent poll in the United States indicate
replacements should last longer, thereby reducing the need that the public, by 80 percent to 17 percent, opposes relaxation
for further hospital stays. of existing regulation of air pollution. Furthermore, not a
Which of the following must be studied in order to evaluate single major segment of the public wants environmental laws
the argument presented above? made less strict. The results of this poll reveal that legislators,
(A) The amount of time a patient spends in surgery versus by voting for renewal of the Clean Air Act, will be responsive
the amount of time spent recovering from surgery to the will of the public without alienating any significant
(B) The amount by which the cost of producing custom special-interest groups.
replacements has declined with the introduction of the new Which of the following pieces of information would be most
technique for producing them useful in evaluating the logic of the argument presented above?
(C) The degree to which the use of custom replacement is (A) The groups in the population that were defined as major
likely to reduce the need for repeat surgery when compared segments of the public and the groups defined as special-
with the use of ordinary replacements interest groups
(D) The degree to which custom replacements produced (B) The length of time that current federal environmental
with the new technique are more carefully manufactured than laws have been in effect and the length of time that states
are ordinary replacements have regulated air pollution.
(E) The amount by which custom replacements produced (C) The probable economic effect of renewal of the Clean
with the new technique will drop in cost as the production Air Act on those opposed to and those in favor of relaxing
procedures become standardized and applicable on a larger environmental laws.
scale (D) The people whom the author hopes to influence by
citing the results of the poll
2. Correctly measuring the productivity of service workers (E) The percentage of those surveyed who choose not to
is complex. Consider, for example, postal workers: they are respond to the questions asked of them
often said to be more productive if more letters are delivered
per postal worker. But is this really true? What if more letters 5. The United States is not usually thought of as a nation
are lost or delayed per worker at the same time that more of parakeet lovers. Yet in a census of parakeet owners in
are delivered? selected comparable countries, the United States ranked
The objection implied above to the productivity measure second, with eleven parakeet owners per hundred people. The
described is based on doubts about the truth of which of the conclusion can be drawn from this that people in the United
following statements? States are more likely to own parakeets than are people in
(A) Postal workers are representative of service workers most other countries.
in general. Knowledge of which of the following would be most useful
(B) The delivery of letters is the primary activity of the in judging the accuracy of the conclusion?
postal service. (A) The number of parakeets in the United States
(C) Productivity should be described to categories of (B) The number of parakeet owners in the United States
workers, not to individuals. (C) The number of parakeet owners per hundred people in
(D) The quality of services rendered can appropriately be the country that ranked first in the census
ignored in computing productivity. (D) The number of parakeet owners in the United States
(E) The number of letters delivered is relevant to compared to the numbers of owners of other pet birds in the
measuring the productivity of postal workers. United States
(E) The numbers of parakeet owners per hundred people
3. The cost of the average computer logic device is falling in the countries not included in the census
at the rate of 25 percent per year, and the cost of the average
computer memory device at the rate of 40 percent per year. 6. A recent state survey of human resources found that
It can be concluded that if these rates of cost decline remain the age to which secretarial school graduates are expected to
constant for a period of three years, at the end of that time live is four years in excess of the age to which other graduates
the cost of the average computer memory device will have of high school are expected to live. One possible conclusion is
declined by a greater amount than the cost of the average that secretarial school attendance is beneficial to one’s health.
computer logic device. To evaluate the conclusion above, it would be most
Accurate information about which of the following would important to know the answer to which of the following
be relevant in evaluating the corrections of the conclusion questions?
above? (A) Have the average age of new high school graduates
(A) The number of logic devices and memory devices and the average age of new secretarial school graduates
projected to be purchased during the next three years recently increased?
(B) The actual prices charged for the average computer (B) Do some secretarial school graduates have college
logic device and the average computer memory device degrees?
(C) The compatibility of different manufacturers’ logic (C) Given that women have a greater life expectancy than
devices and memory devices men, what are the relative proportions of men and women
(D) The relative durability of logic devices and memory among high school and secretarial school graduates?
103
American Education Aids - LSAT - Logical Reasoning
(D) Given that women have a greater life expectancy than (E) Comparing deaths caused by accidents in the United
men, what proportion of all women attend secretarial school? States to deaths caused by combat in the armed forces
(E) Has the proportion of high school graduates who attend
secretarial school increased in recent years? 9. Leaders of a miners’ union on strike against Coalco are
contemplating additional measures to pressure the company
7. The fact that several of the largest senior citizens’ to accept the union’s contract proposal. The union leaders
organizations are constituted almost exclusively of middle- are considering as their principal new tactic a consumer
class elderly people has led critics to question the seriousness boycott against Gasco gas stations, which are owned by
of those organisations’ commitment to speaking out on behalf Energy Incorporated, the same corporation that owns Coalco.
of the needs of economically disadvantaged elderly people. The answer to which of the following questions is LEAST
Which of the following generalizations, if true, would help directly relevant to the union leaders’ consideration of whether
to substantiate the criticism implicit in the statement above? attempting a boycott of Gasco will lead to acceptance of their
(A) The ideology of an organization tends to reflect the contract proposal?
traditional political climate of its locale. (A) Would revenue losses by Gasco seriously affect Energy
(B) The needs of disadvantaged elderly people differ in Incorporated?
some ways from those of other disadvantaged groups within (B) Can current Gasco customers easily obtain gasoline
contemporary society. elsewhere?
(C) Organized groups are better able to publicize their (C) Have other miners’ unions won contracts similar to
problems and seek redress than individuals acting alone. the one proposed by this union?
(D) Middle-class elderly people are more likely to join (D) Have other unions that have employed a similar tactic
organizations than are economically disadvantaged elderly achieved their goals with it?
people. (E) Do other corporations that own coal companies also
(E) People usually join organizations whose purpose is to own gas stations?
further the economic, political, or social interests of their
members. 10. The company should not be held responsible for failing
to correct the control-panel problem that caused the accident.
8. During the Second World War, about 375,000 civilians Although the problem had been mentioned earlier in a safety
died in the United States and about 408,000 members of the inspector’s report, companies receive hundreds of reports of
United States’ armed forces died overseas. On the basis of such problems, and Industry Standard No. 42 requires action
those figures, it can be concluded that it was not much more on these problems only when an accident is foreseeable.
dangerous to be overseas in the armed forces during the If the second sentence in the paragraph above is factually
Second World War than it was to stay at home as a civilian. correct, the answer to which of the following questions is most
Which of the following would reveal most clearly the relevant in helping to determine whether or not the company
absurdity of the conclusion drawn above? violated Industry Standard 42 when it failed to correct the
(A) Counting deaths among members of the armed forces control-panel problem?
who served in the United States in addition to deaths among (A) Was the accident serious?
members of the armed forces serving overseas (B) Was the control-panel problem of a type that is known
(B) Expressing the difference between the numbers of to indicate that an accident is likely?
deaths among civilians and members of the armed forces as (C) Since the accident, has the company done a special
a percentage of the total number of deaths safety check on all control panels?
(C) Separating deaths caused by accidents during service (D) Did the safety inspector mention more than one
in the armed forces from deaths caused by combat injuries problem in the same report?
(D) Comparing death rates per thousand members of each (E) How long was the control panel in use before the
group rather than comparing total numbers of deaths problem was discovered?

104
Chapter 10
'ARGUMENTATION' QUESTIONS
The emphasis in another type of questions under Logical the higher prices they charge for their products.
Reasoning will not be on the content or subject matter of the Which of the following best describes how the consumer
narration, but will be on the process of argumentation used in counters the advertiser’s argument?
it for arriving at a particular conclusion. (A) By alleging something that, if true, would weaken the
The question may ask you to identify the choice that plausibility of the advertiser’s conclusion
describes the process of argumentation, or the choice that (B) By questioning the truth of the purportedly factual
points out a weakness in the process of argumentation, or the statement on which the advertiser’s conclusion is based
choice that names the technique used in the argumentation. (C) By offering an interpretation of the advertiser’s opening
Let us see a few examples of this type: statement that, if accurate, shows that there is an implicit
contradiction in it
Example 1 (D) By pointing out that the advertiser’s point of view is
In the last few years the federal government has imposed biased
so many restrictions and conditions on the building of nuclear (E) By arguing that the advertiser too narrowly restricts
power plants that it takes twice as long to build a nuclear the discussion to the effects of advertising that are economic
power plant in the United States as it does to build one in
either Europe or Japan. It is no coincidence that, during the Analysis
same years, many companies have given up their plans to The advertiser’s argument is that the consumer really
build nuclear power plants in the United States. Clearly, the benefits from advertising, because it is the advertising
decisions to abandon nuclear power plant projects in the revenues that enable newspapers to be sold at a low price to
United States are a direct consequence of government them.
restrictions. The consumer’s counter-argument is that, even if the
The author of the passage above makes her argument by consumer pays less for the newspaper, this gain is more than
(A) denying the plausibility of any other point of view offset by his having to pay more than necessary when he buys
(B) making contradictory assumptions the advertised products, because the manufacturers of these
(C) drawing a generalization from an atypical case products add the cost of advertisements also to the prices of
(D) asserting a causal connection these products.
(E) comparing cases that are unrelated to each other We have been asked to spot the choice which would best
describe how the consumer counters the advertiser’s
Analysis argument.
The words ‘It is no coincidence’ in the second sentence, What the consumer does in his counter-argument is to
and ‘clearly’ in the last sentence, reveal the author’s assertion allege that the advertisers pass along the advertising costs
that government restrictions are directly responsible for the to the consumers by charging higher prices for their products.
abandonment of nuclear power station projects in the United If this allegation is true, the argument of the advertiser is
States. So, she is asserting a causal connection between two thereby weakened.
phenomenon. (D) is what states this, and is the answer. It is (A) which states this, and is the answer.
(You should understand the difference in meanings between (B) is wrong because the consumer does not question the
the words ‘casual’ and ‘causal’. ‘Casual’ means ‘not serious’ or statement that he pays less for the newspaper because of the
‘informal’. ‘Causal’ means ‘being in the nature of the cause presence of advertisements in it.
(for an identified effect)’ (C) is wrong because what the consumer questions is not
The three sentences in the argument are very closely the advertiser’s opening statement, but the conclusion that
related, and no other explanation is plausible. The author follows.
does not have to deny the possibility of any other point of view. (D) is wrong because the consumer does not point out any
So, (A) is not the answer. bias in the advertiser’s argument.
The author only states facts, and no assumptions have been (E) may be a conclusion that may be drawn from the
made by him in the argument. So, (B) is also wrong. consumer’s argument, but is not his argument.
The author’s argument is not based on the abandonment
of any particular atomic plant. Since no single case has been Example 3
referred to in the passage, (C) is wrong. The United States imprisons more people, both in absolute
Restrictions on nuclear plants, and their construction (or numbers and per capita, than any other civilized Western
abandonment) are closely related. So, (E) is also wrong. society. But it has a higher crime rate than any of them. This
shows that imprisonment contributes to a high incidence of
Example 2 crime.
Advertiser: The revenue that newspapers and magazines Which of the following is the most serious weakness in the
earn by publishing advertisements allows publishers to keep argument above?
the prices per copy of their publications much lower than (A) It may be confusing cause and effect
would otherwise be possible. Therefore, consumers benefit (B) It lacks substantiating statistics
economically from advertising. (C) It fails to differentiate among various classes of crime
Consumer: But who pays for the advertising that pays for (D) It rests on the assumption that all Western societies
low-priced newspapers and magazines? We consumers do, are civilized
because advertisers pass along advertising costs to us through (E) It compares phenomena that may not be comparable
105
American Education Aids - LSAT - Logical Reasoning
Analysis not result in as large a reduction in production as is pro-
It is because there is a higher rate of crime in the US that jected.
there is a higher rate of imprisonment. The cause is the higher
crime rate and the effect is the higher imprisonment rate. Analysis
The given passage reverses the cause and its effect, and is The speaker is proposing a progressive cut in production
its serious weakness. So, (A) is the answer. level during the next five years at the rate of 10% of the
The other four choices are also weaknesses in the passage, previous year’s production level in order to reduce the
but are not as serious as that of confusing the cause with the production to exactly 50% of what it is today.
effect and vice versa. We have been asked to identify the logical flaw in the above
reasoning..
Example 4 The flaw is, in fact, in the arithmetic of the reasoning.
In this day and age, The Constitution is a fallible and If the proposal is implemented and if this year’s production
incomplete guide to national policy making. Instead of three is 100 units, the next year’s production will be 100 – 10 = 90
branches of government - legislature, executive, and judiciary units.
- each with its clearly defined sphere of competence and The production in the second year from now will be 90 – 9
activity, as the Constitution prescribes, there may be five = 81 units.
branches of government dealing with any particular subject The production in the third year from now will be 81 – 8.1
matter, or seven, or twenty, or only one, with legislative, = 72.9, or 73 units.
executive, and judicial roles mixed and distributed throughout The production in the fourth year from now will be 73 –
the national government. 7.3 = 65.7, or 66 units.
The persuasiveness of the claim made above is most The production in the fifth year from now will be 66 – 6.6
weakened by its vague use of the phrase = 59.4 units.
(A) this day and age Thus, the production in the fifth year will not be 50% of
(B) fallible and incomplete what it is today, but will be 59.4% of what it is today. So, it is
(C) guide to national policy-making (E) that describes the logical error in the argument.
(D) branches of government The other choices are relevant considerations, but would
(E) mixed and distributed not amount to a logical flaw in the argument.

Analysis Example 6
The three branches of Government laid down in the Any United States flag manufactured outside the United
Constitution are well-defined as the Legislature, Executive States should be banned from importation since some foreign
and Judiciary. In his new theory, the author advocates ‘five manufacturers superimpose images on the United States flag
branches of Government, or seven, or twenty or only one’ and sell some products with relative impunity. United States
without defining what these ‘branches’ could be. manufacturers, on the other hand, would face penalties for
Thus, the vague use of the phrase ‘branches of Government’ such violation of the United States flag code.
is the flaw in the proposal. Which of the following is the best criticism of the argument
So, (D) is the answer. above?
There is no ambiguity about the other phrases in the (A) The argument reiterates its conclusion instead of
answer choices. providing a reason for it.
(B) The argument makes an irrelevant distinction between
Example 5 foreign and United States manufacturers.
“I propose the following strategy for responding to the (C) The reason given for the ban undermines rather than
rapidly shrinking demand for our line of Rota toys: in each of supports the conclusion.
the next five years, let us cut production of those toys by ten (D) The reason given for the ban does not explain why
percent of the preceding year’s production level, and our images superimposed on the United States flag are offensive.
production in year five will be reduced to exactly half of what (E) The reason given for the ban applies only to a part of
it is today”. the group of manufacturers whose flags are included in the
Which of the following is a logical error contained in the ban, not necessarily to all.
proposal above?
(A) The reasons for the decline in the demand for Rota Analysis
toys are not analyzed. For the reason that some foreign manufacturers
(B) The proposal is put forth without an explanation of superimpose images on the United States flags thereby
why the figure of ten percent was set as a goal. violating the US Flag Code, the author argues that any United
(C) The proposal is a conservative response in that it treats States flag manufactured outside the United States (including
the problem of decreasing demand as likely to persist. those that do not violate the US Flag Code) should be banned
(D) The proposal leaves open what is to be done once the from importation.
plan for the next five years has been carried out. The best criticism of this argument is contained in (E),
(E) The strategy as described above, if carried out, would which is the answer.

You can now answer the questions of this type given in the next few pages by yourself.

106
American Education Aids - LSAT - Logical Reasoning
SET 1
1. Statement of a United States copper mining company: not address the fundamental issue of whether space activities
Import quotas should be imposed on the less expensive copper should have priority over other claims on the national budget
mined outside the country to maintain the price of copper in
this country; otherwise, our companies will not be able to 4. Government employment programs can reduce un-
stay in business. employment not just by the jobs that they create. Such
Response of a United States copper wire manufacturer: programs give rise to new jobs in private business as well.
United States wire and cable manufacturers purchase about Each worker in a public mass-transit project, for example,
70 percent of the copper mined in the United States. If the requires tools and materials, the production of which
copper prices we pay are not at the international level, our stimulates employment in the private industries that produce
sales will drop, and then the demand for United States copper them.
will go down. The author’s point is made chiefly by
If the factual information presented by both companies is (A) citing a causal relationship
accurate, the best assessment of the logical relationship (B) presenting an analogy as fact
between the two arguments is that the wire manufacturer’s (C) presenting an overview
argument (D) appealing to emotions
(A) is self-serving and irrelevant to the proposal of the (E) posing a question to be answered
mining company
(B) is circular, presupposing what it seeks to prove about 5. The once widely held perception of intellectuals as the
the proposal of the mining company clarifiers of fundamental moral issues is no longer valid today.
(C) shows that the proposal of the mining company would Intellectuals no longer act as advocates for oppressed groups.
have a negative effect on the mining company’s own business Instead of applying their insights and analyses to the
(D) fails to give a reason why the proposal of the mining problems of these groups, they leave the debate to the
company should not be put into effect to alleviate the concern politicians.
of the mining company for staying in business The logical structure of the passage above depends upon
(E) establishes that even the mining company’s business the author’s closely linking the clarification of fundamental
will prosper if the mining company’s proposal is rejected moral issues with
(A) intellectualism
2. Many pregnant women suffer from vitamin deficiency, (B) advocacy on behalf of oppressed groups
but this is frequently not due to vitamin deficiency in their (C) insight and analysis
diets; most often it is because they have higher requirements (D) debate on contemporary practical issues
for vitamins than do the rest of the population. (E) the role of politicians
The best criticism of the reasoning in the statement above
is that it 6. Some would have you believe that the economic
(A) fails to specify the percentage of pregnant women who problems of Western Europe have been caused by the
suffer from vitamin deficiency Organization of Petroleum Exporting Countries (IPEC) oil
(B) gives insufficient information about why pregnant cartel. This is nonsense. After all, Great Britain is not
women have higher vitamin requirements than do other dependent on OPEC oil and yet Great Britain suffers from
groups the same economic problems that afflict France and West
(C) fails to employ the same reference group for both uses Germany.
of the term “vitamin deficiency” The author’s point is made primarily by
(D) provides insufficient information about the incidence (A) offering Great Britain as a counterexample
of vitamin deficiency in other groups with high vitamin (B) analyzing the economic difficulties of France and West
requirements Germany
(E) uses “higher requirements” in an ambiguous manner (C) pointing out a misconception in reasoning
(D) proposing an alternative explanation
3. Teresa: Manned space flight does not have a future, (E) drawing an analogy between France and West
since it cannot compete economically with other means of Germany
accomplishing the objectives of space flight
Edward: No mode of human transportation has a better 7. Dear Editor: Jones’ new book has the potential to destroy
record of reliability: two accidents in twenty-five years. Thus reputations of persons who have held high governmental
manned space flight definitely has a positive future responsibility during national crises. However, readers
Which of the following is the best logical evaluation of should dismiss Jones’s criticisms. Jones’s antigovernment
Edward’s argument as a response to Teresa’s argument? attitude is well known, and his criticisms will convince only
(A) It cites evidence, that, if true, tends to disprove the those like himself, persons who have never had real
evidence cited by Teresa in drawing her conclusion responsibility and never will, and hence are not qualified to
(B) It indicates a logical gap in the support that Teresa judge.
offers for her conclusion The argument above includes which of the following
(C) It raises a consideration that outweighs the argument questionable techniques?
Teresa makes (A) It employs the term “responsibility” in more than one
(D) It does not meet Teresa’s point because it assumes sense.
that there is no serious impediment to transporting people (B) It assumes that attacking the source of a claim is
into space, but this was the issue raised by Teresa sufficient to disprove the claim.
(E) It fails to respond to Teresa’s argument because it does (C) It assumes that the majority of people share Jones’s
107
American Education Aids - LSAT - Logical Reasoning
attitude of opposition to government policies. the author
(D) It appeals to a person of unreliable authority as a (A) does not adequately link carbon dioxide to the use of
supporter of its position. fossil fuels
(E) It confuses cause and effect. (B) associates a general increase in the temperature of
the atmosphere with a global heat wave
8. Contrary to the assumption that modern technology (C) assumes that an increase in atmospheric temperature
allows us to improve on nature, it is clear that any major can be caused only by an increased use of fossil fuels
technologically induced changes in a natural system are likely (D) fails to define the greenhouse effect and its relation to
to be detrimental to that system. If you were to open the the production of carbon dioxide gases
back of your watch, close your eyes, and poke a pencil into (E) gives no evidence that our use of fossil fuels has
the exposed works, the almost certain result would be damage increased
to the watch. Living organisms too, when subjected to
technological alteration, will almost certainly be damaged 10. Asserting that newspapers should reflect the needs of
rather than improved. their readers, a group of newspaper publishers conducted a
The argument above is developed by survey to determine how readers felt newspapers could be
(A) citing a consensus of opinion about technology among improved. The readers made two recommendations:
informed observers newspapers should emphasize events closer to the readers’
(B) projecting the effects of change in a natural system lives and should feature articles about the reporters. If we
from observation of a representative case take the publishers and their survey seriously, readers will
(C) drawing an analogy between a natural system and a be asked which events in their own lives they wish to read
mechanical device about, and these articles, overlaid with autobiographical
(D) analyzing a natural system in terms of its constituent vignettes, will be produced by the reporters. In this closed
parts world, writers and readers will hold forth in an uninterrupted
(E) examining the results of technological innovations of one-to-one dialogue from which events of the larger world -
the past what used to be called news - are shut out.
The author’s response to the survey is flawed because the
9. Some ecologists predict that a global heat wave will author
result from our increasing use of fossil fuels. They point out (A) fails to consider alternative explanations of the data
that the consumption of these fuels produces carbon dioxide, (B) fails to define what he means by the term “news”
which locks solar heat into the atmosphere through the (C) considers only one of the recommendations made by
“greenhouse effect,” If there is a general increase in the the survey respondents
temperature of the atmosphere in the next decades, we will (D) misinterprets the intention of the survey respondents’
be able to conclude that the ecologists’ prediction was correct. recommendations
The most serious weakness of the argument above is that (E) assumes that newspapers do not need to be improved

SET 2
1. A program of steady, moderate aerobic exercise coupled land previously devoted to farming is income-producing
with a diet low in saturated fats and cholesterol has been (C) ignores the possibility that the average size of farms
associated with reduced risk of heart attacks and strokes. has changed
Therefore, noone who exercises regularly and eats only foods (D) gives data for only 3 of the past 140 years
that are low in saturated fats and cholesterol will have a (E) does not take into account the type of product or crop
heart attack or stroke. each farm yields
Of the following, the best criticism of the argument above
is that the argument does not 3. Miko: Academic products developed at a university are
(A) take into account the possibility of heart attacks and properly considered the results of ‘work for hire’ and really
strokes that occur regardless of diet and level of exercise belong to the institution. Therefore the university should own
(B) take into account all the possible physiological effects the copyright for any computer software developed by its
of saturated fats and cholesterol faculty
(C) specify whether foods high in saturated fats also Kofi: But a copyright policy this restrictive can impede a
contain cholesterol university’s primary mission of generating and disseminating
(D) indicate whether an increased risk of heart attacks knowledge
and strokes is due more to poor diet or more to lack of exercise Kofi’s response has which of the following relationships to
(E) differentiate between the causes of heart attacks and Miko’s argument?
the causes of strokes (A) Kofi contradicts Miko’s evidence.
(B) Kofi points out a hidden assumption required by Miko’s
2. Around 1850 there were about 800 farms in Otsego argument.
county. By the 1950’s the number of farms had dropped to (C) Kofi points out a problematic consequence of accepting
around 400. By 1988 there were only 81 farms in operation. Miko’s argument.
Therefore, the amount of land in the county that is devoted (D) Kofi shows that Miko’s reasoning is circular.
to farming has dropped by about 90 percent in the past 140 (E) Kofi shows that Miko forms a generalization from an
years. atypical case.
A major flaw in the argument above is that it
(A) counts the number of farms only approximately 4. X: When a rare but serious industrial accident occurs,
(B) fails to say whether the use now being made of the people respond by believing that such accidents are becoming
108
American Education Aids - LSAT - Logical Reasoning
more frequent. This belief is irrational. After all, being dealt single event is sufficient to change a belief.
four aces in a hand of poker, a rare event, hardly increases
one’s chances of being dealt four aces in a future hand. 5. The burden of taxation on the back of a people is not
Y: To the contrary, the belief is rational because it results unlike the burden of a weight on the back of a horse. Just as
in people’s sensing a danger to themselves not previously a small burden badly placed may distress a horse that could
sensed and taking precautionary actions to prevent similar carry with ease a much larger burden properly adjusted, so a
accidents in the future. people may be impoverished and their power of producing
Y’s attempt to counter X’s claim is best described by which wealth destroyed by taxation that, if levied another way, could
of the following? be borne with ease.
(A) It questions the aptness of the analogy drawn by X. The author’s point is made by
(B) It makes apparent X’s failure to consider how people (A) pointing out an ambiguity
vary in their responses to a serious accident. (B) refuting a supposed counterexample
(C) It shifts the basis for judging rationality to (C) generalizing from a particular case
considerations of utility. (D) using an analogy
(D) It offers an alternative explanation of why people form (E) appealing to an authority
incorrect beliefs.
(E) It challenges X’s assumption that the occurrence of a

109
Chapter 10
'PARADOX' QUESTIONS
In yet another type of questions asked under Logical Rea- Example 2
soning in LSAT, the given narration will contain two sets of Despite the fact that the health-inspection procedures for
related information which, prima facie, appear to be incon- catering establishments are more stringent than those for
sistent with each other, or paradoxical. ordinary restaurants, more of the cases of food poisoning
Five statements will be given underneath the narration, reported to the city health department were brought on by
among which only one, if true, would imply that the two sets banquets served by catering services than were brought on
of information are not inconsistent with each other, but there by restaurant meals.
is a rational explanation for the apparent inconsistency. Which of the following, if true, helps explain the apparent
paradox in the statement above?
The questions of this type will be worded as: (A) A significantly larger number of people eat in
Which of the following, if true, helps explain the apparent restaurants than attend catered banquets in any given time
paradox in the statement above? period.
Which of the following, if true, would be most useful to an (B) Catering establishments know how many people they
attempt to explain the paradox described above? expect to serve, and therefore are less likely than restaurants
Which of the following, if true, would explain the to have, and serve, leftover food, a major source of food
discrepancy described above? poisoning.
Which of the following, if true, resolves the apparent (C) Many restaurants provide catering services for
contradiction presented in the passage above? banquets in addition to serving individual meals.
Which of the following, if true, best reconciles the seeming (D) The number of reported food-poisoning cases at catered
discrepancy described above? banquets is unrelated to whether the meal is served on the
Let us now see a number of examples of this type: caterer’s or the client’s premises.
(E) People are unlikely to make a connection between a
Example 1 meal they have eaten and a subsequent illness unless the
Currently, the number of first-time admissions of illness strikes a group who are in communication with one
individuals diagnosed as manic-depressives to hospitals in another.
Great Britain exceeds by nine times the number of admissions
of such patients to public and private hospitals in the United Analysis
States, even though the population size of the United States The paradox pointed out is that, even though health-
is many times that of Great Britain. inspection procedures for catering establishments are more
Which of the following, if true, would be most useful to an stringent than those for ordinary restaurants, more cases of
attempt to explain the paradox described above? food poisoning are reported to the city health department by
(A) The term ‘manic-depressive’ refers to a wider range of those served in banquets by catering services than by those
mentally ill patients in Great Britain than it does in the who had eaten restaurant meals.
United States. Those attending banquets are likely to belong to a
(B) The admission rate in the United States includes those homogeneous group who are in touch with each other. So, if a
individuals who visit clinics for the first time as well as those number of persons who had attended a banquet develop
who are admitted directly to hospitals. symptoms of food poisoning the next morning, and they are
(C) A small percentage of patients diagnosed as manic- in communication with each other, they would attribute their
depressive in Great Britain are admitted to private nursing common illness to the food that they had eaten at the banquet,
homes rather than hospitals and make complaints to the health authorities on that basis.
(D) The variety of training institutions in psychology in On the other hand, those who take food in a restaurant
the United States is greater than in Great Britain, reflecting are usually strangers to one another. Therefore, even if a much
the variety of schools of psychology that have developed in larger number of persons eating in a restaurant on a particular
the United States. day develop symptoms of food poisoning the next morning,
(E) Seeking professional assistance for mental health each of them is not likely to attribute it solely to the food
problems no longer carries a social stigma in the United that he had taken in the restaurant, but to some other
States, as it once did. unknown cause. So, they are not likely to make complaints
against the restaurant to the health authorities.
Analysis It is (E) which states this as a resolution of the paradox in
If (A) is true, it implies that many more types of mental the narrative, and is the answer.
illnesses come under the umbrella definition of ‘manic
depression’ in Great Britain than in USA. Example 3
This could logically explain why those diagnosed as having A milepost on the towpath read “21” on the side facing the
‘mental depression’ in Great Britain are many times more hiker as she approached it and “23” on its back. She reasoned
than those in USA, where these same persons would have that the next milepost forward on the path would indicate
been classified as suffering from diseases other than ‘mental that she was halfway between one end of the path and the
depression’. other. However, the milepost one mile further on read “20”
So, (A) is the answer. facing her and “24” behind.
The other choices do not logically explain the paradoxical Which of the following, if true, would explain the
situation pointed out in the passage. discrepancy described above?
110
American Education Aids - LSAT - Logical Reasoning
(A) The numbers of the next milepost had been reversed. end of the path, and not the distance already covered from the
(B) The numbers on the mileposts indicate kilometers, not start. It is (C) which states this, and is the answer.
miles. The numbers on the second milepost were 20 on the side
(C) The facing numbers indicate miles to the end of the facing her and 24 on its back. If they had been reversed as
path, not miles from the beginning. stated in (A), it would mean that, if they had been correctly
(D) A milepost was missing between the two which the written, the number facing the hiker would have been 24.
hiker encountered. This implies that the numbers facing her on two consecutive
(E) The mileposts had originally been put in place for the milestones in the same direction would have been 21 and 24.
use of mountain bikers, not for hikers. This is absurd. So, (A) is not the answer.
The last sentence of the given information says that the
Analysis first milepost read 21, and the milepost “one mile further on”
(Unlike milestones on highways which show the distance read 20. Assuming that the first milepost really meant 21
yet to be covered to reach a destination, milestones on towpaths kilometers and not 21 miles, the next milepost must have
(or hiking tracks) sometimes show the distance that one has been one kilometer ahead, and not one mile ahead as given.
already travelled.) So, (B) is not the answer.
If the milestone read 21 on the side facing the hiker as she If a milepost had been missing between the two that the
approached it, and 23 on its back, it means that the distance hiker encountered {as stated in (D)}, the next milepost must
between the two places connected by the path was 44 miles. have read 23 on the side facing her and 21 on the reverse
If the milestones had been marked so as to show the distance side, and not 20 and 24 as was the case. So, (D) is not the
already covered by the hiker (as she presumed), the next answer.
milestone that she met with, when she proceeded in the same (E) by itself does not explain the given discrepancy, unless
direction, should have shown 22 on the side facing her, and it had stated that mileposts in the paths of mountain bikers
22 on its reverse side, and this was what she had expected. always showed the distance yet to be covered, while those in
If, however, the next milestone showed 20 on the side facing the paths of hikers always showed the distance already
her and 24 on the reverse side, it implies that the facing covered.
number indicated the distance yet to be covered to reach the So, (E) is not the answer.

111
American Education Aids - LSAT - Logical Reasoning
SET 1
1. In October 1987 the United States stock market suffered infants and small children.
a major drop in prices. During the weeks after the drop, the Which of the following, if true, best reconciles the seeming
volume of stocks traded also dropped sharply to well below discrepancy described above?
what had been the weekly average for the preceding year. (A) The businesses specializing in the rental of children’s
However, the volume for the entire year was not appreciably furniture buy their furniture from distributors outside of
different from the preceding year’s volume. Florida.
Which of the following, if true, resolves the apparent (B) The few children who do reside in these communities
contradiction presented in the passage above? all know each other and often make overnight visits to one
(A) Foreign investors usually buy United States stocks only another’s houses.
when prices are low. (C) Many residents of these communities who move
(B) The number of stock buyers in 1987 remained about frequently prefer renting their furniture to buying it outright.
the same as it had been the preceding year. (D) Many residents of these communities must provide
(C) For some portion of 1987, the volume of stocks traded for the needs of visiting grandchildren several weeks a year.
was higher than the average for that year. (E) Children’s furniture available for rental is of the same
(D) The greater the volume of stocks traded in a given quality as that available for sale in the stores.
year, the lower the average price per share on the United
States stock market for that year. 5. Two suits of battle armor worn by King Henry VIII were
(E) The volume of stocks traded rises and falls in discovered, one from the beginning of his reign in 1510 and
predictable cycles. the other from 1540. Although both suits of armor were made
for Henry VIII, the 1540 suit of armor was 40 pounds heavier
2. Because the process of freezing food consumes energy, than the 1510 suit of armor.
many people keep their electric freezers half-empty, using Which of the following, if true, contributes LEAST to an
them only to store commercially frozen foods. Yet freezers explanation of the discrepancy described above?
that are half-empty often consume more energy than they (A) Henry, although slim at the beginning of his reign,
would if they were kept fully stocked. developed a bulky figure because of massive weight gain.
Which of the following, if true, contributes most to an (B) During his reign, Henry increased his arsenal of
explanation of the apparent discrepancy described above? weapons because, despite his popularity in 1510, the English
(A) A given volume of air in a freezer requires much more populace was becoming disenchanted with his rule by 1540.
energy to be maintained at a temperature below freezing than (C) Although the style of armor was plain and severe in
does an identical volume of frozen food. the beginning of Henry’s reign, he started the fashion of
(B) The more often a freezer’s door is opened, the more decorating armor with heavy and elaborate metal pieces
energy is required to maintain that freezer’s normal because of his love for ornamentation.
temperature. (D) Henry ascended the throne while still an adolescent
(C) When unfrozen foods are placed in a freezer, the and grew three inches during his first five years as king.
average temperature of a given volume of air inside that (E) Because of the improved design of battle weaponry
freezer rises temporarily. during the 1530’s, armor was given a multi-layered design
(D) A person who normally maintains a half-empty freezer so that the sharper and stronger weapons could not pierce it.
can cut energy costs considerably by using a freezer that is
50 percent smaller. 6. Passenger boardings on airlines almost doubled between
(E) An electric freezer can operate efficiently only if chilled 1977 and 1987; yet the actual number of trips made by
air is free to circulate within the freezing compartment. passengers increased by only 40 percent over the same period.
The reason for this discrepancy is that at least two boardings
3. In 1960, 10 percent of every dollar paid in automobile are counted when a passenger must take more than one flight
insurance claims went to pay costs arising from injuries to get to the final destination of his or her trip.
incurred in car accidents. In 1990, 50 percent of every dollar Which of the following, if true, best helps explain why the
paid in automobile insurance claims went toward such costs, increase in boardings exceeded the increase in the number of
despite the fact that cars were much safer in 1990 than in airline passenger trips between 1977 and 1987?
1960. (A) Between 1977 and 1987, the number of airports
Which of the following, if true, best explains the discrepancy handling flights by major airlines increased significantly.
outlined above? (B) Between 1977 and 1987, the number of airlines
(A) There were fewer accidents in 1990 than in 1960. decreased significantly.
(B) On average, people drove more slowly in 1990 than in (C) Between 1977 and 1987, the proportion of passenger
1960 flights requiring passengers to change planes en route to their
(C) Cars grew increasingly more expensive to repair over final destinations increased significantly.
the period in question (D) The proportion of business travelers using airlines
(D) The price of insurance increased more rapidly than increased significantly between 1977 and 1987 relative to
the rate of inflation between 1960 and 1990 the proportion of pleasure travelers.
(E) Health-care costs rose sharply between 1960 and 1990 (E) The average seating capacity of passenger aircraft
increased significantly between 1977 and 1987.
4. Some communities in Florida are populated almost
exclusively by retired people and contain few, if any, families 7. I. From 1975 to 1985, the yearly level of polychlorinated
with small children. Yet these communities are home to biphenyls, or PCBs, to which the average person is exposed
thriving businesses specializing in the rental of furniture for decreased significantly.
112
American Education Aids - LSAT - Logical Reasoning
II. From 1975 to 1985, the incidence of disease caused by Which of the following, if true about the period when prices
PCB exposure doubled from previous levels, which were for canned soup were rising, best resolves the discrepancy
already high. This was true even for the group of people whose described above?
exposure was average for each year during that period. (A) Because retail prices fell for both meat and vegetables,
Which of the following, if true, would best resolve the the major ingredients of most canned soups, canned soup
apparent contradiction between Statement I and Statement became more expensive than foods that could be prepared at
II above? home with these ingredients.
(A) The diseases caused by PCB’s are not very severe. (B) Dehydrated soup gained in sales at the expense of
(B) PCB’s are not easily destroyed by environmental forces. canned soup.
(C) From 1965 to 1975, the average level of exposure to (C) Because the retail prices of other foodstuffs rose faster
PCB’s also decreased. than the price of canned soup, canned soup became cheaper
(D) Between 1985 and the present, the average level of relative to other foodstuffs.
exposure to PCB’s increased to pre-1975 levels. (D) Because the retail prices and demand for most other
(E) The time between the period of exposure to PCB’s and foodstuffs remained stable, demand for them decreased
the onset of diseases caused by such exposure is often longer relative to demand for canned soup.
than 10 years. (E) Consumers became increasingly concerned about the
adverse effects of the high salt content in many varieties of
8. During the day in Lake Constance, the zooplankton canned soups.
D.hyalina departs for the depths where food is scarce and
the water cold. The zooplankton D. galeata remains near the 10. During the last twenty years, eleven percent of those
warm surface where food is abundant. Even though D. galeata people who received certification to practice in a particular
grows and reproduces much faster, its population is often profession were women, and all those people who received
outnumbered by D.hyalina. certification during those years obtained full-time positions.
Which of the following, if true, would help resolve the Nevertheless, only five percent of the full-time positions in
apparent paradox presented above? this profession are currently held by women.
(A) The number of species of zooplankton living at the Which of the following, if true, could explain the difference
bottom of the lake is twice that of species living at the surface. in the percentages mentioned in the passage above?
(B) Predators of zooplankton, such as whitefish and perch, (A) It was easier to obtain certification twenty years ago
live and feed near the surface of the lake during the day. than it is currently.
(C) In order to make the most of scarce food resources, (B) The majority of those currently in the profession were
D.hyaline matures more slowly than D.galeata. hired more than twenty years ago, when virtually everyone
(D) D.galeata clusters under vegetaion during the hottest in the profession was male.
part of the day to avoid the sun’s rays. (C) The women certified in the last twenty years have
(E) D.galeata produces twice as many offspring per tended to choose different specialties within the profession
individual in any given time period of time as does D.hyalina. than the men have tended to choose.
(D) Male and female members of the profession have been
9. It is a general rule of economics that, as the price of an paid according to equal pay scales for all of the past twenty
item rises, demand for that item falls and that when the price years.
falls, demand rises. Yet in the United States in 1980, after (E) Although women currently hold five percent of the full-
the price of canned soup rose, demand for canned soup also time positions in the profession, they hold only two percent
rose. of the supervisory positions.

113
Chapter 12
'ANALOGY' QUESTIONS
In another, but less often asked, type of questions in will negotiate with the kidnappers. Moreover, negotiating with
Critical Reasoning, the given narration and the five choices the kidnappers is not, by itself, an illegal act. So, (A) does not
underneath will deal with totally different subjects or have the same logical structure as the given narrative, and is
phenomenon. not the answer.
But one statement among the five choices will incorporate (B) also does not say that if one particular person does
an argument (or a flaw) which is similar in logical structure not refuse to register for military service, someone else will
to the one in the given narration, and you will be asked to refuse. So, (B) is not the answer.
spot this choice as your answer. (C) means that no illegal act was in fact involved in the
Let us see two examples of this type: case discussed. Moreover, it does not follow the pattern “If X
does not do it, Y will do it”. So, (C) is not the answer.
Example 1 (E) says that it is quite habitual for many supervisors to
It is true that it is against international law to sell disobey a rule. But it is not of the form, “If one supervisor
plutonium to countries that do not yet have nuclear weapons. had not disobeyed the rule, some other supervisor would have
But if United States companies do not do so, companies in disobeyed it”, and is not the answer.
other countries will.
Which of the following is most like the argument above in Example 2
its logical structure? Jet fighters have recently been equipped with electronic
(A) It is true that it is against the police department’s improvements enabling the pilot to shoot down an enemy
policy to negotiate with kidnappers. But if the police want to plane while still out of eight. There is, however, the following
prevent loss of life, they must negotiate in some areas. problem: there is no sure way of determining whether a plane
(B) It is true that it is illegal to refuse to register for that is out of sight is friend or foe.
military service. But there is a long tradition in the United Which of the following products suffers from a drawback
States of conscientious objection to serving in the armed that, in its logical features, is most like the problem described
forces. above?
(C) It is true that it is illegal for a government official to (A) A fire alarm systemwith such a high heat and smoke
participate in a transaction in which there is an apparent threshold that it is likely to react too late to a developing fire
conflict of interest. But if the facts are examined carefully, it (B) An improved electronic ignition system whose
will clearly be seen that there was no actual conflict of interest superiority is limited to those rare times when it is perfectly
in the defendant’s case. adjusted
(D) It is true that it is against the law to burglarize (C) A product marketed as a weed killer that kills all
people’s homes. But someone else certainly would have plants to which it is applied before they resume active growth
burglarized that house if the defendant had not done so first. in the spring
(E) It is true that company policy forbids supervisors to (D) A jar lid designed to be childproof that children have
fire employees without two written warnings. But there have little difficulty removing
been many supervisors who have disobeyed this policy. (E) A cold medicine that relieves most symptoms of the
common cold but also causes spells of dizziness
Analysis
The structure of the given narrative is: If X (namely US Analysis
companies) did not commit an illegal act, Y (namely a The given narration implies that new electronic
company in some other country) was likely to commit that instruments in the jet fighters which are intended to destroy
act and get the benefit out of it. enemy planes may kill friendly planes also.
We must therefore look for that choice which has a similar The nearest analogy to this statement is weed killers
logical structure. (which are meant to destroy weeds) killing useful plants also.
(D) says, “If the defendant had not committed the illegal So, (C) is the answer.
act of burglary in that house, someone else would have done (Before spring, both useful plants and weeds would be
so first. This has the same logical structure as the given leafless, and it would be difficult to distinguish between them,
narrative, and is the answer just as it is difficult to distinguish between friendly and enemy
(A) talks of the role of the police in both the cases of planes when they are out of sight.)
‘negotiating’ and of ‘not negotiating' with the kidnappers. It You can easily see that none of the other choices is
does not say that, if the police do not negotiate, someone else analogous to the situation mentioned in the given passage.

You can now try the questions that follow by yourself.

114
American Education Aids - LSAT - Logical Reasoning
SET 1
1. Continuous indoor fluorescent light benefits the health are unsafe.
of hamsters with inherited heart disease. A group of them (C) Some judges are dishonest because they take bribes.
exposed to continuous fluorescent light survived twenty-five (D) Whales are in danger of becoming extinct; therefore,
percent longer than a similar group exposed instead to equal we should not trade with countries that allow whales to be
periods of indoor fluorescent light and of darkness. hunted.
The method of the research described above is most likely (E) Birds are dangerous because they all spread disease.
to be applicable in addressing which of the following
questions? 4. It is sometimes held that computer scientists would
(A) Can industrial workers who need to see their work do make better progress in developing sophisticated artificial-
so better by sunlight or by fluorescent light? intelligence programs if only they knew more about how
(B) Can hospital lighting be improved to promote the human beings think. This view is, however, open to the
recovery of patients? objection that not a single major step forward in airplane
(C) How do deep-sea fish survive in total darkness? design has come from any insights into the nature of bird
(D) What are the inherited illnesses to which hamsters flight.
are subject? The objection above draws on an analogy that assumes that
(E) Are there plants that require specific periods of artificial-intelligence programs are similar to which of the
darkness in order to bloom? following?
(A) Theories of human thought
2. Since 1978 when the copyright law was changed, books (B) Blueprints for airplanes
that are less than fifty years old must not be photocopied (C) Hypotheses about how science achieves progress
without the publisher’s permission. Thus, any book that has (D) Computer simulations of birds in flight
been photocopied since 1978 without the publisher’s (E) Research into the nature of bird flight
permission must be at least fifty years old.
The reasoning above exhibits a flaw similar to one in which 5. It often happens that some crisis or opportunity induces
of the following? people to find a practical use for things that originally had
(A) Any member of the solar system must be either a planet no serious purpose. As an example of this principle, consider
or a moon; so if an asteroid is neither a planet nor a moon, it the dolls and mannequins, programmed to move and built
must not be a member of the solar system for the delight of the wealthy in the eighteenth century, which
(B) Anyone who rides a city bus must buy a bus pass, and were fore-runners of the modern computer. Likewise, it is
since Demetrious has a bus pass, he must be riding on a city almost certain that the first domesticated animals were pets.
bus Domesticating of animals seems to have arisen as an
(C) A driver who turns right must signal, so any driver amusement long before it had practical application.
who did not signal must not have turned right Which of the following, if true, provides another example
(D) Anyone who legally crosses a national boundary must in support of the principle mentioned above?
have a passport; thus anyone who does not have a passport (A) The discovery of America was a by-product of the search
cannot legally cross a national boundary. for ginger, cloves, pepper, and cinnamon.
(E) Any wage earner residing in the state must pay state (B) Children’s games often imitate adult work.
taxes, so since Blodwen pays state taxes, she must be resident (C) The spyglass was simply a source of diversion until its
in the state. commercial and scientific potential was recognized, and its
power of magnification suitably improved.
3. The widespread development of industrial robots is (D) In certain cultures horses are used exclusively for
contributing to unemployment; therefore, ‘intelligent’ pleasure, and never for work, even though in those cultures
machines are dangerous. people are forced to work arduously in the absence of laboring
The argument above is most like which of the following? animals.
(A) Mussolini was an evil man; therefore, he became a (E) The persons who constructed moving the dolls and the
fascist. mannequins in the eighteenth century were also accomplished
(B) Eating bacon is harmful to health; therefore, fatty foods clockmakers.

115
Chapter 13
'MULTIPLE' QUESTIONS
In all the examples that we saw so far, there was only a tion of modern tools needs greater creativity.
single question asked under a single narration. (B) talks of two modern inventions (namely the steam en-
Occasionally, two questions are asked under a single nar- gine and technology of production of iron) and does not com-
ration, and you are asked to answer both of them using the pare them with ancient tools, and is not the answer.
information given in that narration. (C) is a general statement applicable to all inventions, and
Each of these two questions will fall under one of the 11 does not either strengthen or weaken the claim that invention
categories that we have discussed elaborately so far. of modern tools requires more creativity than was involved in
While answering the two questions, you should remember the invention of ancient hand tools. So, (C) is not the answer.
that each is independent of the other, and you should not use (D) weakens the author’s claim and is not the answer.
the information in one for the evaluation or answering of the (E) gives an argument why modern inventions need more
other. creativity than ancient ones, but does not assert that this need
Let us now see a few examples of this type. is fulfilled. So, (E) is not a better choice than (A).

Questions 1-2 are based on the following passage 2. (This is an ‘assumption’ question.) The author’s argu-
The creativity involved in the invention of modern tools is ment is that ‘if we deny that the creativity involved in in-
far greater than that involved in the development of early venting or designing tools is greater than the creativity in-
hand tools. To deny this is to underestimate contemporary volved in the designing of ancient tools, we would be underes-
intellect. timating contemporary intellect.’
Obviously, the author is assuming that the strength of the
1. Which of the following, if true, most strengthens the claim intellect of a population during a particular period can be
presented in the first sentence of the passage above? measured by its capacity to invent new tools.
(A) Early hand tools were discovered accidentally, rather (B) is what brings this out and is the answer.
than developed by imaginative application of knowledge
(B) The steam engine in its most sophisticated and power- Questions 3-4 are based on the following passage
ful form could not be produced until the technology to pro- The recent change in the Quarterly’s book-review format
duce iron was discovered is shortsighted and self-defeating. By adding a new column
(C) Necessity often prompts the creative invention of a tool ‘For the Serious Reader’ opposite the usual column ‘For Your
to meet the needs of a particular job Reading Enjoyment’, the editors think they will stimulate
(D) Modern technology has produced new variations of tools their audience’s interest in serious literature and thus in-
that are more attractive but less durable than the tools they crease the audience’s ability to distinguish significant cul-
replace tural trends from passing fads and diversions. This is not
(E) The technological needs of contemporary culture are the case. The titles of the two columns suggest that serious
more varied and complex than those of the past literature is not enjoyable reading, a suggestion that will al-
ienate the Quarterly’s readers from such literature. Thus,
2. The second sentence in the passage above is based on the audience will be less interested in serious literature and
which of the following assumptions? less able to distinguish cultural trends from passing fads.
(A) The greater complexity of electronic and computer- The editors of the Quarterly should leave well enough alone.
based technology requires greater intellect than does a tech-
nology based on hand tools and simple machines 3. Which of the following, if true, most weakens the author’s
(B) A civilization’s creativity in designing new tools is an argument that the new book-review format will alienate read-
index of that civilization’s intellect ers?
(C) Modern tools meet the requirement of a given job more (A) Only fifty percent of the subscribers to the Quarterly
successfully than do early hand tools read the book reviews
(D) The creative prowess of a civilization may be reflected (B) After the change in format, the Quarterly received nu-
in achievements other than advances over the technology of merous letters from readers who complained that a certain
previous generations book reviewed in one column should have been reviewed in
(E) Each age and civilization expresses its intellect through the other
inventions appropriate to the particular demands of that age (C) The columnist hired to write the Quarterly’s new col-
umn is a nationally respected literary critic
Analysis (D) Studies have established that the Quarterly suffered
1. (This is a ‘strengthens’ question.) The author is compar- no decline in readership after the change in format
ing the creativity of inventors of modern tools with the crea- (E) In a survey taken after the change in format, most
tivity of developers of ancient hand tools, and expresses the Quarterly readers were interested in reading more than half
opinion that the invention of the modern tools calls for greater of the books that had been reviewed in the new column
creativity.
We have been asked to spot that choice which would most 4. The author of the passage above assumes that the Quar-
strengthen the claim of the author. terly’s readers will draw which of the following conclusions
(A), which states that the development of early hand tools from the editor’s change of the book-review format?
was accidental and did not involve any creativity at all, will (A) The editors are attempting to stimulate the market for
most strengthen the argument of the author that the inven- their magazine
116
American Education Aids - LSAT - Logical Reasoning
(B) Books reviewed in ‘For the Serious Reader’ will tend (E) Ms. Dennis does not seem sympathetic to the problems
not to be enjoyable reading of her students.
(C) A reader of serious literature has more discriminating
tastes than other readers Analysis
(D) Other sections of the magazine will be changed in the (This is an ‘assumption’ question.) It is obvious that Ms.
near future Dennis has been a teacher of engineering students for a long
(E) A reader can enjoy serious literature as well as popu- time. According to her, her past students used to do their home
lar literature assignments promptly, but her present students are irregular
in doing such assignments. From this experience, she comes
Analysis to the conclusion that the present day engineering students
3. (This is a ‘weakens’ question.) The ‘alienation’ that the are lazier than the earlier ones.
author talks of is not from ‘the Quarterly magazine’, but from Obviously, her assumption is that laziness of students can
‘serious literature’. be measured by their promptness in doing home assignments.
The author’s argument is that, by naming the new column (B) states this, and is the answer.
as ‘For the Serious Reader’, reviewing serious literature in The other choices are completely off the mark, and are not
that column and printing it opposite the regular column ‘For relevant to the premise and the conclusion in the passage.
your Reading Enjoyment’, the readers of the magazine will
be forced to think that ‘serious literature is not for enjoyment’, 6. (This is an ‘argumentation’ question.) This question asks
and that they will therefore refrain from reading the books us to identify a flaw (or defect) in the reasoning of Ms. Dennis.
reviewed in the new column. She compares her present day students with her past stu-
(E) says that, in an actual survey conducted after the in- dents, and comes to the general conclusion that ‘present day
troduction of the new column, most of the readers of the maga- engineering students are lazier’. She obviously assumes that
zine said they were interested in reading more than half of her students are representative of all engineering students of
the serious books reviewed in the new column. If this is true, today. This assumption may not be correct, and constitutes a
it will weaken the author’s argument. So, (E) is the answer. flaw in her reasoning. So, (D) is the answer.
None of the other choices constitutes a flaw in her reason-
4. (This is an ‘assumption’ question.) The statement in the ing.
passage, “The titles of the two columns (namely ‘For the Seri-
ous Reader’ and ‘For Your Reading Enjoyment’) suggest that Questions 7-8 are based on the following passage
serious literature is not for enjoyable reading, a suggestion No botanist lives long enough to study the complete life
that will alienate the Quarterly’s readers from such litera- cycle of an individual California redwood tree. Nevertheless,
ture” implies that the author thinks that the readers will by observing many trees at different stages, botanists can
draw the conclusion that books reviewed in the column ‘For piece together the evolution of a single tree. Exactly the same
the Serious Reader’ will not be enjoyable reading. principle applies in astronomy to the study of the life story of
It is (B) which states this, and is the answer. globular clusters, huge spherical aggregations of about a mil-
lion stars all swarming about each other.
Questions 5-6 are based on the following passage
“On the whole’, Ms. Dennis remarked, “engineering stu- 7. Which of the following is an assumption made in the
dents are lazier now than they used to be. I know because passage above?
fewer and fewer of my students regularly do the work they (A) The methods of scientists in one field generally carry
are assigned.” over to other fields even if the subject matter is vastly differ-
ent.
5. The conclusion drawn above depends on which of the (B) Observations of the life cycle of a single individual have
following assumptions? little value in scientific studies.
(A) Engineering students are working less because, in a (C) Globular clusters at different stages of development
booming market, they are spending more and more time in- are accessible to astronomers for observation and study.
vestigating different job opportunities. (D) There are globular clusters that have not so far been
(B) Whether or not students do the work they are assigned detected by astronomers.
is a good indication of how lazy they are. (E) Redwoods and globular clusters must both be studied
(C) Engineering students should work harder than stu- intensively now, while they still exist in sufficient numbers.
dents in less demanding fields.
(D) Ms. Dennis’ students are doing less work because Ms. 8. Which of the following studies is most similar, in terms of
Dennis is not as effective a teacher as she once was. both the problem posed by the time dimension and the method
(E) Laziness is something most people do not outgrow. of coping with that problem, to the studies of botanists and
astronomers alluded to in the passage above?
6. Which of the following identifies a flaw in Ms. Dennis’ (A) A study of the annual growth cycle of the sugar maple
reasoning? whose method is to analyze many different individual trees
(A) Plenty of people besides engineering students do not in an effort to derive a composite picture
work as hard as they should. (B) A study analyzing the progression of lakes from their
(B) Ms. Dennis does not consider the excuses her students formation to their end as peat bogs whose method is to exam-
may have for being lazy. ine lakes at many different stages of this progression
(C) The argument does not propose any constructive solu- (C) A study charting developments in automotive engineer-
tions to the problem it identifies. ing whose method is to contrast comparable models from
(D) The argument assumes that Ms. Dennis’ students are many different years
representative of engineering students in general. (D) A study of the impact of computers on industrial soci-
117
American Education Aids - LSAT - Logical Reasoning
ety whose method is to monitor the degree of computeriza- their development and growth over a period of centuries, and
tion by calculating several significant indices every three their final siltation till they end up as peat bogs. (A peat bog
years is a hollow marshy ground that is wet but does not hold much
(E) A study of an extinct creature’s skeletal development of water because it had got silted up over centuries.)
whose method is to compare the extant skeletons of individu- Since water spreads of various stages of growth are avail-
als that died at different ages able on the globe at any one time, this choice satisfies both
the criteria mentioned in the question, and is the answer.
Analysis (A) is not the answer, because it talks of the annual growth
7. (This is an ‘assumption’ question.) The California red- cycle of the sugar maple plant, which can be easily studied by
wood tree lives for a few centuries. The first two sentences a single botanist many times over in his own lifetime.
state that, though no botanist lives long enough to study the (C) is wrong, because the development of automobiles has
complete life cycle of a single tree, scientists can piece together taken place only during the last 50 years and is well docu-
the evolution of a single tree by observing many redwood trees mented, and a single individual can charter their evolution
which are at different stages of growth. with this documented information.
The author then says that, in a similar manner, astrono- (D) also can be done by a single scientist in his lifetime
mers can also study the life story of globular clusters (whose using documented information.
life cycle spans millions of years). (E) appears to be the answer, but is wrong; while the method
We have been asked to spot the assumption that is being of study here is analogous to the cases of redwood trees and
made by the author in coming to the above conclusion. globular clusters, the problem is not caused by the limited
If all the redwood trees that can be observed at a particu- life span of the scientist. In fact, the extinct creature’s skel-
lar time by the botanists are of the same age, the botanists etal development can be fully studied by a scientist in the
will obviously not be able to reconstruct the life cycle of a course of just a few years, if all the samples of individuals
single tree by observing many of them. It is only the existence who died at different ages are made available to him. (Ex-
at any given time of a large number of redwood trees of differ- tinct means dead; extant means available)
ent ages and different stages of growth that makes it possible
for botanists to piece together the life cycle of a single tree. Questions 9-10 are based on the following passage
If this analogy is valid for the exercise of the astronomers The facts show that the fear of flying in airplanes is not
studying globular clusters, the obvious assumption is that rational. In 1986 alone, there were 46,000 fatalities in high-
globular clusters of different ages and at different stages of way accidents, but from 1980 to the present an average of
development, are presently available to the astronomers for only 77 per year died in accidents on major domestic airlines.
observation and study. It is (C) which states this, and is the The rate for regional airlines was only slightly higher.
answer.
The only two fields of study mentioned in the passage are 9. If the evidence cited above is accurate, which of the fol-
botany and astronomy. The existence of one commonalty be- lowing would be most important to know in order to evaluate
tween the methods of research in these two fields need not be the force of that evidence?
based on the assumption that methods in one field generally (A) Whether repeated airplane travel allays fear of flying
carry over to other fields. (For example, even between bota- in airplanes
nists and astronomers, the former use the microscope for their (B) Whether regional and domestic airlines spend the same
studies, while the latter use the telescope.) So, (A) is not the average amount of time per aircraft on maintenance
answer. (C) How many people reported a fear of flying in airplanes
(B) is wrong because, in the case of living organisms with that was strong enough to prevent them from travelling by
short life spans, such as a rabbit, even the observation of the air
life cycle of a single individual can give valuable scientific (D) How many people per year have travelled by highway
information about the full life cycle of the species in general. and how many by air since 1980
(D) may be a factual statement, but does not form a neces- (E) How much higher the accident rate has been for re-
sary assumption by the author for drawing his conclusion in gional airlines than for major domestic airlines since 1980
the passage.
Since redwoods can continue to be planted and preserved, 10. Which of the following, if true, would argue most strongly
and since globular clusters have life spans of millions of years, against the conclusion above?
the statement in (E) that both of them will cease to exist in (A) Since the inventory of spare parts kept at each airport
sufficient numbers in the foreseeable future is inconceivable. is smaller than in earlier years, planes are often delayed at
So, (E) is not an assumption in the passage. an airport while parts are flown in from another airport, and
then repairs are carefully made and checked.
8. (This is an ‘analogy’ question.) This question asks us to (B) Air fatalities from 1980 to the present have been con-
identify that choice which is similar to the study of life sto- centrated in the last two years, with the rate rising sharply.
ries of the redwood tree and globular clusters, in terms of (C) The number of reports of near collisions in midair in
both the problem posed by the time dimension and the method 1986 was less than half those in a typical year of the 1960’s,
of coping with that problem. even with double the traffic of the 1960’s.
We should therefore look for that choice which also involves (D) Many reported near collisions in midair are closer than
an evolution that is much longer than about 75 years (which regulations allow but are nevertheless without actual dan-
is the normal life span of a single scientist), and for which ger.
specimens at different stages of development are available at (E) Between 1980 and 1986, safety improvements in the
any given time. design of automobiles steadily improved their
(B) talks of the analysis of the progression of lakes from crashworthiness.
the time of their initial formation (probably as small ponds),
118
American Education Aids - LSAT - Logical Reasoning
Analysis be less than the number of fatalities per thousand of highway
9. (This is an ‘evidence analysis’ question.) From the fact passengers, and highway travel can be considered to be more
that there were 46,000 deaths in road accidents in 1986, but risky than air travel. In this case, the fear of flying in airplanes
the average number of deaths every year in air accidents on can be considered to be not rational.
major domestic airlines has been only 77 (and only slightly So, the answer to the question in (D) is most relevant for
more in regional airlines), the author draws the conclusion evaluating the evidence presented by the author. So, (D) is
that the fear of flying in airplanes is not rational. the answer.
He implies that, on the basis of the evaluation of this evi- (E) also does not talk of passengers by highways, and is
dence, one should be more afraid of travelling by road than of not the answer.
travelling by air.
We have been asked to identify the choice which seeks in- 10. (This is also an ‘evidence analysis’ question.) This ques-
formation which would be most important for evaluating the tion is also based on the same passage as the last one, but
force of the evidence presented by the author in drawing this does not relate to the evidence presented in the passage but
conclusion. to the conclusion in it, namely, “that fear of flying in airplanes
Since (A) is not relevant to the relative safety levels be- is not rational”.
tween air travel and road travel, it is not the answer. We have been asked to spot the choice which would most
The author himself states that the accident rate in regional strongly argue against this conclusion.
airlines is slightly higher than in major airlines. (B) refers to We should therefore try to locate the choice which implies
the relative levels of efficiency of maintenance of aircraft by that there is justification for the fear of flying in airplanes.
regional and domestic airlines, and does not refer to the acci- (A) implies that airlines take ample precautions for the
dent rates in road accidents at all. So, the answer to (B) is not safety of air travel, even at the cost of punctuality. If this is
a relevant information for evaluating the evidence presented true, it makes flying in airplanes quite safe, and does not jus-
by the author. tify the fear of flying. So, (A) is not the answer.
(C) also does not refer to travel by road, and is not the an- (B) implies that the average number of fatalities in air
swer. accidents in the last two years is much higher than 77 per
The given information is that the annual number of fa- year, and that this figure is even rising sharply. If this is true,
talities in airline accidents (77) is only about 1/600 of the it argues strongly against the conclusion in the passage that
number of fatalities (46,000) in road accidents. So, if the fear of flying in airplanes is not rational. So, (B) is the an-
number of travellers by highway is more than 600 times the swer.
number of travellers by airlines, the number of fatalities per (C) implies that airlines travel is safer today than in the
thousand of air passengers would be more than the number 1960’s. So, (C) supports, and does not contradict, the conclu-
of fatalities per thousand of highway passengers. In this case, sion arrived at in the passage. So, (C) is not the answer.
the fear of flying in airplanes can be considered to be valid (D) also implies that airlines travel is not risky, and is not
and rational. the answer.
On the other hand, if the number of travellers by highway (E) talks about automobiles and not about air travel, and
is less than 600 times the number of travellers by airlines, is not relevant to the conclusion arrived at in the passage.
the number of fatalities per thousand of air passengers would

You can now attempt a number of similar questions.

119
American Education Aids - LSAT - Logical Reasoning
SET 1
Questions 1-2 are based on the following (E) Weddings magazine includeså both articles about
Meteorologists say that if only they could design an catered receptions and articles about preparing food oneself
accurate mathematical model of the atmosphere with all its for one’s wedding reception.
complexities, they could forecast the weather with real
precision. But this is an idle boast, immune to any evaluation, 4. Which of the following, if true, would undermine the
for any inadequate weather forecast would obviously be validity of the investment advice in the paragraph above?
blamed on imperfections in the model. (A) The average wedding reception involves between 50
and 100 guests.
1. Which of the following, if true, could best be used as a (B) Approximately a quarter of all weddings take place
basis for arguing against the author’s position that the without a reception.
meteorologist’s claim cannot be evaluated? (C) Approximately a quarter of all weddings and their
(A) Certain unusual configurations of data can serve as associated receptions are paid for by the couples themselves.
the basis for precise weather forecasts even though the exact (D) Only half of all catered wedding receptions include sit-
caus al mechanisms are not understood down meals.
(B) Most significant gains in the accuracy of the relevant (E) Only half of those who say they want a catered wedding
mathematical models are accompanied by clear gains in the reception actually have one.
precision of weather forecasts
(C) Mathematical models of the meteorological aftermath Questions 5-6 are based on the following
of such catastrophic events as volcanic eruptions are Blood banks will shortly start to screen all donors for
beginning to be constructed NANB hepatitis. Although the new screening tests are
(D) Modern weather forecasts for as much as a full day estimated to disqualify upto 5 percent of all prospective blood
ahead are broadly correct about 80 percent of the time donors, they will still miss two-thirds of donors carrying
(E) Meteorologists readily concede that the accurate NANB hepatitis. Therefore, about 10 percent of actual donors
mathematical model they are talking about is not now in their will still supply NANB-contaminated blood.
power to construct
5. The argument above depends on which of the following
2. Which of the following, if true, would cast the most serious assumptions?
doubt on the meteorologists’ boast, aside from the doubt (A) Donors carrying NANB hepatitis do not, in a large
expressed in the passage above? percentage of cases, carry other infections for which reliable
(A) The amount of energy that the Earth receives from the screening tests are routinely performed
Sun is monitored closely and is known not to be constant (B) Donors carrying NANB hepatitis do not, in a large
(B) Volcanic eruptions, the combustion of fossil fuels, and percentage of cases, develop the disease themselves at any
several other processes that also cannot be quantified with point
any accuracy are known to have a significant and continuing (C) The estimate of the number of donors who would be
impact on the constitution of the atmosphere disqualified by tests for NANB hepatitis is an underestimate
(C) As current models of the atmosphere are improved, (D) The incidence of NANB hepatitis is lower among the
even small increments in complexity will mean large increases potential blood donors than it is in the population at large
in the number of computers required for the representation (E) The donors who will still supply NANB-contaminated
of the models blood will donate blood at the average frequency for all donors
(D) Frequent and accurate data about the atmosphere
collected at a large number of points both on and above the 6. Which of the following inferences about the consequences
ground are prerequisite for the construction of a good model of instituting the new tests is best supported by the passage
of the atmosphere above?
(E) With existing models of the atmosphere, large scale (A) The incidence of new cases of NANB hepatitis is likely
weather patterns can be predicted with greater accuracy than to go up by 10 percent
can relatively local weather patterns (B) Donations made by patients specifically for their own
use are likely to become less frequent
Questions 3-4 are based on the following (C) The demand for blood from blood banks is likely to
Now is an excellent time to invest in the catering business. fluctuate more strongly
A survey conducted by Weddings magazine found that 70 (D) The blood supplies available from blood banks are likely
percent of the magazine’s readers want a catered wedding to go down
reception. An analysis of the catering industry, however, (E) The number of prospective first-time donors is likely
shows that the current number of caterers can serve only 55 to go up by 5 percent
percent of the weddings likely to occur each year.
Questions 7-8 are based on the following
3. Which of the following, if true, reveals a weakness in Record companies defend their substitution of laser-read
the evidence cited above? compact discs (CD’s) for the much less expensive traditional
(A) Catering is a highly labor-intensive business. long-paying vinyl records in their catalogs by claiming that
(B) Caterers are not evenly distributed across the country. the audio market is ruled by consumer demand for ever-
(C) The number of weddings with catered receptions has improved sound reproduction rather than by record
been growing each year for the past five years. manufacturer’s profit-motivated marketing decisions. But
(D) Readers of Weddings magazine are more likely than this claim cannot be true, because if it were true, then digital
most people to want a catered reception. audio tape, which produces even better sound than CD’s,
120
American Education Aids - LSAT - Logical Reasoning
would be commercially available from these same record performances played back on the highest quality disc or tape
companies, but it is not. player.

7. Which of the following, if true, best explains how the Questions 9-10 are based on the following
record companies’ claim about the nature of the audio- In the past, teachers, bank tellers, and secretaries were
reproduction market could be true and digital audio tape predominantly men; these occupations slipped in pay and
nevertheless be unavailable for the commercial market? status when they became largely occupied by women.
(A) Most consumers prefer audio tape to long-playing Therefore, if women become the majority in currently male-
records or CD’s because of the tape’s durability and dominated professions like accounting, law, and medicine,
compactness. the income and prestige of these professions can also be
(B) Prototypes of digital audio tape have been used to make expected to drop.
master tapes of some performances in recording studios.
(C) The manufacturing technology that underlies the 9. The argument above is based on
commercial production of CD’s requires equipment very (A) another argument that contains circular reasoning
similar to that needed for commercial production of digital (B) an attempt to refute a generalization by means of an
audio tape. exceptional case
(D) Record companies have not yet solved several quality- (C) an analogy between the past and the future
control problems that have beset attempts to product digital (D) an appeal to popular beliefs and values
audio tape in commercial quantities. (E) an attack on the character of the opposition
(E) CD’s are more expensive than long-playing vinyl
records by about the same ratio as digital audio-tape cassettes 10. Which of the following, if true, would most likely be
would be more expensive than conventional cassettes. part of the evidence used to refute the conclusion above?
(A) Accountants, lawyers, and physicians attained their
8. Which of the following, if true, would most strengthen current relatively high levels of income and prestige at about
the argument against the record companies’ claim? the same time that the pay and status of teachers, bank
(A) When CD’s were first introduced in the audio- tellers, and secretaries slipped
reproduction market, prices were high and selection was poor. (B) When large numbers of men join a female-dominated
(B) Record companies are reluctant to attempt commercial occupation, such as airline flight attendant, the status and
production of digital audio tape until profits from the sales of pay of the occupation tend to increase
CD’s have enabled them to recover their investments in (C) The demand for teachers and secretaries has increased
compact-disc manufacturing technology. significantly in recent years, while the demand for bank tellers
(C) Some CD’s have been so much in demand that has remained relatively stable
consumers have experienced long delays in obtaining copies. (D) If present trends in the awarding of law degrees to
(D) Because CD’s work according to principles very women continue, it will be at least two decades before the
different from those that govern conventional recordings, majority of lawyers are women
commercial production of CD’s requires new kinds of (E) The pay and status of female accountants, lawyers,
manufacturing technology. and physicians today are governed by significantly different
(E) Any valid comparison of CD audio reproductions to economic and sociological forces than were the pay and status
digital audio tape reproductions must be based on identical of female teachers, bank tellers, and secretaries in the past

SET 2
Questions 1-2 are based on the following 2. Which of the following questions can be answered on the
In the United States, the Postal Service has a monopoly on basis of the information in the passage above?
first-class mail, but much of what is sent first class could be (A) Is the Postal Service as efficient as privately owned
transmitted electronically. Electronic transmittal operators electric transmission services?
argue that if the Postal Service were to offer electronic (B) If private operators were allowed to operate first-class
transmission, it would have an unfair advantage, since its mail services, would they choose to do so?
electronic transmission service could be subsidized from the (C) Do the electronic transmittal operators believe that the
profits of the monopoly. Postal Service makes a profit on first-class mail?
(D) Is the Postal Service prohibited from offering electronic
1. Which of the following, if each is true, would allay the transmission services?
electronic transmittal operators’ fears of unfair competition? (E) Is the Postal Service expected to have a monopoly on
(A) If the Postal Service were to offer electronic electronic transmission?
transmission, it could not make a profit on first-class mail
(B) If the Postal Service were to offer electronic Questions 3-4 are based on the following
transmission, it would have a monopoly on that kind of service Bank depositors in the United States are all financially
(C) Much of the material that is now sent by first-class protected against bank failure because the government
mail could be delivered much faster by special package insures all individuals’ bank deposits. An economist argues
couriers, but is not sent that way because of cost that this insurance is partly responsible for the high rate of
(D) There is no economy of scale in electronic transmission bank failures, since it removes from depositors any financial
- that is, the cost per transaction does not go down as more incentive to find out whether the bank that holds their money
pieces of information are transmitted is secure against failure. If depositors were more selective,
(E) Electronic transmission will never be cost-effective for then banks would need to be secure in order to compete for
material not sent by first-class mail such as newspapers and depositors’ money.
bulk mail
121
American Education Aids - LSAT - Logical Reasoning
3. The economist’s argument makes which of the following Questions 7-8 are based on the following
assumptions? If the airspace around centrally located airports were
(A) Bank failures are caused when big borrowers default restricted to commercial airlines and only those private planes
on loan repayments equipped with radar, most of the private-plane traffic would
(B) A significant proportion of depositors maintain accounts be forced to use outlying fields. Such a reduction in the amount
at several different banks of private-plane traffic would reduce the risk of midair
(C) The more a depositor has to deposit, the more careful collision around the centrally located airports.
he or she tends to be in selecting a bank
(D) The difference in the interest rates paid to depositors 7. The conclusion drawn in the first sentence depends on
by different banks is not a significant factor in bank failures which of the following assumptions?
(E) Potential depositors are able to determine which banks (A) Outlying airfields would be as convenient as centrally
are secure against failure located airports for most pilots of private planes.
(B) Most outlying airfields are not equipped to handle
4. Which of the following, if true, most seriously weakens commercial-airline traffic.
the economist’s argument? (C) Most private planes that use centrally located airports
(A) Before the government started to insure depositors are not equipped with radar.
against bank failure, there was a lower rate of bank failure (D) Commercial airliners are at greater risk of becoming
than there is now involved in midair collisions than are private planes.
(B) When the government did not insure deposits, frequent (E) A reduction in the risk of midair collision would
bank failures occurred as a result of depositors’ fears of losing eventually lead to increases in commercial airline traffic.
money in bank failures
(C) Surveys show that a significant proportion of depositors 8. Which of the following, if true, would most strengthen
are aware that their deposits are insured by the government the conclusion drawn in the second sentence?
(D) There is an upper limit on the amount of an individual’s (A) Commercial airliners are already required by law to
deposit that the government will insure, but very few be equipped with extremely sophisticated radar systems.
individuals’ deposits exceed this limit (B) Centrally located airports are experiencing
(E) The security of a bank against failure depends on the overcrowded airspace primarily because of sharp increases
percentage of its assets that are loaned out and also on how in commercial-airline traffic.
much risk its loans involve (C) Many pilots of private planes would rather buy radar
equipment than be excluded from centrally located airports.
Questions 5-6 are based on the following (D) The number of midair collisions that occur near
Roland: The alarming fact is that 90 percent of the people centrally located airports has decreased in recent years.
in this country now report that they know someone who is (E) Private planes not equipped with radar systems cause
unemployed a disproportionately large number of midair collisions around
Sharon: But a normal, moderate level of unemployment is centrally located airports.
5 percent, with 1 out of 20 workers unemployed. So at any
given time if a person knows approximately 50 workers, 1 or Questions 9-10 are based on the following
more will very likely be unemployed Professor A: We must make a strong moral statement
against Country X’s policies. Only total divestment - the sale
5. Sharon’s argument is structured to lead to which of the of all stock in companies that have factories or business offices
following as a conclusion? in X - can do this. Therefore, the University should divest
(A) The fact that 90% of the people know someone who is totally.
unemployed is not an indication that unemployment is Professor B: Our aim should be to encourage X to change
abnormally high its policies. Partial divestment is the best way to achieve this
(B) The current level of unemployment is not moderate aim. Therefore, the university should sell its stock only in
(C) If at least 5% of workers are unemployed, the result of companies that either sell goods to X’s government, or do the
questioning a representative group of people cannot be the majority of their business in X, or treat their workers in X
percentage Roland cites unfairly.
(D) It is unlikely that the people whose statements Roland
cites are giving accurate reports 9. Professor A’s and Professor B’s arguments differ in which
(E) If an unemployment figure is given as a certain percent, of the following ways?
the actual percentage of those without jobs is even higher (A) They state the same goal but propose different ways of
achieving it.
6. Sharon’s argument relies on the assumption that (B) They state different goals but propose the same way of
(A) normal levels of unemployment are rarely exceeded achieving them.
(B) unemployment is not normally concentrated in (C) They state different goals and propose different ways
geographically isolated segments of the population of achieving them.
(C) the number of people each of whom knows someone (D) They disagree about whether the university should sell
who is unemployed is always higher than 90% of the any stock at all.
population (E) They disagree about whether X’s policies are
(D) Roland is not consciously distorting the statistics he objectionable.
presents
(E) knowledge that a personal acquaintance is unemployed 10. Which of the following, if true, would be evidence that
generates more fear of losing one’s job than does knowledge the university would not be harmed economically if it followed
of unemployment statistics Professor A’s recommendation?
122
American Education Aids - LSAT - Logical Reasoning
(A) Very few of the companies in which the university owns additional cost to the companies.
stocks sell goods to X’s government. (D) The expected financial return to the university from
(B) Most companies that have factories or business offices stocks that the university could own under a policy of total
in X and in which the university owns stock actually do little divestment is approximately the same as the expected
of their business in X. financial return from the university’s current stocks.
(C) Some companies that have factories or business offices (E) If the university sold large blocks of stock under a policy
in X and in which the university owns stock have instituted of total divestment, the prices of the stocks of the companies
fair treatment polices for their workers in X at very little whose stocks were sold would probably decrease somewhat.

SET 3
Questions 1-2 are based on the following (C) If the restriction against advertisements that do not
The average life expectancy for the United State population specify fee arrangements is removed, more lawyers will
as a whole is 73.9 years, but children born in Hawaii will live advertise their services.
an average of 77 years, and those born in Louisiana, 71.7 (D) If more lawyers advertise lower prices for specific
years. If a newlywed couple from Louisiana were to begin services, some lawyers who do not advertise will also charge
their family in Hawaii, therefore, their children would be less than they currently charge for those services.
expected to live longer than would be the case if the family (E) If the only restrictions on the advertising of legal
remained in Louisiana. services were those that apply to every type of advertising,
most lawyers would advertise their services.
1. Which of the following, if true, would most seriously
weaken the conclusion drawn in the passage? 4. Which of the following, if true, would most seriously
(A) Insurance company statisticians do not believe that weaken the argument concerning overall consumer legal costs?
moving to Hawaii will significantly lengthen the average (A) The state has recently removed some other restrictions
Louisianian’s life. that had limited the advertising of legal services.
(B) The governor of Louisiana has falsely alleged that (B) The state is unlikely to remove all of the restrictions
statistics for his state are inaccurate. that apply solely to the advertising of legal services.
(C) The longevity ascribed to Hawaii’s current population (C) Lawyers who do not advertise generally provide legal
is attributable mostly to genetically determined factors. services of the same quality as those provided by lawyers
(D) Thirty percent of all Louisianians can expect to live who do advertise.
longer than 77 years. (D) Most lawyers who now specify fee arrangements in
(E) Most of the Hawaiian Islands have levels of air pollution their advertisements would continue to do so even if the
well below the national average for the United States. specification were not required.
(E) Most lawyers who advertise specific services do not
2. Which of the following statements, if true, would most lower their fees for those services when they begin to
significantly strengthen the conclusion drawn in the passage? advertise.
(A) As population density increases in Hawaii, life
expectancy figures for that state are likely to be revised Questions 5-6 are based on the following
downward. Surveys show that every year only 10 percent of cigarette
(B) Environmental factors tending to favor longevity are smokers switch brands. Yet the manufacturers have been
abundant in Hawaii and less numerous in Louisiana. spending an amount equal to 10 percent of their gross receipts
(C) Twenty-five percent of all Louisianians who move to on cigarette promotion in magazines. It follows from these
Hawaii live longer than 77 years. figures that inducing cigarette smokers to switch brands did
(D) Over the last decade, average life expectancy had risen not pay, and that cigarette companies would have been no
at a higher rate for Louisianians than for Hawaiians. worse off economically if they had dropped their advertising.
(E) Studies show that the average life expectancy for
Hawaiians who move permanently to Louisiana is roughly 5. Of the following, the best criticism of the conclusion that
equal to that of Hawaiians who remain in Hawaii. inducing cigarette smokers to switch brands did not pay is
that the conclusion is based on
Questions 3-4 are based on the following (A) computing advertising costs as a percentage of gross
The fewer restrictions there are on the advertising of legal receipts, not of overall costs
services, the more lawyers there are who advertise their (B) past pattern of smoking that may not carry over to the
services, and the lawyers who advertise a specific service future
usually charge less for that service than lawyers who do not (C) the assumption that each smoker is loyal to a single
advertise. Therefore, if the state removes any of its current brand of cigarettes at any one time
restrictions, such as the one against advertisements that do (D) the assumption that each manufacturer produces only
not specify fee arrangements, overall consumer legal costs one brand of cigarettes
will be lower than if the state retains its current restrictions. (E) figures for the cigarette industry as a whole and may
not hold for a particular company
3. If the statements above are true, which of the following
must be true? 6. Which of the following, if true, most seriously weakens
(A) Some lawyers who now advertise will charge more for the conclusion that cigarette companies could have dropped
specific services if they do not have to specify fee advertising without suffering economically?
arrangements in the advertisements. (A) Cigarette advertisements provide a major proportion
(B) More consumers will use legal services if there are fewer of total advertising revenue for numerous magazines.
restrictions on the advertising of legal services.
123
American Education Aids - LSAT - Logical Reasoning
(B) Cigarette promotion serves to attract first-time (E) It does not acknowledge that nonsmokers, even those
smokers to replace those people who have stopped smoking. who breathe smoke-filled air at work, are in general healthier
(C) There exists no research conclusively demonstrating than smokers.
that increases in cigarette advertising are related to increases
in smoking. Questions 9-10 are based on the following
(D) Advertising is so firmly established as a major business If there is an oil-supply disruption resulting in higher
activity of cigarette manufacturers that they would be international oil prices, domestic oil prices in open-market
unlikely to drop it. countries such as the United States will rise as well, whether
(E) Brand loyalty is typically not very strong among those such countries import all or none of their oil.
who smoke inexpensive cigarettes.
9. If the statement above concerning oil-supply disruptions
Questions 7-8 are based on the following is true, which of the following policies in an open-market
The lobbyists argued that because there is no statistical nation is most likely to reduce the long-term economic impact
evidence that breathing other people’s tobacco smoke on that nation of sharp and unexpected increases in
increases the incidence of heart disease or lung cancer in international oil prices?
healthy nonsmokers, legislation banning smoking in (A) Maintaining the quantity of oil imported at constant
workplaces cannot be justified on health grounds. yearly levels
(B) Increasing the number of oil tankers in its fleet
7. The argument reported above would be most seriously (C) Suspending diplomatic relations with major oil-
weakened if it were true that producing nations
(A) breathing smoke-filled air increases the incidence of (D) Decreasing oil consumption through conservation
headaches and cough in healthy nonsmokers (E) Decreasing domestic production of oil
(B) most nonsmokers dislike the odor of smoke filled air
(C) smoke filled air is a major source of the dirt that 10. Which of the following conclusions is best supported by
damages computers and other sensitive equipment the statement above?
(D) most workers would prefer to have smoking banned in (A) Domestic producers of oil in open-market countries are
workplaces excluded from the international oil market when there is a
(E) legislation banning smoking in workplaces decreases disruption in the international oil supply.
friction between smoking and nonsmoking workers and is (B) International oil-supply disruptions have little effect
easy to enforce on the price of domestic oil as long as an open-market country
has domestic supplies enough to meet domestic demand.
8. Of the following, which is the best criticism of the (C) The oil market in an open-market country is actually
argument reported above? part of the international oil market, even if most of that
(A) It ignores causes of lung cancer other than smoking. country’s domestic oil is usually sold to consumers within its
(B) It neglects the damaging effects of smoke-filled air on borders.
non-smokers who are not healthy. (D) Open-market countries that export little or none of
(C) It fails to mention the roles played by diet, exercise, their oil can maintain stable domestic oil prices even when
and heredity in the development of heart disease. international oil prices rise sharply.
(D) It does not consider the possibility that nonsmokers (E) If international oil prices rise, domestic distributors of
who breathe smoke-filled air at work may become more oil in open-market countries will begin to import more oil
concerned about their health. than they export.

SET 4
Questions 1-2 are based on the following exaggerated claims for their products
If television editors did not exercise control over television
commercials, many commercials would make exaggerated 2. Which of the following, if true, would most strengthen
claims for their products. After all, one third of the the argument above?
commercials reviewed each year by television editors are (A) Nearly half of the commercials submitted by some of
rejected and returned to the advertising agencies for revisions, the advertising agencies are returned to them by television
in many cases because claims made in the commercials are editors for revision
not sufficiently documented. (B) The standards used by television editors for judging
whether a commercial’s claims are sufficiently documented
1. The argument above depends on which of the following have varied widely throughout the history of television.
assumptions? (C) Commercials that contain insufficiently documented
(A) Before television editors began controlling commercials, claims about a product are often brought to the attention of a
most commercials made exaggerated claims for their products television editor by a producer of a competing product
(B) The claims made in many rejected commercials are (D) Many of the commercials accepted each year are
not sufficiently documented because the claims are commercials that were previously rejected by television
exaggerated editors and were revised to make more moderate claims
(C) Most of the claims made in commercials are not initially (E) Of the commercials returned to advertising agencies
well documented by the advertising agencies for revisions each year, many are returned because they are
(D) Most viewers of commercials believe the claims made in poor taste rather than because the claims made in them
in the commercials are not sufficiently documented.
(E) One-third of the commercials reviewed each year made
124
American Education Aids - LSAT - Logical Reasoning
Questions 3-4 are based on the following (D) I and II only
As proof of the effects that astral projection can have on (E) II and III only
society as a whole, a proponent cited studies comparing crime
rates and percentages of the population practicing astral 6. If the scholar discovered an undated fourth manuscript
projection. According to the proponent, benefits are notable D, written in a fourth hand, the wording of which is identical
when even a small percentage of the population practices to C except for the different spelling of a third word wherever
astral projection. In one major metropolitan area last year, it appears, the scholar would probably conclude that the
projectors grew in number from one percent to five percent of fourth manuscript was copied
the population. Concurrently, the crime rate in the area (A) accurately from A
dropped eight percent. In the suburbs of that area, where (B) imperfectly from B
projectors moved up from five percent to twenty percent of (C) imperfectly from C
the population, the crime rate decreased forty percent. (D) accurately from some fifth, lost manuscript
(E) imperfectly from some fifth, lost manuscript
3. In terms of its reasoning, the proponent’s argument is
most like which of the following? Questions 7-8 are based on the following
(A) The officers of the seventh precinct could not be guilty There are experts in this country who claim that there is a
of accepting payoffs. They are the most honest group of police surplus of doctors and that access to medical schools must
officers in the city. consequently be restricted so that the high quality of medical
(B) Insurance rates for medical malpractice continue to care can be maintained. These analysts, though, fail to point
rise far faster than does the cost of living. Consequently, we out that there is a severe shortage of doctors in low-income
can expect that medical school enrollments will drop. areas and that there is only an overabundance of physicians
(C) History has shown that totalitarian states always try where there are patients who can afford to pay high fees.
to extend their boundaries by subjugating their neighbors.
We must build up defenses around totalitarian states. 7. The passage above criticizes the experts by
(D) The entire sociology department at Webley State is (A) showing that their conclusions are merely repetitions
socialist. Every faculty member campaigned to make Marx of their assumptions
required reading. (B) attacking the experts personally rather than their
(E) Television threatens literacy. By all measures, reading arguments
capability in this country has declined since the advent of (C) challenging the inference that restricted access to
television. medical schools will result in a general reduction in the
number of practicing physicians
4. Which of the following statements, if true, most seriously (D) showing that their arguments rely on an ambiguity in
weakens the proponent’s argument for astral projection? the meaning of the word “restricted”
(A) The proponent receives an initiation fee from each new (E) introducing evidence that cases new light on their
astral projector claims
(B) The proponent’s study did not consider that many astral
projectors are former criminals 8. The reasoning of the experts depends on which of the
(C) The proponent’s study did not consider any other following assumptions?
explanations for the drop in the crime rate (A) A surplus of doctors exists in all fields of medicine
(D) The proponent’s study did not distinguish between the (B) Access to medical schools is easily gained
various types of crime (C) The cost of medical care is currently reasonable for all
(E) The suburbs typically have a lower crime rate than (D) A surplus of doctors undermines the high quality of
does the whole metropolitan area medical care
(E) The high quality of medical care has been ensured by
Questions 5-6 are based on the following medical schools
A scholar examines three manuscripts of a nineteenth-
century poem to determine the relationship among them. Questions 9-10 are based on the following
Manuscript A is in the author’s handwriting and was written In a swampy area of South America, the highly poisonous
in 1850. Manuscript B and C are in two different handwritings coral snake and a much less venomous kind of snake both
and date from the period 1855-1860. The wording of B is bear red, black, and white bands. Only 100 miles away, on
identical to A except for the different spelling of one word drier ground, the local type of coral snake lacks the red bands.
wherever it appears. The wording of C is identical to B except And there, the less poisonous species of snake lacks the red
for the different spelling of a second word wherever it appears. bands, too. The explanation is that, in both habitats, close
The scholar concludes that B was copied imperfectly from A imitation of the coral snake affords the less poisonous species
and that C was copied imperfectly from B. a measure of protection it would not otherwise have.

5. Which of the following hypotheses, apparently eliminated 9. Which of the following is most likely to be a major
by the scholar, is (are) consistent with the information given? assumption underlying the explanation in the passage above?
I. B and C were both copied independently and imperfectly (A) Coral snakes claim individual territories for
from A themselves, which they defend against other snakes
II. B was copied imperfectly from C (B) Red coloration is effective camouflage primarily in dry
III. A was copied imperfectly from C habitats, where natural reds are much more common than
(A) I only elsewhere
(B) II only (C) The sight of a coral snake has a strange paralyzing
(C) III only effect on certain of its prey animals
125
American Education Aids - LSAT - Logical Reasoning
(D) Predators whose diets include snakes tend to avoid differentiation among the less poisonous species of snake
those most poisonous and thus most dangerous to them mentioned
(E) Snakes of different species will peacefully coexist only (C) The less poisonous snake species mentioned produces
if they do not compete for the same foods the same venom that the coral snake does, but less of it at a
time
10. Which of the following can be logically inferred from (D) Coral snakes from different habitats also have different
the passage above? types of venom
(A) All varieties of coral snakes have red, black, and white (E) The protection that the less poisonous species of snake
bands mentioned derives from the coral snake means that the less
(B) The differentiation among coral snakes into those with poisonous species of snake must be useful to the coral snake
and those without red bands predates the corresponding in some way

SET 5
Questions 1-2 are based on the following income tax surcharge, which would affect automobile
According to Tristate Transportation Authority, making commuters and rail commuters alike.
certain improvements to the main commuter rail line would
increase ridership dramatically. The authority plans to Questions 3-4 are based on the following
finance these improvements over the course of five years by The program to control the entry of illegal drugs into the
raising automobile tolls on the two highway bridges along country was a failure in 1987. If the program had been
the route the rail line serves. Although the proposed successful, the wholesale price of most illegal drugs would
improvements are indeed needed, the authority’s plan for not have dropped substantially in 1987.
securing the necessary funds should be rejected because it
would unfairly force drivers to absorb the entire cost of 3. The argument in the passage depends on which of the
something from which they receive no benefit. following assumptions?
(A) The supply of illegal drugs dropped substantially in
1. Which of the following, if true, would cast the most doubt 1987.
on the effectiveness of the authority’s plan to finance the (B) The price paid for most illegal drugs by the average
proposed improvements by increasing bridge tolls? consumer did not drop substantially in 1987.
(A) Before the authority increased tolls on any of the area (C) Domestic production of illegal drugs increased at a
bridges, it is required by law to hold public hearings at which higher rate than did the entry of such drugs into the country.
objections to the proposed increase can be raised. (D) The wholesale price of a few illegal drugs increased
(B) Whenever bridge tolls are increased, the authority must substantially in 1987.
pay a private contractor to adjust the automated toll-collecting (E) A drop in demand for most illegal drugs in 1987 was
machines. not the sole cause of the drop in their wholesale price.
(C) Between the time a proposed toll increase is announced
and the time the increase is actually put into effect, many 4. The argument in the passage would be most seriously
commuters buy more tokens than usual to postpone the effects weakened if it were true that
of the increase. (A) in 1987 smugglers of illegal drugs, as a group, had
(D) When tolls were last increased on the two bridges in significantly more funds at their disposal than did the
question, almost 20 percent of the regular commuter traffic country’s customs agents.
switched to a slightly longer alternative route that has since (B) domestic production of illegal drugs increased
been improved. substantially in 1987.
(E) The chairman of the Authority is a member of the (C) the author’s statements were made in order to
Tristate Automobile Club that has registered strong embarrass the officials responsible for the drug-control
opposition to the proposed toll increase. program
(D) in 1987 illegal drugs entered the country by a different
2. Which of the following, if true, would provide the set of routes than they did in 1986
authority with the strongest counter to the objection that its (E) the country’s citizens spent substantially more money
plan is unfair? on illegal drugs in 1987 than they did in 1986.
(A) Even with the proposed toll increase, the average bridge
toll in the tristate region would remain less than the tolls Questions 5-6 are based on the following
charged in neighboring states. Hardin argued that grazing land held in common (that is,
(B) Any attempt to finance the improvements by raising open to any user) would always be used less carefully than
rail fares would result in a decrease in ridership and so would private grazing land. Each rancher would be tempted to
be self-defeating. overuse common land because the benefits would accrue to
(C) Automobile commuters benefit from well-maintained the individual, while the costs of reduced land quality that
bridges, and in the tristate region, bridge maintenance is results from overuse would be spread among all users. But a
funded out of general income tax revenues to which both study comparing 217 million acres of common grazing land
automobile and rail commuters contribute. with 433 million acres of private grazing land showed that
(D) The roads along the route served by the rail line are the common land was in better condition.
highly congested and drivers benefit when commuters are
diverted from congested roadways to mass transit. 5. The answer to which of the following questions would be
(E) The only alternative way of funding the proposed most useful in evaluating the significance, in relation to
improvements now being considered is through a regional Hardin’s claim, of the study described above?
126
American Education Aids - LSAT - Logical Reasoning
(A) Did any of the ranchers whose land was studied use 8. Which of the following, if true, most strengthens the
both common and private land? author’s argument?
(B) Did the ranchers whose land was studied tend to prefer (A) Many workers who have already lost their jobs to
using common land over using private land for grazing? automation have been unable to find new jobs.
(C) Was the private land that was studied of comparable (B) Many corporations that have failed to automate have
quality to the common land before either was used for grazing? seen their profits decline.
(D) Were the users of the common land that was studied (C) Taxes and unemployment insurance are paid also by
at least as prosperous as the users of the private land? corporations that are not automating.
(E) Were there any owners of herds who used only common (D) Most of the new jobs created by automation pay less
land, and no private land, for grazing? than the jobs eliminated by automation did.
(E) The initial investment in machinery for automation is
6. Which of the following, if true and known by the ranchers, often greater than the short-term savings in labor costs.
would best help explain the results of the study?
(A) With private grazing land, both the costs and the Questions 9-10 are based on the following
benefits of overuse fall to the individual user. To protect certain fledgling industries, the government of
(B) The cost in reduced land quality that is attributable to country Z banned imports of the types of products those
any individual user is less easily measured with common land industries were starting to make. As a direct result, the cost
than it is with private land. of those products to the buyers, several export-dependent
(C) An individual who overuses common grazing land might industries in Z, went up, sharply limiting the ability of those
be able to achieve higher returns than other users can, with industries to compete effectively in their export markets.
the result that he or she would obtain a competitive
advantage. 9. Which of the following can be most properly inferred from
(D) If one user of common land overuses it even slightly, the passage about the products whose importation was
the other users are likely to do so even more, with the banned?
consequence that the costs to each user outweigh the benefits. (A) Those products had been cheaper to import than they
(E) There are more acres of grazing land held privately were able to make within country Z’s fledgling industries.
than there are held in common. (B) Those products were ones that country Z was hoping
to export in its turn, once the fledgling industries matured.
Questions 7-8 are based on the following (C) Those products used to be imported from just those
In many corporations, employees are being replaced by countries to which country Z’s exports went.
automated equipment in order to save money. However, many (D) Those products had become more and more expensive
workers who lose their jobs to automation will need to import, which resulted in a foreign trade deficit just before
government assistance to survive, and the same corporations the ban.
that are laying people off will eventually pay for that (E) Those products used to be imported in very small
assistance through increased taxes and unemployment quantities, but they were essential to country Z’s economy.
insurance payments.
10. Which of the following conclusions about country Z’s
7. The author is arguing that adversely affected export-dependent industries is best
(A) higher taxes and unemployment insurance payments supported by the passage?
will discourage corporations from automating (A) Profit margins in those industries were not high enough
(B) replacing people through automation to reduce to absorb the rise in costs mentioned above.
production costs will result in increases of other costs to (B) Those industries had to contend with the fact that other
corporations countries banned imports from country Z.
(C) many workers who lose their jobs to automation will (C) Those industries succeeded in expanding the domestic
have to be retrained for new jobs market for their products.
(D) corporations that are laying people off will eventually (D) Steps to offset rising materials costs by decreasing labor
rehire many of them costs were taken in those industries.
(E) corporations will not save money by automating (E) Those industries started to move into export markets
because people will be needed to run the new machines that they had previously judged unprofitable.

127
Analysis - Chapter 2
'CONCLUSION' QUESTIONS
SET 1 are different kinds of alarm signals for different birds, for
different walking animals and for different reptiles.
Answers
Adult monkeys give many different alarm calls depending
1.D 2.E 3.B 4.C 5.E 6.E 7.D 8.B 9.A 10.E
on which enemy bird, or which enemy animal, or which enemy
Analysis reptile is on the prowl.
1. The given passage states that, for being accepted by the But infant monkeys know only three alarm calls - one for
delivery service, a package must fulfill both of the two any flying bird, the second for any walking animal, and the
conditions: third for any reptile.
(i) it must be within the established size limits; and The obvious explanation for this behaviour of infant
(ii) it must contain a return address. monkeys is that they first learn only to distinguish among
We have been asked to spot that choice which is a logical the three broad groups - birds, animals and reptiles. It is
conclusion that can be drawn from these two statements. only when they mature into adults that they are able to make
The given passage does not refer to the charges of delivery distinctions among different birds, different animals and
at all. So, (A) extends the scope of what is stated in the passage, different reptiles. So, (B) is the answer.
and is not a necessary conclusion from it. (A) contradicts the specific information in the passage that
(B) contradicts the second condition that, for being both adult and infant monkeys do use sound patterns that
accepted, the package must necessarily have a return address communicate information to the other monkeys in the group.
also. (C) is incorrect because, in the case of both adult and infant
(C) looks like a correct answer, but is not valid because, monkeys, the word ‘alarm’ is used. We can therefore infer
the statement that the delivery service will never accept that infant monkeys also know that the significance of
packages which do not fulfill the two conditions does not imply producing the sound is to alert other monkeys of impending
that it will necessarily accept all packages which fulfill these danger, or to seek help. This choice therefore contradicts the
two conditions. (For example, it may yet reject a package for information in the passage, and is wrong.
other reasons such as it being packed loosely, or that it contains (D) does not follow from the given passage, since the
a highly inflammable substance, a poisonous chemical etc.). passage does not mention the relative skills with which the
This choice also extends the scope of the given information. adult and the infant monkeys give the three basic alarm calls
(D) reiterates that a package which conforms to one of the that relate to a flying animal, a walking animal and a long,
rules but not the other will not be accepted by the delivery thin animal. (D) therefore seeks to extend the scope of the
service, and is the answer. information in the given passage, and is wrong.
As pointed out with reference to (A), the passage does not (E) is wrong for the same reason as (C), since the word
talk about charges at all. Therefore, (E) also extends the scope ‘alarm’ is used in the case of infant monkeys also, implying
of the given information, and is not a necessary conclusion that infant monkeys also recognize that eagles, leopards and
from it. snakes posed dangers to them. Thus (E) also contradicts the
information given in the passage.
2. (The ‘incumbent’ means a person who is holding office
as President of USA at the time of the election, and is seeking 4. The first sentence says that, if calcium is taken in
a second-term.) What the passage says is that if, during a recommended doses by teenagers, it gets stored by their bones.
President’s term, there had been an increase in the per-capita The second sentence says that, beyond the age of 35, it is
income of the people, his chances of re-election would be not possible to replenish (meaning to add to the stock of)
brighter. It even quantifies it by stating that, for every 1% calcium in the body, and the body’s needs for calcium beyond
increase in the per-capita income, he can get 2.2% more votes this age are met from the stock already available in the bones.
than he would otherwise have got. It seeks to substantiate The third sentence says that those who have more bone
this argument by pointing out that, in the only election year mass in their system are less likely to suffer from osteoporosis
since 1952 when the per-capita income had declined, the in their old age beyond 50.
incumbent President was defeated in the election. The obvious conclusion from these statements is that, if a
We have been asked to spot that statement among the person wishes to avoid osteoporosis in old age, he should
choices which would logically follow from this passage. consume the recommended levels of calcium when he is in
(A) contradicts the specific instance cited in the passage of his teens, and continue to take them. Neglecting such intake
an incumbent President having lost an election, and is wrong. when one is in his teens, but consuming calcium after one
(B) and (D) are irrelevant to the issue, because the passage crosses 35 (when the body’s capacity to absorb calcium has
talks of the prosperity of the general public, and not of the ceased), will not help him to avoid osteoporosis in his old age.
candidate. In other words, this means that, between a person who
(C) is also irrelevant to the passage because the passage had consumed sufficient quantities of calcium in his teens,
does not talk of the public image of the incumbent at all. and another person who started consuming calcium after he
(E) is what can be directly inferred from the passage, had crossed 35, the former is more likely to escape from
because it states that the result of an election is not solely osteoporosis in old age than the latter.
determined by the ideological positions of the candidates, but It is (C) which states this, and is the answer.
can be influenced by other factors such as an increase in the Since the passage says that calcium in the bone is not
per-capita income of the people. So, (E) is the answer. replenished beyond the age of 35, consumption of calcium after
the age of 35 does not result in the prevention of osteoporosis
3. What the passage means is that, among monkeys, there in old age. So, (A), (B) and (D) contradict the specific
128
American Education Aids - LSAT - Logical Reasoning - Analysis
information given in the passage, and are wrong. Since it is given that the method of dating is accurate
What the first sentence implies is that a person who had within a century, the date when the sculpture was heated
not consumed calcium in recommended levels when he was a could well be 1400, 1450 or 1500. So, (B), (C) and (D) are all
teenager is likely to develop osteoporosis after the age of 50. possible dates when the sculpture was made and also heated
(E) makes a sweeping generalization that persons who develop for the last time.
osteoporosis after the age of 50 had not consumed any calcium It is also possible that the sculpture was made in 1000,
at all when they were teen-agers, and is wrong. but was again heated between 1400 and 1500. So, (A) is a
possible claim.
5. While answering such questions, you should use the If the sculpture had been made only in 1900, it could not
signs ‘>’ to signify ‘greater than’ and ‘<‘ to signify ‘less than’, have been last heated in 1450 or thereabouts. So, it is (E)
and rewrite the given information on your scrap paper using which is not possible, and is the answer.
these symbols.
When names of persons are given in such questions, use 7. (Contentious means quarrelsome). We have been asked
their first letters while writing down the inequalities. (The to state which conclusion can be logically drawn from the
question setters always take care to see that no two names given statements.
start with the same letter.) The first statement means that all good debaters are
The given information can therefore be written as C < J, C intelligent. The second statement says that some good
> E, S < J, S < E, W > S, and W < J. debaters are contentious.
Now, for even better understanding, rewrite all these Combining these two, we can conclude that some intelligent
relationships using the single sign ‘<‘ for ‘shorter than’. persons (who are also good debaters) are contentious. (i)
So, the given relationships become The third statement is that all contentious persons are
C < J, E < C, S < J, S < E, S < W and W < J. boring. (ii)
The first, second and fourth relationships above can be When we combine (i) and (ii), we can conclude that some
combined as intelligent persons (who are both good debaters and
S < E < C < J (i) contentious) are boring.
The third relationship logically follows from the above, Among the choices, it is (D) which states this, and is the
and does not constitute new information. answer.
The fifth and sixth relationships can be combined as
S<W<J (ii) 8. (A) is wrong since Dryden cannot make deliveries after
Now (i) and (ii) are independent of each other, and we cannot dark unless Stan is also with him along with Estey.
determine the relative height of W vis-a-vis either E or C. (B) conforms to the given conditions, and is the answer.
The given question talks of another person B, and asks us (In fact, in addition to Stan, Estey will also be with him in
to identify the choice which will imply that B < C. this case).
Now, B will be < C, if B is < either S or E. (C) is wrong, since, during daytime, Dryden can make
Choice (A) talks of C and W only, and does not refer to B at deliveries even when Stan is not with him.
all. So, (A) is not the answer. (D) is wrong for the same reason.
Since E is found only in the first relationship, and his (E) is designed to trip you. You cannot assume that
relative height vis-a-vis W is not known, (B) cannot help us darkness falls always at 6 p.m. In summer, in northern
to answer the question “Is B < C?” unambiguously. So, (B) is latitudes, there could be daylight even beyond 10 p.m.
not the answer.
(C) means S < B < W. This inequality cannot help us to 9. The first part of the statement says that the imposition
determine whether B < C or not. So, (C) is not the answer. of the speed limit of 55 mph in 1974 resulted in an abrupt
(D) means E < B < J. Comparing this with (i) above, we fall in the accident rate.
cannot say whether B or C is taller. So, (D) is also not the The second part says that, while the speed limit has not
answer. been further reduced, the accident rate has continued to
Choice (E) can be expressed as B < W < E. If B < E, he diminish, though the average speed of vehicles on the
must also be < C, because of the relationship (i) above. So, it highways has risen (but still being less than the speed limit
is choice (E) which enables us to conclude that B is shorter of 55 mph).
than C. We have been asked to spot the logical conclusion that can
So, (E) is the answer. be drawn from these facts.
(In such questions, you can often save time by starting with The obvious conclusion from the given statements is that,
(E), because the question setter is unlikely to have made either while the imposition of the speed limit was certainly a cause
choice (A) or (B) as the answer.) for reduction in accident rate, it had not been the only cause
because, in that case, the continuing fall in the accident rate
6. A clay sculpture requires to be heated for being preserved when the speed limit has not been reduced further cannot be
forever in the future. explained, and so there must be other causes also (Examples:
The given experiment explains a way to determine the date improvements in the safety aspects of vehicles, physical
(within a century) when a particular sculpture that is segregation of vehicles plying in opposite directions etc.).
available today was last heated. Among the choices, it is (A) which states this, and is the
The question, on the other hand, asks us to identify that answer.
year among the choices in which the sculpture could not have Both (B) and (C) extend the scope of the information in the
been made. passage beyond what can be immediately inferred from it,
The second para implies that experts have obtained 1450 and are wrong.
as an estimate of the year in which the sculpture was last (D), if true, would have resulted in an increase, and not a
heated. decrease, in accidents, and thereby contradicts the given
129
American Education Aids - LSAT - Logical Reasoning - Analysis
information. unmotivated and looked down upon by parents, have a
(E) contradicts the first part of the given statement that tendency to make heavy use of marijuana. The author thus
the number of deaths per mile fell abruptly in 1955 as a result advances the theory that the use of marijuana may not be
of the imposition of the speed limit of 55 miles per hour. the cause of depression or amotivation, but may be the result
of it.
10. It is easy to see that the author’s primary concern is So, it is (D) that would be the most logical next sentence of
the continued existence of his country as a nation. He believes the passage.
that, in order to ensure its continuing as a nation, active Having mentioned the popular belief first, the author says
measures must be taken to preserve national security, even in the second sentence that ‘there is no evidence that the
though some of these measures may not be morally acceptable. drug does indeed produce’ the amotivational syndrome. This
We have been asked to spot that choice which follows only shows that he is skeptical of the popular belief, but does
logically from the author’s statements. not positively rule it out as totally untrue. Even in the next
Since the author talks only of security measures and not of sentence, he only says that young people who are already
foreign policy, (A) unnecessarily extends the scope of the depressed are more likely to make heavy use of marijuana.
information in the passage, and is not a logical conclusion Thus, he does not take a positive stand on the question, but
from it. merely advances a theory for consideration. So, (A), which
What the author says is that his country’s continued uses the word ‘concluded’, does not fit into the tenor of the
existence as a nation depends upon preserving national passage, and cannot be the next sentence of the passage.
security. His thinking therefore applies only to those countries The author does not deny that heavy use of marijuana
which wish to continue as a nation. (B) extends the scope of and amotivation syndrome go together. (His only question is
the information to cover all countries, and is therefore wrong. which is the cause and which is the effect.) Therefore (B) is
The author says that only some active measures that are not the answer.
taken to preserve national security may not be morally (C) also makes an assertive statement which is not
acceptable. (C), which implies that all such active measures warranted by the use of the phrase ‘are more likely than
are immoral, is an uncalled-for generalization of the author’s others’ in the third sentence. (Notice that, in the correct
statement, and is wrong. answer (D) which we have already identified, the phrase used
The author says that active steps to preserve the national is ‘may be a symptom’, which is as tentative as the phrase
security must be taken even though some of them may not be ‘more likely’ in the previous sentence, and therefore is a logical
morally justified. He does not say that these immoral steps corollary.)
would become moral just because they help the country to The choice (E) states something which is totally
preserve itself as a nation. So, (D) also extends the scope of contradictory to the syndrome mentioned in the three previous
the author’s statement beyond what can be immediately sentences, and cannot logically follow from them.
inferred, and is not the answer.
What the author means by the last sentence is that 3. (Alumni means old students) At present, it is the general
adherence to morality is less important than ensuring the council (which consists of all old students) that has the
nation’s security. (E) states just his, and is the answer. authority to change the constitution. The senate (consisting
of 20 members) can decide only routine matters by majority
SET 2 vote.
Answers The new proposal seeks to give the power to the senate
itself to change the constitution, provided there is unanimous
1.A 2.D 3.C 4.C 5.C 6.C 7.B 8.B 9.C 10.B
vote in the senate for such change.
Analysis
We have been asked to identify that choice which follows
1. The given passage means that, irrespective of the skills
logically from this proposal.
needed, some jobs traditionally have been paying lower wages
The result of the above proposal is that the power to change
while other jobs have traditionally been paying higher wages.
the constitution, which is now only with the general council,
It then adds that, for historical reasons, women have
will be transferred to the senate, provided the senate is
generally sought employment in jobs paying lower wages, even
unanimous about the proposed change.
if some of these jobs required higher skills.
What this implies is that constitutional changes can then
If this is true, the implication is that some jobs which are
be effected without the general council’s approval. It is (C)
usually taken up by men pay higher wages than some other
which states this, and is the answer.
jobs which require comparable skills but which have been
The proposal itself involves a change in the constitution
traditionally taken up by women.
and, under the rules at present, it will have to be passed by
It is (A) which states this, and is the answer.
the general council before it can be implemented. (A), which
None of the other choices follows from the given statement
says that the proposal will take effect if it receives the senate’s
as a necessary corollary.
unanimous support, is therefore wrong.
(B) is wrong, because the narrative says that, currently,
2. (Marijuana is the name of a habit-forming drug similar
the senate decides routine matters by majority vote, and
to opium and LSD. Syndrome means a group of symptoms
unanimity is not required. So, no individual senator has a
which are associated with a particular illness, or physical or
veto power at present.
mental condition.)
The narrative does not say how the members of the senate
The first sentence of the passage mentions the popular
are selected. So, (D) does not follow from the narrative, and
assumption that Marijuana causes apathy, depression and
is not the answer.
loss of motivation in people who use it heavily. The next two
Since the proposal seeks to transfer the power that is now
sentences go on to dispute this statement and say that, on
with the general body of the alumni to the senate, the voice of
the contrary, young people who are already depressed,
the alumni in the conduct of university affairs will, in fact,
130
American Education Aids - LSAT - Logical Reasoning - Analysis
diminish, and not increase. So, (E) is wrong. James controverts him saying “No, that is not true”, and
seeks to substantiate his rebuttal by saying that he knows a
4. The given narrative first says that an economy which couple of Americans ‘who just love chocolates’.
has a greater division of labour will also need a greater If James had misunderstood Joe as saying ‘Only Americans
coordination. like ice-cream’, his response would not have been ‘I know a
The second sentence gives the reason for this assertion thus: couple of Americans who just love chocolates’. So, (A) is not
A greater division of labour results in a larger number of the answer. (If, on the contrary, James’ response was, “I know
specialized producers, leading to a greater number of a couple of Europeans who just love ice-cream”, (A) would
disruptions of supply and production, thereby necessitating have been the answer.)
greater coordination. If James had misunderstood Joe as saying “I do not like
The third sentence then says, “There is always more ice-cream”, his response would not have been “I know a couple
division of labour in market economies than in planned of Americans who just love chocolates”. So, (B) is not the
economies”. answer.
The question then asks us, “If the statements above are If he had misunderstood Joe as saying, “Americans like
true, then which of the following must also be true?” only ice-cream (and nothing else)”, his counter-statement, “I
In other words, we have been asked to identify that choice know a couple of Americans who just love chocolates (and
which necessarily follows logically from the given statements. therefore what you say is not true)” would have logical force.
The third statement is that “there is more division of labour So, (C) is the answer.
in market economies than in planned economies”. The first Obviously, (D) is not related to what either Joe or James
statement is that “the greater the division of labour in an says.
economy, the greater the need for coordination”. These two Since Joe does not mention chocolates at all, the question
statements together imply that there is greater need for of James misunderstanding him to mean that those who like
coordination in a market economy than in a planned economy. ice-creams will not like chocolates does not arise. So, (E) is
It is choice (C) which states this, and is the answer. not the answer.
(A) contradicts what is stated in the given narrative,
because it is market economies that have greater division of 7. The given statement makes a positive assertion that
labour, and will therefore have more frequent disruptions in any car built after 1965 must have harness-style seat belts in
supply and production than planned economies. So, (A) is the front seats.
not the answer. The question asks you to examine the five choices given
Similarly, (B) and (D) also contradict what is stated in the below and pick out the one from which the given statement
given narrative, and can be discarded. can logically follow.
The passage is not on how effectively the division of labour (A) states that harness-style seat belts first appeared in
functions in different economies, but is on the quantum of cars in the late 1950’s. From this statement alone, we cannot
division of labour present in different economies. So, (E) is logically infer that all cars built after 1965 must have such
not relevant to the given conclusion, and is not the answer. belts. So, (A) has to be discarded.
The second part of (B) specifically lays down that all cars
5. The result of the ‘muscular movement’ mentioned is the built after 1965 were required to have seat belts in the front
expression of smile. seats, and the first part says that no seat belts other than
What the given statement means is that there is no harness-style ones were used in front seats of cars after 1965.
difference between the smile of blind and deaf children and By combining these together, it can be concluded that “if a
the smile of normal children. car was built after 1965, it has harness-type seat belts in the
We have been asked to spot the choice which is a logical front seats”. So, (B) is the answer.
conclusion that can be drawn from this statement. (C) states that some cars built before 1965 have harness-
Children normally learn language as well as patterns of style seat belts in the front seats. The only conclusion that
expression by imitation, and this imitation is generally can follow from this statement is that many cars built before
through seeing and hearing the adults. Therefore, if 1965 did not have harness-style belts in the front seats. You
experiments prove that the expression of smile is the same cannot conclude from this statement that all cars built after
even in children who can neither see nor hear, the conclusion 1965 had such belts in the front seat. So, (C) is not the answer.
must be that smiling comes naturally to children and is not (D) talks generally of seat belts, and does not even mention
the result of imitation through sight or sound. So, (C) is the harness-type seat belts. Secondly, it only implies that many
answer. people who had seen the usefulness of seats belts in racing
Among the other choices, (A) appears to be a plausible cars had them installed in their cars voluntarily. So, we
answer. But it is wrong, because elders do not consciously cannot draw a conclusion from (D) that all cars built after
‘teach’ gestures to the children, but the children voluntarily 1965 definitely have seat-belts, and that too of the harness-
pick up these gestures by imitation, and this imitation is type. So, (D) is not the answer.
normally through sight and sound. Moreover, (A) talks of all (E) can only lead us to infer that, if a car was built after
facial expressions, whereas the given statement relates to 1965, it will have some type of seat belts in the front seats. It
only ‘certain patterns of muscular movement such as those does not follow that the seat belts will be of harness-type only.
involved in smiling’. So, (E) is not the answer.
The other choices are too general, and none of them is an
immediate conclusion that can be drawn from a study of 8. We learn from the given information that (i) all empty
children who are both deaf and dumb. canvasses sold by retailers in USA and Europe to artists were
made in Europe; (ii) some of these canvasses bear the names
6. The simple statement made by Joe is that all Americans and addresses of the retailer, either European or American,
like ice-cream. selling it to an artist; (ii) the wooden stretchers on which the
131
American Education Aids - LSAT - Logical Reasoning - Analysis
art by a US artist was mounted was always made of American municipal staff or leakage of water from a water truck etc.
wood; (iii) finished paintings imported into USA from Europe So I and III need not be true.
were always mounted on European stretchers, though these If it is raining, then both the streets and the sidewalks
were often replaced by American stretchers after import. must be wet. If the streets alone are wet but the sidewalks
In the case of (A), it is not probable that a European retailer are not wet, the wetness of the streets must be due to some
would have sold an empty canvas to an American painter. It other reason (such as the operation of the fire service, cleaning
is more logical to surmise that the painting was done by a by the municipal staff, or leakage of water from a water truck
European artist and the stretcher was changed to American etc.), and not due to rain. So, II must be true.
wood after its import to USA. So, (A) is not the answer. So the answer is (B).
(B) is likely to be true because the American retailer is not
likely to have sold it to a European painter in those ‘pre- SET 3
industrial’ days when trans-Atlantic travel by artists would Answers
have been improbable. So, (B) is the answer
1.D 2.B 3.B 4.E 5.A 6.B 7.C 8.D 9.C 10.C
In the case of (C), it is not likely that an American retailer
Analysis
would have sold a canvas to a European artist. Even if a
1. The statement “If a dormitory room is above the second
European artist had bought one from him, his finished
floor, it has a fire escape” does not mean that a first floor or a
painting would have arrived in USA mounted on a stretcher,
second floor dormitory room should not have a fire escape.
though, in some cases, these stretchers might have been
It only means that while fire escapes are optional for the
replaced by American stretchers after arrival. Consequently,
first and the second floors, it is compulsory for the third,
no European painting is likely to be found without a stretcher.
fourth, fifth floors ....etc.
So, (C) is not the answer.
So (A), (C) and (E) need not be true.
(D) relates to a canvas that is unmarked, and might
(B) contradicts the given statement.
therefore have been sold either by a European retailer to a
(D) is the answer, because the fourth floor dormitory rooms
European painter or by an American retailer to an American
must necessarily have fire escapes as per the given condition.
painter. The painting also might be either by a European or
an American artist, because even European paintings were
2. The given narrative first states that aspirin is capable
often remounted on American stretchers. So, (D) is not a
of eliminating the moderate fever that is associated with some
greater probability than (B).
illnesses.
(E) is also not the answer, because it is given that all
It then points out that, in spite of this property of aspirin,
European paintings arrived in USA on a stretcher and, often,
doctors do not advise its use because a moderate fever helps
were remounted on American stretchers after arrival. So, a
to build antibodies which inhibit the growth of many strains
painting without a stretcher is more likely to be an American
of disease-causing bacteria.
painting than a European painting.
We have been asked to identify that choice which can be
drawn as a conclusion from the above statements.
9. The first sentence says that outlawing (meaning
What the narrative says is that aspirin eliminates moderate
banning) all strikes by its workers is a costly mistake for a
fever. We cannot draw the conclusion from this statement that
local government, because all its labour disputes must then
aspirin is a pain killer. So, (A) is not the answer.
be settled by binding arbitration, without any negotiated
If moderate fever helps in producing antibodies that inhibit
public-sector labour settlement guiding the arbitrators.
the growth of bacteria that caused the illness, we can conclude
The second sentence states that strikes should therefore
that the elimination of the moderate fever through aspirin
be outlawed only for categories of public-sector workers for
will help the disease-causing bacteria to grow uninhibitedly,
whose services no acceptable substitute exists.
thereby prolonging the illness. (B) says just this, and is the
We have been asked to identify that choice which can be
answer.
drawn as a logical conclusion from the passage.
The passage says that moderate fever stimulates the
The first sentence calls the outlawing by a local government
activity of white blood cells. We can only conclude therefrom
of all strikes by its workers as a “costly mistake”, and gives
that the administration of aspirin for eliminating the
the reason as “because all its labour disputes must then be
moderate fever will result in such a stimulation not taking
settled by binding arbitration”. This implies that a recourse
place. We cannot conclude that aspirin will actively inhibit
to binding arbitration always results in less advantageous
the normal growth of the while blood cells. So, (C) is not
results for the local government than it does for the workers.
correct.
This is also confirmed by the statement that, in such cases,
The passage does not talk of ‘white blood cells’ in general,
the arbitrators have to decide without the benefit of guidance
but of ‘disease-fighting’ white blood cells. We cannot conclude
from any negotiated public-sector labour settlements,
that more white blood cells means less severe illness. (In fact,
meaning that such negotiated settlements would be more
uninhibited production of white blood cells will cause blood
advantageous to the local government than the arbitrators’
cancer leading to the patient’s death.) So, (D) is incorrect.
awards. It is (C) which says this, and is the answer.
The focus of modern medicine is in eliminating the disease-
None of the other choices follows logically from the given
causing bacteria in the body, and not merely inhibiting their
information.
growth. So, (E) is wrong.
10. The given statement is “whenever it rains, the streets
3. The first part of the statement says that municipal
and sidewalks are both wet”. It does not follow that the
government finds it politically impossible either to raise fares
converse (namely that whenever the streets and sidewalks are
or to institute cost-saving reductions in the city’s public
wet, it is raining) is true, because the wetness of the streets
transportation service.
and sidewalks might have been caused by reasons other than
The last part of the statement implies that this has
rain - such as operation of the fire service or cleaning by
132
American Education Aids - LSAT - Logical Reasoning - Analysis
necessitated the subsiding of the operating costs of the reason for joining college.
transportation service from government funds. This means that, among the 1964 graduates, it was only a
The author argues that if public transportation is run by minority who had considered this an important reason for
a private firm, it will vigorously pursue profits, and will not their joining college. So, (B) is the answer.
need government subsidy for running the system. You can easily verify that none of the other choices logically
We have been asked to state what logical conclusion can follows as a conclusion from the given information.
be drawn from these statements.
The obvious conclusion from these statements is that 7. After the statement that “to produce seeds, plants must
private companies would not bother about political first produce flowers”, two distinctions are made between
considerations, and would readily raise fares, if necessary, in Russian terragon and French terragon:
order to cover operating costs. (i) The Russian terragon produces flowers, and the French
So, (B) is the answer. terragon does not.
The other choices are not logically related to the problem (ii) The Russian terragon’s leaves do not have flavour, and
that is stated and the solution that is offered. are not as desirable as French terragon for cooking.
We have been asked to identify that choice which can be
4. The author says that praising a historian for factual drawn as a conclusion from the given information.
accuracy in describing events is like praising an architect for From the first distinction, we can conclude that the French
using well-seasoned timber or properly mixed concrete in a terragon does not produce seeds, and is propagated in some
building. other way. Therefore, any seed packet that mentions ‘terragon’
What the author means is that it is the basic duty of an cannot relate to French terragon. So, (C) is the answer.
architect to use well-seasoned timber or properly mixed (A) is wrong, because the second distinction relates to their
concrete in a building designed and constructed by him, and ‘culinary’ (cooking) properties, and not to their ‘decorative’
he does not deserve praise for these alone. On the other hand, properties. Moreover, since it is the Russian terragon that
an architect deserves praise for a distinctive conceptualization produces flowers, it will be more suited as a decorative plant
of a building, its beauty, grace, harmony etc. than the French terragon.
By citing this analogy, the author suggests that it is the From the information that the leaves of Russian terragon
basic duty of a historian to be factually accurate, and that he do not have flavour, we cannot conclude that their flowers
does not deserve any special praise for such accuracy alone, also do not have flavour. So, (B) is wrong.
and that the praise, if any, must be reserved for other (D) and (E) do not necessarily follow from the given
accomplishments such as his innovative interpretation of information.
various historical facts, his relating them cogently and
logically to one another etc. 8. The first sentence says that (i) the wearing of a lap seat
In other words, the author argues that a historian has an belt, and (ii) the wearing a shoulder-and-lap seat belt, both
obligation to be factually accurate, but such accuracy alone protect passengers from major types of accidents in head-on
cannot be considered to be his most notable accomplishment. automobile collisions.
It is (E) which says this, and is the answer. The next two sentences state that, while passengers
wearing lap seat belts alone frequently suffer internal injuries
5. The first law (“plain English” law) is applicable to all caused by the seat belt itself, those wearing shoulder-and-
agreements, and the second (Rent Stabilization Code) is lap seat belts do not suffer such injuries.
applicable only to housing leases. We have been asked to spot the choice which could be a
When the first law is applied to the second, the implications logical conclusion from these statements about passengers
are involved in head-on automobile collisions.
(a) in cases where the expiring lease is in nontechnical (A) is wrong, because the passage states that shoulder-
language, the landlord and the tenant can themselves renew and-lap seat belt does give adequate protection to passengers
it without being represented by attorneys and from major types of injuries in head-long collisions.
(b) in cases where the expiring lease is in technical (B) is wrong, because internal injuries are associated only
language, it must be renewed as it is (because of the second with lap seat belts, and the passage does not say that
law) but, in this case, the landlord and tenant must both be passengers most often wear only this type of seat belt.
represented by attorneys (because of the first law) The given passage is only about protection to passengers
Choice (A) restates the situation described in (b) above, in the case of head-on automobile collisions, and does not
and is a valid conclusion that can be drawn from the given talk about protection in the case of other types of accidents
passage, and is the answer. such as, say, the overturning of an automobile. So, we cannot
(B) is wrong since the two legal requirements are not in infer from the passage that it is only head on automobile
conflict. collisions which cause more injury than any other type of
(C) is wrong because the second law prohibits the accident. In fact, the passage says that, if only shoulder-
renegotiation of the terms of an expiring lease. and-lap seat belts are worn, passengers can escape from major
(D) is wrong in cases where the expiring lease itself is in injuries even in the case of head-on collisions. So, (C) is wrong.
nontechnical language. Obviously, it is (D) which follows directly from the passage,
(E) is prohibited by the second law which states that the and is the answer.
renewal must be on the same terms and conditions as the According to the passage, even the wearing of only a lap
expiring lease. seat belt does protect passengers from major types of injuries
in head-on automobile collisions, though it does cause some
6. The ‘deep-seated’ difference between graduates of 1982 internal injuries. So, non-wearing of a lap seat belt may result
and 1964 is mentioned to be the fact that a majority of 1982 in the passenger suffering major injuries, and cannot be
graduates consider ‘making a good income’ as an important considered to be safer than wearing a lap seat belt. So, (E) is
133
American Education Aids - LSAT - Logical Reasoning - Analysis
wrong. is the answer.
According to the author, any work of literature is
9. (C) is the summary of the second sentence, and is the intertextual if it is written in response to any other work of
answer. literature. The example given in (E) is also of a work of
(A) is wrong, because the author himself concedes that literature that is written in response to another writing, and
some actions of individuals can affect the interests of others must therefore be considered to be intertextual. By saying that
in the society. it is not intertextual, (E) contradicts the information in the
(B) is wrong, because the author is only concerned with a given passage, and is not the answer.
person’s actions which prejudicially affect others, but not with
the actions which benefit others. 3. The first sentence in the passage says that when farmers
(D) is wrong, because the author says that some actions of try to produce high yields of the same crop year after year,
individuals may affect the interests of others. they need to apply high levels of fertilizers and pesticides,
and these high levels pollute water supplies.
10. Use the symbol > to indicate a higher melting point. The second sentence says that, in order to get the benefit
Then the given information can be written as X > P and P > of government’s price support for a crop, the farmers must
Y. These can be combined as X > P > Y have produced the same crop for the past several years.
The question given is “when is S > Y?”. The obvious conclusion that one can draw from these two
The obvious answer is that S will be greater than Y if S is sentences is that the rules for government support for farm
greater than either X or P. prices militate against the efforts to reduce water pollution.
(C) is what states this, and is the answer. So, (A) is the ready answer.

SET 4 4. It is given in the question that the grass is brown. You


Answers should proceed backwards from this statement.
So, from the last condition, we can say that the cherries
1.E 2.D 3.A 4.A 5.A 6.C 7.D 8.C 9.B 10.E
are not ripe.
Analysis
Then, from the previous condition, we can conclude that
1. From the second sentence, we can conclude that any
the berries are blue.
nursing room in the 2nd, 3rd or 4th floor must necessarily
The first condition says that the berries are not blue when
have a fire escape.
and only when the apples are green. So, applying the converse,
So, it is (E) which is a necessary conclusion from the given
we can say that, since the berries are found to be blue, the
statement, and is the answer.
apples are not green.
(A) need not be true, because the statement that a room
So, (A) is the answer.
above the first floor must have a fire escape does not mean
that a room in the first or ground floor should not have a fire
5. This is quite an easy question. The question asks you to
escape.
select that statement from the five given choices from which
(B) is the converse of this; the given statements by
the argument “If George was born in New York State, then
themselves do not mean that a fire escape is allowed for first-
he is citizen of the United States” immediately follows.
floor rooms in nursing homes.
It is quite easy to see that the answer is (A), which
(C) and (D) both contradict the given statements.
stipulates without any ambiguity that every person born in
New York State is a citizen of the United States.
2. The given passage says that masterpieces of literature
We can rule out (B) because it talks of citizenship and the
are written not in response to reality, but in response to other
place of residence, while the given main statement talks of
works of literature, and are therefore poor (or clouded) mirrors
citizenship and place of birth.
of social reality.
(C) can be ruled out because it implies that a person born
We have been asked to spot that choice which follows as a
in New York State may or may not be a citizen of the United
conclusion from the given information.
States. If that be so, it cannot be logically inferred that George
The author’s argument is that if a writing is intertextual
is a citizen of the United States just because he had been
(meaning ‘written in response to other works of literature’),
born in New York State.
it fails to mirror social reality. (A) reverses his argument by
It can be easily seen that the given statement does not
stating that a writing that fails to mirror social reality is
logically follow from (D) or (E).
intertextual. This does not follow necessarily from the given
passage.
6. Note that the given advertisement has been issued by
(B) contradicts the author’s view that all masterpieces are
Megacorp. It implies that, though the products sold by
indeed influenced by other works of literature.
Megacorp are already of high quality, the customers’
The author defines the term ‘intertextual’ as meaning
expectation levels of the quality of products are not static but
‘written in response to other works of literature’. A writing
are always on the rise. It then affirms that ‘meeting or
that is not intertextual may therefore be considered to be
exceeding customer expectation’ is its goal.
having no significant relationship to any other works of
If the customers’ expectation of the product quality is
literature, but it may have a relationship with writings other
always on the rise, such expectation can be successfully met
than literature. So, (C) is not a necessary conclusion from the
only if it is possible to continuously increase the product quality
given passage.
without there being a limit on such quality. It is (C) which
If what is stated in the passage is true, it follows that the
states this, and is the answer.
masterpieces of the past are all intertextual, and therefore
Since the advertisement is about Megacorp’s policy, and
cannot be considered to be works that accurately reflect the
not about its competitors, (A) is not the answer.
social reality of their periods. It is (D) which states this, and
The advertisement is about Megacorp’s policy whereas (B)
134
American Education Aids - LSAT - Logical Reasoning - Analysis
and (D) are general statements concerning all companies. So, it is (C) which is a logical conclusion that can be drawn
Obviously what is stated in them are not logical conclusions from the given information, and is the answer.
from Megacorp’s advertisement. The given passage does not talk of conditions under which
(E) contradicts the statement in the advertisement that a snowstorm will develop into a blizzard (which is a severe
customers are never satisfied with the quality of an existing snowstorm accompanied by strong winds, very heavy
product and always want an improvement in it. snowfall, lightning and thunder) and so, (D) is not a conclusion
that can be drawn from it.
7. The second sentence says that foundations were formerly (E) is also not a conclusion warranted from the information
willing to fund risky projects (meaning projects which did in the given passage, because the ground temperature may
not have much prospects of success) when more money was depend on many other factors such as the season, the latitude
available with them, but now, because of paucity of funds, of the place, its nearness to the sea etc.
they are inclined to support only relatively conservative
projects (meaning projects which have bright chances of 9. The given statements mean that mainland China will
success). allow Hong Kong to retain its capitalist ways only if it
We have been asked to spot that choice which can be drawn continues to stay prosperous. The logical consequence of Hong
as a logical conclusion from the above information. Kong’s failure to stay prosperous will be that mainland China
The quantum of money paid to each project is not referred will not allow it to continue to retain its capitalist ways. The
to at all in the passage. So, (A) is not a necessary conclusion consequence will not be that Hong Kong itself will no longer
from it. remain part of mainland China. So, (A) is not the answer.
‘Conservative’ projects mean projects which have a greater The first sentence says, “Hong Kong can retain its capitalist
chances of success. This word has not been used here in its ways after it becomes part of mainland China in 1997 as long
usual political meaning. So, (B) is not the answer. as a capitalist Hong Kong is useful to China”. The use of the
The first sentence of the passage gives the reason for the phrase ‘can retain’ in this sentence implies that the author is
paucity of funds with the foundations as ‘cuts in federal assuming that, on the day that it becomes part of mainland
spending’. (C) contradicts this fact by citing a different reason, China in 1997, it will have a capitalist economy. This means
and is not the answer. that if Hong Kong, in its status as a British Colony, retains
The fact that the foundations have decided to support its capitalist ways until 1997, it will be allowed to retain it in
relatively conservative projects implies that they do not want the immediately following months, and at least until such
the projects supported by them to fail. (D) states just this, time that it ceases to be prosperous. (B) states this, and is the
and is the answer. answer.
(E) refers to projects not supported by the foundations, and The given narrative does not say that world economic crisis
has no relevance to the given information. will never affect Hong Kong. So, (C) does not logically follow
from the given narration.
8. The first sentence says that dense cover of snow on earth (D) will be true only if Hong Kong remains capitalist as
can cause unusually harsh weather conditions to persist well as prosperous in 1997, at the time it becomes part of
(meaning to continue). The succeeding sentences give an China. Since this is not stated in (D), it does not follow from
explanation and an illustration of this phenomenon. the given narrative.
They state that if a severe snowfall (or winter storm) covers The given narrative relates only to the retention of capitalist
the Great Plains with snow, then this snow cover, being pure ways by Hong Kong, and does not mention anything about
white, reflects the sun’s radiation back into space, and keeps any other restrictions that may be placed on the citizens of
the temperature of the ground low. Consequently, the cold Hong Kong by the government of mainland China on religion,
air that moves down from Canada continues to remain cold, dress, etc.
and causes even more snowfall. This new snow will again So, (E) does not logically follow from the given narrative,
reflect the sunlight into space and will cause still further and is not the answer.
snowfall etc. This is how the harsh weather patterns,
triggered off by the initial severe winter storm, will continue 10. The question is about the expected behaviour of a
to exist. nuclear-missile defence system controlled by a complex
We have been asked to examine which among the choices computer program. The obvious purpose of the defence system
is a conclusion that can be logically drawn from this is to recognize a nuclear blast by the enemy as soon as it
information. occurs, and trigger the counter attack automatically without
(A) talks of normal winter weather, and does not mention human intervention which may delay such counter attack.
any unusual conditions. So, it is not the answer. The first paragraph of the passage says that meteorite
The Great Plains are mentioned only as an illustration of explosions can be as powerful as nuclear blasts, and the
the statement in the first sentence, and the author does not second paragraph says that the response of highly complex
indicate that they are more likely than other areas in the world computer systems to unexpected circumstances is
to suffer unusually harsh weather conditions. So, (B) is not unpredictable. The conclusion can therefore be drawn that
the answer. the computer system, if it is not specially programmed to
If, as stated in (C), the Great Plains get more snow than distinguish between a meteorite explosion and a nuclear blast,
usual early in winter (probably due to a severe snowstorm), might behave in an erratic manner in the event of a meteorite
then this snow will reflect the sunlight back into space and explosion, and recognize it wrongly as a nuclear blast and
keep the ground cool. If so, the cold air moving down from trigger an unprovoked attack on the enemy country.
Canada will remain cold enough to cause more snowstorms. It is (E) which states this, and is the answer.
The result will be that the winter that year is likely to be You can easily verify that none of the other choices is a
colder than usual. necessary conclusion from the given information.
This is exactly what is stated in (C).
135
American Education Aids - LSAT - Logical Reasoning - Analysis
SET 5 conclusion from the information above.
Answers What the passage implies is that different skills are
expected from fighter pilots at different periods depending
1.B 2.D 3.D 4.C 5.B 6.A 7.E 8.D 9.B 10.C
upon the technology at that time. If so, specific training
Analysis
programs for military personnel have to be updated
1. In the worst years of the early twentieth century, the
periodically so as to be in consonance with the changing needs
sequence of events in Southern United States was (i) the price
of actual combat situations.
of cotton went down, (ii) most of the population ate only
It is (C) which states this, and is the answer.
cornmeal, and (iii) the incidence of the debilitating disease
pellagra, which is known to be caused by a deficiency of niacin
5. This is a tricky question. The condition “whenever the
and certain other essential nutrients, went up.
sun is shining and the windchill factor is below zero, Susan
The logical conclusion that can be drawn from this
wears her parka” does not mean “whenever Susan wears her
sequence is that cornmeal is deficient in niacin and certain
parka, the sun must be shining and the windchill factor must
other essential nutrients.
be below zero”. So, (A) is wrong.
So, (B) is the answer.
(B) says that, in spite of one of the two conditions being
None of the other choices is a necessary logical conclusion
satisfied (windchill factor being below zero), Susan is not
that can be drawn from the given information.
wearing her parka. This means that the second condition
(that the sun must be shining) is not satisfied. So, (B) is the
2. You can notice that the given information does not talk
answer.
of trade between Japan and US at all.
(C) need not be true, because the presence of rain does not
What it says is that Japan’s export to all countries in 1980
mean that the sun is not shining, nor does the absence of
was $92 billion, while US’s export to all countries was just
rain mean that the sun is shining.
about $30 billion.
(D) does not fulfill the conditions in the second sentence,
It also says that the import by US from all countries during
and need not be true.
that year was only $26 (there being a $4 billion trade surplus),
(E) also need not be true, because it says that the windchill
while Japan had imported goods worth more than $92 billion
factor is zero, and not below zero, which is the condition for
(causing a trade deficit) from all countries.
Susan to wear parka.
You can easily see that what logically follows from the given
information is that Japan’s payment for imported goods in
6. It is given that, in spite of a ban on tobacco advertising
1980 far exceeded Unites States’ payment for imported goods
since 1975 in Norway, smoking is at least as prevalent among
during that year. It is (D) which states this, and is the answer.
teen-agers in that country as it is in countries that have not
The other choices are either unwarranted conclusions, or
banned tobacco advertising.
are factually wrong.
We can therefore conclude that tobacco advertising is not
the only factor that induces smoking among teenagers, and
3. (The narrative is given in a complicated form just in
that there must be other stronger factors which influence
order to confuse you.) We can paraphrase the given
teenagers uniformly in all these countries.
information as
So, (A) is the answer.
(i) vianzone + sodium chlorate = cloudy
While advertising may not be a factor in the prevalence of
(ii) vianzone + potassium nitrate = cloudy
smoking when the population of teenagers is considered as a
(iii) vianzone + benzene = not cloudy.
whole, it is not possible for us to come to a conclusion from
The question is: If vianzone + x = not cloudy, what could x
the given information that it is not a factor in a particular
be among the five given choices?
individual starting or continuing smoking. So, (B) cannot be
From (i), we can say that the solution did not contain
a valid conclusion from the given information.
sodium chlorate or potassium nitrate. So, (D) is consistent
(C) talks generally of consumption of tobacco, and not of
with the given information, and is the answer.
smoking, and is irrelevant to the issue.
(A) is wrong because of (ii) above.
(D) says that absence of tobacco advertising tempts more
(B) is wrong because of (i) above
teenagers to start smoking, a conclusion that is not only not
From (iii), we can only infer that x could be benzene; we
warranted from the given information but is totally illogical.
cannot assert that x could not be benzene. So, (C) is not the
The teenagers of 1975 would have attained middle age now,
answer.
and would not have been counted during a study of the
While (ii) will enable us to assert that the solution did not
smoking habit of teenagers of today in Norway. So, (E), even
contain potassium nitrate, we cannot say that it did not
though it may be true, is irrelevant to the issue.
contain benzene. So, (E) is not the answer.
7. We learn from the passage that
4. The first two sentences state that, during the second
(i) laws requiring the use of headlights during daylight
world war, an important ability that fighter pilots had to
hours can prevent automobile collisions
develop was the capacity to turn their heads frequently in order
(ii) countries which are farther from the equator than the
to look to their rear, for which they were trained through
continental United States, where daylight visibility is worse,
suitable calisthenic exercises.
have such laws
The last two sentences state that, at present, with the
(iii) continental United States does not have a law that
development of electronic instruments which have obviated
compels automobile drivers to use headlights during daylight
the need for pilots to look to the rear, what is important in a
hours
fighter pilot is the ability to detect even slight changes on
From this information, we can conclude that daylight
electronic dials and gauges.
visibility in continental USA is not so bad as to be considered
We have been asked to spot that choice which forms a valid
a factor in automobile collisions and, therefore, daytime
136
American Education Aids - LSAT - Logical Reasoning - Analysis
headlight laws are less likely to prevent automobile collisions why the number of passengers using public transportation
in continental USA than they do in countries further north. has fallen is that the price of gasoline is still not prohibitive
(E) is what states this, and is the answer. (because of which many persons can still afford to use their
The passage questions the need to make the use of daytime own private vehicles for travel).
headlight compulsory for automobile drivers in continental This implies that, once the price of gasoline becomes
United States, and does not say that use of daytime headlight prohibitive enough, many of them will curtail the use of their
will not play a role at all in preventing collisions. Therefore private vehicles, and resort to the use of public transportation,
(A) is not as appropriate an answer as (E). resulting in an increase in the number of passengers using
The passage itself says that the countries farther away public transportation.
from continental USA have daytime headlight laws which Choice (C) is what states this, and is the answer.
do prevent automobile collisions. So, (B) is a factually The implication of the given statement is that, even though
inaccurate statement, and is not a logical conclusion from the price of gasoline may register some increases, it will not
the given information. (This choice would have been correct have an effect on the number of passengers using public
if there had been no such law in these countries.) transportation, until the price reaches a prohibitorily high
(C) deals with the proportion of the number of accidents level. So, (A), which implies that there is a continuous inverse
during daytime to the number of accidents during nights both correlation between every increase in the price of gasoline
in continental USA and in the countries to its north. Since and the number of passengers using public transportation,
the passage does not deal with accidents at nights at all, (C) is not borne out by the given statement.
is not a necessary conclusion from it. (B) and (D) both contradict what is stated in the given
The passage talks only of collisions that are caused by poor paragraph, and are wrong.
visibility during daytime, whereas (D) refers to all collisions (E) partially supports what is stated in the given
that could arise because of various other factors such as poor paragraph, but does not deal with the contingency when the
maintenance of vehicles, driver negligence, drunkenness of price of gasoline may increase to a prohibitive level.
drivers etc. Obviously (D) cannot be a logical conclusion from
the given passage. SET 6
Answers
8. The passage implies only that, when the percentage of
1.D 2.E 3.D 4.C 5.D 6.D 7.D 8.D 9.B 10.B
expenditure on research and development to the GNP of a
Analysis
country is reduced, the number of inventions by its scientists
1. The first sentence says that, though many of the present
also comes down. It does not say that the number of inventions
buildings use the best of materials, they are still structurally
is related to the size of the country’s GNP. So, (A) is not the
defective because of indifferent design and sloppy construction.
answer.
The second sentence says that this situation is further
The passage talks of only percentages, and does not give
aggravated during maintenance, because the maintenance
the actual figures of GNP of USA, Japan or West Germany.
personnel do not even study the designs of these buildings.
So, (B) is not a valid conclusion that can be drawn from the
What the author implies is that, if the maintenance
given information. (It is quite likely that 3.2% of West
personnel are aware of the design of the building, the further
Germany’s GNP is much less than 2.2% of USA’s GNP.)
damage during the maintenance stage will be much less. If
The passage does not give the number of inventions made
this is valid, the most effective way to contain damages at
by West Germany and Japan during the given years.
the maintenance stage is when the design engineer himself
Moreover, it is quite likely that, even in USA, 2.2% of its GNP
oversees the maintenance. It is choice (D) which states this,
in 1978 was higher than 3% of its GNP in 1964! Therefore,
and is the answer.
(C) is not a valid conclusion that can be drawn from the given
information.
2. The proposed amendment stipulates that every
The passage first says that the expenditure on research
prospective nominee for an office must give a consent to his
and development in USA decreased steadily from 3% of its
nomination, and that, before giving such a consent, he must
GNP in 1964 to 2.2% of its GNP in 1978. It then says that,
be told who the other nominees will be.
during the same period, Japan increased its percentage to 1.6%
If there is only one prospective nominee in the field, he
of its GNP. We can therefore conclude that, during this entire
can give his consent to the nomination without difficulty.
period, USA spent a larger percentage of its GNP (2.2%) on
Consider the case when there are two prospective
research and development than Japan did (only 1.6%). So,
nominees, say M and N. Now M cannot give his consent to be
(D) is the answer.
a nominee unless he is told beforehand whether N has or has
The passage does not give any information that warrants
not given his consent, and, similarly, N cannot give his consent
the conclusion in (E).
before he is told beforehand whether M has or has not given
his consent. This implies that, under the proposed
9. (A) is wrong since, even when a trained examiner asks
amendment, it will be impossible for either M or N to become
questions, the lie detector is said to be accurate in only 70 to
a nominee.
90 percent of the cases.
This situation will hold good even in cases where there
Since the maximum percentage of accurate results is
are more than two prospective candidates, because it will be
mentioned as 90, the conclusion is that, at least in 10% of the
impossible for any of them to give his consent to become a
cases or, in one out of ten cases, the lie detector gave incorrect
nominee. So, (E) is the answer.
results. So, (B) is the answer.
3. The given passage consists of three statements: (i) A
10. Relating the price of gasoline (meaning petrol) to the
violin constructed to have an improved sound would sound
number of riders using public transportation (meaning buses
different from the best-sounding existing violins; (ii)
and trains), the second sentence says that one of the reasons
137
American Education Aids - LSAT - Logical Reasoning - Analysis
Professional violinists would always consider that a violin light is not flashing at the junction of a railroad and a road, a
which sounds different from the best-sounding existing violins bus plying on the road can proceed further. The given passage
sounds less like a violin, and is therefore worse than the best- describes an instance of this not being true.
sounding existing violins; and (iii) Professional violinists are The passage mentions two contingencies - (i) when the
the only accepted judges of the sound quality of violins. warning light is not flashing, and the tracks are clear, and
We have been asked to identify that choice which would (ii) when the warning light is not flashing, and the tracks are
be best supported by these three statements. not clear.
(A) is a suggestion against one of the three statements, and It says that, in the case of the first contingency, the bus
is not one that logically follows from the three statements. So, proceeds further, and that, in the case of the second
(A) is not the answer. contingency, the bus does not proceed further.
(B) also is not a statement that follows logically from the It is implied in this narration that, if the warning light is
three given statements. In fact, if (B) is true, a violin which flashing, the bus will not proceed further.
sounds different from the existing best-sounding violins may So, if the bus does not proceed further, either (i) the warning
not even be constructed at all! light is flashing, or (ii) the warning light is not flashing, but
No information is available on how long the violin itself the tracks are not clear, or (iii) both.
has existed as a musical instrument. Therefore, the conclusion (D) mentions all these three contingencies, and is the
that ‘the best sounding violins have been in existence for answer.
several centuries’ is not warranted from the information in
the three given statements. So, (C) is wrong. 6. The second sentence of the passage says that the carbon
The three given statements imply that the professional dioxide generated naturally by the decay of plants and trees
violinists, who are considered to be the only judges of the sound in forests is balanced by the carbon dioxide absorbed by the
quality of violins, will not rate any violin that sounds different trees which give off oxygen.
from the best sounding violins of today as better than them. This implies that, if a large quantity of carbon dioxide is
This means that it is not currently possible to construct a generated by any other source, the trees in the forests cannot
violin that these persons will evaluate as having an improved absorb such excess, and that such excess carbon dioxide will
sound. So, (D) is the answer. be surplus to what is required for ecological balance.
The statement that the professional violinists will not The third sentence says that the use of plant-based fuels
evaluate a new violin as being better than the best-sounding in industry generates a large amount of carbon dioxide.
existing violins does not logically imply that everyone else Combining both these, we can conclude that a society that
will disagree with them and consider the new violin as a better uses plant-based fuels in industry will contribute a net surplus
one. (In fact, if this is the case, then professional violinists of carbon dioxide to the atmosphere, unless the gas is
would not have been accepted as the only reliable judges of reabsorbed in some other way.
the sound quality of violins.) So, (E) is wrong. So, (D) is the answer.

4. (A polygraph is a lie detector. The purpose of a polygraph 7. The bloods of two persons are said to match only when
test is to determine whether the person who answers the they are both of the same type. What the passage says is that
questions put to him is telling a lie or not. Applicants for jobs Type O blood can be used for everyone. This does not mean
are usually asked to undergo a polygraph test to ascertain that Type O blood matches the blood type of most people. So,
their honesty. But a polygraph test is conclusive only in about (A) is wrong.
90% of the cases and is inconclusive in other cases.) The From the statement that Type O blood is continuously in
passage first points out that, when a polygraph test on an short supply in USA, we cannot come to the conclusion that
applicant is inconclusive, no inference should be drawn at all it is unavailable for emergencies. On the contrary, responsible
about his honesty or dishonesty. It then says that, hospitals will ensure that they always have reserve stocks of
nevertheless, employers sometimes refuse to hire an applicant Type O blood for emergencies. So, (B) is also wrong.
in whose case the polygraph test has been inconclusive. Again, from the fact that 45% of the blood donated in US
We can conclude from this information that employers is of Type O, it is not possible to come to the conclusion that
sometimes interpret an inconclusive polygraph test as if it exactly 45% of the people of US have Type O blood. The only
has given a negative result about the applicant. If so, they are conclusion we can arrive at is that, among those who
holding an inconclusive result unfairly against him. It is (C) contribute blood in US, 45% have Type O blood. So, (C) is
which states this, and is the answer. also wrong.
(A) contradicts what is stated in the passage, and is wrong. From the second and third sentences, we can infer that
The author of the passage is against the unfair use of other types of blood such as A, B etc can only be given to
inconclusive polygraph tests, and is not against conclusive those patients whose blood groups match them, while Type O
polygraph tests being used in recruitment decisions. So, (B) blood can be given to anyone, irrespective of his blood group.
is wrong. So, it is (D) which follows from the information in the passage,
The passage is only about tests which have been and is the answer.
inconclusive, and does not question those which have given a What the passage implies is that, in the case of emergencies
definite result. So, (D) is not a conclusion that can be drawn when there is no time to type the victim’s blood (meaning
from it. ‘determining which group the blood belongs to’), Type O blood
(E) implies that some employers give jobs to applicants can be administered straightaway because it is compatible
who have failed the polygraph test, and reject those who have with every other blood group. We cannot infer from this
passed the polygraph test. Such a conclusion is not warranted statement that any particular blood group takes more time
by the information in the passage. to ‘type’ than any other blood group. So, (E) is not the answer.

5. One would normally expect that, when the warning 8. The second sentence says, “Television news shows report
138
American Education Aids - LSAT - Logical Reasoning - Analysis
more on stories that include dramatic pictures such as fires 1000, it means that, for every 1000 beds available in nursing
and motor vehicle accidents than they do on more common homes, 95 patients were admitted as inpatients in a year.
stories that have little visual drama such as bookkeeping The total number of available occupancy days for all these
fraud”. 1000 beds in a year is 365,000. If the occupancy rate in a
(The underlined phrase implies that crimes of bookkeeping year is 87 percent, it means that the beds were occupied for
fraud are much more common than fires and motor vehicle 365,000 x 87/100 or 3650 x 87 ‘patient days’. This means that
accidents.) the average number of days that a patient was an inpatient
The first sentence says, “Most television viewers estimate in the nursing homes was (3650 x 87)/95.)
how frequently a particular type of accident or crime occurs The passage first says that when the admission rate was
by how extensively it is discussed on television news shows”. 95 per 1000 beds between 1975 and 1985, the occupancy rate
We can conclude from these two sentences that most was 87 percent of capacity.
television viewers would overestimate the number of fires and It then says that, between 1985 and 1988, the admission
motor vehicle accidents that occur because these are reported rate came down to 81 per 1000, but the occupancy rate rose
more by television news shows than are crimes of bookkeeping to 92 percent.
fraud even though the latter are much more common. The conclusion from this information is that, between 1985
(D) is what states this, and is the answer. and 1988, less number of admitted patients spent more days
You can easily see that none of the other four choices is in hospital beds. This means that the average number of days
relevant to the two sentences in the passage. that an inpatient stayed in a nursing home was greater
between 1985 and 1988 than it was between 1975 and 1985.
9. The given passage talks of only three animals: a dog, a (A) is what states this, and is the answer.
cat and a platypus, and refers to only one sensory capacity in The passage does not talk about the age of patients
each of them which is different from that of humans. admitted in nursing homes at all. So, (B) cannot be a logical
So, the sweeping statement in (A) that most animals have conclusion that can be drawn from the given information.
sensory capacities superior to those demonstrated by humans The given passage gives information only about two specific
is not warranted by the passage. periods. It is not possible to draw a general conclusion
(B) is what neatly summarises the information in the therefrom that nursing home admission rates will always
passage, and is the answer. decline whenever occupancy rates rise. So, (C) is not the
You can easily notice that (C) and (D) are far-fetched answer.
statements that do not logically follow from the given The given information has no relevance to the years in
information. which a nursing home was built. So, (D) is irrelevant and
What is stated about the cat is, “a cat has a greater capacity cannot be a logical conclusion from the passage.
to see in dim light than a human normally has”. It is therefore The given information is about the cumulative number of
possible for a specially trained human to see equally well or beds available in all nursing homes and the cumulative
better in dim light than a normal cat does. It is also possible number of patients admitted in all of them. No conclusion
that some cats, because of old age or other medical reasons, can be drawn from the passage relating to the sizes of
are not able to see as well in dim light as a normal cat can. individual nursing homes. So, (E) is not the answer.
So, the statement that ‘any human who can see in dim light (Note that, even though a mathematical principle is used
does so less well than any cat’ is a conclusion that is not in answering the question, there was no need for us to make
warranted by the given information. So, (E) is wrong. any actual calculations.)

10. The passage first says that, early in its development, 2. (This question also involves the same mathematical
the Earth was a rapidly rotating body of molten rock in which principle as in the last question, but in a different context.)
most of the iron had settled to the core. It then says that some The passage first says that husbands of wage-earning women
of this fluid was cast off from the surface of the spinning mass and husbands of women who do not earn wages spend the
and later solidified to form the moon. same average duration of 11 hours per week on household
The passage implies that the surface of the rotating molten work.
Earth contained less iron than the core of the earth, and that It then says that husbands of wage-earning women do
it was from this surface with less iron-content that the Moon household tasks on more number of occasions than do
was eventually formed. Therefore, we can conclude that the husbands of women who do not earn wages.
moon in general, and particularly its core, contains Let us assume that the husband of a woman who does not
proportionately less iron than the Earth does. earn wages does household work on an average of 11 occasions
(B) states just this, and is the answer. a week. This means that he spends one hour on the average
You can easily see that none of the other choices, though on each such occasion.
they may be factually correct, is a logical conclusion from the Then, according to the second piece of information, the
information in the given passage. husband of a wage-earning women must be doing household
work at least on 12 occasions a week. So, on each occasion, he
SET 7 must be spending an average of 11/12 hours or 55 minutes on
Answers household work.
This means that the average time spent on each occasion
1.A 2.C 3.A 4.C 5.D 6.D 7.E 8.D 9.D 10.B
by the husband of a wage-earning woman when he does
Analysis
household work is less than the average time spent on each
1. The passage mentions two sets of statistics relating to
occasion by the husband of a woman who does not earn wages.
nursing homes: (i) admission rate (in terms of number of
(C) is what states this, and is the answer.
admissions per 1000 beds a year) and (ii) occupancy rate (in
terms of percentage of beds capacity). If the former is 95 per
3. The first sentence says that, between 1982 and 1995,
139
American Education Aids - LSAT - Logical Reasoning - Analysis
the increase in the number of people employed in low-paying than $100 millions each.
service occupations will be more than the increase in the So, (D) is what follows from the given passage, and is the
number of people employed in high-paying service answer.
occupations.
The second sentence says that, during the same period, 6. (Citrus is the common name of a group of fruits including
the share of the low-paying service jobs in the total oranges, lemons etc.) The passage says that, in a recent year,
employment will not increase, while the share of the high- while California’s total citrus crop was 23% more than
paying service jobs in the total employment will increase. Florida’s total citrus crop, its orange crop was only 76% of
We have been asked to identify that choice which can be a Florida’s orange crop.
logical conclusion from these two statements. We have been asked to spot the choice which can be
The answer to this question is based on a simple logically inferred from the above information about the citrus
mathematical principle. crops in Florida and California during that year.
If the initial value of M = 1000 and it increases by 80, the This question is also based on a simple arithmetic principle.
increase in ratio is 8%. If, in a particular year, California produced more citrus fruits
If the initial value of N = 100 and it increases only by 15, than Florida, but produced less oranges than Florida, the
the increase in ratio is a higher figure of 15%. obvious conclusion is that the proportion of non-orange citrus
Thus, though M has increased by 80 but N has increased fruits produced by California in that year was more than the
by only 15, the increase in ratio is less for M than it is for N. proportion of non-orange citrus fruits produced by Florida.
Conversely, if, in spite of a larger increase in number, the It is (D) which says this, and is the answer.
increase in ratio is less for M than it is for N, then M must be (A) is wrong because the passage itself states that Florida
greater than N. also produces non-orange citrus fruits.
Applying this principle to the given question, we can There is no information in the passage as to whether the
conclude that the number of persons in low-paying service volumes of the crops were measured in numbers or in weight.
jobs in 1982 must have been less than the number of persons So, (B) cannot be inferred from the passage.
in high-paying service jobs in 1982. (C), while it may well be true, cannot be logically inferred
(A) is what states this, and is the answer. from the given passage.
The fact that the citrus crop in California was more than
4. The first two sentences of the narrative state that, during the citrus crop in Florida cannot lead us to the conclusion
the 1960’s, only 15% of the growth in US population was due that California had more acreage under agriculture than
to immigration while the remaining 85% was due to natural Florida had. It is quite likely that Florida had more land under
increase (namely, the amount by which the birth rate was agriculture but a larger proportion of it was devoted to other
higher than the death rate). crops such as wheat, barley etc. It is also likely that the yield
The last two sentences state that of citrus fruits per acre is greater in California than in Florida.
(i) the population has grown continuously since then So, (E) is not a logical conclusion that necessarily follows from
(ii) immigration accounts for 30% of the population growth the given passage.
today
(iii) the number of people immigrating each year has not 7. The passage concludes that when the entire private
changed significantly business (including farming activity) is considered, the
If we assume that the annual number of migrants in the productivity increase in 1982 over 1981 was 4.8%, but when
1960’s was 1500, the first two sentences state that the annual the non-farm private business alone is taken, the productivity
growth in population was 10,000, with the balance of 8500 increase was 5%.
being attributable to natural growth. This means that the inclusion of farming brought down
According to the last two sentences, the annual number of the percentage increase in productivity in 1982. This would
migrants continues to be 1500, but, now, this forms as much imply that the productivity in the farming sector must have
as 30% of the present population growth. increased by less than 4.8% while the productivity in the non-
This implies that the annual growth of population at farming sector increased by 5%, so that their average was
present is only 5000. So, we can conclude that, compared to 4.8%.
1960’s, the population is growing more slowly now. It is (C) In other words, the increase in the farming sector
which states this, and is the answer. productivity must have been less than the increase in the
non-farming sector productivity. (E) is what states this, and
5. (The statement in this question is deliberately worded in is the answer,
a confusing manner.) The passage deals with the total taxable (If this reasoning is not clear to you, consider the following
income from foreign sources. statement: “There are both boys and girls in a class. The
It first says that 53% of such total taxable foreign income average marks in English of the whole class is 50, while the
came from 38 big companies with more than $100 million average of the marks of the girls alone is 54. What can you
income each. say about the average of the marks of the boys alone?”
It then says that 60% of such income came from 200 Obviously, it must be less than 50. The same reasoning applies
companies reporting income from 10 or more countries. in this case also.)
These two cannot be totally independent groups because, Note that the productivity, as defined, does not depend
in that case, their contribution adds upto 113% of the total upon the number of workers, but is only dependent on the
income, which is not possible. goods or services produced in an hour by each worker. So,
So, the given data should mean that at least a part of this even if (A) is not true, the given statement could still be true.
total income must have been returned by companies which (B) is irrelevant to the statement, because the wages paid
have both the characteristics, namely (i) having income from to the workers has no relevance to their productivity.
10 or more countries and (ii) having a net income of more (C) implies that the increase in the productivity of nonfarm
140
American Education Aids - LSAT - Logical Reasoning - Analysis
private sector must have been less than 2.5% in 1981. This The second sentence says that 26% of the total corporate
need not be true because this increase could have been even gifts to higher education in USA came from 14 Japanese
5%, in which case, if the productivity increase in the farming corporations each of which received income from 27 or more
sector in 1981 had been only 1.5%, the average could still countries.
have been around 2.4% in that year as given in the last The total of these two works out to 103%, which is more
sentence of the given statement. So, (C) is not a necessary than 100% of total corporate gifts to higher education in USA.
conclusion. This means that these two are not totally independent groups
(D) is also irrelevant, because productivity has been defined and that there is necessarily some overlapping between them.
only in terms of the value of the goods and services produced In other words at least one of the 200 corporations having
in an hour, and not in terms of the profits in the business. It income more than $122 million must be a Japanese
is quite possible that the productivity in an enterprise has corporation that had received income from 27 or more
increased even while its profit has declined because of various countries.
other reasons. It is (D) which says this, and is the answer.

8. (This question is similar to Qns. 1 and 2 in this set.) The 10. This is a very simple question camouflaged to sound
given information means that the total duration of the two difficult. From the phrase “Of women in the primary
breaks during the movie program was equal to the total childbearing age range of eighteen to twentyfour who account
duration of the five breaks of the regular programs. for about 40 percent of all births in this country annually
If this total duration is assumed as 10 minutes, each break .....”, we can infer that the remaining about 60% of the annual
during the movie program would have lasted 5 minutes, number of births in the country are accounted for by women
whereas each break during the regular programs would have who are either less than 18 or older than 24. (B) says just
lasted only 2 minutes. this, and is the answer.
So, the former is greater than the latter. It is (D) which (A) looks like the plausible answer, but is incorrect.
states this, and is the answer. According to the last sentence of the passage, women in the
None of the other choices follows as a logical conclusion primary childbearing age range of 18 to 24 account for 40%
from the given information. of all births, and more than 25% of them have no healthcare
insurance. This means that only about 10% (which is 25% of
9. (This question is similar to Qn. 5 in this set.) The first 40%) of all mothers do not have healthcare insurance, and
sentence of the passage says that 77% of the total corporate the rest of nearly 90% of them do have such insurance.
gifts to higher education in USA came from 200 corporations Therefore, (A), which pegs this number at a mere 75%, is
with income more than $122 million each. wrong.

141
Analysis - Chapter 3
'INFERENCE' QUESTIONS
SET 1 following a well-publicized execution, the yearly murder rate
remains the same.
Answers
We can infer therefrom that the lower-than-average murder
1.C 2.C 3.E 4.C 5.B 6.D 7.E 8.B 9.B 10.B
rate during such a week is compensated by a higher-than-
Analysis average murder rate in some other week.
1. What the given passage says is that the pharmaceuticals It is (B) which states this, and is the answer.
division of the company accounted for 20% of its sales and
40% of its profits, while the chemicals division accounted for 6. The author substantiates his assertion in the first
80% of its sales and 60% of its profits. sentence, “Weeds and trees illustrate extreme opposites of
The obvious inference from this information is that the ecological adaptation” by pointing out that the weeds put most
chemicals division realized a lower profit per dollar of sales of their energy into production and dispersal of the season’s
than the pharmaceuticals division did. seeds and then die, while the trees put most of their energy
It is (C) which states this, and is the answer. into sustaining their own growth and competitive ability over
You can easily verify that none of the other choices is a a long life span, producing a relatively moderate number of
necessary inference from the given information. seeds every season.
We can easily notice that, while drawing the distinction
2. The passage asserts repeatedly that there is no between the weeds and trees, the author uses as the criterion
correlation between budget deficits and trade deficits of the ratio between a season’s seed production and the overall
countries. We can therefore infer that a reduction in a nation’s life span of the two species. (D) says just this, and is the
budget deficit does not necessarily result in a corresponding answer.
reduction in its trade deficit, and, conversely, that an increase You can easily verify that the passage does not throw any
in a nation’s budget deficit does not necessarily result in a light on the criteria mentioned in the other choices.
corresponding increase in its trade deficit.
(C) mentions the first of these two, and is the answer. 7. Note that the first part of the sentence refers only to
You can easily verify that none of the other choices is a Wanda’s trip to Albuquerque and asserts that, whenever she
necessary inference from the given passage. visits Albuquerque, she will also visit all three of Santa Fe,
Phoenix and Tucson.
3. The only reason that Bob gives for his ‘knowledge’ that We cannot infer the converse of this statement, namely
Jim is innocent is that Jim and he are ‘good friends’. Therefore, that whenever she visits Santa Fe, Phoenix and Tucson, she
his conclusion about Jim’s innocence is not based on the facts would also visit Albuquerque. So, (A) is wrong.
relating to the incident referred to by Jack. Therefore, in spite On a similar reasoning, none of (B), (C) and (D) can also
of Bob’s assertion that he ‘knows’ that Jim is innocent, it be inferred from the given sentence.
cannot be inferred as necessarily true. So, (A) is not the The statement that whenever she visits Albuquerque she
answer. will also visit Phoenix implies that, if she had not visited
Jack does not give any opinion of his own about whether Phoenix in a trip, she had not visited Albuquerque also during
Jim is guilty or innocent. He only reports the fact that Jim that trip. It is (E) which states this, and is the answer.
has been arrested on the charge of stealing. There is no reason
for Bob to think that Jim is lying while making that report. 8. (“Carotene is a vitamin A precursor” means that it forms
In any case, his response does not exhibit any such distrust. the penultimate stage of vitamin A). The passage implies that
So, (B) cannot be inferred from the conversation. carotene lends a yellowish colour to butterfat, but, if carotene
Jack’s statement is a very simple one, and does not has been converted into vitamin A which is colourless, then
necessary mean that he saw Jim being arrested. He might the yellowness in the butterfat is less.
well have heard the news from someone else. So, (C) is not a So if butter made from the milk of holstein cow is paler
necessary inference from the given conversation. than the butter made from the milk of jersey cows, it means
As stated above, Jack merely reports that Jim has been that there is more vitamin A in the butterfat of holsteins than
arrested for stealing without giving any personal opinion in the butterfat of jerseys.
about Jim’s innocence or guilt. So, (D) cannot be inferred from So, (B) is the answer.
the conversation. (A), (C) and (D) contradict the given information.
From Bob’s response, we can only infer that Bob thinks Since ‘fat solubility’ is not mentioned in the given
that Jim is innocent. So, (E) is the answer. statement. (E) is irrelevant to it.

4. What Belinda emphasizes in her response to Antonio’s 9. (A credo is a firm belief that is put into practice.
strong criticism of modern artists is that “they have all spent “Affirmative-action program” means a voluntary decision to
many years studying and training”. reserve a certain percentage of posts in an organisation to
We can therefore infer that she has misunderstood Antonio persons belonging to deprived classes such as the Blacks and
as saying that modern artists do not have sufficient knowledge women.)
or training in art - in other words, that they are not sufficiently We have been asked to spot that choice which can be
qualified for their work. inferred from the information given in the passage.
So, (C) is the answer. The first sentence of the passage says that fifteen
corporations in the United States that follow a credo of social
5. The passage means that, in spite of the weekly murder responsibility are also very profitable. We cannot infer from
rates falling below average during the weeks immediately
142
American Education Aids - LSAT - Logical Reasoning - Analysis
this statement that the total number of corporations in the people put together.
United States which follow a credo of social responsibility is (C) and (D) both talk only of the total amount of tax paid
only fifteen, and that all these are very profitable. It is quite by each group, but not the proportion of the tax to the income
possible that a much larger number of other corporations also of the respective group.
follow a credo of social responsibility but are not as profitable (E) contradicts the information in the second sentence, and
as these fifteen, due to various other causes. So (A), which cannot be the answer.
states that following a credo of social responsibility helps to
make a corporation very profitable, cannot be inferred from SET 2
the passage. So, (A) is not the answer. Answers
The fact that at least fifteen of the corporations which
1.E 2.B 3.B 4.A 5.E 6.B 7.A 8.A 9.A 10.E
follow a credo of social responsibility are also very profitable,
Analysis
shows that social responsibility and profit making are not
1. The passage says that pilots do not have to respect the
antithetical to each other, and that it is possible for a
wind so much while flying heavier aircrafts as they should
corporation to follow a credo of social responsibility and also
while flying ultralight aircrafts.
be very profitable at the same time. So, it is (B) that can be
This means that, in wind, it is easier to handle a heavier
readily inferred from the given passage, and is the answer.
aircraft than a lighter aircraft. So, (E) is the answer.
(C) looks tantalizingly as a possible answer, but is wrong.
The other choices do not mention the role of wind in the
The statement in the second sentence, “Because of their
flight of aircrafts, and are wrong.
credos, these fifteen corporations give generously to charity”
only means that, in the case of these fifteen corporations, giving
2. The second and third sentences say that a placebo is
generously to charity is based on their credos of social
prescribed “in the hope of instilling in the patients a positive
responsibility, and not on any other reason such as to please
attitude toward prospects for his or her recovery” and that, in
the local politicians, getting cheap publicity etc. We cannot
some cases, the placebo actually produces improvement in the
infer from this statement that every corporation that gives
patient’s condition.
generously to charity must be doing so only because of its
So, in comparing a physician to a placebo, what the
credo of social responsibility. So, (C) is not the answer.
researcher obviously implies is that the very knowledge that
Obviously, (D) is not a necessary inference from the given
he is under a physician’s care can instill in a patient a positive
passage which talks of only fifteen among the thousands of
attitude towards prospects for his recovery, and can
profit-making corporations in the United States. There may
consequently improve the patient's health. It is (B) which
be a number of these corporations that are very profitable but
states this, and is the answer.
do not give generously to charity.
None of the other choices follows as a necessary implication
Once again, from the second sentence of the passage, which
from the comparison of a physician to a placebo.
refers to only fifteen corporations as being charitable as well
as following both stringent environmental-protection policies
3. If A is 0, the last digit in the sum must also be A and not
and affirmative action programs, we cannot come to the
N. Therefore (A) can be inferred as true.
conclusion that all corporations which follow any one of them
From the equation A + A = 2A = N, we can say that N must
follow the other two also. So, (E) is also wrong.
be an even number. So, (C) can also be inferred as true.
If no number had been carried over from the hundred’s
10. The statement made is that, when the sales tax on
position to the thousand’s position, the first digit in the answer
consumer goods is levied at a fixed percentage of the purchase
would have been A, and not J. Therefore, the sum at the
price, poor people pay a larger proportion of their incomes in
hundred’s position must have been more than 10. There are
sales taxes than wealthy people do.
now two possibilities: either L+T is itself 10 or more, or L+T
We have been asked to spot that choice what follows
is 9 and there is a carry over of 1 from the ten’s position to
logically from this statement.
the hundred’s position. In either case, L+T must be greater
If a person with income of $100 and a person with income
than 8. Thus, (D) can also be inferred.
of $1000 both spend the same proportion of their income, say
The maximum that can be carried over from the hundred’s
50%, on essential consumer goods such as food items, the
position to the thousand’s position is 1, and therefore A+1
amounts so spent by them are $50 and $ 500 respectively. If
must be equal to J. Thus (E) can also be inferred.
the tax rate is a fixed rate of 10% of the purchase price, then
It is (B) which need not be true, and is the answer.
the former will pay $5 as sales tax, while the latter will pay
$50 as sales tax. In both cases, the proportion of their income
4. According to the passage, the important distinction
paid as tax is the same 5%.
between historical research and anthropological research lies
But the second sentence of the given passage says that
in the fundamental questions that each pose: the historian’s
the poorer people pay a higher proportion of their income as
question is “Why did the change occur?”, while the
sales tax than wealthy people do. This means that poor people
anthropologist’s question is “What is the meaning of actions
spend a higher proportion of their income on essential
and systems?”.
consumer goods than the wealthy people do. (For example, if
The obvious inference from the statement that these
a person with an income of $100 spends 70% of his income
questions are distinct is that the answer to the one does not
(or $70) on essential consumer goods, he pays a sales tax of
constitute the answer to the other. In other words, the meanings
$7, which is 7% of his income. If a wealthier person with an
of actions and systems do not fully explain why change
income of $1000 spends only 50% of his income, or $500, on
occurred.
essential consumer goods, he pays a sales tax of $50, which
So, (A) is what is implied by the passage, and is the answer.
is only 5% of his total income.) So, (B) is the answer.
(A) is wrong because the passage does not say that all the
5. (A), (B) and (C) are wrong because the given conditions
poor people put together pay more tax than all the wealthy
143
American Education Aids - LSAT - Logical Reasoning - Analysis
do not preclude Manuel, Ken, Lou and Jan eating together in innovative products other than computer programs qualify
a restaurant in Jan’s neighbourhood. for either copyright or patent, and not for both.
(D) is wrong because the given conditions do not preclude Since the passage does not mention the number of
Manuel and Ken alone eating together in a restaurant in the innovative computer programs that have been patented and
downtown area. copyrighted vis-a-vis the number of other products that have
From the first two sentences, we can infer (E), which is been either patented or copyrighted, it is not possible for us
the answer. to infer that the former is only a few compared to the latter.
So, (D), while it may be a fact, cannot be inferred from the
6. The first statement is that, in a political system which given passage.
has only two major parties, the entrance of a third party While the passage says that computer programs are
candidate has the effect of affecting the votes of only one of capable of being protected both by patent and copyright, it
the candidates, and not those of the other. does not indicate which between the two can be more easily
On the basis of this statement, the author says that this is obtained. So, (E) cannot be inferred from the narration.
unfair to the first candidate, and argues that the contest by
the third candidate should not therefore be allowed. 9. The first part of the second sentence says that, in the
Since the author feels so strongly about the unfairness suburban regions, housing availability has not kept up with
that he even advocates the banning of the entry of the third the growth of jobs.
candidate into the election arena, he implies that the entrance It then says that this has resulted in long commutes
of the third candidate would lead to the defeat of the candidate (meaning daily travels from house to workplace and back)
who would otherwise have won. for middle-income workers, and inaccessibility of job
We have been asked to spot that choice which can be a opportunities for lower-income workers.
logical inference from the author’s statements. A long commute necessarily involves a considerable
If one of the two original candidates is likely to get 85% of expenditure on travel. The information that middle-income
the votes and the other is likely to get only the remaining workers are able to undertake such long commutes for the
15%, the entrance of the third candidate (taking away even sake of their jobs implies that the income that they get from
10% of the votes that would have been polled by the winning their jobs makes it affordable for them to undertake such
candidate) is not likely to make any difference to the final long commutes.
outcome of who wins and who loses. The information that the lack of housing availability in
But, the third candidate’s entry into the area will make a suburban areas has resulted in inaccessibility of job
difference to the final result if the votes polled by the two opportunities for lower-income workers implies that this class
major candidates in a direct fight is something like 51% and of workers are not able to afford these long commutes with
49%, and the third candidate diverts even 3% of the votes the salary that they would get in the jobs in the suburban
that might have been polled by the winning candidate. areas. It is (A) which states this, and is the answer.
So, it is (B) that can be inferred from the author’s (B) would mean that suburban housing is available in
statements, and is the answer. plenty, which contradicts the information in the first sentence.
(C) contradicts the information that middle-income
7. The implication of the passage is that objects of workers have to undertake long commutes for reaching their
experiments change with the changes in the problems which work spots.
become of interest to biochemists. The first sentence of the passage implies that, while
Since experiments on insects’ biochemistry have started locating or relocating industries, no consideration was given
only recently, it can be inferred that a new problem has to the availability of housing for the workers. (D) contradicts
attracted the interest of biochemists and, for solving it, study this information, and is not the answer.
on insects has become relevant. (E) talks of affordable housing in urban areas, whereas
So, (A) is what is implied in the paragraph. the passage is about the availability of affordable housing in
(B) does not follow since the paragraph does not say that suburban areas. So, (E) cannot be logically inferred from the
the experiments have been successful as yet. given information.
(C) and (E) are obviously far off the mark.
(E) is wrong since, even if biochemists had studied the 10. The first two sentences state that the technique of radio
biochemistry of insects in the past, some other problem interferometry requires ultraprecise timing, exact knowledge
relating to the insects (such as their breeding habits, their of the locations of the different radio telescopes used in the
resistance to diseases etc) could be of fresh interest to them technique and sophisticated computer programs.
today. The next sentence says that, in a significant technological
achievement, an earth-based radio telescope has been
8. The last sentence of the narration says that, in order to interferometrically linked with a radio telescope on an orbiting
qualify for either a patent or a copyright, the idea behind an satellite.
innovation must be clearly distinguished from its expression. We have been asked to identify that choice which can be a
Since computer programs are protected both by patent and correct inference from these statements.
by copyright, we can infer that the idea behind innovative One of the two radio telescopes mentioned in the passage
computer programs must be capable of being clearly is earth-based, and static. But the other is mounted on an
distinguished from their expression. orbiting satellite whose position varies from moment to
It is (A) which states this, and is the answer. moment. If it has been found possible to connect them
(B) is wrong because most computer programs do not interferometrically, it implies that the location of an orbiting
involve any innovation, and a person who writes it cannot be satellite relative to locations on earth at any given instant
considered to be an ‘inventor’. can be calculated precisely. It is (E) which says this, and is
(C) contradicts the first sentence in the narration that all the answer.
144
American Education Aids - LSAT - Logical Reasoning - Analysis
(A) merely talks of the calculation of the orbit of the replaced by normal rains. This would be favourable to the
satellite, but not of the exact location of the satellite in this gypsy moths and would not create a biological stress in them.
orbit at any given moment. So, (A) is not the answer. (B) talks of stress suffered by the trees and not by the gypsy
Since the relative strengths of the signals received by moths, and is irrelevant to the given information.
different radio telescopes has not been mentioned as a factor In the case of (C), the gypsy moth larvae will be decimated
in radio interferometry, (B) is not a relevant conclusion from by the parasitic wasps and flies and not by the nuclear
the given passage. polyhedrosis virus.
(C) may be a factual statement, but it cannot be inferred Since the virus is already present in a latent form in the
from the information in the given passage. gypsy moth larvae, the spraying of more virus will not result
(D) also may be a factual statement, but it cannot be in the decimation of the moths, unless the larvae suffer a
inferred from the information in the given passage. biological stress. So, (E) is not the answer.

SET 3 3. (A school of fish means a group of fish swimming together.


Answers A predator is a larger fish which eats the smaller fish.) The
passage says that a fish is liable to be attacked by a predator
1.A 2.D 3.A 4.A 5.E 6.C 7.B 8.C 9.A 10.E
Y if it comes within the sphere of vision of Y.
Analysis
Since all the fish in a school swim close together, if one of
1. The first sentence of the given paragraph implies that
them falls within the sphere of vision of Y, most of the others
investment in expensive machinery can increase the
in the school are also likely to fall within the same sphere.
productivity of large farms. Yet, according to the next two
Therefore the vulnerability of a single fish to attack by Y would
sentences, if the owner of a large farm borrows money for
be more or less equal to the vulnerability of the entire school
making this investment, the consequent liability to pay
to attack by Y.
interest can lead to operating losses for him in spite of such
It is (A) which implies this, and is the answer.
higher production, if the market prices for their products drop.
Since the fish in a school swim close together, whether the
We have been asked to identify the choice when can be
school consists of three or four fish should not make any
logically inferred from the given paragraph regarding a large
difference so far as the sphere of vision of Y is concerned. So,
farm.
(B) is not a necessary inference from the given passage.
What the passage implies is that a large farm can be highly
A predator is more likely to chase a school of fish than a
productive because of investment in modern machinery, but
single fish. So, (C) is not the answer.
could still be unprofitable to the owner (during those years
The passage says that a fish will be visible to the predator
when the market prices of the products drops) because of the
as soon as it comes within the sphere of vision of the predator
heavy interest cost on the investment. (A) states just this, and
Y, irrespective of its size or whether it is swimming alone or
is the answer.
in a school. So, (D) is wrong.
(B) is wrong because, what the last sentence states is that
Each predator Y will have its own sphere of vision inside
the farms stand to incur losses if the prices of the products
which it can spot its prey, irrespective of whether it is
drop. This sentence does not say that higher production by
swimming alone or is swimming in a school of Y’s. So, (E) is
large farms inevitably results in reduced market prices.
not a logical inference from the given passage.
(C) also looks like a possible answer, but is wrong because
the given paragraph does not talk about rates of interest at
4. (‘Fallow acreage’ means the land left uncultivated out of
all. If the investment that has to be made is heavy, the interest
land that is capable of being cultivated). Let the total crop
liability will be considerably high even at a low rate of interest,
yield in the United States and Soviet Union be M and N
and the farm is still likely to incur operating losses during
respectively. What the passage says is that when the total
periods of low market prices.
yield is divided by the planted acreage in each case, the United
(D) is wrong, because a farm which is already incurring
States has a lead over the Soviet Union but, when the total
operating losses due to the cost of interest on earlier
yield is divided by the total agricultural acreage in each case,
investments cannot be expected to invest in additional
the Soviet Union fares better. This means that the new
machinery thereby incurring still more interest liability.
denominator (which consists of Planted Acreage + Fallow
(E) makes a definitive statement that a big farmer
Acreage) shows a proportionately higher increase in the case
depending on credit will never make a profit. This statement
of USA than in the case of the Soviet Union. This means that
will not be true in normal years when the market prices of
there is a higher percentage of fallow acreage in USA than in
the products have not fallen below economic levels. So, (E) is
the Soviet Union.
not a necessary inference from the given information.
It is (A) which states this, and is the answer.
2. The narration says that the virus is always present in
5. From the last sentence, “In fact, nothing ruins scientific
the larvae, but is triggered only when the larvae experience
research faster than mixing it with political considerations”,
biological stress.
we can readily infer that the author believes that scientific
We have been asked to identify that condition among the
research must be apolitical. So, (E) is the answer.
choices which is most likely to trigger the virus to become
Obviously (A) and (B) can be discarded straightaway as
active.
totally far-fetched.
We should therefore look for that choice which can
What the author emphasizes is that, while pursuing their
contribute to a biological stress in the moth’s larvae.
research, scientists should not be swayed by political
It is only (D) which describes a condition which will cause
considerations. This does not mean that scientific conclusions
acute hunger in the larvae, thereby causing a biological stress
discovered by them do not have political implications. (For
in them. So, (D) is the answer.
example, the scientific discovery of atomic fission by US
(A) mentions a situation in which drought conditions are
scientists during World War 2 had immense political
145
American Education Aids - LSAT - Logical Reasoning - Analysis
implications.) made to remember its name. (A) is what is implied in the
In the first sentence, the author says that scientists toil passage, and is the answer.
also in government laboratories. It is quite legitimate for a (B) is wrong, because an advertisement only seeks to make
government to fund only such research projects in its people remember a particular product, and cannot improve
laboratories that will subserve its own political aims. their memories generally.
(Example: funding research projects on missile development (C) is wrong because the passage says merely that it is
by Indian and Pakistani governments). So, (D) is not a valid necessary to attract a potential customer’s attention; it does
inference from the given passage. not assert that achieving this is an easy job.
(D) is wrong because the passage merely says that
6. The given information consists of two parts: (i) if a advertisements need not write in detail about the qualities
household buys more of canned and prepackaged foods, it or substance of a product; it does not imply that the
throws away less food; (ii) But, among the items of food that advertising agency itself need not know anything about the
such a household does throw away, a major part consists of quality of the product. So, (E) also does not follow logically
fresh produce. from the passage.
We have been asked to spot the choice which follows
logically from these two statements. 10. (Rule 1 of Game X implies that it is possible for a player
Since the given information does not give any statistics who has become a participant in the game to refuse to continue
regarding the number of households switching over to to play at some point in the course of the game, in which case
standardized diet, (A) cannot be inferred from it. he will be awarded a penalty of ten points. A person who
Since the given information is only about food-waste and refuses to become a participant in the game even in the
the component of standardized and fresh produce in such food beginning cannot obviously be awarded a penalty.)
waste, the statement in (B) which talks of non-food waste (A) need not be true, because a player who is awarded a
cannot be inferred from it. penalty of 10 at a fairly late stage of the game may already
The second sentence of the passage says, “The more have a lead of, say, 500 points over his nearest rival.
standardized a household’s diet is, the greater the quantities (B) is wrong because the penalty is leviable at the moment
of fresh produce the household throws away”. It logically of refusal after the game is well under way.
follows from this sentence that “the less standardized a The statement relates only to Rule 1 of the game and no
household’s diet is, the less are the quantities of fresh produce information about other rules are available. So we cannot
that it throws away”. (C) states just this, and is the answer. deduce (C).
(D) contradicts the information in the first sentence of the (D) does not follow from Rule 1, because it implies that a
passage. person who refuses to become a player will only lose 10 points.
(E) contradicts the information in the second sentence. If, in spite of this penalty, his score is still the highest, he has
the right to be declared as the winner.
7. The last sentence in the argument says that, by 1985, From the argument above, it follows that, even after a
the price of oil will dictate the price of corn. Therefore, if there player had declined to play, he continues to be in the game
is a slump in the oil price, there will be a corresponding slump because he can still be declared the winner if his total score
in the price of grain. So, (B) is the answer. upto the point of withdrawal, minus the penalty, is higher
(A) is wrong, since the price of corn that is used for oil than that of all other players. So, (E) is the answer.
production might still be more than the price of corn used as
food. SET 4
(C), (D) and (E) are farfetched statements not directly Answers
following from the facts mentioned.
1.B 2.C 3.E 4.C 5.A 6.E 7.A 8.C 9.B 10.D
Analysis
8. The phrase ‘extrapolating inference across species’ in
1. (Unlike a broadcast TV service which can be received
this sentence means ‘administering toxic substances to
through an antenna, a private cable TV service transmits
animals, studying the effects of these substances on them,
programs through cable that is connected to the households
and then presuming similar effects of these substances on
on monthly hire charges. This narration talks of someone
human beings also.’
who had an unauthorised connection to his house TV without
The author says that this procedure has a disadvantage
the knowledge of the cable TV company, and was caught
which is avoided by epidemiology, implying that epidemiology
subsequently.)
involves direct study on human beings. (The disadvantage is
(Restitution means recovery.) The man who was caught for
that the effects of these substances on human beings may
unauthorised viewing of the cable TV exclaims that it is
not always be identical to the effects on the experimental
ridiculous to ask him to pay the full charges, since he did not
animals.)
enjoy some of the programmes. This assertion by him
So, (C) is the answer.
establishes that he had indeed watched the programs that
were shown on the cable service without paying for them.
9. The purpose of advertising a product is to persuade
So, (B) is the answer.
people to buy it. If this purpose is to be achieved, one would
(A) is incorrect, since there is no sufficient information to
normally assume that people must be made to know not only
assert that the sum of $662 has been arrived at by the cable
the name of the product but also its positive qualities, and
company correctly.
must be made to remember these qualities when they actually
(C) is incorrect, because we do not know the tariff, and we
go for shopping.
certainly cannot agree with this person that he does not have
But this passage says that it is not necessary that people
to pay money for the programs which he says he did not enjoy.
should know something about the product before they are
(D) need not be correct, since this person might have just
persuaded to buy it, and that it is enough if they are somehow
146
American Education Aids - LSAT - Logical Reasoning - Analysis
moved into a house in which the cable service had been tapped (B) contradicts the author’s conclusion, and is not the answer.
by the previous tenant. What the author says is that, while it is easy to teach the
We cannot be sure of (E), since, while the person may like computers the formal rules of language, it is not easy to teach
to have the service so long as he gets it free, he may not be them how to extract the meanings of ordinary sentences. We
willing to have it on payment. cannot infer from this statement that it is futile (meaning
useless) to teach computers the formal rules of a human
2. (We can infer that ‘front’ is the meteorological term for a language. (In fact, there are a number of widely used computer
type of cloud. ‘Precipitation’ means ‘rain’.) programmes which check the correctness of spellings and
(A) is incorrect because the passage implies that high grammar in typed passages.)
pressure fronts are by themselves not a sign of rain unless While (D) may be a factual statement, it cannot be inferred
they meet low pressure fronts. from the given passage.
The passage merely says that meteorologists are able to (E) also contradicts the view of the author, and is not the
predict precipitation with reference to the movement of answer.
pressure fronts. The statement in (B) that the chief function
of meteorologists is the prediction of precipitation is not 6. The sentences, “She thinks managers get too much credit
warranted from this information. So, (B) is not the answer. for the upswings in their business. A fickle public assumes
(C) neatly paraphrases what is stated in the second that managers are brilliant when demand is strong, and
sentence of the passage, and is the answer. idiotic when it weak” imply that the upswings and
(D) contradicts the information in the first sentence of the downswings in businesses are caused by strong and weak
passage, and is not the answer. demands for their products in the market. This, in turn,
The information in the passage does not necessarily mean means that the managers themselves cannot significantly
that the low pressure and high pressure fronts always travel control the levels of the demands through their management
towards each other. So, (E) is not the answer. practices. It is (E) which states this, and is the answer.
You can easily see that none of the other choices logically
3. The point made in the passage is that “when the number follows from the reaction of the manager.
of women in the child bearing age was a record low between
1946-66, the number of children produced was a record high. 7. The third sentence says that the lack of genetic diversity
These children are now in the child bearing age (we can infer could allow a significant portion of a major crop to be destroyed
that their number must be very high), but the number of overnight. The next sentence gives the illustration of nearly
children produced by them is a record low”. 20% of the US corn crop having been destroyed in 1970 by
What we can readily infer from the passage is that the the southern leaf blight, leaving very few varieties of corn
birth rate is not directly proportional to the number of women unaffected in the areas over which the disease had spread.
in the child bearing age. So, (E) is the answer. This implies that plants with certain specific genetic
(A), (C) and (D) contradict the information given in the properties were affected by the plant disease, while plants
passage. without these genetic properties were not so affected. So, we
(B) is an absurd statement, and does not follow logically can infer that susceptibility to certain plant diseases is
from the given information. genetically determined. So, (A) is the answer.
(B) is wrong because, from the fact that 80% of the crop
4. The ‘critic’ is obviously critical of the adherents of what grown in US in 1970 had been resistant to southern leaf
he calls ‘the doctrine of unshakable foundations’. blight, we cannot come to a general conclusion that it will
These adherents first design a policy based on their always be so, because the genetic property of the major crop
philosophy and, if that policy fails in practice, their ‘only grown could be different in different years.
reaction’ is to design another policy based on the same (C) is not a necessary inference from the passage, because
philosophy. the extinction of wild food plants could have been due to many
By calling them dogmatic adherents to their philosophy, other reasons.
the critic implies that they do not brook the thought that While (D) may be true as a general proposition, it does not
there could be a fundamental flaw in their philosophy itself, necessarily follow from the given passage.
and that any revised policy based on the same philosophy is (E) is wrong, because the information in the passage does
likely to fail again. What he would approve of is their not rule out the possibility that the southern leaf blight might
willingness to reexamine the foundation of their philosophy have destroyed other food crops also.
itself. It is (C) which states this, and is the answer.
8. The first sentence says that, for twenty years prior to
5. The first sentence states that, for understanding the 1984, there had been a regular tendency on the part of US
meanings of even ordinary sentences, one requires to know residents older than sixty to migrate from urban areas to small
not only the rules of that language, but also a general towns and rural regions, but that a sample population study
knowledge of the world. in 1984 showed that there had been a dramatic reversal of
The second sentence concludes that, though computers can this tendency during that year.
be taught the rules of a language, they cannot be supplied This implies that, among the sample that was studied in
with a general knowledge of the world, and it is, therefore, that year, a large number had been seen to have newly settled
very difficult to develop computers which can extract the down in the urban areas in USA.
meanings of even ordinary sentences. The subsequent part says that this finding was in error,
We can infer from these sentences that extraction of the because, in this sample study, people who had moved to the
meanings of ordinary sentences is dependent on a general United States from abroad were accidentally counted as part
knowledge of the world. (A) states just this, and is the answer. of interstate migration within the United States.
In the light of the above explanation on (A), the next choice We have been asked to spot the choice which can be
147
American Education Aids - LSAT - Logical Reasoning - Analysis
logically inferred from this passage. buyers of books and by narrowing the range of books made
It is obvious from the information in the passage that available to the public.
migrants from abroad had been mistakenly counted as It then draws the conclusion that the losers in the process
interstate migrants within the United States, and that this have been the majority of authors or aspiring writers and all
had swelled the number of persons counted as interstate discriminating readers.
migrants to urban areas. This implies that migrants from This implies that commercial publishing used to cater to
abroad had predominantly settled down in urban areas, and discriminating readers earlier, but is now catering mostly to
not in small towns and rural regions. It is (C) which states the vulgar tastes of new buyers of books. In other words,
this, and is the answer. (If the foreign migrants in the sample commercial publishing is now catering to a different
had distributed themselves between urban areas and the community of book readers from those whom the book business
rural areas in the same proportion as the local interstate had served in the past. It is (B) which states this, and is the
migrants, the error would not have arisen.) answer.
The passage is about where the foreign migrants above the From the phrase “management’s new methods have
age of 60 settled down, and not about the proportion of old increased the financial profits of commercial publishing”, we
people among foreign migrants. So, (A) cannot be inferred can infer that commercial publishing was profitable even
from the passage. earlier. So, (A) is wrong.
If (B) were true, there would have been a discontinuance (C) contradicts the assertion in the last sentence of the
in the tendency of the migration of people over 60 from urban narration that the losers from the new practices of the
to rural areas, and not a reversal of that tendency. So, (B) publishers are ‘the majority of authors or aspiring writers’.
cannot be inferred from the given passage. (D) contradicts the information in the first sentence that
The passage itself states that the problem was not in the the narrow range of the new books, in fact, caters only to the
size of the microdata sample that was used in the study, but vulgar tastes of the new buyers of books.
in the inclusion of wrong people in the sample. If the sample (E) does not follow logically from the information given in
had been larger, and if the larger sample had also included the passage.
the wrong people, the same error could have resulted. So, (D)
is wrong. 10. We have seen questions involving the same
The passage itself states that the twenty-year tendency in mathematical principle in the previous chapter. This question
migration was noticed both within and across state seeks to divide all the households into three categories: lower-
boundaries. There is no further information in the passage income, middle-income and upper-income {The last is not
which can lead to the inference that moves over relatively stated in the narration, but is available in answer choice (D)}.
short distances, mostly within the same state, had The initial narration states that, while the average income
predominantly contributed to this trend. So, (E) is not the for all households increased by 2.4% in 1983 compared to
answer. 1982, the average income of two of the three categories had
declined during the period.
9. The information contained in the passage is: Publishing This implies that the average income of the third category,
concerns have recently been taken over by communications- namely upper-income households, must have increased by
entertainment firms, and they have increased the profits of more than 2.4% so that the average was 2.4%.
commercial publishing by catering to the vulgar tastes of new It is (D) which states this, and is the answer.

148
Analysis - Chapter 4
'ASSUMPTION' QUESTIONS
SET 1 You can easily see that none of the other choices constitutes
a logical link between the first and the second sentences of
Answers the passage.
1.D 2.A 3.E 4.A 5.D 6.D 7.C 8.B 9.D 10.B
Analysis 4. It is a fact that there had been an increase in the use of
1. The proposed ban relating to retired high-level officials chemicals in the US since 1940, and the life-expectancy of the
is only on their employment as lobbyists, and not on their US population had also increased during this period.
employment in any other capacity. If the views of those ‘who argue that the introduction of
So, if an official concludes that such a ban would prevent new chemicals has caused severe health problems’ were to be
these officials from earning a livelihood, he obviously assumes true, their logical assumption would be that, if the chemicals
that retired high level officials are capable of earning their had not been introduced, the increase in life-expectancy would
livelihood only as lobbyists and are unfit for any other jobs. have been still greater than what had been actually recorded.
It is (D) which states this, and is the answer. On the contrary, the author of this passage, who disagrees
You can easily verify that none of the other choices is a with the above class of people, concludes that the increased
necessary assumption for the official to come to his conclusion. use of chemicals has had no adverse effects on the health of
the society Obviously, his assumption is that the increase in
2. (“Rotating in a disorienting manner” means “rotating life expectancy would not have been greater, even if the use of
haphazardly”.) From the fact that 2/3 of the mice whose cages chemicals had not increased. So, (A) is the answer.
were rotated in a disorienting manner developed cancers (B) is wrong, since the author makes no reference either
(while only 1/10 of the mice in stationary cages developed directly or indirectly to other countries at all.
cancers), the researchers came to the conclusion that stress (C) is incorrect, since the author does not attempt to find
enhances the development of cancer in mice. reasons for the increase in the life expectancy in US. (Note
The obvious assumption made by the researchers is that that he does not say that the increase in the use of chemicals
rotating cages in a disorienting manner produced stress in has resulted in the increase in life expectancy. He only argues
the mice. It is (A) which states this, and is the answer. that little expectancy has increased in spite of the increase in
If (B) was the assumption, the conclusion of the researchers the use of chemicals).
would have been that it is the injection of a higher dosage of (D) is wrong since, according to the author, there had been
cancer cells, and not stress, that enhances the development a dramatic increase in the use of chemicals in US since 1940,
of cancer in mice. So, (B) is not the assumption of the which does not rule out there having been a less dramatic
researchers. increase before 1940.
If (C) was the assumption, both groups of mice would have (E) is incorrect, since the author does not discuss the factors
been subjected to equal stress, and would have shown similar contributing to the growth of industry between 1940 to 1979,
results in terms of the percentage among them that developed but is only concerned with the health level of the US
cancer. So, (C) is not the answer. population.
The observation was made only after the mice had been
injected with cancerous cells and their cages rotated in a 5. (The reasoning for this question is similar to that for the
disorienting manner. Whether mice that are kept in cages previous question.) We learn from the narration that, in 1981,
which are rotated in a disorienting manner would develop the increase in expenditure by US businesses on research
cancer even without the prior injection of cancerous cells into and development was 16.4% over what was spent in 1980,
them will have to be tested in another separate experiment. and that this annual increase has been steadily coming down
(D) is therefore not a necessary assumption for drawing the since then, and has touched just 8% in 1984.
conclusion from the present experiment. The author then comes to the conclusion that the tax
(E) also has to be tested in a separate experiment in which incentive of 25% introduced in 1981 on expenditure on
the cages are rotated in a manner that is not disorientating. research and development by US companies has not had the
It is therefore not a necessary assumption for drawing the desired effect of stimulating expenditure on research and
conclusion from the present experiment. development.
The author is obviously assuming that, even if the tax
3. The first sentence says that, whereas most universities incentive had not been introduced, the expenditure on
retain the royalties from faculty member’s inventions, the research and development by US businesses would have been
faculty members themselves retain the royalties derived from of the same order, and not less, from 1981 and 1984. He does
the books and the articles they write. not consider the possibility that, if the tax incentive had not
A conclusion is drawn from this practice that, therefore, been introduced in 1981, the expenditure on research and
faculty members should retain the royalties from the development by US businesses would have been even lower
educational computer software that they develop. than what it actually was during these years, and that,
We have been asked to identify that choice which would therefore, the tax incentives had, in fact, served the desired
form the missing premise on which the above conclusion could purpose.
be more reasonably drawn. It is (D) which states this, and is the answer.
Obviously, what the author has assumed while drawing None of the other assumptions has a nexus between the
his conclusion is that educational software programs are more tax incentive introduced in 1981 and the author’s conclusion.
comparable to books and articles than to inventions. It is (E)
which explicitly states this missing assumption, and is the 6. It is obvious, while reading the first and the second
answer.
149
American Education Aids - LSAT - Logical Reasoning - Analysis
sentences together, that the author considers the increase in SET 2
the price of postage stamps as an effort to reduce the number Answers
of persons using the postal system. Obviously, he assumes I.
1.C 2.A 3.C 4.A 5.C 6.C 7.E 8.C 9.D 10.C
Since the author holds that one of the improvements
Analysis
anticipated by such measure is the reduction in the percentage
1. The ‘plan’ (whose assumption we have been asked to
of letters lost, he obviously assumes II also.
spot among the given choices) relates to the cost to insurance
Since the purpose of the increase in the price of postal
companies in meeting the claims relating to the medical
stamps is not to increase the revenue of Government, but to
treatment of stress-related disorders.
discourage people from using the postal system unnecessarily,
So, (A), (B) and (D), which do not even mention insurance
III is neither an assumption nor a conclusion of the author.
companies, can be discarded straightway.
So, III is not true.
Since the suggested plan is that, for lowering their costs,
Therefore, the author assumes only I and II, and (D) is the
the insurance companies must subsidize psychotherapy, the
answer.
obvious assumption made is that, in the case of stress-related
disorders, the cost to the insurance companies for reimbursing
7. The passage says that the research was undertaken on
the expenditure on conventional medical treatment will be
behalf of the Bureau of Tourism of Florida State, and the
higher than the suggested subsidies for psychotherapy for these
researcher’s interest was only to find out the attitude of people
disorders. So, (C) is the answer.
towards spending their vacations in Florida. But she did not
(E) is an unwarranted assumption that is not logically
tell the interviewees that her survey was being conducted on
related to the suggested plan.
behalf of the Florida organisation.
The reason for her strategy of suppressing this fact is
2. The statement in the second sentence, “their enthusiasm
contained in the phrase “in an effort to obtain valid
would be dampened” shows that the author assumes that
information”. Obviously, the interviewer thought that, if she
those who favour the cutting of hospital costs and diverting
revealed the name of the sponsor of the survey, the
the money so saved to important social purposes would oppose
interviewees might not give their honest opinion, and might
its use towards other purposes such as the expansion of the
give opinions which they thought would please the sponsors.
military, for balancing the budget, revitalizing the space
If this had happened, the information that she gathered would
program etc. Thus, (A) is the answer.
not have been true and valid. It is (C) which states this, and
You can easily see that the other choices are totally off the
is the answer.
mark.
None of the other choices is directly related to why she
was suppressing the name of the sponsor during her
3. The meaning of the first sentence is that foreign cars
interviews.
are cheaper than US-made cars and, therefore, the imposition
of quotas on imported cars forces US citizens to buy the more
8. (‘Taking advantage of bargains’ means ‘buying at a time
expensive local cars, and that it will leave less money with
when things are sold at discounted and reduced prices’).
them for expenditure on other things.
Whenever things are bought on credit, interest has to be paid
The second sentence draws the conclusion that this will
on it. The author says that one can save money by buying on
result in unemployment in other domestic industries.
credit to take advantage of bargains.
We have been asked to spot the assumption that has been
This is will be true only if the interest payable for buying
made in this line of reasoning.
on credit does not exceed the discount that one gets during
Obviously, what the author assumes is that any person
bargain sales. (B) is what states this, and is the answer.
who pays less price for the imported car will not keep the
difference in price in savings or use it for travel or recreation
9. When Zeuxis painted a picture of grapes, they were so
etc, but will use it to buy other goods produced in USA, thereby
realistic that birds pecked at them.
enabling those industries to create more employment
It is given that, when he painted the picture of a boy
opportunities within USA.
carrying grapes, even then the birds pecked at the grapes.
It is (C) which states this, and is the answer.
If Zeuxis had presumed that birds would not peck at fruits
if they see a human being also present, then he would have
4. The first sentence estimates the reserves of oil that exist
been disappointed with his later picture because his painting
underground in the unsampled areas as 10,000 times the
of the human figure was apparently not realistic enough to
estimated reserves in the sampled areas.
scare away the birds. So, (D) is the answer.
On this basis, the second sentence draws the conclusion
(A), (B), (C) and (E) do not constitute a valid reason for the
that the oil needs of the next five centuries can be easily met,
disappointment of the painter.
even if the rate of consumption grows rapidly.
Obviously, the author assumes that the estimated reserves
10. The last sentence of the passage implies that the transit
in the unsampled areas can be recovered, because, if they
time and the transportation cost are the only two
cannot be recovered (because they are located too deep below
considerations that the farmers have while making a decision
the earth to make their recovery economically viable, or they
between choosing the truck and choosing the train.
are under deep ocean beds, or they are under heavily
So, the author assumes that there is no other significant
populated areas etc.), the mere fact that they exist
differences between these two modes of transportation from
underground cannot meet the future oil needs of humanity.
the farming region to New Mexico.
So, (A) is the answer.
So, the answer is (B).
None of the other choices is a necessary assumption for
None of the other choices is germane to the conclusion of
drawing the conclusion from the given premises.
the author given in the last sentence of the passage.

150
American Education Aids - LSAT - Logical Reasoning - Analysis
5. (To stunt means to suppress) In the second sentence, the by merely using the trace amounts left after the vein has been
author says that the reason why television stunts a child’s cut.
powers of imagination, while the pages of a book enhance its We have been asked to identify that choice which forms a
powers of imagination, is that the book can be read as slowly necessary assumption in coming to the above conclusion.
or as quickly as the child wishes, while television images Since the conclusion is only about the insects’ ability to
appear with a speed which cannot be controlled by the child. eat milkweed leaves, the question whether they are able to
So, his assumption is that a child’s imagination can develop depoison other weeds is not relevant to the conclusion. So,
more fully when the child is able to control the speed of its (A) is not a necessary assumption for arriving at the
entertainment. (C) says just this, and is the answer. conclusion, and is not the answer.
(A) makes no mention of a child’s powers of imagination, (B) is wrong for the same reason as (A), because the
and cannot be the answer. conclusion is only about milkweed plants.
(B) is wrong because, according to the author, books alone One presumption that has been specifically stated in the
are sufficient for the development of a child’s powers of second sentence is that the trace amounts of poison left after
imagination. the veins have been cut will never be strong enough to kill the
(D) and (E) also do not make any mention of a child’s powers insects. The other presumption implied here is that it will
of imagination, and can be discarded. always be possible for the insects to cut the veins of the
milkweed plant. This again implies that the milkweed plant
6. We have been asked to spot the choice which, together will never be able to evolve veins which cannot be cut by
with the first statement (“Neither Peter nor Sarah has any insects. (C) says just this, and is the answer.
common sense”), will logically lead to the conclusion in the (D) also relates to depoisoning of other plants, and is not
second statement (“Neither Peter nor Sarah is able to run the relevant to the conclusion in the given narrative.
factory”). The conclusion in the given narrative relates to only those
Obviously, the connecting statement must be one which insects which are capable of cutting and draining the veins
says that common sense is required to run the factory. that secrete the poison in the milkweed. (E) relates to other
It is (C) which makes this statement, and is the answer. insects which use other strategies while eating the leaves of
None of the other choices is in the nature of a missing other poisonous plants, and is therefore not relevant for the
logical assumption between the two given statements. arrival of the conclusion in the given narrative. So, (E) is not
the answer.
7. The conclusion of the art experts that The Man with the
Golden Helmet is not a true Rembrandt is based on an analysis 9. (It is the immune-system activity in our body that protects
of stylistic features, especially details both of shading and of us from disease-causing agents such as bacteria and viruses.)
brushwork. This obviously implies that they could recognize It is known that persons with low immune-system activity
some consistent stylistic features in other genuine Rembrandt are more susceptible to physical ailments than those with
paintings which are absent in The Man with the Golden normal or high immune-system activity.
Helmet. It is (E) which states this, and is the answer. The researcher mentioned in the narration has noticed
Since the time element relating to the period of the painting that persons with low-immune system activity score much
is not referred to in the passage, (A) is not a necessary lower on mental health also than people with normal or high
assumption leading to the expert’s conclusion. It could well immune-system activity.
have been drawn by a contemporary of Rembrandt. He has therefore concluded that a good immune system
The passage is about the authenticity of only The Man protects a person not only from physical disease but also from
with the Golden Helmet as a Rembrandt painting, which does mental illness.
not depend upon the authenticity of other Rembrandt We have been asked to spot the choice that forms an
paintings. So, (B) is not a necessary assumption for the assumption made by the researcher while coming to this
experts’ conclusions. conclusion.
(C) in fact contradicts the artists’ assumption because, if The fact noticed by the researcher is that persons with
the painting that exists today of The Man with the Helmet is mental illness also have low immune system activity. There
a copy of Rembrandt original, it must contain the stylistic are now two possibilities: (i) low immune-system activity is
features contained in other Rembrandt paintings. the cause and the mental illness is the result; or (ii) mental
The experts’ only concern is whether The Man with the illness is the cause and the low immune-system activity is
Golden Helmet was painted by Rembrandt or not, and they the result.
are not themselves sure of their tentative conclusion that it The researcher’s conclusion that a good immune system
was not painted by Rembrandt. This is the reason why they protects a person against mental illness implies that a bad
have proposed to subject the paintings to a series of further immune system leads to mental illness, which is based on
sophisticated tests. So, it is premature for them to suppose the first possibility mentioned above. He is therefore ruling
that the original ascription of the painting to Rembrandt was out the second possibility. In other words, he is assuming that
a deliberate fraud. So, (D) is not a necessary assumption by mental illness does not cause a reduction in people’s immune-
the experts. system activity.
(D) is what states this, and is the answer.
8. The first sentence in the narration states that some The researcher makes a distinction only between ‘low
insects are able to feed on the leaves of the poisonous immune-system activity’ and ‘normal or high immune-system
milkweed plant by first cutting and draining the vein that activity’. The narration does not indicate that he had
secretes the poison. The second sentence draws the conclusion discovered that a person with normal immune-system activity
that this method of depoisoning guarantees that some insects is more susceptible to mental illness than a person with high
will always be able to eat milkweed leaves, because the plant immune-system activity. So, (A) is not a necessary assumption
will never be able to produce a poison that is capable of killing made by him while coming to his conclusion.
151
American Education Aids - LSAT - Logical Reasoning - Analysis
The researcher’s conclusion does not depend on the stopped, and would not have been allowed to take place.
assumption that mental illness and physical disease have If (E) is an assumption that is made, then the conclusion
the same effect on other body systems. So, (B) is not the answer. logically follows that the government must have known about
From the correlation that he has found between low the smuggling of the weapons also. So, (E) is the answer.
immune-system activity and mental illness, the researcher
has come to the conclusion that immune system protects a 3. The first para says that therapists have found that their
person against mental illness. That a person with high treatments for smoking and overeating for those who
immune-system cannot develop mental illness may be a approach them for such treatment are rarely successful.
further conclusion that may be arrived at him, but it is not But the second para quotes an independent survey to show
an assumption by him for coming to the earlier conclusion. that millions of people have in fact given up both smoking
So, (C) is not the answer. and overeating.
The passage does not mention the methods of treatment If both these statements are true, it follows that those who
for mental and physical illnesses, and so (E) is irrelevant to had cured themselves of smoking and overeating had done
the researcher’s conclusion. so by themselves without going to the therapists, and that
their cases had not been included in the therapists’ data. So,
10. After pointing out in the second sentence that the only (E) is the answer.
cyclists seriously interested in innovation and willing to pay (A) is wrong, because some successes in therapy cannot
for it are bicycle racers, the author draws the conclusion in add up to millions and, if they had, then the therapists would
the next sentence that “therefore, innovation in bicycle not have come to a negative conclusion about the effect of
technology is limited by what authorities will accept as their treatment.
standard for purposes of competition in bicycle races”. (B) is wrong, because the contradiction referred to in the
He obviously assumes that even bicycle racers do not sentence is not between smoking and overeating.
generate a demand for innovations which are not officially (C) is wrong, because it is not given that giving up smoking
recognized as standards for purposes of competition. and overeating by using one’s own will power is an easy job.
(C) is what states this, and is the answer. (D) is not the answer, because the survey does not say that
The other choices are not related to the limitation on the all those who were surveyed had given up smoking and
innovation in bicycle technology as apprehended by the overeating.
author.
4. The narrative says that aspirin and acetaminophen
SET 3 together shared 100% of the non prescription pain-reliever
Answers market till 1984 and that, in 1984, ibuprofen was expected to
steal 15% of the share of this market.
1.B 2.E 3.E 4.E 5.B 6.E 7.D 8.A 9.B 10.B
On this basis, a prediction is made that the combined sales
Analysis
of aspirin and acetaminophen was likely to register a 15%
1. The first sentence says that Child’s World, a chain of
decrease between 1983 and 1984.
toy stores, has already implemented the computerized
We have been asked to identify that choice which is a
inventory control and customer self-service in the toy section,
necessary assumption in making the above prediction.
and has eliminated the category of sales clerks there.
The first sentence talks of 15% market share for the new
The fact that it is now proposing to extend the scheme to
drug, while the second talks of decrease of 15% in the total
the selling of children’s clothes implies that the scheme of
sales. This would be true only if the overall size of the market
‘customer self-service’ has been successful in the toys’ section,
remained the same.
and that the sale of toys had not been affected by its dispensing
If, on the other hand, the overall market size for non-
with the sales clerks.
prescription pain relievers in 1983 was $100 million and it
In planning to extend the scheme to the children’s clothes
had increased to $120 million in 1984, the sale of the new
section, the company is obviously assuming that personal
drug would be 15% of $120 million, which is equal to $18
service by sales personnel is not required for selling children’s
million. So, the total sale volume of the two older drugs would
clothes either.
have constituted $102 million, which is an increase of $2
So, (B) is the answer.
million.
None of the other choices is a necessary assumption for
Therefore, in making the prediction that there will be an
the company’s decision to extend the scheme to the children’s’
exact 15% drop in the combined sales of the two older drugs,
clothes section.
an assumption has been made that there will be no increase
in the total market volume of nonprescription pain relievers.
2. From the two pieces of facts that country Y is a closed
So (E) is the answer.
society, and that a weapons smuggling incident recently took
place in Y, the author comes to the definite conclusion that
5. From an agreement among investigators regarding
the government of Y must have known about the weapons.
maternal behaviours (towards their children) which can be
We have been asked to spot the choice which is an
classified as signifying acceptance and signifying rejection,
assumption made by the author in coming to this conclusion.
some psychologists have concluded that there are two
(A) and (B) talk about government in general, and not about
distinctive sets of parental behaviours in the United States.
‘the government of a closed society’, and are not germane to
We have been asked to spot the assumption on which this
the author’s conclusion.
conclusion is based.
(C) contradicts the given statement as well as the
We can easily notice that a conclusion regarding the
conclusion that weapons had in fact entered country Y, and
behaviors of both parents has been drawn from an
that Y’s government also knew about it.
investigation of the behavior of the mothers alone. Obviously,
If (D) is true, smuggling of weapons would have been
the assumption made is that fathers also behave in the same
152
American Education Aids - LSAT - Logical Reasoning - Analysis
manner as the mothers. It is (B) which states this, and is the farmers can realize greater savings than small farmers. So,
answer. (A) contradicts what is stated in the given narrative, and is
not the answer.
6. The only reason given by the author for concluding that The suggestion that large farmers must also adopt the
the sharp reduction imposed this year on research programs practice of using manure instead of chemical fertilizers
relating to social and behavioral sciences is not dictated by assumes that a sufficient amount of manure will be available
financial constraints is that, even last year, support for these for use in the fields of large farmers. So, (B) is a necessary
programs amounted to only three percent of the total research assumption for the argument in the given narrative, and is
funds in the United States. the answer.
He obviously assumes that a saving of three percent in the The given narrative restricts its scope to comparing the
total budget for research funds makes no difference to cost of using chemical fertilizers as against organic manure.
budgetary balancing or, in other words, three percent is an (C), which talks of the cost effectiveness of using sewage
insignificant portion of the research budget. So, (E) is the sludge, is irrelevant to the argument in passage. So, (C) is
answer. not the answer.
If (D) is true, large farmers would already have adopted
7. The two arguments advanced by the optimists (to counter the use of natural manure in place of chemical fertilizers,
the doomsayers’ warning that long term cooling or warming and there would therefore have been no need for the author
trends in weather pattern will drastically reduce grain to exhort them to do so. So, (D) is not a necessary assumption
production) are: (a) changes in weather pattern are not for the author’s argument.
generally accompanied by fluctuations in rainfall patterns, (E) would, in fact, contradict the argument of the author,
and (b) technological improvement will compensate for any because it would imply that the lower operating costs in a
likely shortfall in grain production that may arise from small farm by the use of manure may not necessarily be
changes in weather patterns. achieved in a big farm. So, (E) is not the answer.
The first argument implies that grain production will
indeed be affected if the changes in weather pattern are 10. According to the first sentence, the optical illusion of
accompanied by fluctuations in rainfall patterns. (D) is what ‘velocity and size’ means that a truck, being bigger, would
states this, and is the answer. appear to move slower than it actually does, while a car, which
The other choices are not relevant to either of the two is smallerr, would appear to move faster than it actually does.
arguments of the optimists. The assertion in the second sentence (that a motorist’s
estimate of the time available for crossing a highway with a
8. The author’s argument is that it is the scope of a small car approaching him is bound to be lower than it would
negotiating party’s aims that determines the energy and be with a large truck approaching) will be true only when it
perseverance that will be brought by him to the negotiations. is assumed that both the truck and the car are approaching
He then concludes that a party with unlimited aims will him at the same speed.
win in the negotiations, and the party with limited aims will Otherwise, if the approaching car is running faster, the
lose. lesser time available for crossing it would be due to the car’s
Obviously, the author assumes that the chances of success own higher speed and not due to the optical illusion.
or failure in a negotiation depend not so much on the merits On the contrary, if the truck is running faster, the lesser
of each negotiating party’s case, but on the energy and time available for crossing it may be compensated by the
perseverance with which each party conducts the negotiation. operation of the illusion of velocity and size, and the car and
(A) states just this, and is the answer. the truck would appear to move at the same speed,
The author’s argument relates only to those negotiations contradicting the conclusion in the second sentence.
in which the scope of the aims of the two parties differs. The So, (B) is the answer.
author does not assume that there will be no negotiations at (A) and (C) are wrong because of the above reasoning.
all between parties both of whom have limited aims. So, (B) (D) and (E) are wrong because, in both cases, the effect of
is not an assumption by him. the optical illusion will be to make the estimate of the driver
(B) may be a true statement, but is not a necessary inaccurate.
assumption of the author.
(D) is in the nature of a conclusion by the author, and not SET 4
an assumption by him. Answers
(E) is also not an assumption that is necessary for the
1.B 2.D 3.E 4.D 5.E 6.A 7.D 8.D 9.B 10.B
author’s conclusion.
Analysis
1. From the facts that, among all those who became
9. The first sentence says that small farmers save on
teachers in 1968, 30% of those who had subsequently left the
operating costs by eliminating chemical fertilizers and
teaching profession are earning above $35,000 a year, but
pesticides in favour of crop rotation and twice-yearly use of
only 15% of those who have remained in the profession earn
manure as fertilizer.
salaries above $35,000 a year, the author comes to the
The next two sentences argue that, if the large farmers
conclusion that teachers are underpaid today.
also adopt the same measures, they will realize greater total
We have been asked to state the assumption that the author
savings than the smaller farmers.
is making to validate his conclusion.
We have been asked to identify that choice which forms
The author himself says that not all, but only 35% of those
an assumption while advancing the above argument.
who had left the teaching profession, are now drawing more
If it is more cost-effective for small farmers than for large
than $35,000 a year. This implies that the higher salaries
farmers to eliminate the use of chemical fertilizers and
drawn by them cannot be attributed to their qualifications
pesticides, one cannot come to the conclusion that large
153
American Education Aids - LSAT - Logical Reasoning - Analysis
that had enabled them to become teachers earlier, but must relating to a limited number of clients of a single consultancy
have been due to other factors such as their higher firm. Moreover, (D) is in the nature of a conclusion (though
qualifications, aptitudes, capabilities etc, which the others invalid), and not a presupposition warranting the conclusion
who had either remained in the teaching profession or had drawn by the president of the consulting firm. So, (D) is not
left it do not possess. the answer.
Therefore, the author’s conclusion will be valid, only if he Unless there are some valid criteria which can be applied
has assumed that those who had left teaching and who did to marketing decisions to classify each of them as either
not leave teaching are comparable in terms of criteria that “right” or “wrong”, one cannot analyze them and come to a
determine how much the people outside the teaching conclusion that half of them were right and half of them were
profession are paid. So, (B) is the answer. wrong. So, (E) is a necessary presupposition for the conclusion
arrived at by the president of the consulting firm, and is the
2. The author specifically says in the last sentence, “Only answer.
when investigators find out why the error was made .... can
changes be made to ensure that the same type of error does 4. The concluding sentence in the given narrative is, “For
not recur and cause another accident”. this reason, cabinet making is not an art”. The immediately
Obviously, the presupposition of the author is that accident preceding sentence which contains the reason is: “Since
investigation should concentrate not only on examining why furniture must be useful, cabinet-makers must exercise their
an accident occurred in the past, but should also contribute craft with an eye to the practical utility of their product”.
to the prevention of similar accidents in future. So, (D) is the The obvious assumption that connects these two sentences
answer. is that an object, in the making of which a craftsman has
The sentence, “Instead of blaming an airline accident on paid attention to its practical utility, is not an art object.
pilot error, investigators should find out why the error was It is (D) which says this and is the answer.
made by analyzing airplane design .....” implies that the Since only art objects are placed in museums, a craftsman
author concedes that the accident was due to pilot error, but who makes a cabinet that is meant to be exhibited in a museum
implies that the error might have been made not necessarily should be considered to be an artist. So, (A) would contradict,
because of a personal drawback of the pilot, but because of a and not constitute an assumption for the conclusion that
defective airplane design etc. So, (A) is not an assumption cabinet making is not art.
made by the author. (B) would mean that, at least for those cabinetmakers who
The passage talks of ‘investigators’ in general, and there are not concerned with the practical utility of their products,
is no indication that the author means that the investigators cabinet making would be considered as an art. This again
must all belong to the airline companies. So, (B) is not the would contradict, and not constitute an assumption for the
answer. conclusion in the given narrative that “cabinet making is not
Stricter government regulation on airline companies may art”. So, (B) is not the answer.
improve airline management and pilot training programs, (C) says that cabinetmakers should be more concerned with
but cannot improve airplane design which the author the practical utility of their products than they currently are,
considers as one of the possible causes of airline accidents. and would mean that present-day cabinetmakers can at least
So, the author does not assume (B). marginally be considered as artists as per the author’s
Since the passage implies that airline accidents can also yardstick. So, (C) would also not constitute an assumption
be due to defective airplane design or airline management, behind the conclusion that cabinet making is not art.
we cannot say that retraining of pilots alone can avoid all The yardstick that is sought to be applied in the narrative
future airline accidents. So, (E) is also not correct. (to decide whether an article is an object of art or not) is its
practical utility and not its monetary value. So, (E) is
3. The conclusion of the president of the consulting firm, irrelevant to the conclusion in the given narrative, and does
based on her analysis of a number of marketing decisions by not constitute a necessary assumption for drawing that
her clients, is that these decisions “were correct only about conclusion.
half of the time”.
We have been asked to identify the presupposition on which 5. From the result of a study which found that top managers
such conclusion is based. used intuition significantly more than did most middle-or
Since the narration does not say that all the clients of the lower-level managers, the author comes to the conclusion that
consulting firm whose marketing decisions were analyzed intuition is actually more effective than careful, methodical
were successful companies, (A) is not the answer. In fact, even reasoning.
if they were all successful companies, (A) could only be a We have been asked to identify that choice which
conclusion to be drawn from the analysis, and not a constitutes an assumption while reaching the above
presupposition. conclusion.
The consulting firm’s study is related solely to the What the given narrative implies is that (i) both top
marketing decisions made by its clients, and its president’s managers and middle-or lower-level managers are involved
conclusion has nothing to do with all companies in general. in decision making; (ii) the middle-or lower-level managers
So, (B) is not a necessary presupposition for the conclusion generally use careful, methodical reasoning for decision-
arrived at by the president of the consulting firm, and is not making, while top managers mostly use intuition for decision-
the answer. making; and (iii) intuition is actually more effective than
(C) also has no relevance to the conclusion of the president careful, methodical reasoning.
of the consulting firm about the percentage of correct decisions Obviously, (iii) above will follow from (i) and (ii) only if it is
made by its clients, and is not the answer. assumed that top managers are more effective at decision-
(D) is a general statement covering all companies making than middle-or lower-level managers. (If, on the other
universally, and is not warranted on the basis of the finding hand, it is assumed that middle-or lower-level managers are
154
American Education Aids - LSAT - Logical Reasoning - Analysis
more effective at decision-making, it would follow that careful, necessary assumption on which the above argument is based.
methodical reasoning is more effective than intuition for In the first sentence, the author concedes that the press
arriving at a decision, which contradicts the conclusion should cover the factors in the campaign which bear on the
mentioned in the given narrative.) It is (E) which states this, eventual outcome of the election. While arguing that the press
and is the answer. should not cover the campaigns of candidates of minor parties,
You can easily see that none of the other choices is a he obviously assumes that the campaigns of candidates of
necessary logical link among (i), (ii) and (iii) mentioned above. minor parties are not factors that would bear on the eventual
outcome. Since the number of votes polled by the candidate
6. (Aesthetic merit means artistic merit.) In the first of a minor party would be based on his campaign, this means
sentence, the author states his conclusion that the financial that the author is additionally assuming that the number of
value of any particular painting is only indirectly connected votes polled by a candidate of a minor party is not likely to
with its own aesthetic merit. As evidence of his conclusion, he affect the eventual outcome of the contest between the
states in the second sentence that there is always an candidates of the two major parties. (D) states just this, and
enormous difference in financial value between the original is the answer.
of a painting and a forgery of it even when the naked eye cannot (To understand this argument clearly, suppose that the
distinguish between the two (implying that only scientific candidates of the Republican and Democratic parties are
experts can tell the difference, using methods such as carbon likely to get 51% and 49% of the votes polled in a straight
dating etc). fight, resulting in the victory of the Republican candidate. If
We have been asked to spot that choice which is a necessary a candidate belonging to a minor party enters the field as a
assumption made by the author while arriving at the third contestant and takes away 5% out of the Republican
conclusion based on the evidence cited by him. vote and 2% out of the Democratic vote, the eventual polling
Citing the vast difference in prices between the original of would be 46%, 47% and 7% in favour of the three candidates,
a painting and its forgery (even when they are resulting in the victory of the Democratic candidate. In this
indistinguishable by the naked eye), the author comes to the case, it is the campaign of the candidate belonging to the
conclusion that the price of a painting is only indirectly minor party that has had a bearing on the eventual outcome,
connected with its aesthetic merit. Obviously, he assumes that and must therefore be covered by the press even according to
there is no difference in aesthetic merit between the original the view of the author as stated in the first sentence. In arguing
of a painting and its visually indistinguishable forgery. In to the contrary, the author is obviously assuming that the
other words, he assumes that the authenticity of a painting votes polled by the candidate belonging to a minor party does
does not, by itself, contribute anything more to its inherent not affect the eventual outcome.)
aesthetic merit. (A) states just this, and is the answer. (A) states a reason why the candidates of major parties
The conclusion of the author is not based on the number of should be covered by the press, but does not give any reason
paintings which have accurate forgeries. So, (B) is not a objecting to the coverage of candidates of minor parties.
necessary assumption in the author’s conclusion. Therefore (A) is irrelevant to the argument of the author, and
If the forgery of a painting does not have the same financial does not form a necessary assumption by him.
value as the original as stated by the author, there is no The use of the plural phrase ‘minor parties’ in the last
incentive to forge it. So, (C) in fact weakens the conclusion of sentence implies that there is more than one minor party in
the author, and is not a necessary assumption made by him. the country. This means that there could be more than three
The author’s argument is based on the financial values of candidates in any given election (one each from the two major
only those paintings whose originals are distinguishable from parties, and one each from the many minor parties). So, (B)
their forgeries at least by experts. Otherwise, there would be contradicts the information in the given narrative, and is not
no basis for determining which is the original and which is a necessary assumption made by the author in advancing his
the forgery, and consequently there would be no basis for the argument.
difference in their prices. The author does not refer to Since the opinions of those who do not vote do not alter the
paintings whose authenticity has not been determined even eventual outcome of the elections, (C) is irrelevant to the
by experts. So, (D) is irrelevant to the conclusion of the author, argument of the author, and is not a necessary assumption
and is not a necessary assumption made by him. made by him.
The author merely implies that any original painting and If (E) is true, the author need not at all object to the
its visually indistinguishable forgery have equal aesthetic coverage of the campaigns by the candidates of the minor
merit (whether such merit is very high or very low). He makes parties. So, (E) does not constitute a necessary assumption
no statement about how the relative aesthetic merits of two made by the author while advancing his argument.
entirely different paintings can be determined. So, (E) is not
a necessary assumption made by him while drawing his 8. (Hittites were one of the ancient people like Greeks and
conclusion from the evidence cited by him. Romans, who had once lived in Northern Syria and adjacent
areas. Their language was also known Hittite. There are a
7. The first sentence of the narrative says that the proper large number of ancient stone inscriptions which give
role of the press during elections in the United States is to information about these people. These inscriptions have been
cover only those factors in the campaign which bear on the referred to here as ‘Hittite tablets’. The Iliad is a Greek epic,
eventual outcome. similar to the Ramayana).
On the basis of this premise, the second sentence argues The author’s main contention is that since The Iliad
that, since the outcome in the US elections is invariably a records historical facts, it cannot be considered to be creative
victory for the candidate of one of the two major parties, the literature. So, he certainly assumes I.
press should not cover the campaigns of candidates of minor The only reason why the author thinks that The Iliad
parties at all. record historical events and not legends is that these events
We have been asked to spot that choice which forms a have been recorded in Hittite tablets also. Obviously, he
155
American Education Aids - LSAT - Logical Reasoning - Analysis
assumes II also. consumer goods. It may as well be on other activities such as
Since, on the basis of some facts which are common to travel, holidaying etc. So, (E) is not a necessary assumption
Hittite tablets and The Iliad, the author has already come to by the author.
the conclusion that The Iliad is pure history, he assumes that
all cities and events mentioned in The Iliad must be true 2. James’ response “If you are not able to hear anything,
places and true events. He therefore does not assume III. you could not know that someone was calling you” implies
So, (D) is the answer. that he assumes that Harry would not have known that
someone had called him unless he had heard that call.
9. What has declined is not the disease Q itself, but ‘reported It is (B) which states this, and is the answer.
instances’ of Q, and this is what surprises the author, since,
according to him, it has come about after a great deal of 3. In spite of stating that there had been sharp drops in
publicity about the disease in newspapers, radio and TV. the number of reported volcanic eruptions during the two
Obviously the author must have presumed that, after such world wars covering a total period of ten years, the author
a wide publicity to the disease, there must have been an comes to the conclusion that volcanic activity has been ‘steady’
increase in the reported instances of it. So, (B) is the answer. during this century.
The other choices do not explain the surprise of the author Apparently, he assumes that the number of volcanic
logically. eruptions during the war periods had also conformed to long-
time averages, but many of these had gone unreported,
10. The author says that, in every Presidential election, perhaps because of the preoccupation of the scientists with
the number of voters who abstained from voting has been other war-related activities, or because of breakdowns in
greater than the number of persons who voted for the winner, communication. In other words, he presumes that the
and therefore ‘None of the candidates’ is the favourite of most variations in the frequency of reports of volcanic eruptions
voters. can reasonably be attributed to factors (namely wars) that
He obviously assumes that a voter abstains from voting had impinged on the reporting activity. So, (D) is the correct
because he/she does not like any of the candidates. (He does answer.
not consider the possibility that failure to vote could be due The other choices do not explain the inherent contradiction
to other reasons - the voter being away from his normal place between the two given sentences.
of residence on some work, his being ill, his being in jail etc.).
So, (B) is the answer. 4. From the first sentence, “Much of what passes for
You can easily see that none of the other choices is an political dialogue in this country is really a veiled discussion
assumption made for arriving at the author’s conclusion that of whether the distribution of wealth in our society is
‘none of the above’ is the perennially favorite candidate of equitable”, we can infer that the author assumes that
most citizens. economic policy has implications for the distribution of wealth.
So, (D) is the answer.
SET 5 (A) in fact contradicts the assertion of the author in the
Answers last sentence that questions of today’s economic policy are so
complicated that they are beyond the attention span of most
1.C 2.B 3.D 4.D 5.A 6.D 7.C 8.B 9.D 10.C
voters. So, (A) is not the answer.
Analysis
The passage is about how economic issues can be phrased
1. The first sentence says that when there were limitations
in simple terms so that they can be understood by the voters,
on nuclear-arms testing, people tended to save more of their
and not about the fairness of the economic policies themselves.
money and that, when nuclear-arms testing increased, people
So, (B) is not the answer.
tended to spend more of their money.
(C) also contradicts the argument of the author that it is
From this phenomenon, the author draws the conclusion
possible to phrase basic economic policy issues in such simple
that a perceived threat of nuclear catastrophe makes people
terms that most voters can understand them.
to spend now rather than to save for the future.
What the passage says is that the short attention span of
We have been asked to identify the choice which constitutes
most voters is the result of political dialogue being based on
an assumption in drawing this conclusion.
complicated phrasing of questions of economic policy. (E)
Obviously, the author assumes that people’s perception of
reverses the cause and the effect, and is not the answer.
the threat of nuclear catastrophe increases or decreases with
the amount of nuclear-arms testing in the world. It is (C) which
5. The author says that there are fundamental distinctions
states this, and is the answer.
between historical research and anthropological research, and
(A) can at best be a conclusion that can be drawn from the
mentions one such distinction.
given passage, and is not an assumption made by the author.
According to him, historians are interested in knowing why
The given passage mentions nuclear-arms testing, and not
a particular change occurred. (The word ‘whereas’ in the next
the development of nuclear arms. So, (B) has no relevance to
sentence means ‘on the contrary’). One the contrary,
it.
anthropologists are interested in knowing the meanings of
(D) implies that the extent of one’s support to nuclear-arms
actions and systems. The author thus implies that meanings
testing is directly proportionate to one’s capacity to save money,
of actions and systems cannot explain why a change occurred
implying that those who are capable of saving more money
because, if they can, then there will be no fundamental
are more in favour of nuclear testing. This is not only not an
distinctions between historical and anthropological research.
assumption in the passage, but is against commonsense.
So, (A) is the answer.
The statement that people spend more money when
nuclear-arms testing increases does not necessarily mean that
6. One of the beliefs of the ‘doctrine’ is that “an individual’s
such additional expenditure is incurred only on buying more
personality can be changed by management of the social
156
American Education Aids - LSAT - Logical Reasoning - Analysis
world”. So, the doctrine believes that, if a person has been a lawful.
criminal, he can be corrected through social reform. So, (D) is It is (D) which states this, and is the answer.
the answer. The other choices are all off the mark.
(C) appears to be a possible answer, but is wrong, because
the last sentence says that crimes will continue even if all 10. In the first sentence, the author says that a play
individuals are treated well by the society. becomes successful by reflecting the concerns and values of
the immediate audience.
7. According to the author, boxes packed with cereals (such According to this reasoning, the successful plays of the
as corn-flakes) used to contain parts of toys as gifts, so that Restoration period must have reflected the concerns and
children could first assemble them and then play with them, values of the audience who actually attended the plays.
but, nowadays, these toys come in fully assembled condition. But, in the last sentence, the author says that the
The author cites this as an example of the giving up of the successful plays of this period reflected (“was a good index
sense of ‘working now for later pleasure’. So, according to to”) “the tastes and attitudes of its time” meaning the concerns
him, ‘assembling the toys’ can be equated with work and and values of the whole population.
‘playing with them’ can be equated with pleasure. Obviously, in coming to this conclusion, the author assumes
This would have been so only if the same child had that the Restoration audiences were representative of the
assembled the toys first and played with them later. (The whole population of that time.
criterion ‘working now for later pleasure’ is obviously not It is (C) which states this, and is the answer.
applicable to a case in which one child had assembled the toy
but another child plays with it.) It is (C) which states this SET 6
and is the answer. Answers
1.A 2.D 3.C 4.B 5.C 6.C 7.C 8.D 9.E 10.C
8. (“Tenure system of employment” in Universities is the
Analysis
system in which a person who has been offered ‘tenure’ on
1. If, according to the first side, imprisonment would ‘deter’
the basis of his proven competence during the initial period
(meaning discourage) further rebellion, they obviously
of probation can serve uninterruptedly till his age of retirement
assumed that imprisonment was a harsh penalty, and the
without the risk of being fired for subsequent incompetence.
future rebels would therefore like to avoid it and will therefore
‘Grant of tenure’ in USA is equivalent to an employee being
not indulge in rebellion at all.
‘confirmed’ in his job in India.) The passage says that the
The second side, who argued that imprisonment would
best argument in favour of the tenure system is that senior
discourage future rebels from surrendering, also assumed that
faculty members are able to recruit younger teachers who
these rebels would like to avoid being imprisoned after such
are smarter than themselves, without any fear of losing their
surrender, considering imprisonment as a harsh penalty. They
own jobs.
would therefore prefer not to surrender.
The author obviously assumes that, if the tenure system
So, both sides are assuming that imprisonment is a harsh
did not exist, senior faculty members will be reluctant to
penalty. So, (A) is the answer.
recruit younger members whom they feel to be smarter than
themselves and who may subsequently threaten their own
2. The conclusion that we have to examine is “The brands
jobs. So, the answer is (B).
of gasoline for sale on Haibei may be different in name and
(A) is neither an assumption nor a conclusion of the author.
price, but they are identical in quality”. And we have to locate
The “beliefs” of the university scholars are not mentioned
that choice which is a necessary assumption for drawing the
in the passage, and so (C) is irrelevant.
above conclusion.
The author mentions only in passing about moral
If, after drawing gasoline from the storage tank, one
turpitude, merely to emphasize that a senior faculty member
company alters its quality before sale, it will be different in
who has been offered tenure cannot be fired for relative
quality from the gasoline sold by the other companies. This
incompetence (though he can be dismissed for moral
will contradict the assertion by the author that ‘the brands
turpitude). (D) is not therefore a necessary assumption for
of gasoline sold in Haibei are identical in quality’. Therefore,
his argument.
he assumes that, if one company makes such a change in
(E) contradicts the statement in the passage, and is not
quality, the other companies will also follow suit and make
the answer.
identical changes in quality in the gasoline drawn by them
from the same storage depot. So, (D) is a assumption
9. Presently, if police officers obtain any evidence against
necessarily made by the author, and is the answer.
an accused person by unlawful search or seizures, such
Since the conclusion is not about consumer awareness but
evidence will not be admitted in the criminal trial against
of actual quality, (A) is irrelevant to the issue.
him.
The argument is on the quality of the gasoline sold, and
The author first says that some people assert that such
not on the quantity. So, (B) is not a relevant issue for the
evidence must also be allowed in the courts, at least in cases
stated conclusion.
in which the concerned police officer was not aware that the
The narrative itself says that the prices of different brands
practice indulged in by him was unlawful.
of gasoline are different. The range of these prices is not of
The author then criticizes this argument by stating that,
relevance to the argument on the quality of the gasoline on
if this is allowed, it will lead to “less careful police practices,
sale.
and also lying in court by police officers”.
The size of the storage tank is relevant to the frequency of
This means that the author believes that the present
refilling it, and is not relevant to the quality of the different
prohibition ensures “more careful practices” by police officers
brands on sale. So, (E) is not relevant to the issue.
regarding searches and seizures. This would, in turn, mean
that the present searchers and seizures are, by and large,
157
American Education Aids - LSAT - Logical Reasoning - Analysis
3. From the fact that the world’s annual food production for the maintenance of their vehicles. Moreover, the passage
at present exceeds the amount of food required to provide a is about regulations and procedures to be enforced by the
minimum adequate diet for the world’s population, the author States, and not of safety consciousness of the drivers. So, (D)
comes to the conclusion that any hunger crisis in future will is wrong.
result from a distribution problem rather than a production The first sentence does not say that New Jersey has one of
problem. the lowest fatalities in the country, but says that it has one of
He obviously assumes that, forever in future, the world’s the lowest fatality rates in the country. So, if the number of
food production will either meet, or exceed, the population’s motor vehicles running in New Jersey is very much higher
food needs. So, (C) is the answer. than the number of motor vehicles running in most other
(A) implies that the population of the world will come down States in USA, it is quite likely that a larger number die on
in future which is obviously an unwarranted and unreal the roads in New Jersey than in most other States, but the
assumption. fatality rate per thousand of automobiles is lower in that state.
What the author says is that a hunger crisis that may be Moreover, the number of fatalities is a matter of fact, and
caused by a possible shortfall in food production may be cannot be an assumption of the author. So, (E) is also wrong.
prevented by a better distribution system. He does not say or
imply that the shortfall in food production may itself be 7. While fixing a specific rate of income tax, the tax
prevented by a better distribution system. So, (B) is not the authorities target to get a particular level of revenue.
answer. What the narration says is that when some people evade
If the author had presumed that the distribution of food the tax because of which this target is not reached, tax
supply will be improved in future, he would not have predicted authorities increase the rate of tax in order to ensure the receipt
that any hunger crisis will be caused by a distribution of the targetted level of revenue, and this increased rate tempts
problem. So, he does not assume (D). more people to evade tax, because of which the revenue falls
(E) is wrong for the same reason as (D), because the author even shorter than the targetted level, making the tax
does foresee the possibility of a hunger crisis arising becausing authorities increase the rate of tax still further ....., thereby
of a distribution problem. causing a vicious circle.
We have been asked to spot that choice which is an
4. (A) cannot be the view of the author, because, if he had assumption that has been made by the author in forecasting
held this view, he would not have described Stanislavski’s such a vicious circle.
method for training actors as ‘justly praised’. According to the author, the vicious circle starts when the
The last sentence says that, despite the pretensions of some tax authorities raise the rate of tax for the reason that evasion
of his disciples in the United States, the Stanislavski’s method by some people of the existing rate of tax has resulted in less
was never intended to offer rigid solutions to acting problems. tax being collected than targetted. He obvious assumes that,
This means that Stanislavski’s method incorporated flexibility while fixing the target of collection at the present rate of tax,
and innovation, and that these qualities have been lost the authorities had not made adequate provision for possible
because of the manner it has been adopted by some of his evasion of the tax by some people. (On the other hand, if they
own disciples. (B) says this, and is the answer. had fixed the target after providing for such evasion, the
None of the other three choices can be inferred from the target would have been achieved even when some people
information in the given passage. evaded the tax, and there would have been no need for the
rate to be increased. In this case, the vicious circle would not
5. The only basis for the hypothesis that much of the matter have started at all.)
in the universe is unseen is that scientists have made studies (C) is what states this, and is the answer.
which have shown that the actual pace of movement of
galaxies is faster than what it would be if visible stars alone 8. The conclusion reached in the passage is “Therefore,
constituted all their mass. secretaries who smoke should be offered enclosed offices’. This
It therefore follows that scientists have a fairly accurate conclusion is reached as a consequence of the previous
measure of the aggregate mass of the visible stars in the galaxy sentence, “Virtually all employees with enclosed offices are
and this is what has enabled them to make the calculations at the professional level, and virtually all secretarial
and arrive at their hypothesis. employees lack enclosed offices”, meaning that the new
(C) is what states this, and is the answer. smoking regulations allow all professional-level employees
to smoke, while preventing all secretarial-level employees
6. By recommending to other states that they should adopt from smoking. So, the demand that the secretarial staff who
“similar” annual safety inspection procedures as in New smoke must also be offered enclosed offices is based on the
Jersey, the author obviously assumes that annual automobile assumption that all employees who smoke must be given an
safety inspection in New Jersey have significantly contributed equal opportunity to do so, without an invidious distinction
to its registering one of the lowest rates of motor vehicle fatality being made based on their job levels. It is (D) which states
rates in the country. (C) states just this, and is the answer. this, and is the answer.
The passage talks not of rules relating to driving, but of (A) is obvious because, under the new regulations, among
annual inspection of motor vehicles. So, (A) is not a necessary the professional-level employees, even those who smoke can
assumption for arriving at the conclusion in the passage. retain their enclosed offices. But (A) is not a necessary
The passage is about what the author feels about the assumption for the conclusion that non-professional-level
connection between annual automobile safety inspections and employees should be given enclosed offices.
reduction in fatality rates. What most States feel about this (B) contradicts the new regulation, and is not an
connection is not relevant to the author’s logic. So, (B) is not assumption made by the author while arriving at his
the answer. conclusion.
(D) talks of drivers, and not of owners who are responsible There are two ways of maintaining equality between the
158
American Education Aids - LSAT - Logical Reasoning - Analysis
professional and secretarial staff in the matter of smoking: The definitive statement in (D) is not warranted by the
(i) the secretarial staff should be exempt from the new information in the passage in which the author only guesses
regulation, and be allowed to smoke at their desks; or (ii) the that ‘many of the suns in the galaxy could be accompanied by
secretarial staff who smoke should also be given enclosed earth-like planets’.
offices. The author is advocating the latter as his conclusion. By linking the possibility of existence of life to the possibility
(C), which proposes the former, is not a necessary assumption of ‘earth-like planets’ in other solar systems, the author
made by the author for recommending the latter. So, (C) is obviously assumes that earth-like conditions are necessary
not the answer. for life to thrive on any other planet. So, (E) is the answer.
(E) talks of the right of the non-smokers in the office, and
is not relevant to the conclusion of the author regarding the 10. When a person tells a lie, he undergoes some stress of
right of the secretarial staff who smoke. mind. A lie-detector is useful because, by measuring and
revealing such stress, it indicates whether a person is telling
9. (A) is wrong because the passage is about the possibility a lie or not.
of any form of life existing on another planet, and is not about The first two sentences say that a widely used tranquilizing
how they look. (In fact, even on earth, there are millions of drug suppresses the stress that one undergoes while telling a
forms of life, each looking very much different from the lie, and therefore can be used effectively to mislead a lie-
others!) detector.
The statement that ‘the greatest chance for the existence From this statement, the conclusion is reached in the third
of extra-terrestrial life is on a planet beyond our solar system’ sentence that the same tranquilizer can be used for reducing
does not preclude some chance of existence of life on another stress in everyday situations also.
planet in the solar system itself. So, (B) is wrong. The obvious assumption that is made while coming to this
The phrases ‘greatest chance’ and ‘suitable abodes of life’ conclusion is that the stress that one undergoes while telling
indicate that the author believes that, if appropriate physical a lie is similar to the stress that one undergoes in everyday
conditions exist, existence of life is a great possibility. He does situations. Otherwise, the same tranquilizer will not have
not presume that, in such a case, life is an inevitable the same effect on that person.
consequence. So, (C) is wrong. Thus, (C) is the answer.

159
Analysis - Chapter 5
'WEAKENS' QUESTIONS
SET 1 (D) would mean that both australopithecies and other
Answers animals might have been hunted and eaten by another species
1.E 2.C 3.D 4.C 5.B 6.A 7.C 8.D 9.A 10.E of larger animals which were living in these caves. If this is
Analysis true, australopithecies would have been victims of a predator,
and not great hunters themselves. This would seriously
1. The given argument is that the contributions made to
weaken the author’s conclusion. So, (D) is the answer.
charities by corporations reduce the profits of the corporations,
(E) would only mean that australopithecines did not hunt
and consequently reduce the dividends that their
this particular elephant-like animal for some reason (example:
shareholders, who are the real owners, are entitled to receive;
they did not like its meat). It will not weaken the author’s
therefore corporations should not contribute to charities.
conclusion that they were great hunters. (In fact, no great
This is prime-facie a logical and valid statement, but we
skill is required to hunt down a slow-moving, elephant-like
have been asked to identify the choice which would weaken
animal.)
the reasoning in this argument.
We should therefore look for that choice which states or
4 The first sentence attributes the improvement in the
implies that contributions made by corporations to charities
future prospects for success of the children (who were subjects
in fact increase their profits, and not reduce them. It is (E)
of a research study) solely to a particular educational program
which specifically points this out, and is the answer.
that these children had recently undergone.
The other choices do not directly relate corporate charity
We have been asked to locate that statement among the
to increase in their profits.
choices which will weaken this argument.
We should therefore look for the choice which give another
2. The given passage says that Tocqueville’s biographers
probable reason for such improvement in their prospects.
claim that his dislike of centralized government arose from
(C) provides such a reason, because it says that a
the fact that he considered the centralized rule of Napoleon
substantial number of the children who had been the subjects
responsible for his own childhood poverty in Normandy.
of this research had previously been exposed to another
We have been asked to identify that choice which would
educational enrichment program. If this is true, their
cast the most serious doubt on the biographers’ claim.
improved prospects for success may well be due to this
We should therefore look for that choice which says or
previous program, and not to the recent program whose
implies that Tocqueville himself considered his childhood
efficacy was the subject of the research study. So, (C) is the
poverty to be due to some reason other than Napoleon’s rule.
answer.
(A) refers to the knowledge of current historians, and not
The other choices do not constitute an alternative
of the nineteenth-century Tocqueville, who might well have
explanation for the expected result, and do not weaken the
agreed with the popular view of his time. So, (A) does not
given argument.
weaken the claim of his biographers.
(B) neither strengthens nor weakens the claim of
5. The speculation of the researchers is that people arrived
Tocqueville’s biographers.
in South America from the old world after voyaging across
(C) implies that Tocqueville himself believed that his
the Pacific some 32,000 years ago, and then spread northward.
childhood poverty was due to his father’s ill health, and was
This speculation is based on the discovery of human shelters
not because of Napoleon’s rule. If true, it is this choice that
in South America dating from 32,000 years ago.
will cast the most serious doubt on his biographers’ claim.
We have been asked to spot that choice which would be
So, (C) is the answer.
pertinent evidence against this speculation.
While (D) is irrelevant to the biographers’ claim, (E) would
Choice (B) says that some North American sites of human
strengthen their claim because it implies that there was an
habitation predate any sites found in South America. If this
all pervasive poverty in Normandy during Napoleon’s rule
is true, it is pertinent evidence against the theory that people
and such poverty could have been blamed on that rule.
arrived in South America first and then spread northwards.
So, (B) is the answer.
3. From the fact that bones of australopithecines (who are
(In fact, (B) would also provide strong evidence against
considered to be ancient ancestors of human species) and some
the earlier theory that people first reached America less than
bones of animals are found together in caves, the author comes
20,000 years ago.)
to the conclusion that australopithecines must have been
The other choices are not relevant to the researcher’s
great hunters.
speculation that people arrived first in South America and
We have been asked to identify the choice which weakens
then spread northwards.
this conclusion most.
If (A) is true, it would only explain why animal bones are
6. The passage suggests that private environmental groups
found not in just one cave but in many caves. It will not weaken
may be put in charge of operating and financing the national
the author’s conclusion that australopithecines were great
park system so that conservation objectives might in general
hunters.
be better served.
(B) would imply that, just like modern human beings,
We have been asked to identify the choice, assuming that
australopithecines were living as families. It will not in any
it is a realistic possibility, that argues most strongly against
way weaken the conclusion that they were great hunters.
this suggestion.
(C) would imply that australopithecines did not cook the
(A) says that persons who wish to exploit the natural
meat of animals before eating it. This will also not weaken
resources of the national parks may infiltrate into the private
the conclusion that they were great hunters.
160
American Education Aids - LSAT - Logical Reasoning - Analysis
environmental groups, and eventually take over their go specifically looking for dead birds and conclude that every
leadership. If this possibility is real, the administration of bird that had died even due to natural causes had suffered
the natural parks would ultimately fall into the hands of these the fate only because of the use of pesticides by the farmers.
undesirable persons, and they would start exploiting the We have been asked to spot the choice which would refute
natural resources of the park. In this case, the very aim behind this argument of the pesticide manufacturers.
the proposal, namely, “conservation objectives would be better The argument of the pesticides manufacturers will be
served” would be defeated. So, (A) itself is a strong argument refuted if it is proved that, even during the times or in areas
against the suggestion, and is the answer. where there was no publicity to this theory, the number of
Since, in case of disagreements among members of any dead birds reported had registered an increase. It is (D) which
group, it is the majority view that is always allowed to prevail, states this, and is the answer.
(B) is not a strong argument against the suggestion, and is (A) does not mention either pesticide spraying, or the
not the answer. publicity given to the theory, and is therefore not the answer.
The proposal in the passage is about the administration of (B) talks of the insects killed by the pesticides, and not of
the present national park system, and not about financing the birds killed by them, and is therefore irrelevant to the
the extension of the system. So, (C) is irrelevant to the given argument.
suggestion, and is not a strong argument against it. (C) means that a single dead bird would be reported by
(D) is not an argument against the suggestion at all; in many volunteers and would be counted as more than one
fact, if there is competition among various groups, the number; this will, in fact, strengthen and not weaken the claim
responsibility could be vested with the most resourceful among of the pesticides manufacturers that there has really been no
them so that the intended aim is served best. increase in the actual number of bird deaths recently.
(E) implies that some endangered species may die out in (E) would also strengthen and not weaken the argument of
spite of the private environmental groups having sufficient the pesticides manufacturers that pesticides were not the real
resources to try to protect them. If so, they may die out even cause of more bird deaths being reported.
if the administration of the national parks continues to be
with the government. So, (E) is not a strong argument against 9. The politician’s refutation of the Governor’s claim that
the given suggestion. the State’s economy is prospering is based only on his assertion
that the State has lost 26,000 manufacturing jobs during her
7. Based on the fact that machine tool imports from two term of office.
major exporters to US have been restricted recently, the We have been asked to spot that choice which, if it has
analysts have predicted that domestic sales of locally made happened during the Governor’s four year tenure, would most
machine tools are bound to increase substantially in the very seriously weaken the politician’s conclusion.
near future. (A) says that an influx of non-manufacturing industries in
We have been asked to identify the choice which is likely the State has led to a net gain of 200,000 new jobs in the
to render this prediction inaccurate. State. If this is true, the politician’s conclusion that the State’s
The fact that a bill is being debated in parliament cannot economy is not doing well (just because it has lost 26,000
by itself adversely affect the sale of machine tools. So, (A) is manufacturing jobs) will be very seriously weakened, because
wrong. the economy has, in fact, added 226,000 new jobs during the
(B) talks of the proportion of the purchase of one type of same period. So, (A) itself is the answer.
machine tools vis-a-vis the others in the last year, and can (B) would only imply that such of those workers who are
neither support nor invalidate the analysts’ prediction. still holding their jobs are not worse off than before, but the
(C) refers to the worldwide sales of US machine tools, while 20,000 persons who had lost their jobs are suffering from
the prediction relates to sales within the US. So, (C) is unemployment. So, (B) does not weaken the politician’s
irrelevant to the prediction. conclusion that the State’s economy is not doing very well.
(E) talks of trends in demand for machine tools in many So, (B) is not the answer.
countries, and is irrelevant to the prospects of US-made If 26,000 jobs have been lost, and the average level of
machine tools registering higher sale within US itself in the training required for entry level jobs in the State also
very near future. increased, it would only strengthen the politician’s conclusion,
From the fact that machine-tool imports from two foreign and will not weaken it. So, (C) is not the answer.
countries into US has been substantial, we can infer that (D) would only imply that the level of social security in the
many of the users of machine tools in US prefer these imported State had increased during the governor’s tenure, and does
tools to locally-made ones for some unstated reason. So, if a not by itself weaken the politician’s conclusion that the State’s
substantial inventory of these imported machine tools had been economy is not doing very well. In fact, (D) may even imply
stockpiled in the US during the past year, US buyers are likely that the unemployment created by the loss of jobs had been
to continue to buy them till these stocks get exhausted. In the main reason for the increase in the expenditure on social
that case, the prediction that sales of locally made machine security, in which case it would strengthen the politician’s
tools are bound to rise considerably in the very near future is conclusion, and not weaken it. So, (D) is not the answer.
likely to prove inaccurate. So, (D) is the answer. (E) talks of the opinion of the majority of the employed
workers in the state, and not of the 26,000 workers who had
8. The claim of the pesticides manufacturers is that there lost their jobs. So, (E) by itself does not weaken the conclusion
has been really no increase at all in the number of birds which of the politician, and is not the answer.
die in the agricultural areas because of the spraying of the
pesticides. 10. The view of the soil scientist quoted is that decaying
They argue that, formerly, these dead birds went unnoticed organic mater on forest floor is a greater source of acidity in
but, because of the publicity given to the theory that the use of mountain lakes than acid rain (presumed to be caused by
pesticides resulted in more bird deaths, more number of people industrial pollution) and, therefore, reducing acid rain will
161
American Education Aids - LSAT - Logical Reasoning - Analysis
not significantly reduce the acidity of mountain lakes. argued that, if this is done, the teen-age unemployment rate
We have been asked to identify the choice which most will no longer rise.
seriously weakens this argument. We have been asked to spot the choice which will weaken
We should therefore look for that choice which says that this argument.
the acid rain itself contributes to an increase in the amount of This argument will obviously be weakened if it is shown
decaying matter on the forest floor. that teen-age unemployment rate had risen even when the
(E) says just this, and is the answer. minimum wage for them was either constant, or had been
The other choices do not strengthen or weaken the reduced. It is (B) which states this, and is the answer.
argument of the soil scientists. (A) will support, and not weaken the argument in the
passage.
SET 2 (C) refers to seasonal employment, and implies that such
Answers employment is available to teenagers in spite of the higher
minimum wage. This choice therefore weakens the argument
1.E 2.C 3.B 4.E 5.A 6.C 7.E 8.C 9.A 10.B
in the passage that, for teenagers to get employed, it is
Analysis
necessary for the minimum wage to be reduced.
1. The first sentence says that the average weight of a 3-
(D) and (E) do not mention the levels of minimum wage
month old baby born in the United States is between 12 and
during the periods mentioned and so, neither support nor
14 pounds. From this statement, the second sentence draws
weaken the given argument.
the conclusion that, if a 3-month old baby weighs only 10
pounds, its weight gain is below the United States average.
4. Governments are today attaching a lot of importance to
We have been asked to identify the flaw in this reasoning.
environmental purity, and they periodically bring new
The flaw is in the words italicized above. The only
regulations which industries must comply with in order to
conclusion that can be drawn from the given statement is
keep down pollution.
that if a 3-month-old baby weighs only 10 pounds, its weight
The first sentence says that industries can and will build
is below the United States average. The statement that its
new plants conforming to environmental standards, but the
weight gain is below United States average does not
second sentence argues against the uncertainty in this regard.
necessarily follow from the first statement.
What the narration implies is that if, after permission is
(For example, if the average weight at birth of a US-born
given for the construction of a factory as per the existing
baby is 9 pounds, and it increases to 12 to 14 pounds in 3
regulations, the Government changes the regulations and
months, the average weight gain in 3 months is between 3 to
asks the factory to suspend further construction till the new
5 pounds. It is quite possible that the baby in question was
regulations are also satisfied, then it will affect the growth of
born prematurely and had a weight of only 6 pounds at birth.
industry because the industry may not be able to meet the
If it weighs 10 pounds at the end of 3 months, its weight gain
additional cost required to meet the new standards. In such
is 4 pounds, which is not below the United States average.)
a case, entrepreneurs will not even attempt to invest in new
So, the flaw in the reasoning is that average weight is wrongly
factories.
equated with average weight gain.
This threat to the growth of industry would be minimized,
It is (E) which states this, and is the answer.
if it is laid down that environmental standards for any factory
will not be altered after the construction has started, and that
2. The given passage says that, because shrimp can tolerate
the new standards would be applicable only to new
salinity in water much better than can other types of fish,
constructions.
increased concentration of salt in the bay will not hurt the
It is (E) which says this, and is the answer.
shrimp industry.
We have been asked to spot the statement which will
5. The passage says that parents have no longer much say
weaken this argument.
in the kind of education that their children receive, since the
We should therefore look for that choice which points out
function has been taken over by professional educators, who
a fact relating to concentrated salinity which will have an
have themselves become ineffective because of the
adverse effect on shrimp’s survival.
centralization and bureaucratization of schools. We have been
Among the choices, it is (C) which points out such a fact,
asked to spot that choice which weakens the claim that there
because, if the other organisms on which shrimp usually feed
is continuing erosion of parent’s control over their children’s
cannot thrive in water with high salinity, the shrimp will die
education.
of starvation, thereby affecting the shrimp industry adversely.
(B) and (E) do not even mention parental control, and can
So, (C) is the answer.
be rejected straightaway.
Choices (D) and (E) will strengthen the argument of the
(C) and (D) will in fact strengthen the given claim and not
author, while the other choices will neither strengthen nor
weaken it.
weaken it.
(A) says that growing numbers of school administrators
follow the recommendations made by parents. If this is true,
3. The passage implies that the minimum wage fixed by
the claim that there is continuing erosion of parents’ control
the government for teenagers (who are capable of being
over their children’s education gets weakened.
employed to give ‘extra help’ to the adult workers) is too high
So, (A) is the answer.
and too disproportionate to their productivity levels.
Consequently, employers cannot afford to use them, and this
6. From the statement that squinting gives at least as much
has resulted in the rate of teenager unemployment increasing
protection from ultraviolet rays as do nonprescription glasses,
since 1960.
the author has come to the conclusion that there is, therefore,
The solution offered in the passage is that Government
no health advantage to be gained by wearing nonprescription
must fix a still lower minimum wage for teen-agers, and it is
sunglasses rather than by squinting.
162
American Education Aids - LSAT - Logical Reasoning - Analysis
We have been asked to locate the choice which will weaken 9. The passage first says that the treatment for
this conclusion. hypertension prevents strokes and heart disease, thereby
We should therefore look for that choice which implies that saving the medical expenses that would otherwise have been
squinting leads to some other health disadvantage which is incurred in treating them. It then points out that the money
not caused by wearing nonprescription sunglasses. so saved forms only one-fourth of the expenditure required
(A) can straightaway be ruled out, because it talks of to treat the hypertensive population (implying that treating
prescription sunglasses, and not nonprescription sunglasses. all the hypertensive patients would cost as much as four times
(B), if true, will strengthen and not weaken the conclusion the expenditure incurred in treating strokes and heart
in the passage that nonprescription sunglasses do not provide diseases that some of them may suffer as a result of
any health advantage over squinting. So, (B) is not the hypertension.) It therefore concludes that there is no economic
answer. justification for the preventive treatment of hypertension.
(C) says that squinting strains facial muscles and causes We have been asked to spot the choice which is most
headaches and fatigue. If this is true, there is certainly a damaging to this conclusion.
health advantage in wearing sunglasses rather than The conclusion relates to ‘economic justification’ for the
squinting. So, it is (C) which weakens the conclusion of the preventive treatment of hypertension, and we must therefore
author, and is the answer. look for that choice which points out an economic justification
(D) neither strengthens nor weakens the conclusion of the for it.
author that sunglasses do not offer any health advantage to (A) points out that though the many fatal strokes and heart
the wearer that is not offered by squinting, and is not the attacks resulting from untreated hypertension cause
answer. insignificant medical expenses (perhaps because, by the time
(E) is a mannerism of only “some people”. So, it neither they are admitted in the hospital, their condition is hopeless),
strengthens nor weakens the general and universal conclusion they cause large economic losses of other sorts. (Two examples
of the author that sunglasses do not offer any health could be: the loss of expertise available to the community
advantage to the wearer. So, (E) is also wrong. because of the death of a rare expert in a highly strategic
field; the loss of a bread-earning parent of a newborn baby).
7. The conclusion that the quality of care in university So, it is (A) which weakens the conclusion drawn in the
hospitals is lower than that in private hospitals is arrived at passage, and is the answer.
only on the ground that the death rate in the university (C) looks like a possible answer, but it is only a contrary
hospitals is higher. opinion and is not a fact.
This argument will be undermined if it is known that more 10. (Spouse means the husband or the wife.) In the first
number of patients whose chances of survival are bleak usually sentence, the author says that a study of the medical records
get admitted in the university hospitals than in private of people below the age of forty shows that those whose
hospitals. (E) is what states this, and is the answer. spouses have died have a significantly lower chance of
(D) would appear to be plausible answer but is not true, surviving upto the age of forty than those with living spouses.
because even the research activity in a hospital must be aimed From this statistics, the author comes to the conclusion
towards curing the patients. that the emotional stress of losing one’s spouse affects the
(A), (B), and (C) do not logically weaken the given argument. health of the surviving widow or the widower leading to her
or his own premature death.
8. What the given para states is that, since there has We have been asked to spot that choice which, if true, casts
actually been an increase in the number of job-related most doubt on the author’s conclusion (that it is the emotional
accidents under OSHA, it has been an ineffective agency. stress of having lost the spouse that had affected the health of
The choice which weakens this argument must be one the surviving partner leading to his or her death before
which implies that OSHA has, in fact, helped in the reduction reaching the age of forty.)
in the number of possible accidents. (A) strengthens, and does not weaken, the author’s
(A) would imply that there may have been no reduction in conclusion, and is not the answer.
the number of accidents in jobs which are outside OSHA’s (B) says that the medical records on which the author has
jurisdiction. This would neither strengthen nor weaken the based his conclusion included cases of accidents in which both
conclusion that OSHA has been ineffective in reducing of the married couple had received fatal injuries (meaning
accidents under its jurisdiction. So, (A) is not the answer. injuries that had ultimately led to their death). If this is true,
The statement that OSHA has been assigned a greater it means that, in the statistics relied on by the author, a
number of workplace activities to monitor, implying that it is number of deaths of the surviving widow or widower were
too understaffed to monitor all of them effectively, would only due to the injuries sustained in the accidents, and not due to
strengthen the given argument, and not weaken it. So, (B) is the emotional stress of having lost the spouse. So, it is (B)
not the answer. that weakens the conclusion of the author, and is the answer.
(C) says that “the ratio of work-related deaths and injuries (C) would strengthen, and not weaken, the author’s
to size of work force has fallen”. Let us presume that, before conclusion that the separation from the spouse causes
OSHA was established, there were 1000 workers and the deterioration in one’s health. So, (C) is not the answer.
number of accidents was 15 and that, after the establishment (D) would also strengthen, and not weaken, the author’s
of OSHA, the number of workers has increased to 5000, while conclusion that it is the separation from the spouse that had
the number of accidents has increased to 30. If this is true, it caused deterioration in the health of the surviving partner.
implies that, if OSHA had not been established, and the same So, (D) is not the answer.
ratio of accidents had continued, the actual number of (E) would also strengthen, and not weaken, the author’s
accidents would have been 75, and not a mere 30. So, it is (C) conclusion that the separation from the spouse causes
which weakens the argument in the passage that OSHA has deterioration in the health of the surviving partner. So, (E)
been ineffective, and is the answer. is not the answer.
163
American Education Aids - LSAT - Logical Reasoning - Analysis
SET 3 their operations through the sale of the seized cargo. If the
Answers Florida man’s proposal is accepted, the new privateers must
also be able to finance their operations through the sale of
1.D 2.B 3.C 4.C 5.B 6.E 7.D 8.D 9.A 10.E
the seized cargo. But the seized cargo in this case will be
Analysis
“illegal drugs”. So, the newly licensed privateers will not be
1. The first sentence says that the recent decline in the
able to sell them without violating the law. It is (C) which
value of the dollar was triggered by (meaning started by) a
states this, and is the answer.
prediction of slower economic growth in the coming year. The
(A) is not the answer, because the privateers can very well
second sentence, however, concludes that such prediction by
acquire ships which are faster than those of the drug
itself would not have resulted in the decline in the value of
smugglers.
the dollar, if there had not been a huge budget deficit of the
(B) in fact strengthens and does not weaken the Florida
government.
man’s proposal, because the Constitution does not have to be
We have been asked to identify that choice which, if true,
amended in order to implement the proposal.
would most seriously weaken the conclusion in the narrative.
(D) does not represent a flaw in the Florida man’s proposal,
We should therefore look for that choice which points out
because the new privateers can also plunder ships of
an instance in which a prediction of slower economic growth
smugglers belonging to countries other than USA.
had in fact caused a decline in the value of the dollar, even
(E) also does not constitute a flaw in the Florida man’s
when there was no significant budget deficit of the government.
proposal, but may make it easier for the privateers with more
It is (D) which makes such a statement, and is the answer.
sophisticated ships to plunder them.
(A) does not even refer to the decline in the value of the
dollar, and is not the answer.
4. The given passage argues against the proposed high-
The conclusion under examination is that a prediction of
speed train line in favour of buying a plane stating that the
slower economic growth and a huge budget deficit have
former, in addition to being three times as costly as a plane,
together caused a decline in the value of the dollar, and not
is a fixed-linear system in a world in which consumers prefer
that the budget deficit, by itself, had caused a slower economic
the free-wheel system consisting of cars, buses and planes
growth. So, (B) is irrelevant to the conclusion in the narrative,
which can go anywhere.
and is not the answer.
We have been asked to spot the choice which severely
The conclusion that is under examination is that dollar
weakens this argument.
values will not decline unless there is a huge fiscal deficit of
(A) strengthens the author’s argument that the train is a
the government. (C) merely gives an instance when dollar
fixed-linear system that is not favoured by consumers, and
values had declined when there was no prediction of slower
does not weaken it.
economic growth. So, (C) is also irrelevant to the conclusion
The author’s argument is about the superiority of airplane
under examination, and is not the answer.
to the proposed high-speed train line. (B), on the other hand,
(E) in fact strengthens, and does not weaken, the conclusion
talks of the superiority of the high-speed train line to cars
in the given passage.
and buses, and is not relevant to the author’s argument.
(C) controverts the author’s basic premise that the planes
2. The argument states that, at higher altitudes, the
are a free-wheel system, by pointing out that they can fly only
human body has more red blood cells per unit volume of blood
between airports, which are less convenient for consumers than
than at sea level, and this improves the performance of the
the high-speed train’s stations would be. So, it is (C) which
athletes.
severely weakens the author’s argument, and is the answer.
We have been asked to spot that choice which is most
(D) and (E) strengthen the author’s argument against the
damaging to the above conclusion.
proposed high-speed train line, and do not weaken it.
If (B) is true, and the volume of blood in the body is less by
25% in higher altitudes than at the sea level, then even a
5. Arguing that, in a free society, people have the right to
25% increase in the red blood cells per unit volume will be
take personal risks as long as such risks do not harm others,
offset by such decline. If this percentage is less, then the
the opponents of laws that require automobile drivers and
athlete’s performance at high altitude will be even worse than
passengers to wear seat belts conclude that each person
his performance at sea level. So, (B) is the answer.
should be left to decide for himself whether or not to wear a
(A), (C), (D) and (E) have no relevance to the scientific
seat belt.
statement made in the argument and do not weaken it.
We have been asked to spot that choice which weakens
Even if (A) is true, the athlete can start his practice after
this conclusion.
he had been acclimatized to the higher altitude.
We should therefore look for that choice which implies that
The given argument only says that any particular athlete
the risk taken by those who do not wear seat belts while
can perform better at high altitudes than he can at sea level,
driving or travelling in a car affects not only themselves, but
and does not say that all runners who trained at high level
also others.
will beat all runners who trained at sea level.
Obviously, (A) and (C) do not have any such implication.
(D) neither strengthens nor weakens the given argument.
(D) and (E) emphasize the risk that non-wearers of seat
(E) in fact strengthens the given argument, and does not
belts expose themselves to, and do not imply that others are
weaken it.
adversely affected thereby.
(B) says that, because the insurance companies are obliged
3. Note that we have to identify the drawback, or flaw, in
to pay compensation for the increased number of injuries and
the Florida man’s proposal, in order to explain why the
deaths of people not wearing seat belts, they have been forced
proposal is not practicable.
to increase the insurance premium rates for all automobile
The first two sentences state that the privateers who had
owners (including those who wear seat belts voluntarily). If
been licensed to plunder enemy ships in 1812 could finance
this is true, the argument that whether a person wears the
164
American Education Aids - LSAT - Logical Reasoning - Analysis
seat belt or not affects only him and not others will be etc, and people had avoided living in those places till recently.
considerably weakened. So, (B) is the answer. If, in the recent decades, people had occupied these areas
because of population pressure and poverty, they would have
6. The proposed ordinance will make it compulsory for all been affected by these disasters. From this we cannot draw
new homes to install sprinklers that will automatically go the conclusion that the earth, as a whole, has become more
into action if there is a fire and quench it. dangerous for habitation in the recent decades. So it is (D)
A home builder questions the necessity of this ordinance which weakens the given conclusion, and is the answer.
by pointing out that, even now, ninety percent of residential
fires are put out by a household member, and the installation 8. From the facts that the life-cycle of a red blood cell is
of automatic sprinklers would only marginally decrease only 120 days, and that a malarial-fever parasite that resides
property damage caused by residential fires. in a red blood cell is not capable of travelling to a new
We have been asked to identify the choice which would generation of red blood cells, the author draws the conclusion
most seriously weaken the home builder’s argument. that any fever that a person develops beyond 120 days of his
We should therefore look for that choice which implies moving over to a malaria-free region could not have been
that it is the remaining 10% of the fires, which are not caused by the malarial parasite.
extinguished by a household member for some reason or other, We have been asked to identify the choice which would
which cause the maximum property damage in household most seriously weaken the above conclusion.
fires, and that the automatic sprinklers would be effective in (A) is irrelevant to the information in the given passage,
extinguishing these fires and averting the consequential and neither strengthens nor weakens the given conclusion.
damage. (B) merely implies that some parts of the world are still
(E) says that the largest proportion of property damage in vulnerable to malarial epidemic, and neither strengthens nor
residential fires in caused by fires that start when no weakens the conclusion in the given passage.
household member is present. If this is true, the installation While the given conclusion talks of fever, (C) talks of
of automatic sprinklers will prevent such damage, thereby symptoms other than fever. So, it neither strengthens nor
weakening the home builder’s argument. So, (E) is the answer. weakens the given conclusion.
Even if (A) is true, it only implies that no formal training (D) implies that the malarial parasite can reside not only
is required for extinguishing most household fires, because, in the red blood cells of a person but also in his spleen. It adds
even without such formal training, 90% of the household fires that, if the parasites had reached the spleen, they are not
are now being extinguished by householders themselves. So, eliminated within 120 days. If this is true, a person, in whose
(A) does not weaken the home builder’s argument. So, (A) is case the malarial parasite had reached the spleen, can develop
not the answer. malarial fever even beyond 120 days of his moving to a
(B), in fact, strengthens, and does not weaken, the home malaria-free region. So, it is (D) which weakens the conclusion
builder’s argument that the proposed ordinance would have in the given passage, and is the answer.
only a marginal effect in reducing overall property damage While the given passage is about persons who are
in household fires. So, (B) is not the answer. vulnerable to malaria, (E) talks of persons who are immune
The argument of the home builder is not related to the to malaria. So, (E) also neither strengthens nor weakens the
cost of installing the sprinklers at all. So, (C) neither given conclusion.
strengthens nor weakens his argument.
The response time of the city fire service in meeting 9. The argument of the judicial experts is that multiple
emergency calls is not an issue at all in the home builder’s opinions by different judges accompanying a single ruling
argument. So, (D) also neither strengthens nor weakens his create problems for lower courts and other branches of
argument. government which require guidance. We are asked to state
which of the three statements, if true, would weaken this
7. The given argument first states the fact that more argument of the judicial experts.
number of people have been affected by natural disasters I means that the lower courts and other government
during the past decade than during the previous decades. branches can just follow the ultimate ruling in the case
It then draws the conclusion that Earth has therefore ignoring the opinions accompanying it, and the ruling itself
become more inhospitable and dangerous to human beings would constitute sufficient guidance to them. If this is true,
than before. We have been asked to identify the choice which, it would weaken the argument of the judicial experts.
if true, will weaken this conclusion. II means that the (Supreme) Court is as much divided as
(A) and B, if true, will undermine the truth of the fact the rest of the nation itself on controversial issues. This, if
stated in the first sentence itself, and will not either weaken true, will strengthen and not weaken the argument of the
or strengthen the conclusion. judicial experts, because it is in such a situation, when there
(C) talks of indefinite periods such as ‘distant past’ and in a widespread controversy over an issue, that Supreme
‘recent past’ which may even mean centuries, and does not Court is expected to give an unambiguous opinion for the
mention the number of people affected by these disasters in guidance of the lower courts.
the last decade or in the immediate previous decades. So it III, if true, will lead to confusion and misunderstanding
also does not weaken the conclusion. among the lower courts and other government agencies, and
(E) is only relevant to the weather conditions, and cannot will only strengthen the argument of the judicial experts.
explain why more people have been affected by earthquakes Thus, (A) is the answer.
and volcanic eruptions during the last decade. It does not
therefore weaken the conclusion. 10. The police chief of the city attributes the fall in crime
If (D) is true, it would mean that certain areas of the earth rate in 1982, as compared to that in 1981, solely to the
have always been dangerous and inhospitable to people innovative police program put into effect by him in the
because of being prone to earthquakes, volcanic eruptions beginning of 1982.
165
American Education Aids - LSAT - Logical Reasoning - Analysis
In trying to locate the choice which seriously weakens this (that privatisation of services will improve their quality) on
conclusion, we should look for that statement which cites the basis of only one sample, which may not be representative.
another possible cause (unconnected with the innovative police (It is quite likely that there are other instances where the
program) for the reduction in the crime rate in the city. privatisation either did not improve the quality of the services,
(A) tantalizingly looks like the correct choice, but is wrong or may even have resulted in their deterioration.) So, (D) points
because it talks of increased expenditure on police programs out a logical flaw in the author’s argument, and is the answer.
in other cities, whereas the police chief of this city attributes (E) is wrong, because there is no word in the narrative
the reduction in crime rate of ‘the innovative police program’ which has been used with two different meanings in two
put into effect by him. If the programs implemented in the different places.
other cities had not been similar to the one implemented in
this city, their success or failure can neither strengthen nor 3. The ruling of the court is that it is legal for a company
weaken the argument of this city’s police chief. to reject a job applicant if ‘working on the job would entail a
Among the other choices, (E) states that the size of the 90 percent chance of the applicant suffering a heart attack’.
age-group most likely to commit crimes decreased considerably We have been asked to spot that choice which, if true, would
in the city in 1982 as against 1981, because of a declining render the ruling of the court ineffective in regulating
birth-rate. If this is true, this gives another possible employment practices.
explanation for the reduction in the crime rate in 1982 as (B) says that no legally accepted methods exist for
compared to that in 1981, and will thereby weaken the calculating the risk of a job applicant’s having a heart attack
argument of the police chief. So, (E) is the answer. as a result of being employed in a particular occupation. If
this is true, a company which wants to reject a job applicant
SET 4 for some other legally unacceptable reason may falsely claim
Answers that working on the job would entail a 90 percent chance of
the applicant suffering a heart attack, and the rejected
1.B 2.D 3.B 4.C 5.C 6.A 7.E 8.C 9.C 10.C
applicant would have no legal way of proving the fraudulent
Analysis
intention of the company.
1. From the fact that 86% of the 5000 teachers who
So, it is (B) which, if true, will render the ruling of the
responded to survey questionnaires expressed a high level of
court ineffective in regulating employment practices, and is
interest in microcomputers, the author comes to the
the answer.
conclusion that the popular notion that teachers are generally
You may be tempted to choose (E) as the answer. But (E)
apathetic (meaning indifferent) to microcomputer technology,
merely points out a practical difficulty that a company may
is false.
face in attracting applicants, but the passage deals with cases
We have been asked to locate the choice which would be
where there is an applicant for a job, but the company rejects
most damaging to this conclusion.
him on the basis that working on the job would entail a
(A) will not damage, but will strengthen the conclusion,
possibility of heart attack for him. So, (E) is not the answer.
because it would mean that there was overwhelming interest
for the microcomputers among all the teachers irrespective
4. On the basis of the assertion that only forty-five percent
of whether they had previous exposure to computers or not.
of the owners of jogging shoes jog more than once a year, and
(B) implies that the number of questionnaires actually sent
only seventeen percent of such owners jog more than once a
out was much larger than 5000, but only those teachers who
week, the author comes to the conclusion that “most
had previous exposure to computers chose to reply. If this is
consumers do not get much use out of the sports equipment
true, the conclusion reached by the author would be based on
that they purchase”.
minority opinion, and would not be valid. On the contrary, it
We have been asked to identify that choice which, if true,
will confirm the popular notion that teachers are generally
casts most doubt on this conclusion.
apathetic about microcomputer technology. So, (B) is the
If (A) is true, it only gives a plausible reason why a majority
answer.
of consumers who buy jogging shoes stop using them, but will
(C), (D) and (E) neither strengthen, nor weaken, the author’s
strengthen the conclusion of the author that most consumers
conclusion.
do not get much use out of the sports equipment purchased by
them.
2. (Turnpike means a highway for using which the motorists
If (B) is true, it means that even the figures of 45% (for
have to pay a toll.) On the basis of the statement that the
those who jog at least once a year) and 17% (for those who jog
turnpike system became a more reliable system after it was
at least once a week) are exaggerated figures. So, (B) would
taken over from the government by private organizations, the
strengthen, and not weaken, the conclusion of the author that
author argues that private ownership of services will improve
most consumers do not get much use out of the sports
the quality of those services.
equipment purchased by them.
We have been asked to spot that choice which will describe
If (C) is true, and many consumers in fact purchase jogging
a significant flaw in the argument of the author.
shoes for use in activities other than jogging, it implies that
(A) is wrong because there is no “appeal to a person of
the question whether they get much use of these shoes cannot
authority” in the given narrative.
be judged on the basis of the percentage of those among them
(B) is wrong, because the author mentions only his own
who actually jog. It is quite likely that more than 90% of the
view on the subject, and does not even refer to any “opposing
jogging shoe owners are actually using the shoes for the real
view”.
purpose for which each one of them had bought them. In this
(C) is factually incorrect, because the author considers the
case, the claim of the author would be wrong. So, it is (C)
handing over of services to private institutions not as a wrong
which casts most doubt on the claim of the author, and is the
action, but as a right action. So, (C) is wrong.
answer.
The author makes a generalized and conclusive statement
Obviously, (D) and (E) neither strengthen nor weaken the
166
American Education Aids - LSAT - Logical Reasoning - Analysis
claim of the author, and are both wrong. retail prices of cotton clothing should fall in the immediate
future. So, (E) is not the answer.
5. The given passage refers to the compilation of lists of
hospitals on the basis of their patient death rates (namely, 7. The engraving work has to be done by the artisan by
how many patients died in the hospital out of every 100 keeping the coins within a foot from the eye.
patients admitted). It also says that the data have been If it is proved that there is no difference between the
adjusted to allow for differences in the ages of patients. eyesight of near sighted individuals and those with ordinary
One may be tempted to draw the conclusion from this list vision if the objects viewed are less than a few feet away, the
that the quality of treatment in a hospital, where the death researchers’ argument that nearsightedness is an advantage
rate is higher, is inferior to the quality of treatment in another for work on coins will be weakened.
hospital where the death rate is lower. So, (E) is the answer.
The given question implies that four of the five given None of the other choices weakens the conclusion of the
choices are logical grounds for hospitals to object to such author.
interpretation, while one of them is not. We have been asked
to spot this particular choice. 8. The athletic director adds all the numbers; and comes
(A) will be a valid objection because, if the 10th ranking to the conclusion that his program serves 250 different
hospital has a death rate of 26 per hundred, and the 11th individuals. The one condition given is that no athlete is on
ranking hospital has a death rate of 27 per hundred, one two teams during any one season. But there is no stipulation
cannot come to the conclusion that the quality of health care that a single athlete cannot be on different teams in different
in the latter is worse than the quality of health care in the seasons - for example, a footballer in the first season being in
former (because the number of deaths could depend on the the swimming team in the second season. If this be the case,
relative seriousness of the condition in which the patients had then the program will serve less than 250 different individual
been admitted initially). athletes, because there will be overlapping of at least some
(B) will also be a valid objection, because a hospital which athletes. It is (B) which the athletic director fails to consider,
has a policy of compulsorily discharging incurable patients and is the answer.
(thereby avoiding their deaths within its premises) cannot be (A) is wrong, because it is prohibited by the given rule.
considered to be more efficient in health care provision than (C) is already implied in the numbers which add up to
another hospital which has a policy of allowing them to remain 250.
as inpatients till the end (thereby having a higher death rate). (D) is also implied in the given numbers which add up to
It is (C) which cannot be a valid objection, because the 250.
passage specifically states that the death rates had been (E) contradicts the penultimate sentence. But, even if an
adjusted to allow for differences in the ages of patients. So, athlete does not participate in all practice sessions, but
(C) is the answer. participates in only some of them, the fact remains that the
(D) and (E) are valid objections for the same reason as (B). program serves him also.

6. From the fact that wholesale price of raw cotton fell 9. The argument underlying the two given sentences is
considerably in the last year, the author argues that the retail that “only those who know computer programming can use
prices of cotton clothing at clothing stores are likely to fall computers effectively”.
soon. We have been asked to identify that choice which weakens
We have been asked to locate the choice which will seriously this argument.
weaken the argument above. We should therefore look for that choice which gives an
We should therefore look for that choice which points out example of persons who do not know computer programming
that the expense incurred in an intermediate stage (from the but are still able to use computers effectively.
time that the raw cotton is purchased at the wholesale level It is (C) which gives such an instance, and is the answer.
and the time that it is converted into cotton clothing and made
available for sale at the retail level) have increased in such a 10. On the plea that the number of crimes reported had
way as to offset last year’s fall in raw cotton prices. increased after the installation of the closed-circuit television
Among the choices, it is (A) which states this, and is the in the two subway stations, city officials had decided to remove
answer. the television, stating that the installation of the television
The reference to the price of raw wool in the given passage had led to an increase in the number of crimes.
is irrelevant, and has been given only to confuse you. The We have been asked to identify that choice which most
argument is limited to the prices of cotton clothing, without seriously weakens this claim of the city officials.
comparison with the corresponding prices of woolen clothing. A), even if it is true, makes no reference to the increase in
So, (B) is irrelevant to the issue, and is not the answer. the number of crimes reported after the installation of the
(C) would support (and not weaken) the argument that the closed-circuit television, and does not either strengthen or
retail price of cotton clothing would shortly fall because of weaken the claim of the city officials. So, (A) is not the answer.
the lower price of raw cotton last year, and is not the answer. (B), if true, only supports the claim of the city officials that
The argument is not that the retail price of cotton clothing the installation of the closed-circuit television had led to an
will fall immediately on the fall of the wholesale price of raw increase in crime, and does not weaken it. So, (B) is not the
cotton; it only says that ‘it will inevitably fall (in future)’. So, answer.
(D) supports and does not weaken the author’s argument, and (C) says that the percentage of all crimes that were
is not the answer. committed at the two subway stations and which were
Even if (E) is true, it only means that, in spite of such a reported rose because of increased instances of observations
rise in the cost of harvesting, the wholesale cotton price had of crime on the short-circuit televisions. If this is true, it means
fallen last year. (E) is irrelevant to the argument why the that the increase in the number of crimes reported was not
167
American Education Aids - LSAT - Logical Reasoning - Analysis
because of increase in the number of crimes committed, but were studied, of whom 501 pairs had been raised together
because of their being observed through the closed-circuit and 499 pairs had been raised separately. The results of this
television. If so, the experiment must be deemed to have been study cannot be considered to be invalid merely because the
a success and not a failure. In this case, the decision of the number of pairs who had been raised separately was
city officials to discontinue the experiment with the claim marginally less than 50%. So, (A) cannot be stated to cast the
that the installation of the closed-circuit television had led to most doubt on the results of the study, and is not the answer.
an increase in the number of crimes, can be clearly described By the very nature of the experiment, the twins who were
as a wrong decision. So, (C) is the answer. the subjects of the study must have been in a narrow age
range for the results to be valid. (For example, the results of
SET 5 the study will not be valid if one pair of twins are each aged
Answers 5, and another are each aged 75.) So, the fact that the ages of
all the twins studied fell within a 10-year range would validate
1.D 2.C 3.C 4.E 5.A 6.B 7.E 8.A 9.E 10.A
the results of the study, and would not cast doubts on them.
Analysis
So, (B) is not the answer.
1. The passage first says that early buyers of video recorders
(C) says that even the twins who were raised separately
lost interest in watching video movies after about six months.
grew up in similar families. This means that they had grown
It goes on to say that video movie sellers and hirers have
up in similar environments. So, if they had shown some
not so far been affected by this fact, because more and more
similar personality traits, these traits could have been
new people are still buying video recorders, and there has till
contributed by the similar environments in which both of them
now been a net expansion for the market for video movies.
had grown up, and may not be attributable solely to hereditary
The passage then predicts that, once all the people who
factors. So, it is (C) which casts the most doubt on the study’s
can afford to buy video recorders have purchased them, their
results, and is the answer.
interest in video movies is likely to be sustained for only about
What the study concluded was that 7 of the 11 traits
six months thereafter, and the market for video movies will
studied were hereditary. For this purpose, a comparison of
thereafter start shrinking, thereby adversely affecting the
the nature of these traits among themselves is irrelevant. So,
businesses of video movie sellers and hirers.
(D) does not cast any doubt on the study’s results, and is not
We have been asked to identify the argument among the
the answer.
choices which will weaken this prediction.
(E) merely states the usual mechanism of selection of the
The prediction that video movie business will shrink is
traits taken up for study, and does not cast doubt on the results
based on the behaviour of the early buyers of video recorders
of the study. So, (E) is not the answer.
who got tired of watching movies in them after six months.
The argument that will weaken this prediction is the one
3. The first sentence does not talk of the number of people
which differentiates between the behaviour of early buyers
who had been affected by the intestinal disease in that rural
and that of later buyers of the video recorders. (D) says that
county, but of those diagnosed as having been affected by it.
early buyers of a novel product are always people who are
The second sentence says that the health officials of the
quick to acquire novelties, but also often as quick to tire of
rural county claim that it is the improvement in the sanitary
them. If this is true, the early buyers of video recorders are
conditions at the water treatment plants in that county that
not representative of the general run of consumers, and the
has led to the reduction in the incidence of the disease.
behaviour of the latter cannot be predicted on the basis of the
We have been asked to identify the choice which would
behaviour of the former. So, it is (D) which weakens the
weaken this claim of the county’s health officials.
prediction of the author that businesses distributing videos
The claim of the health officials is not about increase in
face hard times, and is the answer.
the number of new water-treatment plants during the last
2. What the first sentence means is: if identical twins who
year, but about the improvement in the sanitary conditions
are raised separately show certain similar traits of personality,
in the existing water-treatment plants. So, (A), which talks
it can be surmised that these traits are hereditary, and are
of new water treatment plants built in the last five years,
not influenced by the environment in which they were raised.
neither strengthens nor weakens the claim of the county’s
If, on the other hand, some other traits of their personality
health officials.
are different, it can be surmised that those traits are not
Since one can contact an intestinal disease by consuming
hereditary, but are influenced by the environment in which
contaminated water even once, the total quantity of piped
they were raised.
water or spring water consumed in a year by a person is not
The second sentence then refers to an experimental study
relevant to the health officials’ claim. So, (B) neither
of 11 key traits of different pairs of identical twins, some of
strengthens or weakens their claim.
whom were raised together and the rest of whom were raised
(C) implies that many people who had been diagnosed as
separately, and says that it was found that 7 out of the 11
having the particular intestinal disease in the previous year
traits are primarily products of heredity, implying thereby
did not really suffer from that disease at all, but had intestinal
that these 7 traits were found to be the same even in pairs of
ulcers. This means that the claim that there has been a
twins who had been raised in different places.
reduction in the incidence of the intestinal disease is itself
We have been asked to identify the choice which would
not real, but is imaginary. If so, the claim of the health officials
cast the most doubt on the study’s results.
that there has been a reduction in the incidence of the disease
We should therefore look for that choice which states that
because of improvements in the sanitary conditions in the
the fact that identical twins who were raised separately
water treatment plants would be baseless. Thus, it is (C)
showed the some traits does not necessarily mean that these
which weakens the claim of the health officials, and is the
traits are hereditary, but can also be attributed to the
answer.
environments in which they grew up.
Obviously (D) and (E) are not relevant to the health officials’
(A) may refer to a situation in which totally 1000 pairs
claim, and are wrong.
168
American Education Aids - LSAT - Logical Reasoning - Analysis
4. The given narrative implies that free import of steel We have been asked to identify the choice which would
from foreign steel mills into USA is adversely affecting the weaken this argument.
interests of big American steel mills and, therefore, a proposal Obviously, the choice must be the one which says that the
to impose quotas on the import of foreign steel into USA is increasing consumption of cigarettes by Italian teenagers is
under consideration. in fact due to their continued exposure to cigarette
The narrative then argues that such imposition will not advertisements, in spite of the ban by the Italian government.
help the big American steel mills at all but, on the contrary, (B) says that most Italian teenagers watch television
these big mills are likely to be affected even more because channels which are broadcast from countries where cigarette
the “mini-mills” in USA will take even more business from advertisement is not banned. This means that they are even
big American mills than what has been taken by foreign steel now getting exposed to cigarette advertisements in spite of
mills till now. the ban by their own government. So, (B) is the answer.
We have been asked to identify that choice which would
cast the most serious doubt on the conclusion arrived at in 7. The given passage argues that major stockholders in
the narrative. corporations are interested only in short-term increases in
Since the given narrative does not make any specific their values, and therefore pressurize the management to
reference to the relative quality and price of the three neglect the corporation’s long-term growth.
categories of steel - produced by the big American steel mills, We have been asked to identify the choice which will
produced by the “mini-mills” in USA, and imported from weaken this argument.
abroad, (A) neither confirms nor casts doubt on the claim made Obviously, what will weaken the argument is a statement
in the given narrative, and is not the answer. that persons holding huge stocks in these corporations know
(B) talks of the relative qualities of the steel produced by that there can be no short-term growth in the value of their
foreign steel mills and the big American mills, and is therefore stocks unless there are prospects of long-term growth for the
irrelevant to the claim that the small American mills will corporation itself. If this were true, these stock holders would
take away business from the big American mills. not compel the management to neglect the long-term growth
(C) talks of American “goods” in general, and is therefore of the corporation even when their prime concern is short-
irrelevant to the claim that the small American steel mills term increase in stock value.
will take away business from the big American steel mills. (E) states just this, and is the answer.
So, (C) is not the answer.
(D), if true, would strengthen, and not cast doubt, on the 8. The given paragraph states that, because the bovine
claim made in the last sentence of the given narrative. growth hormone has the effect of increasing the milk yield of
(E) says that the domestic mini-mills and the big American cows by 10-40%, the FDA of US Government is proposing to
mills produce totally different types of steel. If this is true, license the commercial production of this hormone.
then the mini-mills will not be able to “take away” the We have been asked to identify the strongest criticism of
business of the big American mills if import quotas are this move of the FDA.
imposed. So, it is (E) which casts the most serious doubt on The aim of the FDA in the proposed licensing of commercial
the claim made in the last sentence of the passage, and is the production of the hormone is increase in milk production. The
answer. strongest criticism of this move will therefore be one which
points out that the commercial production of the hormone,
5. The prediction made in the passage is that, since tropical and administering it to a large number of cows would, in fact,
forests produce a large percentage of the Earth’s supply of result in a decrease in milk production, and not in its increase.
oxygen, the continued destruction of these forests is likely to It is (A) which points out that, while the milk-yield from
reduce the Earth’s oxygen supply to levels that will endanger cows may initially increase because of the administration of
the existence of all oxygen-dependent life on earth. the hormone, the cows are likely to develop a variety of diseases
We have been asked to identify the choice which would soon, and this may either lead to their mortality or reduction
most seriously weaken this argument. in their milk-yield. If this is true, the aim of the FDA is likely
If (A) is true, it implies that oxygen-dependent life-forms to be defeated by its own action.
living in parts of the world other than tropical forests are So, (A) is the answer.
even now not dependent on oxygen produced by tropical forests. None of the other choices gives a reason which will defeat
Therefore, the destruction of the tropical forests will have no FDA’s main aim of increasing milk yield.
adverse effect on them. The prediction of the author will
therefore be seriously weakened if (A) is true. So, (A) is the 9. From the mere fact that a larger number of newspapers
answer. are sold in Town S than in Town T, the author comes to the
(B) merely implies that destruction of tropical forests would conclusion that the citizens of Town S are better informed
deplete the oxygen content of the upper atmosphere, but this about major world events than are the citizens of town T.
will not affect the oxygen-dependent life on earth either way. We can understand from the question that follows that
(C) and (E) will strengthen, and not weaken, the prediction each of four of the five given choices weakens the above
of the author. conclusion, and only one choice either strengthens it or is
(D) is an information which merely confirms a part of the irrelevant to it.
author’s statement. We have been asked to identify this choice and select it as
the answer.
6. (‘Tenuous’ means ‘weak’) The author’s argument is that, If (A) is true, it means that the only reason that more
since cigarette consumption among teenagers in Italy, where newspapers are sold in Town S than in Town T is that the
cigarette advertising is prohibited, has been rising, it can be former has a larger population. So, (A) will weaken the
concluded that there is very little connection between cigarette conclusion that citizens of Town S are better informed on
advertisement and the increased consumption of cigarettes. world affairs than are the citizens of Town T.
169
American Education Aids - LSAT - Logical Reasoning - Analysis
If (B) is true, it implies that a sizable number of newspapers SET 6
sold in Town S are actually sold to and read by citizens of Answers
Town T. This will also weaken the conclusion that citizens of
1.B 2.B 3.C 4.B 5.E 6.E 7.E 8.D 9.C 10.C
Town S are better informed about world affairs than the
Analysis
citizens of Town T.
1. On the basis of the information that the new Environ-
(C) will also obviously weaken the conclusion that citizens
mental Party, which had won 2 seats out of 7 in the town
of Town S are better informed about world affairs than the
council in 1988, had lost both of them in 1992, the author
citizens of Town T.
argues that the decline in the party’s fortunes clearly demon-
If (D) is true, and a considerable part of the total circulation
strates that environmental concern faded in significance in
of newspapers in Town S consists of the circulation of this
the town during this period.
newspaper which covers only local events, it will also weaken
We have been asked to locate that choice which most seri-
the conclusion that citizens of Town S are better informed
ously weakens this argument.
about world events than those of Town T.
(A) merely says that both the number of voters and the
(E) does not dispute the fact that more copies of newspapers
number who voted increased between 1988 and 1992, but does
are sold in Town S than in Town T, and gives a possible reason
not give a reason which strengthens or weakens the author’s
why it is so. Therefore, if the number of newspapers sold in a
theory why the Environmental Action party lost its seats.
town can be taken as an index of the awareness of its citizens
(B) says that the town’s leading political party (which had
of major world affairs, (E) would strengthen, and not weaken
won the majority of the other 5 seats in 1988) had revised its
the conclusion drawn by the author. So, (E) is the answer.
platform adopting a strongly environmental stance. If this is
(Even if no such correlation is assumed, we can only say that
true, voters who are in favour of environmental action might
(E) neither confirms nor weakens the conclusion, and would
have voted for the leading political party itself this time in-
still be the answer.)
stead of for the Environmental Action party. So, the defeat of
the members of the EA party would not mean that the envi-
10. On the basis of the fact that there was a slowdown in
ronmental concerns had faded in significance in the minds of
the growth of industrial productivity in USA between 1968
the voters. On the contrary, the reason why the dominant
and 1978, in spite of the investment in industrial research
party adopted a strong environmental stance may itself be
by private industrial firms remaining steady as a percentage
because the public concern for environment had become more
of sales (after correcting for inflation), the author argues that
intense, and the dominant party wanted to go along with that
the growth of industrial productivity is not directly
popular sentiment. So, it is (B) which seriously weakens the
proportional to the amount invested in industrial research.
argument of the author, and is the answer.
We have been asked to locate that choice which weakens
(C) neither strengthens nor weakens the argument, and is
this argument.
not the answer.
The narrative says that investment in industrial research
(D) strengthens the argument of the author, and does not
by private industrial firms remained steady. But if, as stated
weaken it.
in (A), federal government funds, which constituted a
(E) would give a reason why the voters could have been
significant portion of the support for industrial research, fell
disillusioned with the Environmental Action party, because
annually and substantially during that period, then the total
of which environmental concerns could have faded in signifi-
amount invested in industrial research during that period
cance. So, (E) strengthens the argument of the author, and
(by private industries and the government together) would
does not weaken it.
have been less during that period than in the previous periods.
This would satisfactorily explain why there was a slowdown
2. The program described is in the nature of insurance
in industrial productivity during that period, consistent with
against future increases in college tuition fees.
the notion that the growth of industrial productivity is directly
It suggests that the parents participating in the program
proportional to the amount invested in industrial research.
should pay an amount equal to the present day tuition fee
So, (A) is the answer.
now itself, and the administrators of the program will then
(B) is immaterial to the argument, because the narrative
meet the actual enhanced tuition fees when the child grows
specifically says that the figure of the amount spent on
older and enrolls in any public college in the state.
industrial research by private industrial companies has been
We have been asked to identify the choice which is the
adjusted for inflation.
most appropriate reason for parents not to participate in the
The given argument refers to the total expenditure on
program.
industrial research, and does not differentiate between
Obviously, attraction of any such program would lie in the
expenditure on product development and on basic research.
probability that the rate of enhancement in tuition fees will
So, (C) is not relevant to the argument. (In fact, if the emphasis
be greater than the rate of interest that a person can earn by
had been on short term product development, the productivity
depositing an amount equal to the present day tuition fees in
increase must have accelerated, and not slowed down, during
appropriate savings bonds. If the rate of interest that can be
the period.)
earned is likely to be more than the rate of enhancement of
(D) will strengthen the argument, and not weaken it.
the tuition fees, it will be more prudent for a parent not to
The argument relates to the period 1968-78; what
participate in the program, but to invest the money in such
happened after 1978 is irrelevant to the issue. So, (E) is not
interest-bearing bonds. (B) says this, and is the answer.
the answer.
(A) need not deter a parent from participating in the pro-
gram, because the program will cover the fees in any of the
public colleges in the state.
(C) is an argument in favour of, and not against, joining
the scheme.
170
American Education Aids - LSAT - Logical Reasoning - Analysis
(D) is also an argument in favour of joining the scheme. aged men who drank more than two cups of coffee a day also
The scheme covers only the tuition fee in any public col- ate more foods which are high in cholesterol, then the high
lege. So, if the student is to stay in a hostel, the parent has to level of cholesterol in their blood would have been caused by
bear the cost of room and board, irrespective of whether he these foods and not by coffee. If true, it is (E) that weakens the
joins the scheme or not. So, (E) is a consideration neither in conclusion that coffee increases the risk of heart disease, and
favour nor against joining the scheme. is the answer.

3. Ecological balance is best maintained by ensuring that 6. What is being doubted by the author is the hypothesis
existing species of life in their present environment are not that the daily biological rhythm in the functions of many
threatened with extinction. plants and animals is stimulated by the alteration in the in-
(C), if it is true, would mean that, if people ate abalone tensity of incident light.
instead of tuna, all abalones will be destroyed sooner or later. We have been asked to identify that choice which is in the
Thus, if (C) is true, the author’s suggestion would defeat the nature of an evidence that contradicts this hypothesis.
very purpose for which it is made, namely, preservation of eco- We should therefore look for that choice which describes
logical balance. an experiment in which the daily biological rhythms in func-
The other statements are not directly relevant to ecologi- tions was noticed even while the intensity of light was kept
cal balance, and are not correct. constant.
Photosynthesis is normally noticed in plants, including
4. The conclusion drawn by the author is that, since it is algae, during daylight hours. (E) describes an experiment in
the insurance companies which reimburse the expenditure which some algae were exposed to constant light intensity
incurred by health-care providers, such providers are under both during the day and the night, and it was found that they
the control of the insurance companies, and insurance compa- displayed rates of photosynthesis which was much greater
nies can therefore decide who shall take to the profession of during daylight than at night (even though they were ex-
providing health-care. This is the meaning of the last sen- posed to the same intensity of light at night also), thereby
tence which says, “it is the insurers who decide who is a healer conforming to their natural biological rhythm. This experi-
in modern society’. ment weakens the hypothesis that the daily biological rhythms
(A) implies that all expensive health services are covered are stimulated by the alterations in the intensity of incident
by health insurance. This will strengthen and not weaken the light. So, (E) is the answer.
argument of the author that it is the insurance companies (A) would strengthen the theory that biological rhythms is
which decide who is the healer in modern society. So, (A) is not controlled by intensity of light because there is much more
the answer. light during the afternoon than just before dawn.
(B) points out that it is the state agencies in charge of regu- The hypothesis does not say that, in respect of all animals
lation of health-care who license health providers, and health and plants, biological activity is directly proportional to the
insurance companies are bound to reimburse such providers incident light. In fact, for the class of nocturnal animals, bio-
for the services rendered by them to the insured persons. This logical activity is greater during night than during day, but
means that it is the state agencies, and not the insurance com- this also conforms to the hypothesis that their biological ac-
panies, who decide who are the healers in modern society. So, tivity is controlled by the incident light. So, (B) does not weaken
it is (B) which weakens the argument of the author, and is the the hypothesis.
answer. (C, in fact, conforms to the hypothesis and does not repudi-
(C) talks of a situation in which the insurance companies ate it.
are not bound to reimburse a health-care expenditure, and is (D) does not talk of incident light at all, and is irrelevant
therefore not relevant to the author’s argument. So, (C) is not to the hypothesis.
the answer.
(D) and (E) are also not relevant to the author’s argument, 7. The only reason advanced by the author to support his
and can be discarded. statement that the deregulation of airlines has harmed the
economy of the United States is that it has resulted in a cut
5. The passage says that, though coffee does not itself con- by major airline companies of 3000 of their employees.
tain cholesterol, middle-aged men who drink more than two We have been asked to identify the choice which will
cups of coffee a day have been found to be more prone to cho- weaken this conclusion.
lesterol-related heart disease than are other middle-aged men. We should therefore look for that choice which implies that
We have been asked to identify the choice which will the same deregulation of the airlines has contributed to the
weaken the drawing of the logical conclusion from the above creation of more than 3000 jobs in some other sector in the
finding that coffee is responsible for heart attack in sedentary United States.
middle-aged men. (E) says that smaller carriers of passengers have thrived
(A) does not mention the drinking of coffee at all, and nei- as a result of deregulation, and they now provide more new
ther weakens nor strengthens the given conclusion. jobs than the major airlines have eliminated since 1978. This,
(B) strengthens, and does not weaken, the conclusion that if true, weakens the argument of the author that the deregu-
coffee increases the risk of heart attack, and is not the an- lation of airlines has harmed the economy of the country. So,
swer. (E) is the answer.
(C) also does not weaken the conclusion that coffee increases (B) looks like a possible answer, but is wrong, because the
the risk of heart attack, and is not the answer. fact that fewer number of passengers now travel on commer-
(D) neither strengthens nor weakens the given conclusion, cial airlines may itself be a direct consequence of the deregu-
and is not the answer. lation of the airlines, and this will strengthen, and not weaken,
The last sentence of the passage says that coffee by itself the given argument.
does not contain cholesterol. If, as stated in (E), the middle- (B) is wrong because, it only says that several of the major
171
American Education Aids - LSAT - Logical Reasoning - Analysis
airlines in US have higher levels of employment today than agency. So, (A) states the obvious, and is not relevant in evalu-
they had before the deregulation, but it does not refute the ating the effect of the delay in reimbursement by 40 days. So,
statement that, when all major airlines are considered to- (A) is not the answer.
gether, there has been a net elimination of 3000 jobs. (B) means that the agency is not in a position to save on
its administrative costs. This, if true, will justify, and not
8. (Metabolism is the process by which chemical reactions weaken, the agency’s plan to make savings through other
in plants and animals create the energy needed by them to means such as a deliberate delay in making reimbursements
live. Palate is the roof of the mouth.) The author’s assertion is and earning interest thereby. So, (B) is not the answer.
that breathing while eating is necessary to maintain the high (C) means that these clients will be financially harmed by
rate of metabolism of mammals, and that this facility is pro- the planned delay by the government agency in reimbursing
vided by the secondary palate. their claims, thereby contradicting its claim that the plan
We have been asked to identify the choice which will most would not harm its clients financially. So, (C) would be the
weaken this assertion. best criticism of the agency’s plan, and is the answer.
The author’s assertion relates to mammals with high rates (D) only means that, in respect of such delayed claims, the
of metabolism. So, (B), (C) and (E), all of which relate to agency has already been benefiting by interest-earning. There-
animals with a low rate of metabolism, can straightaway be fore (D), by itself, does not constitute a criticism of the agen-
ruled out. cy’s present plan to delay reimbursement of claims by an-
(A) will only conform to the general property of mammals other forty days.
referred to by the author, and will strengthen, and not weaken The given passage does not claim that the interest of $180
his assertion. million earned by the deliberate delay in the reimbursement
(D) would prove that the lack of a secondary palate and of claims was intended to cover the budget cut fully. So, (E)
the consequential incapability to breathe while eating are even if it is true, is not a valid criticism of the agency’s plan.
not necessary for a high rate of metabolism in all mammals,
and will therefore weaken the author’s assertion. So, (D) is SET 7
the answer. Answers
1.D 2.C 3.D 4.E 5.A 6.A 7.D 8.C 9.A 10.B
9. From a study of the average ages of CEOs in ‘a large
Analysis
sample of companies’ a general conclusion has been arrived
1. What W says is that the rapid rise in the rate of unem-
at, which is then presumed to be applicable to all companies.
ployment will cost the ruling party a lot of votes in the im-
We have been asked to identify that choice which would
pending election.
cast the most doubt on this conclusion.
What W means is that, if the rate of unemployment when
We should therefore look for that statement which implies
the present ruling party came to power was 5%, and it re-
that the sample companies studied did not represent the
mains at 5% even now, the ruling party may not suffer. But
majority of companies in some important respect, and that it
the rate of unemployment has risen to a much higher figure
is therefore unscientific to arrive at a general conclusion based
in the course of the last 4 years when the ruling party has
on this unrepresentative sample. (A ‘large sample’ may still
been in power, this fact is likely to affect its fortunes adversely
constitute a minority of the total number of companies.)
in the next election.
The phrase ‘in those same companies 20 years ago’ in the
Z counters this argument stating that statistics of the last
third line of the passage implies that the companies taken
40 years show that there is no significant relationship be-
up for the sample study were limited to those which have been
tween the prevailing level of unemployment and the political
operating for more than 20 years. If a vast majority of existing
fortunes of the ruling party.
companies are, in fact, less than 20 years old, the phenom-
What he means is that, according to his statistics, a ruling
enon prevailing in the older companies cannot be inferred as
party has been returned to power even while the prevailing
being applicable generally to all companies. It is quite likely
unemployment level was high. His statistics would cover even
that the average age of the CEO’s of newer companies is much
those cases in which the unemployment level was high when
less than 57.
the ruling party was elected to power and continued to re-
It is (C) which implies this, and is the answer.
main at the same high level during its tenure in office, but
this is not a situation that W was talking about.
10. With a view to offset the budgetary cut that it had
(D) correctly points out that the weakness in Z’s argument
faced, the government agency plans to earn an interest of
is that he focuses on the level of unemployment, rather than
$180 million a year through a deliberate delay in the reim-
the direction of change in that level, on which W's statement
bursement of medical expenses to its clients by forty days. It
is based. So, (D) is the answer.
claims that, by implementing this plan, it would neither cut
reimbursements nor otherwise harm its clients financially.
2. The Corporate Officer first says that the previous year
We have been asked to identify that choice which can be
had been an “unusually poor one” for the chemical division
described as the best criticism of the agency’s plan.
which traditionally contributed about 60% of the corporation’s
Since the agency’s claim is that its plan will neither result
profits. Then, he says that the fact that, in the same year, the
in cutting reimbursements nor otherwise harming its clients
pharmaceutical division had contributed 45% to the corpora-
financially, we should look for that choice which implies ei-
tion’s profits, up from 20% of the previous year, was evidence
ther that the reimbursement is in fact reduced, or that the
that the pharmaceutical division was growing stronger.
clients are otherwise financially harmed by this plan.
We have been asked to identify that choice which would
The first sentence of the passage specifically states that
form the best critique of the “evidence” cited by the corporate
the schemes of reimbursement means that the clients have
officer.
to first pay the bills to the hospitals or the physicians, and
We should therefore look for that choice which implies that
only later can claim reimbursement from the government
172
American Education Aids - LSAT - Logical Reasoning - Analysis
the increase in the percentage of the profits of the corpora- as pollen or household dust, and that it is this unnecessary
tion attributable to the pharmaceutical division does not nec- activation that gives rise to asthmatic attacks.
essarily mean that the pharmaceutical division has grown It then talks of a possible development of a medication
stronger in the last two years. that will block the receipt of any messages sent by the messen-
(A) merely gives a possible reason why the pharmaceuti- ger molecules.
cal division contributed a higher percentage of profits of the We have been asked to identify the choice which, if true,
corporation in the previous year. If the new product intro- will constitute the most serious flaw in the plan to develop
duced is an important one, it is quite likely that its sales such a medication.
would increase further in the future years. In that case, the Since the problem is not how the messenger molecules are
statement of the Corporate Officer that the pharmaceutical produced, but how they act after they are produced, (A) by
division is growing stronger would be strengthened, and not itself does not constitute a flaw in the proposed plan. Prob-
weakened, by (A). So, (A) cannot be considered to be a criti- ably, in the course of further research, the researchers will be
cism of the evidence cited by him, and is not the answer. able to even close the gap in knowledge that is mentioned in
Even if (B) is true, it only means that the pharmaceutical (A). So, (A) is not the answer.
division of this corporation, which had been much weaker (B) also talks of the limitation in the present knowledge of
than the pharmaceutical divisions of other corporations, had the researchers. Probably, in the course of further research
grown stronger during the previous year, thereby approach- during the development of the proposed medication, this gap
ing the norm in the other corporations. So, (B) is not a criti- would be filled. So, (B) by itself does not constitute a flaw in
cism of the evidence presented by the corporate officer, and is the proposed plan.
not the answer. If (C) is true, it only emphasizes the need for starting the
(C) says that the percentage of the corporation’s profits research without any further delay, and does not constitute a
attributable to the pharmaceutical division could have in- flaw in the proposal.
creased even if that division’s performance had not improved, If (D) is true, and if the proposed medication will be un-
If this is true, the statement that the increase in the percent able to distinguish between messages triggered by pollen and
of contribution by this division from 25% to 40% to the over- household dust and messages triggered by noxious air, and if
all profits of the corporation is evidence of the growth in it suppresses both of them, it means that the proposed medi-
strength of this division, would be wrong. (Consider the situ- cation will totally nullify the very purpose of the existence of
ation in which the previous year’s overall profit of the corpo- the messenger molecules. This is because a person to whom
ration was $100,000, of which the chemical division contrib- the medication is administered will inhale even the poison-
uted 60% ($60,000) and the pharmaceutical division contrib- ous air without any hindrance, and will suffer adverse conse-
uted 25% ($25,000). If, last year, which has been an “unusu- quences that may even prove fatal to the person. This will be
ally poor year” for the chemical division, the chemical divi- a much more serious consequence than occasional asthmatic
sion had not made any profit at all, its contribution to the attacks that are caused by the unnecessary activation of the
overall profits of the company would have been nil. So, the messenger molecules. So, it is (D) which, if true, constitutes
overall profits of the corporation could have come down to a serious flaw in the proposed plan, and is the answer.
$40,000. The 45% contribution of the pharmaceutical divi- Since prevention is better than cure, (E) is not an argu-
sion to this figure would be $18,000, which is even less its ment against the development of the proposed medication,
profit during the previous year. This means that the pharma- but supports the proposal. So, (E) is not the answer.
ceutical division has not grown stronger as inferred by the
Corporate Officer, but has, in fact, grown weaker.) So, it is (C) 4. The first sentence states that Ms. Friedman noticed that
which constitutes the best criticism of the evidence and the the average productivity of day-shift workers fell at about 3
inference presented in the passage, and is the answer. p.m. The next sentence says that, she hypothesized that, since
What the Corporate Officer claims is that the pharmaceu- this is the time that many children leave school, the reason
tical division had grown stronger between the last year and for this fall in productivity was that employees’ minds turned
the year previous to it. For evaluating this claim, it is not to their children at this time instead of focussing on their jobs.
necessary to determine whether there had been an improve- We have been asked to spot that choice which casts the
ment between the year previous to the last year and the year most doubt on the truth of this hypothesis.
even previous to that. So, (D) does not constitute a criticism of If (A) is true, it would strengthen and not weaken Ms. Freed-
the Corporate Officer’s claim, and is not the answer. man’s hypothesis, because the workers could have been wor-
(E) is factually wrong, because the passage does give infor- rying about the safety of their children who are alone at home,
mation about the percent of total profits attributable to the thereby losing concentration on their work. So, (A) is not the
pharmaceutical this year as 45%. There is no need to know answer.
the actual percent of profit attributable to the chemical divi- (B) would also strengthen, and not weaken Ms. Friedman’s
sion to evaluate the claim that the pharmaceutical division hypothesis, and is not the answer.
was becoming stronger. So, (E) is not a valid criticism of the Even if (C) is true, the employees could be worried about
evidence or the inference of the Corporate Officer, and is not the safety of their children from the time they leave school
the answer. and reach the after-school centers. So, (C) by itself does not
weaken Ms. Friedman’s hypothesis, and is not the answer.
3. The given narrative says that the purpose of the mes- (D) talks of some difference in average daily productivity
senger molecules is to fight the damage to the lungs from nox- between employees with one child and employees with sev-
ious (meaning poisonous) air by telling the muscle cells (en- eral children. This choice neither strengthens nor weakens Ms.
circling the lungs’ airways) to contract, thereby sealing off Friedman’s hypothesis about the reason for the average fall
the lungs partially. in productivity of all employees at about 3 p.m. So, (D) is not
It adds that these messenger molecules are also sometimes the answer.
activated unnecessarily, in response to harmless things such (E) implies that the average fall in productivity after 3
173
American Education Aids - LSAT - Logical Reasoning - Analysis
p.m is observed in all workers irrespective of whether they 8. It can be easily seen that the generalization is true only
have children or not. If this is true, it most certainly weakens if the birth years had ended in ’12, ‘23’, ’34, ’45, ....etc, in which
the hypothesis that the fall in productivity is mainly due to the last digit is one greater than the second last digit. So, (C)
the workers worrying about their children returning home is the answer.
alone. So, (E) is the answer. You can easily verify that none of the other choices is ar-
ithmetically correct.
5. The author of the statement makes the point that the
death rate of persons who are ten percent underweight is al- 9. The author’s conclusion is that, since large companies
most twice what it is for persons of normal weight. On this will have to incur more expenditure in altering their equip-
basis, he derives the conclusion that being underweight is ment, governmental safety regulations create more serious
‘dangerous to health’, because it can lead to death. hardships on them than on smaller companies.
If it is known that many of the underweight persons (who This will not be true if the real situation is that big compa-
subsequently died) had lost weight as a result of illness, it is nies have large funds at their disposal that are sufficient to
possible to conclude that what really caused their death was meet the expenditure, while smaller companies do not have
the illness which they were suffering from. In that case, their even the smaller cash resources which are needed to comply
being underweight would only have been an intermediate re- with government requirements.
sult of the illness which led to their ultimate death. If this is It is (A) which states this, and is the answer.
true, it follows that persons who are naturally underweight
because of genetic reasons, but are otherwise active and healthy, 10. The argument that we have to examine is: “The (new)
need not be afraid of the findings mentioned in the given pas- regulations (stipulating that manufacturers should draw up
sage. detailed plans for reducing the pollutants by at least 50%
So, it is (A) that makes the danger of being underweight over a period of five years) will not result in significant pollu-
look less serious. tion reduction since the regulations do not force the manu-
The phrase ‘supported by the statistics above’ in the ques- facturers to implement the plan”.
tion implies that the author is not doubting the validity of (A) will strengthen the argument that there will not be a
the statistics themselves. So, (B), which casts doubts on the significant reduction in pollution, and will not weaken it.
definition of ‘underweight’ and ‘overweight’, and therefore on (B) implies that the stipulated preparation of detailed plans
the validity of the quoted statistics, is not the answer. for reduction of pollutants will reveal to the industries that
Both (C) and (D) would, in fact, support the conclusion in they can simultaneously reduce the costs for materials, waste
the passage and do not cast doubt on it, and are not the cor- disposal and legal services. If this is true, industries are likely
rect choices. to implement the plans in their own interest even when the
(E) would imply that the words ‘underweight’ and ‘over- regulations do not force them to do so. This will weaken the
weight’ were also defined equally narrowly (or more precisely), argument of the author, and is the answer.
and would support the conclusion and not cast doubt on it. (C) refers to only direct pollutants, and not the indirect
ones. In the case of industries where the pollutants are re-
6. This question raises the issue of which is the cause and leased into the environment indirectly, the argument of the
which is the effect between two observed phenomena. author will be strengthened, and not weakened.
According to the given statement, researchers found that (D) and (E) do not weaken the given argument.
those using the drug amphetamine became schizophrenic (a
form of madness), while those using drugs known as depres- SET 8
sants became depressed. The researchers therefore concluded Answers
that amphetamine causes schizophrenia, while depressants
1.B 2.E 3.D 4.E 5.C 6.D 7.A 8.C 9.A 10.B
cause depression.
Analysis
But, if it is proved that, it is those who are already prone to
1. The point made by the author is that, since nuclear phys-
schizophrenia who always choose amphetamine as their fa-
ics has made it possible to make copper from other metals,
vourite drug in preference to other drugs, and it is those who
copper available for industrial use need not be considered to
are already prone to depression who always choose depres-
be limited by the quantity of copper deposits, known or un-
sants as their favourite drugs in preference to other drugs,
known.
then it follows that the choice a particular drug is a result of
We have been asked to locate the choice which will form
a particular psychiatric illness, and is not the cause of that
the strongest argument against the author’s conclusion.
illness. In this case, the researchers’ conclusion will be con-
(A) is not an argument against the author’s conclusion,
siderably weakened.
because it does not refute the possibility of copper made from
Among the given choices, it is (A) which states exactly such
other industrial metals after fifty years.
a situation, and is the answer.
(B) says that the cost of copper produced from other met-
als will be prohibitively expensive. Since the cost of the input
7. The statement ‘only a Japanese woman wears Kimono’
is an important consideration in any industrial venture, com-
does not mean that ‘a woman who does not wear Kimono is
panies using copper today for their manufacturers will not
not a Japanese’.
be able to afford the proposed substitute. So, if (B) is true,
Similarly, the statement that ‘only a member of a Region-
copper available for industrial use will continue to be limited
alist party will oppose the bill’ does not mean that ‘a person
by the quantity of copper deposits directly contradicting the
who supports the bill cannot belong to a Regionalist party’,
author’s conclusion. So, (B) is the answer.
because the first sentence does not rule out the possibility
You can easily see that none of the other three choices con-
that some members of a Regionalist party may be in favour of
tradicts the author’s conclusion.
the bill. It is (D) which points this out, and is the answer.

174
American Education Aids - LSAT - Logical Reasoning - Analysis
2. Normally, children are expected to live with both the come best sellers.
natural parents. This means that these periodicals do publish reviews of
But, from the fact that the ratio of divorces to marriages likely bestsellers also, though such reviews may be short.
has increased since 1940, the author comes to the conclusion So, the conclusion in the last sentence that buyers of
that the proportion of children living with only one natural bestsellers do not get any guidance whatever from these peri-
parent must also have increased. We have been asked to lo- odicals is unwarranted. It is (E) which says this, and is the
cate the weakness in this argument. answer.
What the author has assumed is that the contingency of (A), (B), (C) and (D) do not even refer to the statement in
children having to live with only one natural parent arises the first sentence from which the conclusion in the last sen-
out of only divorce between the parents. tence is drawn, and are therefore wrong.
We should therefore look for that choice which cites an-
other possible reason for children having to live with only one 5. The argument given in the passage is that, because it is
natural parent, and any developments which might have af- cheaper and faster than any currently existing model, Re-
fected this other reason during the last 50 years. gent microcomputer will quickly establish itself as the fast-
Another obvious reason why a child is forced to live with est-selling computer in the market. We have been asked to
only one natural parent is the death of the other parent. (E) locate the choice which will LEAST WEAKEN the above ar-
states that, during the last 50 years, the proportion of chil- gument.
dren who must be raised by only one parent because of the (A) will greatly weaken the argument, because, if the re-
death of the other parent has decreased as a result of medical tailers refuse to stock the Regent computer, it cannot estab-
advances. If this is true, such decrease might have either lish itself in the market as the best selling model.
offset, or even exceeded, the number of children who have to (B) will also result in the Regent computer being pushed
live with only one natural parent because of divorce between out of the market, and will therefore weaken the argument of
them. So, it is (E) which will weaken the inference drawn by the author greatly.
the author. (D) implies that the potential market for Regent is already
(A) tantalizingly looks like the answer, but is wrong be- saturated, which will again weaken the argument in the pas-
cause, even if less number of women got married during this sage greatly.
period, the proportion of greater divorces among them would (E) means that the Regent computer is not perceived by
still result in a greater proportion of children having to live the prospective buyers as being superior in speed to the other
with only one natural parent. computers. If so, there is no possibility of the Regent comput-
ers becoming the best sellers in the market. So, (E) also con-
3. The example cited by the author is that we normally siderably weakens the given argument.
say that a person who has N hairs on his head, and another The passage first says that the new Regent computer is of
person who has (N–1) hairs on his head both have hair, be- high quality, is fast, and costs less than any currently existing
cause one hair less does not make any difference. If we kept competing model, and then predicts that it will quickly es-
on comparing such pairs of persons each with just one hair tablish itself as a fast-selling, low-priced alternative to cur-
less than the previous person (say, one with (N – 2) hairs and rently available microcomputers. (C) says that this computer
another with (N – 1) hairs), we should continue to say that can be used in conjunction with higher-priced microcomput-
each of them has hairs. ers manufactured by other companies. Therefore, anyone who
But, starting from the other end, a person with no hair at wishes to buy a second computer is likely to choose Regent
all on his head, and another person with just one hair, are computer because of its high quality and low price, because
both called bald. Similarly, a person with one hair on his he can easily exchange files between his older computer and
head and another with two hairs on his head are also both the new, less-expensive one. So, even if Regent computer does
called bald, because one hair more also does not make any not establish itself as a low-priced alternative to currently
difference. If we kept on comparing such pairs of persons, available microcomputers, it has good chances of establish-
each with just one hair more than the previous person, we ing itself as a low-priced addition to the existing computers.
should continue to say that each of them is bald. So, at some So, it is (C) which least weakens the given argument, and is
point of time, the same person will be called both ‘with hair’ the answer.
and ‘bald’, because there is no precise value of N for which we
can say that a man is either having hair, or is bald. 6. The argument of the author is that, because both union-
The author therefore argues that words are meaningless, ized and non-unionized women workers earn the same wages
and there is really no distinction between the seemingly in those industries in which only a part of the workers are
antonymous phrases ‘having hair’ and ‘being bald’. unionized, unionization does not raise women’s wages.
The example cited by the author is one in which there is We have been asked to identify that choice which most
lack of precise definitions for the phrase ‘having hair’ and the seriously weakens this argument.
word ‘bald’. This example does not prove that there is no Note that (A) does not say that unions succeed in getting
meaning at all to the word ‘bald’ or to the phrase ‘has hair’. benefits such as medical insurance of its members, but only
The argument of the author is wrong because he confuses says that unions bargain for such benefits. So, (A) by itself
absence of precision with total lack of meaning. It is (D) which does not constitute a statement that can be said to most seri-
points this out, and is the answer. ously weaken the argument that unionization does not result
None of the other choices counters the argument of the in wage increases for women. So, (A) is not the answer.
author. The given argument is based on wages earned by compa-
rable employees in the same type of industries, of whom some
4. It can be inferred from the first sentence that respect- are unionized and some are not. But (B) is a statement relat-
able periodicals give less space for the review of likely ing to one class of employees, namely executives, none of whom
bestsellers than they do for books which are not likely to be- are unionized. So, (B) neither strengthens nor weakens the
175
American Education Aids - LSAT - Logical Reasoning - Analysis
argument that unionization does not raise women’s wages, ucts from these cows.
and is not the answer. We have been asked to spot that choice which most strongly
The phrase “their non-unionized counterparts in these in- suggests that the mining company’s plans will fail.
dustries” in the second sentence of the given passage implies (A) merely implies that the profit made by the mining com-
that the author is comparing unionized and non-unionized pany by selling the dairy products may not be much, but does
workers in the same geographical areas. So, (C), which com- not mean that the company’s plan will totally fail. This is
pares the wages in industries in different geographical areas, because the company is now getting no revenue at all from
is not relevant to the author’s argument, and is not the an- these lands, and even a little revenue from them will be wel-
swer. come to it. So, (A) is not the answer.
(D) says that whenever unionized women workers win a (B) is irrelevant to the plan of the company, because the
wage increase in an industry through union activities, such company wishes to grow only that variety of Agrostis tenuis
increases are made applicable to non-unionized workers also that grows in contaminated areas. So, (B) is not the answer.
in that industry. This implies that unionization of even part (C) says that dairy products from cows that eat Agrostis
of the women workers in an industry results in an increase in grown in contaminated areas cannot be sold at all because
the wages of all women workers in that industry including they will contain unacceptably high levels of heavy metals. If
those who are not unionized. If this is true, the parity in wages this is true, the mining company’s plan of selling dairy prod-
between the unionized and the non-unionized women work- ucts from these cows will most certainly fail. So, (C) is the
ers that the author relies upon to buttress his argument is, answer.
in fact, at a higher level than what it would have been if (D) talks of the growth of Agrostis in ‘normal’ soil, and is
there had been no partial unionization of women workers. not relevant to the plan of the mining company to grow it in
So, (D) is the answer. contaminated soils. So, (D) is not the answer.
(E) talks about spread of unionization, and does not say (E) is also irrelevant to the plan of the mining company to
anything about increase in wages. It neither strengthens nor sell dairy products from cows grazing on Agrostis grown in
weakens the argument that unionization does not result in the soil contaminated by heavy metals. So, (E) is not the
increase in women’s wages, and is not the answer. answer.

7. The given passage says that, through the method of “his- 9. The first sentence says that people who marry have a
torical costing”, the government allows military contractors longer average life span than do people who never marry.
(meaning those who supply standard weapons to it) to protect From this information, the second sentence advises the
their profits by allowing a percentage increase in their prices reader that if he/she gets married, he/she can live longer.
based on the current rate of inflation. We have been asked to spot the choice which, if true, would
We have been asked to identify that choice which would cast the most serious doubt on the validity of that advice.
form the best basis for a criticism that “historical costing” is (A) says that the group of “people who never marry”, re-
an economically unsound pricing method for military con- ferred to in the first sentence (whose average life span is less
tracts. than the average life span of those who marry), includes also
The phrase “past inefficient use of funds” in (A) implies those who had died before they were old enough to get mar-
that the price agreed to be paid last year was higher than ried. In other words, this group also included babies, infants,
what was warranted, and had allowed the contractors to make children and teenage boys and girls. If the age of each one of
undue profits. If this is true, protecting their profits by con- these had been included while working out the “average life-
ceding even higher prices this year (in order to compensate span of people who never marry”, it would obviously be less
them for the inflation) would result in the continuance of the than the “average life-span of people who marry”, because
inefficient use of funds. So, (A) itself would form a basis for each of the latter would have been older than 18 or 19. Arriv-
the criticism that historical costing is not an economically ing at a conclusion, based on this information, that an adult
sound pricing method, and is the answer. who never marries will live for a shorter period than an adult
(B) merely implies that the prices paid to the contractors who marries would therefore be absurd. So, (A), if true, would
would have varied considerably during the last twenty years. seriously weaken the validity of the advice tendered in the
This does not constitute a criticism of the historical costing second sentence, and is the answer.
method. The given narrative does not distinguish between the life-
(C) forms a justification, and not a basis for the criticism of spans of men and women. So, (B) is irrelevant to the advice in
the historical costing method, and is not the answer. the second sentence, and neither strengthens nor weakens it.
Whatever is the money spent on military contracts, a pro- Again, the given narrative does not distinguish between
cedure has to be adopted for pricing the weapons. (D) neither the life-spans of those who marry only once and of those who
supports nor criticizes the historical costing method for such marry more than once. So (C) is also irrelevant to the advice
contracts, and is not the answer. in the second sentence, and neither weakens nor strengthens
The given passage is about “standard weapons”, and not it.
about “innovative” weapons. So, (E) is irrelevant to what is For a similar reason, (D) is also irrelevant to the advice in
stated in the passage, and is not the answer. the second sentence, and neither weakens nor strengthens it.

8. The first three sentences of the narrative give the infor- 10. (Surge means a sudden increase; lull means stagna-
mation that a particular variety of Agrostis tenuis, a wild tion.) In the first sentence, the author says that the increase
grass, is capable of growing even in areas covered by mine in consumer spending has benefited also the service indus-
waste where the soil is contaminated by heavy metals. tries in USA, meaning that people have been spending more
The last sentence says that a mining company has there- money not only on goods but also on services. As proof of his
fore decided to plant this variety on contaminated areas and statement, he cites the fact that, after a lengthy lull, pay-
turn them into cow pastures, planning to sell the dairy prod- ments to medical doctors have increased significantly in the
176
American Education Aids - LSAT - Logical Reasoning - Analysis
past year, implying that the medical doctors have benefited ing the required funds to meet its cost. So, (B) does not consti-
thereby. tute a reason why the program is likely to fail to achieve its
We have been asked to spot that choice which would most aim.
weaken the assertion that medical doctors have drawn such The phrase “as needed” in the second sentence of the nar-
a benefit from the surge in consumer spending. rative implies that the different varieties of soybean seeds
(A) would only strengthen the author’s assertion that the will be replanted before each of them becomes infertile. So,
end of the period of economic recession (as evidenced by the even if (C) is true, the program plans to take care of it by
increase in consumer spending) has enabled consumers of replanting the short-life varieties more often than the long life
medical care to go for the treatment of minor medical prob- varieties.
lems which they had postponed earlier. If this is true, then The narrative states that all the different varieties of seeds
the surge in consumer spending would indeed have benefited will be replanted at the storage center. (D) states that re-
the medical doctors. So, (A) does not weaken the author’s as- planting the seeds in a single environment can eventually
sertion, but strengthens it, and is not the answer. lead to the loss of genetic traits. So, if (D) is true, the very
(B) states that the increases in the cost of malpractice in- purpose of the program (namely, the conservation of the ge-
surance (payable by the doctors to the insurance companies) netic diversity of soybean seeds) would be defeated. So, it is
have required the doctors to increase their fees accordingly. If (D) which most strongly indicates that the program will not
this is true, it means that the increase in payments to medi- achieve its intended result. So, (D) is the answer.
cal doctors last year by the consumers had been passed on by (E) talks of programs to conserve the genetic diversity of
the latter to the insurance companies which had raised their seeds other than soybeans, and is not relevant to this passage.
premia. In this case, it is not the medical doctors but the
insurance companies which had been benefited by increased 2. The author’s contention is that, by reducing the number
consumer spending. So, it is (B) which weakens the author’s of cases in which oral argument is heard, the court can ‘dra-
assertion that the medical doctors had benefited from the re- matically increase’ the number of cases that it decides each
cent surge in consumer spending. So, (B) is the answer. year. This will be true only if a large amount of time spent in
(C) neither strengthens nor weakens the author’s assertion, deciding cases now is on hearing oral arguments. But if, even
and is not the answer. now, the time spent on orally hearing a case is only a small
(D), if true, would even contradict the factual statement part of the time spent on deciding it, then the discontinuance
that payments to medical doctors have increased last year. It of oral hearing will not produce a ‘dramatic’ improvement in
neither strengthens nor weakens the author’s assertion that the disposal of cases. So, (A) is the most effective argument
consumer spending has benefited medical doctors. against the given suggestion, and is the answer.
The given passage talks of “increase in payments to medi- (B) is not the best argument against the author’s solution,
cal doctors”, and does not distinguish between the proportion because he calls it a ‘long range solution’ which does not de-
of the medical fees paid by individuals and the proportion pend upon the position this year or next year.
met by insurance companies. So, (E) also neither strengthens You can easily see that (C), (D) and (E) are completely off
nor weakens the author’s assertion. the mark.

SET 9 3. From the fact that bones scratched with stone tools pro-
Answers duced scratches similar to those found on fossils, the author
argues that stone tools had been used on bones that became
1.D 2.A 3.B 4.A 5.D 6.D 7.E 8.A 9.B 10.E
fossilized.
Analysis
We have been asked to locate that choice which seriously
1. The first sentence says that the purpose of the program
weakens this argument.
is to “conserve the genetic diversity of soybean seeds” and, in
(B) says that trampling on sand in which a bone is buried
order to achieve this purpose, all known varieties of soybean
produces similar scratches as found on the fossils. If this is
seeds will first be collected and centrally stored.
true, the scratches found on the fossils could well have been
The second sentence says that all seeds tend to become in-
caused by people trampling on the sand above the bone for
fertile in the long run and, therefore, the seeds would periodi-
many centuries, and not necessarily by the ancient stone tools.
cally be replanted at the storage center to produce new seeds
So, it is (B) which undermines the argument of the author,
as needed.
and is the answer.
We have been asked to identify that choice which, if true,
The other choices do not have direct relevance to the argu-
would most strongly indicate that the program, if carried out
ment of the author, and can be discarded.
as planned, would not achieve its intended result.
The intended result of the program is to “conserve the ge-
4. The question implies that four of the five given choices
netic diversity of soybean seeds”. We should therefore look
support the experts’ claim that appliances produced by com-
for that choice which states or implies that the procedure
puterized technologies would be “better built”, while one choice
suggested is not likely to result in the conservation of the ge-
does not support it. We have been asked to spot this particu-
netic diversity of soybean seeds.
lar choice.
(A) gives, in fact, a valid justification for the program be-
We can easily see that choices (B), (C), (D) and (E), if true,
ing undertaken, because all the other varieties of soybean
would all lead to an improvement in the quality of the appli-
are likely to become extinct in the long run. (A) does not state
ances, or in the appliances becoming “better-built”.
a reason why the procedure proposed to be adopted is likely
If choice (A) is true, it would merely lead to greater effi-
to fail in its aim of conserving the other varieties.
ciency in the productivity of the factory, and might not result
The given narrative does not mention the cost aspect of
in a better-quality product.
the program at all, and we can therefore presume that those
So, it is (A) which would LEAST support the experts’ claim
who have formulated the program are confident of mobiliz-
that the use of computerized technology would necessarily
177
American Education Aids - LSAT - Logical Reasoning - Analysis
result in better-built products. So, (A) is the answer. been “only children”. This means that both the groups whose
behaviour had been studied had all been “only children” for
5. (‘Contraband’ means items that are legally prohibited most of the period that their behaviour was under observa-
from being imported into a country by a passenger.) The only tion. So, the observed uniformity in their behaviour could
fact before the customs inspector is that, whenever he sus- only be expected, and cannot lead to the conclusion that be-
pected a passenger of carrying contraband, a search of the ing an “only child” does not affect a child’s social develop-
latter’s baggage always yielded the prohibited stuff. ment. So, it is (D) which very much weakens the given con-
From this fact, he comes to the conclusion that nobody can clusion, and is the answer.
cheat him. (If the second group of children had consisted of those whose
We have been asked to spot that choice which constitutes parents had their second child when the first child was just
a logical flaw in his reasoning. about a year old, then the conclusion would have been valid.)
If (A) is true, tourists who might otherwise have carried Obviously, (E) neither weakens nor strengthens the conclu-
contraband did not do so because of the fear that they might sion in the passage.
be searched and found out. So, they were not trying to deceive 7. (Cartographers are specialists in drawing maps.) The
the inspector at all. (A), therefore, does not constitute a logi- inference drawn by the scholars is that, since modern explor-
cal flaw in the inspector’s claim that he always caught the ers had never seen Antarctica till 1820, but since some six-
passengers who were actually carrying contraband. teenth-century European maps show a body that resembles
The inspector’s claim has no relevance to the percentage of the polar landmass, this continent must have been discovered
passengers who intentionally carry contraband. So, (B) does and mapped by the ancients whose maps had been copied by
not amount to a logical flaw in his reasoning. 16th century European cartographers. We have been asked
The inspector does not claim that he is more efficient than to spot the choice which is most damaging to this inference.
all his other colleagues. So, (C) also does not amount to a (A) talks of the identity of the modern explorer who had
logical flaw in his claim. first sighted Antarctica around 1920, and can neither support
If it is true that some tourists whom the inspector decided nor damage the inference that the ancients had discovered
not to search had intentionally carried contraband but still the continent.
passed through the customs, it implies that he was indeed The passage does not define the “ancients”, and so could
deceived by these persons. Therefore coming to a conclusion very well refer to people who had lived just 2500 years ago,
that nobody could deceive him merely because he was always when the polar mass was as big as it is now. Nor does the
right whenever he suspected a person of carrying contraband passage say that the 16th century maps represent the size of
is logically flawed. (D) says this and is the answer. the polar mass as the same as what we know of it now. So,
Tourists mentioned in (E) were not trying to cheat the in- even if we assume that the polar mass had been smaller in
spector at all. So, (E) is not relevant to his claim that he could ancient times, the inference in the passage that the ancients
not be cheated by anyone. knew about its existence would still be valid. So, (C) does not
weaken the inference drawn by the scholars.
6. (Peers means persons of the same age group, or social Since the existence of even a single 16th century map show-
class.) The conclusion that “being an only child does not af- ing the existence of a polar land mass is sufficient to support
fect a child’s social development” is stated to have been ar- the scholars’ inference, (C) does not weaken, but strengthens
rived at on the basis of the study of the behavior of 30 “only- it, and is not the answer.
children” and 35 “first-born children” upto the time each at- The fact that most other attributions of surprising accom-
tained the age of three, because this study showed that both plishments to ancient civilizations have eventually been dis-
groups behaved in the same manner toward their peers, par- credited will not, by itself, lead to the conclusion that every
ents and other adults. attribution of accomplishment to them is wrong. So, (D) by
We have been asked to spot that choice which would weaken itself does not damage the inference of the scholars.
the conclusion as drawn from this observation. (E) concedes the possibility that the maps drawn by the
Even if the behaviour of five children in the second group ancients had indeed shown a southern polar mass, and that
had been left out of consideration, the same conclusion as in these had been copied by the 16th century cartographers. But
the passage could be logically arrived at. So, the fact that the it says that the depiction of a large polar mass in the maps of
strength of the second group of children was larger by 5 does the ancients had not been based on the discovery of that fact
not weaken the conclusion drawn in the passage. So, (A) is by explorers, but had been based on a belief by philosophers
not the answer. that the world was basically symmetrical and should there-
Even if (B) is true, and even if it is surmised therefrom fore have a southern mass in order to balance the northern
that some significant differences could have been observed continents. If this is true, the depiction of a southern land
between the behavior patterns of the “only children” and the mass in the ancient maps would have been based on an un-
“first-born children” towards their fathers, it would not very scientific speculation, and would be most damaging to the
much weaken the argument that “being an only child has lit- inference of the scholars that Antarctica must have been “dis-
tle to do with a child’s social development”, because the be- covered and mapped by the ancients”. So, (E) is the answer.
haviour patterns of the two groups had been observed to be
the same towards their peers, mothers and other adults. 8. From the fact that younger children, unlike older chil-
The behaviour of the two groups of children, or the obser- dren, assigned similar punishments for similar harms irre-
vation of such behaviour, had no relationship to the fact spective of whether the harm was caused intentionally or acci-
whether the observers had any brothers or sisters of their dentally, the author comes to the conclusion that younger
own. So, (C) neither strengthens nor weakens the conclusion children do not consider people’s intentions as relevant to the
in the given paragraph. quantum of punishment to be awarded to them.
(D) implies that, during a major part of the period of obser- We have been asked to spot that choice which could most
vation by the researchers, the “first-born” children had also seriously weaken this conclusion.
178
American Education Aids - LSAT - Logical Reasoning - Analysis
While arriving at his conclusion, the author obviously pre- their resumes.
sumes that the younger children who participated in the ex- We have been asked to spot the choice which casts the most
periment were capable of deciding which among the harms serious doubt on the value of the ‘simultaneous interview’ tech-
had been caused intentionally and which had been caused nique.
accidentally. If, as is pointed out in (A), it requires a relatively (A) talks of resumes and not of the new method, and is
mature sense of human psychology in order to tell whether therefore not the answer.
harm was caused intentionally or accidentally, which the (B) is not the answer, because, the passage specifically says
younger children would not have possessed, then the author’s that the ‘simultaneous interview’ method will be confined to
conclusion that younger children consider people’s intentions the top candidates only, and not to all candidates who had
as irrelevant to punishment would be seriously weakened. applied for the job.
Therefore (A) itself is the answer. (C) will support, and not cast doubt, on the new method.
(B) does not talk of the intention of the perpetrator of the The objection in (D) can be easily met by extending the
harm, but of the severity of the harm itself, and is not relevant time duration of each interview.
to the conclusion of the author. (E) states that the simultaneous interview distorts each
The passage is about the reactions of younger and older candidate’s response style because it induces stresses in him,
children to the stories told to them; (C), which talks of the the like of which he is not likely to meet in his subsequent
capacity of the younger and the older children to cause harm appointment. This means that the performances of the can-
by themselves, is not relevant to the author’s conclusion. didates in the simultaneous interview are not relevant to the
(D) is implicit in the narration, and only strengthens the practical needs of the job. If this is true, this will cast a seri-
author’s conclusion, and does not weaken it. ous doubt on the value of the simultaneous interview.
The issue is whether the punishment assigned by the So, (E) is the answer.
younger children for a particular harm depended upon the
intentional or unintentional nature of the harm; the issue is SET 10
not whether the punishment assigned by the younger chil- Answers
dren was proportionate to the severity of the harm or not. So,
1.A 2.A 3.A 4.D 5.B 6.D 7.D 8.E 9.D 10.B
(E) is irrelevant to the conclusion of the author of the pas-
Analysis
sage.
1. The given passage says that when a recession (meaning
a period of economic depression) deepens, the stock market
9. (The phrase ‘property crime without contact between per-
goes up, and that, later, after the economy has strengthened,
petrator and victim’ means ‘crimes such as housebreaking and
the stock market often appears to become shaky.
theft when the owners of the houses are away’.) From the fact
From these statements, the author comes to the conclu-
that cities with the densest populations have the highest ra-
sion that the stock market is a poor indicator of economic
tio of police officers to citizens, and that, in these cities, the
trends.
rates of ‘property crimes without contact between perpetra-
There is an obvious flaw in the author’s conclusion. He
tor and victim’ are also the lowest, the author comes to the
himself says that, often the stock market goes up when a
conclusion that maintaining a high ratio of police officers to
recession deepens. So, the only conclusion that can be logi-
citizens can serve as an effective deterrent to some types of
cally drawn is that the stock market does not indicate the
crime.
current state of the economy.
We have been asked to locate the choice which, if true,
His statement that the economy had strengthened later
would weaken this argument.
shows that the soaring of the stock market even in the midst
The correct choice must therefore be the one which gives
of a recession really foreboded this subsequent strengthening.
another possible explanation, unconnected with the strength
So, even according to the evidence presented by him, the
of the police, as to why the rates of burglaries and house-
stock market is a reliable indicator of the future trends in the
breakings are low in densely populated cities.
economy, and his conclusion to the contrary is wrong.
(B) says that a high population density, by itself, makes it
It is (A) which states this, and is the answer.
difficult to commit such burglaries and housebreaking (be-
None of the other choices point out such a basic flaw in the
cause neighbours are being forced to live close together be-
author’s conclusion.
cause of the high population density, and this deters burglars
and housebreakers.). It is this choice which grossly under-
2. Note that we have been asked to identify the choice that
mines the argument in the given passage, and is the answer.
would weaken the given argument.
(A) and (C) do not even refer to either the strength of police
The premise of the author is, “Young people know that ciga-
force, or property crimes, and can straightaway be discarded.
rettes exist, and they also know how to get them, and they do
If (D) is true, the number of property crimes should show
not need the advertisements to supply that information”. From
an increase, and not a decrease, because the unarrested crimi-
this premise, he argues, “Banning cigarette advertisements
nals will be emboldened to repeat their crimes.
in the mass media will not reduce the number of young people
You may be tempted to choose (E) as the answer, but it is
who smoke.”
wrong, because the conclusion reached by the author is not
Obviously, his assumptions are that the purpose of ciga-
that overall crime rate has come down in these cities, but
rette advertisements is to enable people to know about the
only some kinds of property crime have become less.
existence of cigarettes and how to get them, and that anyone
who knows that cigarettes exist and where they can buy it will
10. (In this question, the word ‘resume’ means ‘bio-data’ or
start smoking them.
a ‘write up’ by a job applicant about himself.) The leading
If, on the other hand, it is true that mere knowledge about
company referred to in the passage has preferred to adopt
the existence and availability of cigarettes is not sufficient to
the method of ‘simultaneous interview’ of all top candidates
make a person start smoking, but it is the repeated exposures
to the earlier method of judging their merits on the basis of
179
American Education Aids - LSAT - Logical Reasoning - Analysis
to cigarette advertisements that instill a desire in him to smoke, (A) says that couples who follow the same sleeping and
the author’s argument that cigarette advertisements play no waking cycles also ‘occasionally have arguments’ which can
role regarding smoking habits would be most weakened. It is jeopardize their marriage. Since the author does not assert
(A) which states this, and is the answer. that couples with the same sleeping and waking cycles will
If the places in which cigarettes are sold are found in many never argue nor have their marriage threatened, (A) does not
places, (B) will repeatedly expose many persons to these ad- weaken his argument, and is not the answer.
vertisements, and would persuade them to smoke. So, (B) (B) by itself does not either strengthen or weaken the au-
does not weaken, but strengthens the author’s argument, and thor’s argument regarding couples whose sleeping and wak-
is not the answer. ing cycles are not the same.
(C) may have an effect on the profits of cigarette compa- In developing his argument, the author is merely concerned
nies, but neither strengthens nor weakens the argument of with the quarrels between the husband and the wife. Whether
the author. they tend to quarrel with their office colleagues or not is irrel-
(D) talks of advertisements against cigarette smoking, and evant to his reasoning. So, (C) does not weaken the argument.
not of advertisements promoting cigarette smoking, and is (D) says that people in unhappy marriages express hostil-
not relevant to the author’s argument. ity by adopting a different sleeping and waking cycle from
(B) is a comparison between older and younger people, and that of their spouses. If this is true, the mismatched sleeping
has no relevance to the author’s argument which talks only and waking cycles are a result of the unhappy marriage, and
of younger people. are not the cause of the unhappiness. This seriously weakens
the author’s argument that mismatched sleeping and wak-
3. (‘Toxic’ means ‘poisonous’.) Two reasons are given for ing cycles can seriously jeopardize a marriage, because the
the need for passengers to leave airplanes swiftly after acci- marriage has already been jeopardized. So, it is (D) that is
dents: (i) the gases that are released inside the aircraft fol- the answer.
lowing an accident are poisonous; and (ii) these gases often (E) neither strengthens nor weakens the author’s argument,
explode soon after being released. and is not the answer.
The safety officials’ recommendation of providing smoke
hoods to passengers is limited to the prevention of their deaths 5. The given passage says that the only reason why the ice
due to inhalation of the toxic gas. on the front windshield melted quickly was that the defrost-
We have been asked to locate that choice which mentions ing vent had been turned on full force on it.
the strongest reason for not implementing this recommen-da- We have been asked to spot the choice which would weaken
tion. this explanation for the quick melting of the ice on the front
(A) says that, during test evacuations, it was seen that windshield.
putting on the smoke hoods added considerably to the overall We should therefore look for that choice which says either
time it took for passengers to leave the cabin of the aircraft. that ice on another part of the car, which did not have a de-
Therefore, while the smoke hoods may protect passengers frosting vent blowing on it, melted equally quickly or that ice
from inhalation of the poisonous gases, the delay in putting on another part of the car, which also had a defrosting vent
on the hood exposes them to the greater risk of dying in a gas blowing on it with equal force, melted much more slowly.
explosion, because it has been stated that the gases often ex- (B) mentions the first of the two contingencies stated above,
plode soon after being released. So, (A) is a strong reason for and is the answer.
not implementing the recommendation of the safety officials, (A) is not the answer, because the question is about the
and is the answer. melting of ice that has already been formed, and not about its
(B) cannot be a strong reason for not implementing the initial formation.
recommendation, since passenger safety should be the pri- (C) also relates the speed of melting of ice to the tempera-
mary consideration irrespective of the cost incurred. ture of the air blown on it, and will strengthen, and not weaken,
If putting on the gas hoods does not result in any delay in the explanation in the passage.
evacuation, the fact that passenger hoods can do nothing to (D) means that the defrosting vent acting on the front
prevent the gases from igniting will not be an argument for windshield will have no effect on the ice on other parts of the
not using them, because the hoods can avert at least one of car, and will neither strengthen nor weaken the explanation
the two mishaps that can befall the passengers, namely the given in the passage for the quick melting of the ice on the
inhalation of poisonous gases and consequent death. So, (C) is front windshield.
not the answer. (E) also strengthens, and does not weaken, the explanation
(D) advocates a reason for stricter measures of safety and in the passage.
not for giving them up. So, (D) is not the answer.
(E) would support the proposal for wearing of the smoke 6. The contention of the environment group is that ‘bats
hood, and not weaken it. are feared and persecuted solely because they are shy ani-
mals that are active only at night’.
4. The narrative first states that a study has shown that, We have been asked to spot the choice which would cast
if the sleeping and waking times of the two partners in a the most serious doubt on the accuracy of the group’s conten-
marriage are different, they share fewer activities with each tion.
other, and also have more violent arguments than couples who We should therefore look for that choice which states that
share the same waking and sleeping hours. another type of animal or bird, which is also equally shy and
Based on this study, the author argues that mismatched is active only at night, is not feared and persecuted by people.
sleeping and waking cycles can seriously jeopardize a mar- It is choice (D) which makes such a statement, and is the
riage. answer.
We have been asked to identify that choice which most
seriously weakens this argument. 7. The passage cites just one instance of fraud, that too
180
American Education Aids - LSAT - Logical Reasoning - Analysis
relating to a year as early as 1870, and then draws a general higher accident rate for experienced workers also.
conclusion that ‘this example demonstrates that the doctrine (D) states just this, and is the answer.
of spiritualism is worthless’. This is like concluding that all The other choices neither weaken nor strengthen the con-
college students are addicted to drugs on the basis of the drug clusion of the author.
addiction of just a single student.
The flaw in the conclusion is that it draws a general con- 10. The author says in the first sentence that, though the
clusion on the basis of a single cited instance. (D) is the best human population in Middlesex County has increased, the
argument against the conclusion, and is the answer. extent of forest land in the county has not decreased. He con-
(A) is wrong because it is not in the nature of a general cludes from this information that the decrease in the county’s
conclusion drawn on the basis of a single instance. songbird population cannot be attributed to the growth in
(B) questions the truth of the report regarding the 1870 the county’s human population.
incident, but this argument will be valid only if a general We have been asked to spot the choice which would most
conclusion can be legitimately drawn from a single instance seriously weaken this conclusion.
even if its truth has been verified beyond doubt. The argu- The obvious assumption behind the author’s conclusion is
ment in (D) which says that a general conclusion cannot be that it is only a decrease in the extent of forest land that can
drawn from a single instance is a better argument than the lead to a decrease in the songbird population. If there can be
one in (B). some other logical reason for the diminution of the songbird
The evidence cited is intended to ‘prove’ that spiritualism population, which can be directly attributed to the growth in
is false, and does not presuppose it. So, (C) is also wrong. human population, it would weaken the author’s conclusion.
(E) is wrong for the same reason as (B) - even if the report We should therefore look for that statement among the choices
had not been biased and had stated only the truth, it cannot which mentions such a reason.
lead to the general conclusion that the doctrine of spiritual- (A) does not give a logical reason to relate the increase in
ism is worthless. the number of shopping malls to a decrease in the songbird
population, and is not the answer.
8. (Beef is cow meat.) The second sentence states that fish (B) says that an increase in human population results in a
are an excellent source of omega-3 polyunsaturated fat (for corresponding increase in the number of garbage cans, and
human beings). that such increase in the number of garbage cans ensures the
The next sentence does not say that beef is an excellent survival of more raccoons (a species of small hairy animals)
source of omega-3, but says that cows obtain omega-3 which prey on songbird eggs whenever available.
polyunsaturates when they eat grass. Thus (B) gives a logical reason connecting an increase in
So, the second sentence can only mean that cows have less human population to a decrease in the songbird population
risk of heart disease for themselves by eating grass. even when there has been no diminution in the extent of forest
If, on the other hand, it is true that the omega-3 land. So, if (B) is true, one can conclude that it is the increase
polyunsaturates that the cows eat are converted into satu- in human population in Middlesex County which has resulted
rated fats in their own body, then the beef obtained by slaugh- in a decrease in the county’s songbird population, thereby
tering them and consumed by human beings would contain weakening the author’s conclusion.
these saturated fats, and will increase the risk of heart dis- So, (B) is the answer.
ease in human beings. (E) states just this, and therefore weak- If (C) is true, the decrease in the songbird population in
ens the conclusion in the passage that a diet which is rich in Middlesex could be attributed to the decrease in the extent of
beef and a diet which is rich in fish are equally effective in rain-forest land in Central and South America, and not to the
reducing the risk of heart disease. So, (E) is the answer. increase in human population in Middlesex County. This
The ultimate effect on human beings is not decided by what would only strengthen, and not weaken, the author’s conclu-
the fish or the cows themselves eat, but what the meat ob- sion.
tained by killing them contains. (D) merely confirms the author’s statement that songbird
So, (A) neither strengthens nor weakens the conclusion in population in Middlesex County is decreasing, and does not
the passage. add any more information which either strengthens or weak-
(B) does not refer to either the fish meat or beef, and nei- ens his conclusion that such decrease has not been caused by
ther strengthens nor weakens the conclusion in the passage. the increase in human population in the County. So, (D) is
(C) differentiates between people who eat fish and people not the answer.
who do not eat fish, and not between people who eat fish and (E) predicts that a decrease in the songbird population
people who eat beef. So, it also neither strengthens nor weak- could ultimately lead to a decrease in the extent of forest land.
ens the conclusion in the passage. This is, however, irrelevant to the author’s conclusion that
(D) talks about future research, and is not relevant to the the decrease in songbird population so far has not been caused
conclusion reached in the given passage which is based on by an increase in the human population. So, (E) is not the
currently available knowledge. answer.

9. The conclusion of the author is that the higher rate of


injuries which usually accompanies the increased workload
of a firm is caused by a higher accident rate for inexperienced
workers who are temporarily hired to cope with such increased
workload.
We have been asked to spot the choice which would most
weaken this conclusion.
We should therefore look for that choice which says that
increased workload of a firm is usually accompanied by a
181
American Education Aids - LSAT - Logical Reasoning - Analysis
SET 11 Since many colleges are apprehending reduced enrollments
Answers because of the decrease in the college-age population, we can
presume that admission in these college has become easier
1.C 2.D 3.D 4.B 5.E 6.C 7.A 8.E 9.D 10.A
than usual, and most of these applicants are likely to be
Analysis
offered admission in other colleges which are their first
1. We learn from the passage that the Mayor of the town
preferences. If so, most of these students who have applied
started a publicity campaign for its bus service six months
for admission in Nice College are not likely to join it eventually,
back and that, since then, there has been a 7% reduction in
and Nice College would therefore not require any additional
the morning automobile traffic entering the city’s midtown
faculty. So, it is (D) which suggests that the decision of the
area, along with a 7% rise in the number of persons travelling
management of Nice College is flawed, and is the answer.
by bus to the midtown area. The author of the passage
None of the other choices is germane to the decision of the
concludes that the mayor’s campaign has successfully
New College management to appoint additional faculty
persuaded people to leave their cars at home and travel by
members for all courses taken by freshmen.
bus to their places of work in the midtown area.
We have been asked to spot that choice which, if true, casts
3. The passage says that a drug that is highly effective for
the most serious doubt on this conclusion.
treating many types of infection is at present obtained only
We should therefore look for that choice which gives
from the bark of the ibora tree which is a rare species that is
another equally convincing reason why people are leaving
found in the wild, and that it takes the barks of as many as
their cars at home and taking the bus to the downtown area
5000 trees to obtain a mere one kilogram of the drug. The
for work.
passage therefore argues that continued production of the
(C) says that road construction has greatly reduced the
drug will ‘inevitably lead to the ibora’s extinction’.
number of lanes available to commuters in major streets
We have been asked to spot that choice which most
leading to the midtown area during the last six months. If
seriously weakens this argument.
this is true, there would have been consequent traffic jams
We must therefore look for that choice which indicates that,
which would have repeatedly prevented car-owners from
in spite of the continued production of the drug, ibora is not
reaching their work spots on time, resulting in their preferring
likely to become extinct.
to travel by buses which, apparently, have right of way in
(D) says that the ibora can be propagated from cuttings
other lanes. This is likely to have happened even if the mayor
and can be grown under cultivation. If this is true, ibora will
had not started his publicity campaign. So, it is (C) which
no longer be a rare tree that is found only in the wild, and
casts the most serious doubt on the conclusion in the passage,
many farmers will undertake its cultivation because of its high
and is the answer.
financial value, and it will not become extinct. So, it is (D)
If car-owners have preferred to travel by bus in spite of a
which weakens the argument in the passage, and is the
5% increase in the bus fare in the last six months, the mayor’s
answer.
campaign must be considered to be even more successful than
None of the other choices will weaken the prediction that
what the author of the passage thinks. So, (A) would
the continued exploitation of ibora’s barks will lead to the
strengthen, and not weaken, the conclusion in the passage,
extinction of the species.
and is not the answer.
(B) is irrelevant to the conclusion in the passage, and
4. The mayor proposes to levy an entry fee of 5 dollars per
neither strengthens nor weakens it.
day on private vehicles entering the city from outside areas
(D) means that these buses had been running with at least
with the view to alleviate the city’s traffic congestion. His
7% empty seats six months ago necessitating the campaign
reasoning is that, since the fee will exceed the cost of a round-
undertaken by the mayor to popularise bus travel, and that
trip bus fare from many nearby points, many people will
these seats are now being occupied. This choice would
switch from using their cars to using the bus.
strengthen, and not weaken, the conclusion in the passage.
We have been asked to spot the choice which ‘provides the
(E) implies that there has been no improvement in the
best evidence that the mayor’s reasoning is flawed’.
efficiency of bus service in the last six months. The increase
With the expected increase in the price of gasoline, together
in the number of users of the bus service in spite of this
with the proposed entrance fee, the cost of using one’s own
perception strengthens the conclusion that the mayor’s
car for trips to the city will further increase, and car-owners
publicity campaign has been effective, and does not weaken
may be tempted to use the bus instead. (A) will therefore
it.
strengthen the mayor’s reasoning, and not weaken it, and is
not the answer.
2. The first sentence says that, because of a decrease in
The mayor’s reasoning is that, if the cost of travelling to
the college-age population, many colleges are anticipating
the city by car costs more than travelling by bus, many car-
that there will be decreasing number of enrollments in
owners will switch over to travelling by bus. (B) contradicts
freshman classes in the coming years. But Nice College has
this reasoning by pointing out that, even now, travelling to
surprisingly received 40% more applications from qualified
the city by car costs considerably more than travelling by
applicants this year, and therefore plans to appoint more
bus, but that has not deterred car owners from preferring to
faculty members to cope with the increased enrollment.
use their own cars. (There could be valid reasons for such
The five choices in the question contain statements about
preference such as having to wait inordinately for the bus,
these applicants, and we have been asked to spot that choice
overcrowding in the bus, bus-stops being located far away
which indicates that the plan of the college management is
from the work spots etc.). Thus, (B) provides factual evidence
flawed.
that the mayor’s reasoning is flawed, and is the answer.
(D) states that a substantially lower percentage of the
The mayor’s plan is to persuade people owning cars to use
applicants than usual have indicated Nice College as their
the bus instead. (C) relates to those who do not own cars, and
first choice among the colleges to which they are applying.
is irrelevant to the success of the mayor’s plan.
182
American Education Aids - LSAT - Logical Reasoning - Analysis
(D) does not say ‘most commuters but ‘many commuters’, be replaced at least once a month. So, (A) is not the answer.
without quantifying what is meant by ‘many’. It is equally The conclusion is about the desirability of using
likely that ‘many other commuters’ may support the mayor’s toothbrushes contaminated with bacteria that cause
proposal. So, (D) does not provide evidence that seriously pneumonia and strep throat. Lack of research about similar
weakens the mayor’s reasoning. contamination by viruses, yeasts and other micro-organisms
(E) indicates that 80% of the traffic congestion in the city does not, by itself, weaken this conclusion. So, (B) is not the
is caused by vehicles from outside the city. This choice justifies answer.
the mayor’s proposal, and does not constitute evidence of a (C) says that the same researchers found that the
flaw in his reasoning. contaminated toothbrushes did not expose the user to any
greater risk of pneumonia or strep throat than the use of new
5. The given narration says that, in order to take advantage toothbrushes. If this is true, then the conclusion that one
of the increase in the sales of telephones in the last year, should necessarily change one’s toothbrush at least once a
Mammoth Industries plans to expand the production its own month loses its force. So, (C) is the answer.
model of telephone and also to continue the very extensive (D) means that even the washing of toothbrushes in hot
advertising of this product. water after each use did not prevent them from getting
We have been asked to identify the choice which provides contaminated. This will strengthen, and not weaken, the
most support for the view that Mammoth Industries cannot conclusion in the passage that toothbrushes should be
increase its sales of telephones by adopting the above plan. changed at least once a month.
The two elements of Mammoth Industries’ plan are (i) to The conclusion in the passage is that the use of a
expand the production of its model; and (ii) to continue the contaminated toothbrush is risky after a 4-weeks’ use. The
very extensive advertising of the model. finding that the risk remains stationary after a 6-weeks’ use
We must therefore look for that choice which indicates that does not weaken this conclusion. So, (E) is not the answer.
these two elements of the plan, together, are not likely to result
in the increased sale of the model. 7. The author first says that “putting patients on diets
(E) points out that, in spite of the dramatic increase in the that eliminate those foods to which the patients have been
sale of phones in the market, in spite of a reduction in its demonstrated to have allergic migraine reactions frequently
retail price, and in spite of very extensive advertising, sales does not stop headaches”. He then comes to the conclusion
of Mammoth Industries’ telephone had fallen last year. This that ‘obviously, some other cause of migraine headaches
shows that the company has not benefited from the boom in besides food allergies must exist’.
the telephones market and has not been able to sell even the We have been asked to identify the choice which most
existing production of its model in spite of very extensive weakens the above conclusion.
advertising. (The reason could be that its model is outdated, (A) says that, because most common foods elicit an allergic
or its price, even after reduction, is higher than those of more response only after several days, it is difficult to observe links
popular models etc.). So, unless these other causes are between specific foods that the patients eat and the headaches
identified and corrected, the mere increase in production of that they develop. If this is true, eliminating foods, the eating
the same model and continuance of the same level of of which is followed immediately by migraine headache (as is
advertising, is not likely to produce the intended result. So, meant by the phrase ‘foods to which the patients have been
(E) is the answer. demonstrated to have allergic migraine reactions’), is not likely
(A) would imply that the company had lost its market share to stop the headache, because these foods may not have been
only because its production was not sufficient to meet the the cause for the headaches at all, and the headaches may
demand in the market. It will therefore justify the plan of have been caused by some other food that the patient had
the company to increase production in order to regain its eaten many days earlier. If this is true, the conclusion that
market share and also maintain its level of advertising, and the cause of the migraine headache is not food allergy would
is not the answer. be wrong. So, (A) is the answer.
(B) will also justify the plan of the company to increase You can easily verify that none of the other choices leads
the production of its model. to a weakening of the conclusion in the last sentence of the
The purpose of the advertisements of consumer products narration.
is to popularise the brand name, and, according to (C), the
present advertisement policy of the company has achieved 8. After stating that robot satellites, which relay important
this purpose very well. The sale of the phones does not depend communications and identify weather patterns, would
on the consumers knowing the name of the company that eventually malfunction and can be repaired only in orbit by
produces it. So, (C) does not constitute a flaw in the company’s astronauts, the author argues that space flights carrying
plan to continue the same level of advertisement. astronauts must continue.
(D), by itself, neither strengthens nor weakens the proposal We have been asked to identify the choice which would
of the company. most seriously weaken this argument.
(A) would strengthen, and not weaken, the author’s
6. In the light of the discovery that, after four weeks of argument, because malfunctioning satellites, because of their
use, toothbrushes become contaminated with bacteria that importance in communications and weather-forecasting, must
cause pneumonia and strep throat, the passage concludes that be repaired before they fall from orbit and burn up in the
people should replace their toothbrushes at least once a month. atmosphere.
We have been asked to spot that choice which would weaken (B) emphasizes the continued importance of weather
this conclusion. satellites, and strengthens the argument of the author.
The lack of knowledge on why it takes four weeks (and not (C) is irrelevant to the argument of the author.
less) for toothbrushes to get contaminated with bacteria does (D) merely implies that repairing a malfunctioning satellite
not, by itself, weaken the conclusion that toothbrushes should in orbit is expensive. But if the cost of replacement of a
183
American Education Aids - LSAT - Logical Reasoning - Analysis
malfunctioning satellite is much higher than the cost of council.
repairing it, it will be prudent to undertake the repairs. So
the information in (D), by itself, does not weaken the argument 10. After stating that delays at the nation’s busy airports
of the author. have increased by 25% after the deregulation of airlines, the
(E) points out that repairing a malfunctioning satellite is author says that the problem can be alleviated by allocating
much more expensive than sending a new and improved more of the take-off and landing slots to commercial airlines
satellite into orbit. If this is true, there will be no need to send (in preference to private aircrafts).
astronauts into orbit to repair the malfunctioning satellites. We have been asked to spot that choice which casts the
So, it is (E) which most seriously weakens the argument of most doubt on the effectiveness of the proposed solution.
the author, and is the answer. (A) says that the major causes of the delays at the nation’s
busiest airports are bad weather and overtaxed air traffic
9. To achieve its laudable aim of reducing the generation control equipment. If this is true, then the mere allocation of
of non-biodegradable waste and to preserve the environment, more landing and take-off slots for commercial airlines is not
the council of the small town proposes to ban the sale of likely to reduce the delays, and other steps will have to be
disposable plastic goods within its jurisdiction. taken to solve these two problems. So, (A) itself is the answer.
We have been asked to identify the choice which indicates The conclusion is related to traffic in the busiest airports
that the plan to ban the sale of disposable plastic goods is not of the nation. (B) and (D), which do not refer to traffic in the
likely to lead to the fulfillment of the town council’s busiest airports, are not relevant to this conclusion.
environmental goal. (C) says that, even now, over 60 percent of the take-off
(D) says that most townspeople prefer plastic goods to paper and landing slots in the nation’s busiest airports are reserved
goods in many instances, and are likely to continue to purchase for commercial airlines. This information, by itself, does not
them from shops in neighbouring towns where such a ban is cast doubt on the effectiveness of the proposed solution,
not imposed. If this is true, the town council’s aim to preserve namely, that more of these slots must be reserved for
its environment will not be achieved by its proposed action. commercial airlines. So, (C) is not the answer.
So, (D) is the answer. (E) also emphasizes the need for some meaningful steps to
None of the other choices gives a reason why the plastic be taken to reduce the delays, and does not cast doubt on the
goods will continue to be bought and used by the residents of effectiveness of the proposed solution. So, (E) is not the
the town even after a ban on its sale is imposed by the town’s answer.

184
Analysis - Chapter 6
'STRENGTHENS' QUESTIONS
SET 1 (B) is not the answer.
Answers (C) says, “the greater the performance demands placed on
satellites, the more frequently they break down”. If this is
1.B 2.B 3.C 4.E 5.E 6.B 7.C 8.A 9.B 10.D
true, “squeezing more performance out of currently operating
Analysis
satellites” will result in their breaking down more frequently,
1. The author says first that, between personal merit and
resulting in still further increases in their insurance
money, the society respects money more than personal merit.
premiums. If such a spiral continues, the cost of television
In the experiment that he suggests, he forecasts that a
satellites will continue to increase. So, it is (C) that best
man to whom you give money will respect you more than a
supports the conclusion in the passage, and is the answer.
man to whom you give a lecture on morality. His argument
will be validated if it can be stated that the existence of
4. In this question, we have been asked to select the choice,
personal merit is demonstrated by giving a lecture on morality.
which, if true, is not a valid objection to the given argument
It is (B) which states this, and is the answer.
that the banning of the use of fireworks by the legislature
deprived the entire population of liberty and happiness, which
2. (Be careful about double negatives like the ones in this
is a violation of the Declaration of Independence, and
question. The phrase ‘valid objections’ and ‘except’, ‘together,
therefore amounts to dictatorship.
mean ‘confirmation’.) The author of the original statement
(A) contradicts the statement in the second sentence in
seeks to refute the theory that off-shore blasting in oil
the given argument that the entire population derives
exploration would hurt fishing. He points out that this year’s
happiness from fireworks and is a valid objection to the
salmon catch has been the largest in a long time in spite of
author’s conclusion.
there having been off-shore blasting during the period.
(B) is also a valid objection to the author’s argument that
The question states that four of the five choices would
banning of fireworks amounts to dictatorship, because such
disprove the author’s contention while just one of them will
banning was something voted by the people themselves
support it. We have been asked to choose that statement
indirectly through their representatives, and not imposed on
which supports the author’s contention that off-shore blasting
them by an arbitrary authority.
is in fact beneficial for fishing.
(C), if true, is also a valid objection because, if the use of
It is easily seen that (B) is the correct choice, because it
fireworks deprives many people of their lives, it directly
implies that blasting would result in an increase in the salmon
violates the right to life that has been guaranteed by the
population which, in turn, would lead to larger catches of
Declaration of Independence.
salmon during the fishing season.
(D), if true, is also a valid objection, since it means that
(A) would mean that coming to the conclusion that blasting
banning of fireworks does not offend the Declaration of
is beneficial for fishing on the basis of the catches of just one
Independence.
species of fish alone would be wrong, and that the catch of
The statement that few legislators profit financially from
other species of fish might have been less during the year. This
the sale of fireworks is irrelevant to the argument in the
would weaken the conclusion in the passage.
passage that banning of fireworks amounts to dictatorship,
If (C) were true, the salmon population must have
and does not constitute a valid objection to it. So, (E) is the
dwindled, and the author’s statement would not have been
answer.
substantiated. So, (C) would weaken the conclusion in the
passage.
5. The trend indicated in the argument is that the number
(D) also implies that there could have been other reasons
of persons living together as a single family unit has been
for the largest salmon catch this year (such as more number
steadily decreasing from 1930’s.
of fishermen fishing in the area) and therefore the author’s
Five choices have been given below, of which four are
conclusion could be wrong.
capable of being identified as factors contributing to this trend,
(E) would also directly result in the dwindling of the fish
while the fifth is not such a factor. We have been asked to
population and would go counter to the author’s theory.
spot this choice.
The increase in the divorce rate can certainly be considered
3. The first sentence says that there has recently been a
as one of the contributing factors for the reduction in the
spate (meaning undue increase) of failures in the launching
average household size. So, obviously is the decrease in the
and operation of television satellites, resulting in their
birth-rate. So, neither (A) nor (B) is the answer.
insurance premiums going up.
What (C) means is that old persons (beyond the age of 65)
The last sentence says that, because of this increase, users
are nowadays more commonly left in Homes for the Aged for
of currently operating satellites are trying to squeeze more
proper care, instead of being allowed to stay in the homes of
performance out of them.
their sons and daughters. If this is true, this will also be a
We have been asked to identify the choice which best
contributing factor for the reduction in the average household
supports the conclusion that the cost of television satellites
size, and is not the answer.
will continue to increase.
(D) says that there used to be substantial tax advantages
(A) merely implies that the insurance premiums on
earlier for keeping dependent family members with oneself,
television satellites will continue to remain high; it does not
but these have been reduced now. If this is true, this could
give any valid reason why they should increase further. So,
also be a contributing factor for the reduction in the average
(A) is not the answer.
household size, because earning members would no longer
(B) neither strengthens nor weakens the statement that
have any financial incentive to support dependents living with
the cost of television satellites will continue to increase. So,
185
American Education Aids - LSAT - Logical Reasoning - Analysis
them. 8. The first part of the given narrative says that extinction
What is left is (E), which says that enrollment of children is a process that depends on a variety of ecological,
at day-care centers has increased. These are centers in which geographical and physiological variables which affect different
working mothers leave their children during daytime, but species of organisms in different ways, and that, therefore,
pick them up on their way back home from their office. Since different species of organisms must logically be expected to
these children will continue to be counted as members of the become extinct in different periods of time.
mother’s household, the proliferation of such day-care centers The second part of the narrative points out that, on the
does not contribute to a reduction in the average household contrary, fossil record shows that many species had vanished
size. So, it is (E) which does not help to account for the trend at the same time in the past.
of reduction in the number of persons living in each household, We have been asked to identify that choice which, if true,
and is the answer. will constitute at least a partial explanation for the fact that
many species had become extinct at the same time in the past.
6. The passage argues that, if high speed ground (A) says that widespread environmental disturbances can
transportation is provided between major cities which are result in major episodes of extinction affecting numerous
between 200 and 500 miles apart, airport congestion can be different species at the same time. This, if true, will give an
reduced considerably. explanation for the fossil record referred to in the given
We have been asked to identify the choice which would narrative. So, (A) itself is the answer.
form a valid evidence supporting the above argument. (B) will confirm what is stated in the first part of the
We should therefore look for that choice which implies that narrative, but will not provide an explanation for the
a considerable proportion of the present airline traffic consists contradictory result mentioned in the second part. So, (B) is
of travel between cities which are less than 500 miles apart. not the answer.
Among the choices, it is (B) which makes such a statement, (C) also does not give an explanation for “many species
and is the answer. vanishing at the same time”. So, (C) is not the answer.
What is required is a possible explanation for the
7. We learn from the given information that new collision- extinctions during the early periods for which we have fossil
avoidance systems have been developed for aircrafts, but they records. (D), which does not refer to this early period at all
have not yet been fully tested to discover potential but talks of ‘geologically recent times’, is not relevant to the
malfunctions. issue, and is not the answer.
Yet, the airline management wants these systems to be (E) also confirms the statement in the first part of the
installed immediately in its passenger planes, arguing that narrative, but does not provide an explanation for the
their mechanical warnings would enable pilots to avoid contradictory statement in the second part that many species
crashes. had become extinct at the same time. So, (E) is not the answer.
The pilots, on the other hand, refuse to fly the planes in
which these collision-avoidance systems (which have not been 9. The narrative states that researchers, who have noted
fully tested) are installed, arguing that, if they malfunction, that different local populations of bowerbirds of the same
they would mislead the pilots, thereby causing crashes. species build bowers that exhibit different building and
We have been asked to identify the choice which, if true, decorative styles, have concluded that the bowerbirds’ building
would strengthen the objection of the pilots. styles are culturally acquired (meaning ‘learnt by watching
(A) is a statement that is applicable to any mechanical other bowerbirds’) and not genetically transmitted.
device, and does not, by itself, strengthen the argument of We have been asked to identity that choice which will most
the pilots, unless it is asserted that a malfunctioning collision- strengthen this conclusion of the researchers.
avoidance system will cause a crash in a situation in which a If (A) is true, it would imply that the common features in
plane without such a system will not be involved in a crash. the bowerbuilding styles are due to the fact that
Since this assertion has not been made, (A) is not the answer. bowerbuilding skills are genetically acquired. This would, in
(B) is a statement about jet engines, and is irrelevant to fact, weaken, and not strengthen, the conclusion of the
the controversy about collision-avoidance systems. In fact, if researchers. So, (A) is not the answer.
the collision-avoidance devices also behave in the same (B) first says that young male bowerbirds lack the skills
manner as the jet engines, and result in exemplary required for bowerbuilding, implying that bowerbuilding
performance and safety records, the objection of the pilots skills are not genetically transmitted. It then says that they
will be weakened, and not strengthened. So, (B) is not the however become accomplished in bowerbuilding after
answer. watching their elders for years. This implies that
(C) states that the collision-avoidance systems, when they bowerbuilding is a culturally acquired skill. So, (B)
are functioning properly, will enable pilots to avoid some strengthens the conclusion drawn by the researchers, and is
crashes, but, if they malfunction, they will cause even more the answer.
crashes. If this is true, it will undoubtedly strengthen the If the bowers of one particular species of bowerbirds are
objection of the pilots to fly the planes fitted with these significantly different from the bowers of all other species, it
systems until they are fully tested against possible implies that their bowerbuilding skill is genetically inherited.
malfunctioning. So, (C) is the answer. This weakens, and does not strengthen, the conclusion of the
If (E) is true, any help from a mechanical device should be researchers.
welcomed by the pilots, and not resisted. So, (E) does not Neither (D) and (E) refers to the building styles of
strengthen the argument of the pilots, but weakens it. bowerbirds. So, they neither strengthen nor weaken the
Since the controversy is regarding the immediate conclusions of the researchers.
installation of the new devices in passenger planes, (E) will
weaken, and not strengthen, the objection of the pilots. So, 10. The first sentence says that bats detect, locate and
(E) is not the answer. catch their prey (or victims) by using the echoes of the sounds
186
American Education Aids - LSAT - Logical Reasoning - Analysis
produced by them as reflected by such prey. If (D) is true, it will itself form a valid and alternative
The second sentence says that the capacity of bats to catch explanation for the reduction in the population of native fish
their prey through this method is not as effective in the case in the Emerald river, and will weaken the scientists’
of certain categories of moths which are able to hear their hypothesis. So, (D) is not the answer.
sounds. (E) is just a factual statement, and neither strengthens nor
What will best support the above claim is an assertion weakens the scientists’ hypothesis. So, (E) is not the answer.
which states that bats find it more difficult to catch the
categories of moths which can hear their sounds (and can 2. (Antibiotics are drugs that kill bacteria.) The first part
therefore take evasive action to escape from them) than those of the narrative says that routine use of antibiotics can give
categories of moths which cannot hear their sounds. rise to resistant bacteria that are capable of surviving any
It is (D) which states this, and is the answer. further administration of antibiotics, and that the presence
(A) talks of moths living in an environment which is of such resistant bacteria in people is probably due to their
continuously free of such bats, and is therefore irrelevant to having used such antibiotics repeatedly.
the given statement. The second part says that some scientists believe that this
(B) implies that the efficiency of the bats is not affected by is not the real cause, but that most of the resistant bacteria
the behaviour of the moths which are able to hear their in people are derived from their consumption of bacterially
sounds. This contradicts the claim in the passage, and is not infected meat.
the answer. We have been asked to identify that choice which, if true,
(C) is also irrelevant to the given statement, because it will most significantly strengthen the belief of these scientists.
refers to the sound made by the moths, and not about the (A) says that antibiotics are routinely included in livestock
reflection of the sound made by the bats. feed to increase the rate of growth of animals. If this is true,
If (E) were true, it will weaken, and not strengthen the it could result in the presence of antibiotic-resistant bacteria
given statement. in the meat of these animals, and consequently in the people
who consume such meat. Thus (A) itself strengthens the belief
SET 2 of these scientists, and is the answer.
Answers Even if (B) is true, it means that it is not the eating of
bacterially infected meat, but the subsequent administration
1.A 2.A 3.D 4.E 5.D 6.E 7.A 8.A 9.A 10.C
of the antibiotics to get rid of the poison from the infection,
Analysis
that causes the development of resistant bacteria in people.
1. We learn from the given narration that the range of
This strengthens what is stated in the first part of the
water temperature (meaning the difference between the
narrative, and not the belief of the scientists. So, (B) is not
highest temperature in summer and the lowest temperature
the answer.
in winter) in the portion of the Emerald river below the dam
Since it is not the “quality of meat” that is at issue, but the
had been reduced from 50º (prior to the construction of the
presence of resistant bacteria in it, (C) is not relevant to the
dam) to 6º (after its construction).
belief of the scientists.
We also learn that, two decades after the construction of
(D) strengthens not the scientists’ belief but what is stated
the dam, it has been found that none of the eight fish species
in the first sentence of the passage, and is not the answer.
native to the river are reproducing adequately below the dam.
If the claim of the livestock producers mentioned in (E) is
The author states that scientists have hypothesized that
true, it will weaken, and not strengthen, the scientists’ belief.
“sharply rising river temperatures” (resulting in a larger
So, (E) is not the answer.
temperature difference) is required to signal the native fish
species to begin the reproductive cycle, and that it is the
3. {You should understand that the first sentence gives
narrowing of the temperature difference after the construction
Panovich’s conclusion, but the second sentence (which is the
of the dam that is responsible for the reduction in the
view of the author) refutes it.} The only evidence on which
population of these fish species in the river below the dam.
Panovich argues that dinosaurs (the huge animals which lived
We have been asked to identify that choice which, if true,
on earth millions of years ago) were cold-blooded is that
would most strengthen the scientists’ hypothesis.
dinosaur teeth (which have been discovered in archaeological
(A) says that, in the side streams of the river below the
explorations) exhibit annual growth rings just as those of
dam, where the annual temperature range remains
percent day crocodiles which are cold-blooded.
approximately 50º, the native fish species continue to
The author, in the second sentence, refutes this conclusion
reproduce as before. If this is true, it can be concluded that
by stating that desert bighorn sheep (which are warm-
the major factor in the reproductive capacity of the native
blooded) also exhibit annual growth rings in their teeth, and
fish species is the wide range of temperature difference in the
therefore Panovich’s conclusion is incorrect.
water, thereby strengthening the scientists’ hypothesis. So,
We have been asked to spot that choice that would
(A) itself is the answer.
strengthen Panovich’s reasoning.
(B) will imply that it is the existence of backwaters, and
Now, we can agree with Panovich, provided there is some
not an adequate temperature difference that is needed for the
other valid explanation why desert bighorn sheep exhibit
breeding of the native fish species in the Emerald river. If
annual growth rings in spite of being warm-blooded.
this is true, it will weaken, and not strengthen, the scientists’
(D) says that warm-blooded animals exhibit annual growth
hypothesis. So, (B) is not the answer.
rings in their teeth only if they are living in areas with extreme
While the scientists’ hypothesis relates to “sharply rising”
seasonal variations in temperatures, whereas cold-blooded
water temperature, (C) talks of the lowest recorded
animals (such as dinosaurs) exhibit such rings wherever they
temperatures before and after the construction of the dam.
live. If this is true, this gives a valid explanation why desert
So, (C) by itself does not either strengthen or weaken the
bighorn sheep (which live in deserts where the temperature
scientists’ hypothesis, and is not the answer.
is very high in summer and very low in winter) exhibit annual
187
American Education Aids - LSAT - Logical Reasoning - Analysis
growth rings in their teeth in spite of being warm-blooded. In So, she assumes that the potential number of contributors
this case, the objection to Panovich’s theory by the author of among the readers of the magazine is not less than 70 + 140
the passage will not be valid. So, (D) is the answer. = 210
This means that, according to her, the advertisements
4. According to the passage, there are two methods of which appeared in the three issues were noticed by less than
producing ethanol – from sugarcane and from corn. one third (namely 70) of the potential contributors to the
The former method requires more energy than the latter, Alliance (210 plus).
but yet the producers prefer the former method. So, (A) is the answer.
We have been asked to spot the choice which gives a valid None of the other choices strengthens the expectation of
reason for such preference, in spite of the higher energy the president.
requirement in the case of the former.
When will this preference of the producers be justified? 9. The given statement says that the political power of
Obviously when, in spite of the higher requirement of energy, retired people will expand faster than their numbers.
the overall cost of producing ethanol from sugarcane is less We have been asked to spot the choice which does not
than that in the second method. support this statement.
Among the choices, it is (E) which provides this reason, The given statement will be valid only if a number of people
since the required additional energy can be obtained who had not taken any interest in politics before their
practically free of cost by burning the sugarcane residue. retirement start taking such interest after their retirement.
So, (E) is the answer. Any choice which states the opposite of this will be the one
which does not support the given statement.
5. What the scientists wished to show was that (i) (A) says that people who had not taken interest in politics
hummingbirds pollinate only red flowers, and that too only before their retirement are not likely to take interest in it even
during day, and (ii) hawk moths pollinate only white flowers, after their retirement. If this is true, it will weaken the logic
and that too only during night. of the given statement. So, (A) is the answer.
Their contention will be strengthened if red flowers which All the other choices support the statement in the passage,
are covered during day, and white flowers which are covered and do not weaken it.
during night, do not get pollinated. It is (D) which says this,
and is the answer. 10. The question implies that, of the five given choices,
four represent a benefit to the farmers arising from the new
6. If the census had been perfect, and the separated cloning technique, and only one choice does not represent a
husband and wife were both living in the US, then the number benefit. We have been asked to spot this particular choice.
of separated men must be equal to the number of separated (A) will be beneficial to farmers, since they will then have
women. (Because of the law of bigamy in the US, a separated much quicker access to superior strains of plants.
man or woman cannot remarry, unless there is a legal divorce, Since identical copies of a plant are likely to grow at fairly
in which case each of them would be counted as divorced and uniform rates, (B) means that farmers will find it less difficult,
not as separated). or more easy, to care for them; this is also beneficial to the
It is explanations II and III which will account for the fact farmers.
that the census recorded more number of separated women If (C) is true, genetic uniformity among the plants grown
than of separated men. by a farmer would enable the diseases and pests to spread
I is not relevant for the census of separated couples. rapidly, and will adversely affect his economic interests. So,
So, (E) is the answer. it is (C) which is not only not beneficial, but is even detrimental
to farming, and is the answer.
7. From the fact that Luis has just seen two ravens, the Since identical clones will be uniform in size, (D) will also
author predicts that the next bird that Luis sees will also be be beneficial to the farmers.
a raven. (E) enables desirable traits being introduced in the clones,
This assumes that there are more than two ravens in the and will again be beneficial to the farmers.
vicinity, and that they fly in close formation, or in flocks. It is
(A) which states this, and is the answer. SET 3
(B) in fact, will weaken the author’s prediction, because, if Answers
it is true, there will be a considerable time-lag before the
1.A 2.A 3.B 4.B 5.A 6.A 7.D 8.B 9.D 10.B
next raven comes into the field of vision of Luis, and, in the
Analysis
meantime, other types of birds may pass by her.
1. The belief till recently was that Y causes Z. It has since
(C) says that ravens are seen only occasionally, and not
been seen that both Y and Z are often observed to be preceded
frequently, in Luis’ state of residence, and this will again
by X. The suggestion in the new report is that it is X, and not
weaken, and not strengthen, the author’s prediction.
Y, that causes Z.
(D) neither strengthens nor weakens the author’s prediction.
We have been asked to spot that choice which would best
(E) does not imply that the next bird that Luis sees will
support the suggestion in the new report.
only be a raven and not any other species.
The new suggestion will be strengthened if a number of
cases are observed in which Z has directly followed X, without
8. If the total number of contributors to the Alliance is 100
there being an occurrence of Y in between.
at present, 70 of them have enrolled by seeing three
It is (A) which states this, and is the answer.
advertisements in the Grange Report.
(B) and (C) would strengthen the old theory that Y causes
By advertising in 9 more issues, the president of the
Z, and not the new theory that X causes Z.
Alliance hopes to increase the number of contributors to at
(D) merely means that X causes Y, but does not give any
least double this number, namely 140.
188
American Education Aids - LSAT - Logical Reasoning - Analysis
information about whether X or Y causes Z. (A) will explain only (ii).
(E) can be an argument equally in favour of both the old (C) and (D) will explain only (i).
and the new theories, and does not specifically confirm the (E) is not relevant to any of the three trends.
new theory. (B) mentions that there is still a large market for some
drugs, and this explains (iii).
2. The last two sentences of the passage together mean (B) also says that research competition has increased
that laws requiring employers to provide parental leave and among different pharmaceutical companies, which means
day-care centers for children would lead to ‘happier, better- that the research staff can exploit this situation to demand
adjusted families’ because such laws would decrease the stress much higher salaries and amenities. This explains (i) also.
levels of employees who have responsibility for small children. (B) also says that, because of its high cost, research is now
We have been asked to locate a statement which will being restricted to only a few drugs for which there is a large
strengthen this conclusion. market. This explains (ii) also.
The statement that will strengthen this conclusion is one So, (B) is the answer.
which asserts either that more stress levels result in unhappy
families, or less stress levels result in happier families. (A) 5. The passage may be summarized as follows: “In America,
states the first, and is the answer. Black Americans as well as White Americans consume the
(B) does not relate the higher stress level to unhappiness same type of food. Yet, studies show that Black Americans
in the family, and therefore does not support the conclusion are more prone to high blood pressure than White Americans.
in the last two sentences in the passage. In Africa itself, westernized Black Africans and the White
The other choices also do not relate the stress level of Africans take more or less the same type of food. Again, the
employees to the happiness in their family lives. former are more prone to high blood pressure than the latter.
From these facts, researchers hypothesize that high blood
3. The author first gives the factual information that the pressure results when a group of people who have been fed
average age and average years of experience of race drivers at for generations on low salt food (meaning Black Africans in
Indianapolis race increased every year between 1965 and 1980. general) suddenly take to high-salt food.”
He then attributes such increase to the fact that high-speed We have been asked to spot the choice which relates to the
racing drivers were living longer than their predecessors, and present-day westernized Black Africans and which would
this, in turn, was because race-car safety features had reduced confirm this hypothesis.
the severity of the crashes of the kind which used to result in We should therefore look for that choice which says that
the deaths of race drivers earlier. westernized Black Africans who have descended from people
We have been asked to identify the choice which would who have had salt-rich diet for generations continue to have
strengthen this conclusion of the author. low blood pressure.
If, because of improvement in the race tracks or in the Of the choices, (A) gives an instance of the former, and is
design features of the racing cars, the number of accidents the answer.
that occurred on the race tracks had diminished substantially (B) talks of the general population, and not of westernized
between 1965 and 1980, then the reason why race drivers Africans, and is not the answer.
are living longer now must be attributed to these factors, and (C) does not state whether the blood pressure level of these
not to the race-car features that had reduced the severity of persons subsequently came down or not, and is therefore not
the crashes of the kind that used to be fatal to the drivers the answer.
earlier. So, the choice that strengthens the author's conclusion (D) refers to the general Yoruba population, and not to the
must be the one that states that there has been no significant westernized ones among them, and neither confirms nor refutes
reduction in the number of major crashes on the race track. It the researchers’ hypothesis.
is (B) which states this, and is the answer. (E) refers to people whose food habits have not changed,
If (A) is true, and these younger drivers had lost their lives whereas the hypothesis concerns those whose food habits have
in these accidents, the average age of race-drivers must have changed in this generation. So, (E) also neither confirms nor
increased and not decreased during this period. So, (A) even refutes the researchers’ hypothesis.
contradicts a factual information given in the narration, and
is not the answer. 6. The author first states that, on the basis of a study of
(C) refers to the age of persons attempting to qualify as the ratio of the number of rider injuries to the number of riding
race car drivers, and not to those who have actually become hours of each of the two brands of cycles Xylo and Zenon
such drivers, and is not the answer. during the period 1981-84, a consumer agency had concluded
(D) talks of accidents on highways in USA, and is irrelevant that Xylo was safer than Zenon. (This implies that this ratio
to a conclusion about race track safety. was smaller in the case of Xylo than in the case of Zenon.)
If the reduction in the fatalities in racing is attributable He then states that, since 1985, the number of injuries
to the factors mentioned in (E), it would weaken the conclusion among the riders of Xylos has been twice as great as the
of the author, and not strengthen it. So, (E) is not the answer. number of injuries among the riders of Zenons, and then
asserts that, consequently, the agency’s conclusion would have
4. Three trends are mentioned in the given statement: (i) been different for the period subsequent to 1985.
the research and development expenditure in the This conclusion could either be that both Xylo and Zenon
pharmaceutical industries has been going up, (ii) the number are equally safe, or Zenon is a safer cycle than Xylo.
of new drugs invented and patented has come down steadily, We have been asked to identify that choice which would
and (iii) profits of the pharmaceutical industry have been support the author’s claim.
steadily increasing. Note that the consumer agency’s conclusion was not based
We have been asked to spot the single choice which will on the actual number of injuries to riders of Xylo and Zenon,
explain, at least partly, all these three trends. but on the ratio of the number of rider injuries to the number
189
American Education Aids - LSAT - Logical Reasoning - Analysis
of riding hours on each brand. Therefore, if the agency’s The first part of the second sentence then refers to the
conclusion had to be different for the period after 1985, the traditional assertion that it was the residents of the remote
ratio of the number of rider injuries to the number of rider rural areas in the State who were the supporters of the
hours in the case of Xylos must have been equal to or greater temperance movement during the mid-nineteenth century. The
than the corresponding ratio in the case of Zenons. second part of the second sentence mentions the conclusion
If, for example, the number of rider injuries and the that, on the contrary, the supporters of the temperance
number of riding hours in Xylos had been 100 and 1000 movement were concentrated in “centers of economic
respectively, and the number of rider injuries and the number opportunity”.
of riding hours in Zenon had been 50 (half as that in Xylos) We have been asked to identify that choice which would
and 400 respectively, the ratio in the first case would have best support the latter conclusion.
been 1/10 and the ratio in the second case would have been 1/ (A) talks of what happened during the second half of the
8. In this case, Xylo would still be considered to be a safer nineteenth century, whereas the conclusion drawn in the
vehicle than Zenon, and there would be no change in the article is about the situation in the mid nineteenth-century.
consumer agency’s conclusion. Moreover, (A) does not also throw any light on whether the
But if the figures had been 100 and 1000 in the case of support for the temperance movement during this period was
Xylo, and 50 and 500 in the case of Zenon, then the in the remote rural areas or in the centers of economic
corresponding ratios would be 1/10 and 1/10, and the opportunity. So, (A) neither supports nor refutes the conclusion
conclusion of the consumer agency would have been that both drawn in the article, and is not the answer.
cycles were equally safe. (B) states that support for the ban on alcohol was strongest
If the figures had been 100 and 1000 in the case of Xylo, in those towns of New York State where the economy was
and 50 and 600 in the case of Zenon, the corresponding ratios based on new, growing industries. Since the areas where the
would be 1/10 and 1/12 respectively, and the consumer agency economy was based on new, growing industries would have
would have concluded that Zenon was safer than Xylo. been the same as those which were the centers of economic
So, while asserting that the conclusion of the consumer opportunity, (B) strongly supports the conclusion reached in
agency would be different for the period since 1985, the author the article, and is the answer.
is assuming that the number of riding hours for Zenon was (C), in fact, supports the traditionally held view that
at least half the number of riding hours for Xylo. support for the temperance movement was mainly among
It is (A) which says this, and is the answer. residents of the rural areas, and contradicts the conclusion
(B) looks like a possible answer, but is wrong because it reached in the article. So, (C) is wrong.
does not mention the number or ratio of riding hours on each (D) implies that it was not support to the temperance
cycle. movement, but opposition to the temperance movement, that
had been concentrated in the centers of economic opportunity.
7. The assertion in the given passage is that people often This also contradicts, and does not support, the conclusion
make irrational decisions, without analyzing which choice reached in the passage. So, (D) is wrong.
would bring them maximum benefits and minimum harm. (E) also contradicts the conclusion drawn in the article,
The given question implies that, of the five choices, four and does not support it.
constitute evidence in support of this assertion, while the fifth
either weakens the assertion or is irrelevant to it. We have 9. The statement in this question has been phrased in a
been asked to identify the last one. deliberately confusing manner. The author first says: People
We should therefore look for that choice which implies that first predict that a certain result will not take place unless a
the decision taken is rational, and is likely to bring maximum certain action is taken. They then believe that their prediction
benefits and minimum harm to the person taking that was correct if the prescribed action was taken and the
decision. predicted result also takes place.
(A) gives an instance of an irrational decision which is likely The author then disagrees with this belief (as can be
to harm the decision maker, and is not the answer. understood from the use of the word “however”) because,
(B) is also an instance in which the decision maker is taking according to him, the result can admit of more than one
an unnecessary risk, and is not the answer. interpretation.
(C) is also an instance of a person taking a potentially We have been asked to identify that choice which supports
dangerous decision in spite of being cautioned by others, and this claim of the author.
is not the answer. We should therefore look for that choice which implies that
An illustration of (D) is a person preferring to go in a the linking of a particular action with a particular result, or
crowded road by a bicycle than by a bus, on the assumption the linking of the absence of a particular action with the
that more number of people are likely to be hurt if there is an absence of a particular result may not always be correct.
accident to the bus. This decision is irrational because a cyclist It is (D) which throws doubt on the correctness of such a
is more likely to be involved in an accident on a crowded road linkage, and is the answer.
than a bus is. So, (D) is also an illustration of an irrational
decision, and is not the answer. 10. Mr. Primm’s argument is that teaching hospitals are
(E) is what any cautious or rational person should do, and very expensive to run and, therefore, if hospitals depend only
does not form an illustration of the author’s assertion that on profits for their survival, teaching hospitals will cease to
people often take irrational decisions. So, (E) is the answer. exist.
He implies that it is not possible for teaching hospitals to
8. (“Temperance movement” means the movement that charge high enough fees to cover their cost.
supported the prohibition of alcoholic drinks.) The first Ms. Nakia disagrees with Primm, and argues that teaching
sentence mentions a fact: that about 80% of the towns in New hospitals attract the very best physicians, and also treat non-
York State had banned the sale of alcoholic beverages in 1846. routine cases successfully.
190
American Education Aids - LSAT - Logical Reasoning - Analysis
But these two arguments of Nakia, by themselves, do not imports. This gives a possible reason why, as a last resort,
meet Primm’s point that teaching hospitals will find it difficult the industry has seen it fit to raise the issue of national security
to balance their budgets. in support of its petition. So, (C) is the answer.
The additional argument that will meet Primm’s point is The other choices do not explain why this particular
that, because teaching hospitals attract the very best of industry should emphasize the national security aspect while
physicians, they are capable of treating non-routine cases pleading for its own interests.
which command a high price. This would enable the teaching
hospitals to increase their revenue and balance their budgets. 3. The reason given by the local board of education why
It is (B) which states this, and is the answer. physics classes no longer attract high school students is that
None of the other choices meets Primm’s argument. the current physics curriculum has little direct relevance to
today’s world.
SET 4 It is in order to rectify this defect that the board has
Answers proposed a curriculum that emphasizes the principles of
physics involved in producing and analyzing visual images.
1.D 2.C 3.E 4.B 5.E 6.C 7.B 8.B 9.E 10.D
We have been asked to locate that choice which provides
Analysis
the strongest reason to expect that the new curriculum will
1. What we have been asked to do is to spot that choice
be successful in attracting students.
which can provide evidence for determining whether physical
Since the reason for the present curriculum not attracting
fitness helps one react better to stress.
students is that it is irrelevant to today’s world, the new
We should therefore look for that statement among the
curriculum, to be successful in its aim, must be relevant to
choices which cites an instance of a person’s capacity to face
today’s world.
situations of stress having either increased or decreased after
In other words, the production and analysis of visual
a period of physical fitness training.
images, which the new curriculum emphasizes, must be
(A) does not mention how many among the men and how
relevant to today’s world. The correct choice must therefore
many among the women who participated in the laboratory
be the one which emphasizes this point.
tests had earlier undergone the program of aerobic exercises.
(E) states, “In today’s world, the production and analysis
So, (A) does not furnish the required evidence.
of visual images is of major importance in communication,
(B) looks like a plausible answer, but is wrong because it
business and recreation”. This means that a person who has
says that a group of men who underwent a weight-lifting
studied this subject is likely to obtains a job easily in these
training program for six-months encountered fewer stressful
fields. Therefore, it is choice (E) which provides a strong
situations during a subsequent period. This does not help us
reason to expect that the proposed curriculum will be
to state whether the weight-lifting program had improved
successful in attracting students, and is the answer.
their capacity to cope better with the stressful situations when
While (A) is true, it will not, by itself, persuade students
they actually encountered them.
to study physics, unless it is known that producing and
(C) talks of persons who had undergone a course in
analyzing visual images is relevant to today’s world. Since
‘relaxation techniques’ and is irrelevant to this passage which
(A) does not state this, it is not the answer.
is concerned with ‘physical fitness’.
(B) is a general statement about ‘technology used in today’s
(D) specifically refers to a group of college students who
world’, and does not explain why the new physics curriculum
showed a markedly reduced reaction to stressful events after
should lay particular emphasis on the principles involved in
training in aerobics for six months. Thus (D) furnishes
producing and analyzing visual images. So, (B) is not the
evidence for concluding that physical fitness really helps one
answer.
to react better to stress. So, (D) is the answer.
(C) merely implies that the school administering the new
(E) says that persons with high self-esteem participated
curriculum may save some money, but it does not explain
more in physical exercise programs than persons with low
why the students should opt to study physics under the new
self-esteem. This statement does not provide evidence on
curriculum.
whether physical fitness helps one to face situations of stress
(D) merely restates the problem, which is the reason why
better. So, (E) is not the answer.
the board of education had decided to change the curriculum,
but this choice makes no reference to the proposed solution
2. (Industries in USA want their government to restrict
which lays special emphasis on visual images laid in the new
imports from abroad, so that indigenous industries are able
curriculum. So, (D) is not the answer.
to have unfettered business locally. The ‘petition for import
quota’ in the last line of the passage means ‘request to the
4. The given narrative states that the proposed legislation
government to impose restrictions on imports’). Normally,
(i) seeks to substantially raise the minimum wages to be paid
when petitioning for import quotas, the machine-tool
to employees, and (ii) will simultaneously exempt, for the first
manufacturing industry should have stressed commercial
time, all small business from paying any minimum wage.
aspects as well as the need for protecting the employment
The narrative then expresses surprise why small business
opportunities for local citizens in the industries. But the
groups are still lobbying to get the legislation defeated.
passage says that the industry had raised the issue of national
We have been asked to locate that choice which would best
security in its petition for import quotas.
explain why small business groups are opposed to the new
We have been asked to spot the choice which gives a
legislation even though, on the face of it, it is beneficial to
plausible explanation as to why the machine tool industry
them because it exempts them from paying even the existing
raised the national security issue.
lower statutory minimum wage for the first time.
Among the choices, (C) states that the other grounds that
(B) says that, in order to attract workers, small businesses
had been raised earlier by the industry did not convince the
must match the wages offered by their larger competitors,
government of the need for imposing quotas for machine-tool
and that these larger competitors have to necessarily pay the
191
American Education Aids - LSAT - Logical Reasoning - Analysis
higher minimum wages stipulated under the new legislation. explains the disparity between the wages in the auto and
This implies that, though the new law would not compel small garment industries.
businesses to pay the higher minimum wages, the market A manufacturer is not likely to increase the wages of his
conditions would force them to do so. As a result, the proposed workers if such an increase is likely to make a big dent in his
legislation would harm small businesses. Thus, (B) best profit margin. If, for example, the wages form only 4% of the
explains the rationale behind the opposition of small selling price of $1000 of a car (or just $40), an increase of
businesses to the proposed legislation, and is the answer. 25% in the wages will reduce his profit by only $10 out of a
(A), by itself, should lead to the conclusion that small sale price $1000.
businesses would support the new legislation which exempts But, if the wages form 60% of the selling price of $100 of a
them from paying the minimum wage. It does not explain garment, or $60, an increase of 25% in the wages will reduce
why they are opposing it. So, (A) is not the answer. his profit by $15, out of a sale price of only $100. So, it is this
The absence of information about the number of small aspect which can be an important contributing factor to the
businesses that will benefit by the exemption cannot by itself low wage levels in the garment industry.
explain why small business groups are against the proposed So, (B) is the answer.
legislation. So, (C) is not the answer. (A) talks of the variations in wages within the garment
(D) refers to state legislation and not to federal legislation, industry and within the auto industry, and is not relevant to
and is irrelevant to the given issue. the issue which deals with variation in wages between the
The narrative says that ‘small business groups’ are workers in the garment industry and the workers in the auto
lobbying to defeat the proposed legislation. If service industry.
companies make up the majority of small businesses as stated You can easily see that none of (C), (D) and (E) mentions a
in (E), this implies that they are also an important part of factor which is relevant to the existence of the disparity in
these lobbying groups. (E) does not explain why they are wages between auto and garment industry workers.
opposing the new legislation when it should, in fact, be more
beneficial to them than to the other small businesses. So, (E) 8. (An incumbent member means a sitting member.) The
is not the answer. author’s claim is that since incumbent members of the
Congress get a lot of press publicity, and are also entitled to
5. (A cluster of commercials means a group of consecutive free mail privileges and generous travel allowances, they enjoy
commercial advertisements.) The second statement says that an overwhelming advantage over their challengers in elections
television viewers are able to recall the first and the last in a for the United States Congress.
cluster of advertisements more easily than they can recall We have been asked to identify that choice which, if true,
the other advertisements broadcast in between. best supports this claim.
The first statement says that the proportion of brand names (A) says that incumbent Congressmen had met with a
covered in TV advertisements that the viewers are able to larger number of lobbyists than the challengers did during
recall is slowly decreasing. the last congressional election. Since nothing had prohibited
We have been asked to identify that choice which, if true, the challengers also from meeting with an equal number or
would render the second statement a logical conclusion of more of lobbyists, (A) by itself does not support the claim that
the first. the incumbents enjoy an overwhelming advantage over the
We should therefore look for that choice which implies that challengers, and is not the answer.
the proportion of the number of intermediate advertisements The author’s claim is that the incumbents enjoy an
to the number of first and last advertisements has been overwhelming advantage over their challengers in the
showing an increase recently. It is (E) which implies this, elections for the US Congress. If 98% of the incumbents in
and is the answer. (E means that, if the average number of the Congress who were seeking re-election actually won the
advertisements in a cluster was, say 5, till recently, it has election, as stated in (B), it will certainly be a conclusive proof
been increased to, say 9, now. If so, viewers would have been of the existence of such an advantage. So, it is (B) which
able to recall 2/5 of the advertisements earlier, while they strengthens the author’s claim, and is the answer.
are now not able to recall only a reduced proportion of 2/9 of The fact that incumbent members of the Congress are often
the advertisements.) critical of the attention given to them by the news media
implies that, much of the time, such media attention is adverse
6. The question implies that, of the five given choices, four to them. If so, the author’s contention that such media
constitute valid reasons why treatment in an emergency room attention gives them an overwhelming advantage over their
costs more than treatment in a physician’s private office, and challengers will be weakened, and not strengthened. So, (C)
the remaining one does not constitute such a reason. We are is not the answer.
asked to identify this choice. (D), if true, in fact weakens, and does not strengthen, the
You can easily see that (A), (D) and (E) are valid reasons claim of the author that incumbent members have an
why hospital emergency room treatment is costlier. overwhelming advantage over their rivals. So, (D) is not the
(B) is a valid explanation why treatment in a private answer.
physician’s office is cheaper. Since 98% of the incumbent members actually getting
(C) compares the variety of illnesses treated in emergency elected is a much stronger proof of their advantage over their
rooms and by private physicians, but does not give a reason contenders than 78% of them having hoped to win the
why, when the same illness is treated in an emergency room elections, (E) is not a better answer than (B).
and by a private physician, the former should be costlier. So,
(C) is the answer. 9. After indicating in the first sentence that, at present,
government relies more on general tax revenue than on
7. In the light of the information given in the narration, passenger fares to finance public bus and train service, the
the question asks us to spot that choice which is a factor which author argues that, to be fair, the users should be made to
192
American Education Aids - LSAT - Logical Reasoning - Analysis
meet the full cost of running these services through increase SET 5
in fares if necessary, since it is only the users who benefit Answers
from public transportation.
1.D 2.C 3.B 4.E 5.A 6.E 7.D 8.D 9.C 10.D
We have been asked to identify that choice which
Analysis
constitutes a principle underlying the author’s argument.
1. The passage implies that there is a shortage of teachers
Obviously, the principle that the author relies on is that
in public schools at present.
general tax revenue should be used only for purpose which
In the first sentence, the author makes a categorical
benefit the general public, and not for purposes which benefit
statement that the primary reason for this shortage is not
only a section of the public. It is (E) which states this, and is
the toughening of the hiring standards (meaning prescription
the answer.
of higher qualifications for appointment). In the next
(A) would imply that the more the number of people using
sentence, he makes an equally categorical statement that the
a public service, the more should be the amount of government
primary causes for such shortage are: (i) there have been no
financial support for that service. This would contradict, and
improvements in working conditions of teachers in recent
not form the basis for, the author’s argument that transport
times; and (ii) the salaries of teachers have not kept pace
users should pay all the operating costs of the transportation
with salaries in other professions.
service. So, (A) is not the answer.
We have been asked to identify that choice which would
(B) would imply that, if the operating cost of a
most support the author’s statements.
transportation service is high, the government should restrict
(A) implies that many teachers already in the profession
that service. This is not what the author of the passage says.
had been recruited on the basis of lesser qualifications than
His argument is that government can provide as much service
what have been prescribed at present. This statement neither
as is necessary, provided the cost of such service is recovered
supports nor disproves the author’s contentions, and is not
from its users. So, (B) is not a necessary assumption in the
the answer.
author’s argument.
(B) merely implies that the new recruiting regulations are
Both (C) and (D), in fact contradict, and do not form the
being strictly enforced. This also neither supports nor
basis for, the author’s argument.
disproves the author’s thesis, and is not the answer.
(C) and (E) both cite new recruiting standards as a reason
10. The given narrative is: Since the export of vanilla beans
for the shortage of teachers. These weaken and not strengthen
from a particular island in the Indian Ocean had dropped
the author’s theory that tougher recruitment standards are
drastically after the secret formula of the soft drink Voltage
not the primary reason for the shortage of teachers.
was changed, analysts have concluded that the original secret
(D), if true, confirms the author’s theory that the shortage
formula of Voltage contained vanilla from these beans and
of teachers is due to inadequate pay scales and unsatisfactory
the new formula does not.
working conditions. So, (D) is the answer.
We have been asked to identify that choice which, if true,
would most strengthen the conclusion of the analysts that
2. The author’s conclusion is that there is a critical early
the earlier formula of Voltage contained vanilla as an
period in the life of a living being (namely, the very first weeks
ingredient.
after it was born) when the living being must be exposed to
If (A) is true, and if this second island has started to
light for it to develop normal vision for the rest of its life. He
successfully compete with the first island in exporting vanilla
draws this conclusion from experiments in which one of the
beans to the rest of the world, it will form a different and
two eyes of newborn kittens or monkeys were covered with
probably valid explanation for the drop in the export of vanilla
patches for two weeks and they never developed normal vision
beans from the first island. So, (A) weakens, and does not
for the rest of their lives.
strengthen, the conclusion of the analysts that the drop in
His argument will be strengthened if it can be proved by
vanilla export from the first island was only due to the change
experiment that, once normal vision has already been
in the formula of Voltage. So, (A) is not the answer.
developed during such a critical early period, subsequent
(B) will also constitute another different and valid
wearing of a patch over the eye does not affect the future vision.
explanation for the drop in the export of vanilla beans from
(C) describes an experiment which proved this, and is the
the Indian Ocean island, and will weaken, and not strengthen,
answer.
the conclusion of the analysts. So, (B) is not the answer.
(A) talks of adult men and women, and neither strengthens
(C) will also constitute another different and valid
nor weakens the author’s argument.
explanation for the drop in the export of vanilla beans from
(B) is already an assumption contained in the argument
the Indian Ocean island, and will weaken, and not strengthen,
of the author, and does not add to the strength of his
the conclusion of the analysts. So, (B) is not the answer.
argument.
The given narrative states that the export of vanilla beans
(D) is wrong, since ‘young children’ is too vague an
from the Indian Ocean island has drastically come down ever
expression and may mean the age of one day to 7 years.
since the formula of Voltage was changed. Choice (D) says
(E) is in fact a conclusion that can be drawn from the
that, simultaneously, the import of vanilla beans has dropped
author’s argument, and is not an independent evidence to
in countries where Voltage is made. (D) will obviously
support his argument.
strengthen the conclusion of the analysts that the original
secret formula of Voltage had contained vanilla, while the
3. According to the passage, the reason why some residents
new formula does not contain it.
of large cities move to small towns and small cities is that
So, (D) is the answer.
they are fed up with big government. Their objections against
(E) will also independently constitute a valid explanation
‘big government’ are that (i) big government is unresponsive
for the drop in export of vanilla beans from the Indian Ocean
and (ii) it interferes with their lives.
island, and will weaken the conclusion of the analysts. So,
We have been asked to identify the choice which will imply
(E) is not the answer.
193
American Education Aids - LSAT - Logical Reasoning - Analysis
that moving to a small town will not, by itself, solve these Obviously, (A) strengthens the author’s conclusion, and is the
problems. answer.
Obviously, the correct choice is the one that would imply Since the author does not refer to meanings of words and
that, even after moving into small towns, these people will phrases, (B) is irrelevant to the conclusion of the author.
have to continue to deal with the same type of government (C) would imply that the authorship of an unknown work
agencies. cannot be determined on the basis of similarity of verbal
(A) refers to people who have decided to continue to live in patterns with known works of an author. This, if true, would
big cities, and is not the answer. weaken, and not strengthen, the conclusion of the author. So,
(B) says that the government agencies which are often (C) is not the answer.
perceived as being unresponsive are national agencies whose (D) implies that some verbal patterns will occur in every
policies are uniform across the nation, meaning that they are work irrespective of who its author is. If this is true, works by
the same in large cities as well as in small towns. If this is different authors will also exhibit the same verbal pattern.
true, moving into small towns is not a solution for the problem This will weaken, and not strengthen, the conclusion of the
of having to deal with unresponsive government agencies. author. So, (D) is not the answer.
So, (B) is the answer. The author’s conclusion is based on the assumption that
(C) talks of migrants from rural areas to small towns, while word choice is a conclusive indicator of authorship. (E)
the given passage refers to migrants from big cities to small contradicts this, and therefore weakens the conclusion of the
towns. So, (C) is not the answer. author. So, (E) is wrong.
(D) would imply that there is no problem at all in big cities,
and this contradicts the very basis of the given narration, 6. The first sentence states that, as an energy source, wood
and is not the answer. is preferable to coal, oil and gas because of the consideration
(E) does not talk of government interference at all, and of cost. The second sentence says that, nevertheless, wood
can be rejected for that reason. will never supply more than a modest (or small) fraction of
the nation’s energy needs.
4. (A county is a geographical unit similar to a municipal We have been asked to spot that choice which does not
ward in India). From the statistics that the incidence of support this claim.
encephalitis is least in counties which had the greatest number The choice must therefore be one which gives a valid reason
of TV sets per hundred persons, the researchers have why wood is capable of supplying a considerable portion of
concluded that the persons in these counties stay indoors the energy needs of US.
more, and thus are less exposed to the mosquitoes which only Among the choices, it is (E) which gives such a reason and
thrive outdoors and which cause this disease. is the answer.
We have been asked to select that statement among the All the other choices will strengthen the claim of the author
choices which will strengthen this conclusion. that wood will never be able to supply more than a small
The mere possession of TV sets by itself does not mean fraction of the nation’s energy needs.
that people spend more time indoors than outdoors, unless
this fact is supplemented by the information that the persons 7. In the first sentence, the author states that, in the early
in this county also watch TV for a longer hours per day than 1980’s, many makers of car radiators switched over from
those in the other counties which have less number of TV copper to aluminum because aluminum was then cheaper
sets per capita. It is (E) which states this, and is the answer. than copper.
(A) is wrong because it does not even mention the number In the next sentence, he first gives the information that
of TV sets per capita. aluminum prices have risen sharply this year, but predicts
(B) implies that the occupations of county residents is more that, in spite of such increase, radiator manufacturers are
outdoors than indoors, but it does not connect the number of not likely to revert to copper, because car makers have used
TV sets per capita to the incidence of the brain disease. So, aluminum radiators to design better-performing cars, and are
(B) is not the answer. likely to insist on the continued use of aluminum radiators
(C) may be the result of other protection measures taken in spite of their higher price.
independently by the authorities of the county, and may not We have been asked to spot that choice which would
at all be related to TV watching by the citizens. So, this choice, support the prediction of the author, but for a different and a
by itself, does not strengthen the conclusion of the researchers. more valid reason.
(D), in fact, contradicts the conclusion of the researchers, If the price of aluminum radiators goes up because of the
because it implies that encephalitis will be least prevalent in increase in the price of aluminum, and if all car makers
counties in which people spend more time outside their homes. continue to use aluminum radiators, the price of all cars will
go up simultaneously. So, the buyers’ preference for a lower
5. According to the author, the similarity in the use of verbal priced car will play no role in the radiator manufacturers’
patterns (including the frequency of specific words and anticipated decision either to stick to aluminum or to revert
recurrence of certain phrases) in four known works of an to copper. So, (A) is not a valid reason that would support the
author, and in a work of uncertain authorship establishes that author’s prediction. So, (A) is not the answer.
the same author wrote all the five works. If (B) is true, it will give an added reason for the radiator
We have been asked to identify that choice which, if true, manufacturers to revert to copper, and will contradict the
most strengthens the conclusion of the author. author’s prediction that the radiator manufacturers are likely
(A) says that no two authors are likely to display similar to continue to use aluminum. So, (B) is not the answer.
verbal patterns in their works. If this is true, the fifth work (C) implies that a great majority of car owners who have
which displays verbal patterns similar to the four works of been used to better performing cars (whose design had been
the known author could not have been written by anyone else, made possible because of the use of lighter aluminum
and must have been written only by that author himself. radiators) come to value superior performance highly, and
194
American Education Aids - LSAT - Logical Reasoning - Analysis
are therefore likely to insist on the continued use of aluminum We have been asked to locate that statement among the
in spite of its higher price. This is the same reason given by choices which does not support this argument.
the author for his forecast, and is therefore not the answer. (D) means that State support can be given only to artists
(D) says that this year’s high aluminum prices reflect a who already have gained public approval. If this is true, this
temporary shortage, and that its prices are likely to collapse weakens the given argument, because the same artist cannot
and stabilize at levels even below those of the early 1980’s. If have two basically contradictory styles. So, (D) is the answer.
this is true, it is obvious that radiator manufacturers will (A) strengthensthe last sentence in the argument, and is
continue to use aluminum for economical reasons, thereby not the answer.
confirming the author’s forecast. (D) thus gives a much more (B) strengthens the second sentence in the argument, and
valid reason to support the author’s prediction than what is not the answer.
has been given in the passage. So, (D) is the answer. (C) follows from the first and second sentences and is not
Whether radiator manufacturers had reduced the price of the answer.
their products or not in early 1980’s is not relevant to their (E) also is a restatement of the second sentence, and is not
present option either to continue to use aluminum in spite of the answer.
its present higher price or to revert to copper. So, (E) does not
constitute a reason for the continued use of aluminum in spite SET 6
of its present higher prices. So, (E) is not the answer. Answers
1.D 2.C 3.B 4.A 5.D 6.B 7.D 8.D 9.D 10.D
8. From the fact that the weekly spendable earnings of
Analysis
nonsupervisory workers in the private sector has steadily
1. The narration refers to the recent proposal of a number
declined during the 1970’s, the author draws the conclusion
of writers that ‘our ancestors were influenced by visits from
that economic disparity among various classes in the USA
extra-terrestrials (meaning creatures from other planets), and
has been widening.
that it is these visitors who had built such monuments as the
The author’s argument would be weakened if someone
pyramids of Egypt.
points out that the weekly spendable incomes of supervisory
We have been asked to spot that choice which will
workers and owners of companies have also declined equally
constitute the strongest evidence in favour of this proposal.
or more during this period, because it would mean that the
(A) will, in fact, contradict the proposal, since it would mean
disparity either remains as it was, or is in fact getting
that our ancestors were themselves capable of building these
narrowed down. If, on the other hand, it is pointed out that
monuments and had probably done so.
there has been no decline in the earnings of supervisors and
Since all structures (including those built by us during
owners during this period, the author’s argument would be
our own generation) generally point to the sky, (B), by itself,
strengthened. Thus, (D) is the answer.
cannot prove that such structures were built by extra-
(A) is irrelevant to the argument, and neither strengthens
terrestrial visitors.
nor weakens it.
While (C) may be an argument in favour of the theory of
(B) can be said to strengthen the argument, only if it is
visits by extraterrestrial beings to various parts of the earth,
simultaneously mentioned that the earnings of supervisors
it cannot by itself lead to the conclusion that monuments such
and owners have either increased or remained the same.
as pyramids were built by them.
(C) and (E) would in fact weaken the author’s argument
(D) is the answer, since it would mean that our ancestors
that the trend was most noticeable in the 1970’s.
did not possess the technology which would have enabled them
to build these monuments by themselves and, therefore, these
9. What the author says is that robots will not only
monuments must have been built by persons who had visited
eliminate jobs, but will also diminish the quality of working
the earth from other heavenly bodies.
life, even in factories where they do not eliminate all the
(E) also has the same effect as (C), and cannot prove that
workers.
pyramids were built by extraterrestrial visitors.
This argument of the author will be strengthened if it is
shown that, even where robots do not eliminate all the
2. The passage says that forestry experts are beginning to
workers, they take over all the jobs which the workers usually
question the policy of attempting to extinguish all forest fires
find enjoyable, and leave for the workers those jobs which
instead of allowing some of them to run their course. This is
the workers find difficult or boring, thus diminishing the
based on the observations about the results of forest fires
‘quality of working life’ of the workers. It is (C) which states
which have indicated that their effects may, in the long run,
this, and is the answer.
be beneficial to the overall ecology of the forest.
(D) weakens the conclusion that robots diminish the
We have been asked to spot that choice which could be one
‘quality of working life’ of the workers.
of the ‘observations’ which had persuaded the forestry experts
None of the other choices has a nexus with the ‘quality of
to come to their conclusion.
life’ of the workers, and is the answer.
(A) mentions the cause of most forest fires, and does not
give a reason why some of these fires, once they start, should
10. The question implies that four of the five given choices
be allowed to ‘run their course’. So, (A) is not the answer.
strengthen the given argument, while the remaining choice
(B) may constitute a reason for not extinguishing the forest
either weakens it, or is irrelevant to it. We have been asked to
fires in order to save human lives, but not in order to preserve
spot this remaining choice.
the overall ecology of the forest, which is the concern of the
The argument in the statement implies that one artist who
forestry experts. So, (B) is not the answer.
is supported by the State and another artist who has to please
(C) says that frequent small forest fires prevent the
the public for earning his income would have basically
accumulation of highly flammable dead brushwood and
different styles, the former resorting to self-indulgence and
timber, which can subsequently support massive,
waste, and the latter catering to the pleasure of the public.
195
American Education Aids - LSAT - Logical Reasoning - Analysis
uncontrollable fires. It is obviously the observation of such imagination.
small fires that has persuaded the forestry experts to conclude (A) refutes the hypothesis and does not support it, and is
that not all forest fires are harmful to the ecology of the forest, not the answers.
and that the small fires are in fact beneficial to it, and should (B) furnishes evidence that chiru was, in fact, one of the
be allowed to run their course. So, (C) is the answer. inhabitants of the river valley at some time in the past. If the
(D) and (E) do not constitute reasons why some fires should fossil that has been discovered is 16,000 years old, it is quite
be allowed to run their course. possible that the species had lived in that valley during the
next 100 years also, in which case the artist could have
3. The ‘paradox’ mentioned is that, even though Japanese personally seen it. So, it is (B) which strengthen the given
factory owners have to continue to pay the workers the earlier hypothesis, and is the answer.
rates of wages even after they install ‘labour-saving’ (C) and (D) neither support nor weaken the hypothesis that
machinery, they still find the installation of such machinery the particular artist who had drawn the chiru had actually
advantageous. seen it.
We have been asked to spot that choice which logically The hypothesis is not that the artist had painted the
explains why this is so. Himalayan chiru, but that he had painted the chiru that he
When ‘labour-saving’ machinery is installed in one section was familiar with. The given narrative does not say that the
of the factory, the employees in that section have to be painting is identical with the Himalayan chiru, but only says,
retrained to work in some other section, and such retraining “it seems to resemble the chiru, a rare antelope of the
results in expenditure to the management. But, if the labour- Himalayas”. So, the fact that the antlers (the horns) in the
saving machinery increases the productivity of the factory painting are longer than those of the Himalayan chiru neither
and yields an additional profit which is more than the cost of supports nor weakens the given hypothesis. So, (E) is not the
such retraining, then the installation of the machinery is answer.
advantageous to the owners.
(B) states just this, and is the answer. 7. To support his argument that colour-tinting of movies
The other choices do not resolve the ‘paradox’ of factory which had been originally photographed in black and white
owners installing labour-saving machinery and willing to is inappropriate, the author gives the reason that, if the
continue to pay the employees their earlier wages. movies had been originally filmed in colour, hundreds of the
original artistic choices would have been made differently. As
4. The view expressed in the passage is that all members one of the artistic choices, he mentions the lighting on the
of a community are interdependent, and, therefore, any sets, which had been so arranged that highlights and shadows
particular member cannot argue that, if he does not protect looked right in black & white. He thereby implies that the
himself against accidents or illnesses, none else in the lighting parameters are different for black & white movies
community suffers thereby. and colour movies.
We have been asked to spot that choice which would We have been asked to locate that choice which gives a
strongly support this view. similar illustration to support the author’s argument.
(A) states that the cost of avoidable accidents and illnesses We should therefore look for another of the artistic choice
increases the insurance rates for everyone. If this is true, a that had been made keeping the requirement of black and
person who exposes himself voluntarily to accidents or white filming in view.
illnesses affects others also adversely. Therefore (A) itself Choice (D) says that makeup for the artistes in the old
supports the argument of the author, and is the answer. movies had been applied so as to look best in black and white
(B) contradicts the author’s view, and is not the answer. films, implying that they significantly differ from the makeup
(C) says that a life without minor risks may be boring to that has to be applied if the movie is to be shot in colour. If
the person concerned, and does not say how one person taking this were true, giving colour tinting to these films will not
such risks will affect other persons. So, it neither strengthens give the best results, and such tinting will indeed be
nor weakens the author’s view, and is not the answer. inappropriate. So, (D) is the answer.
(D) is also irrelevant to the author’s view, and is not the While (A) also gives a reason why colour tinting is not
answer. appropriate for black & white films, it does not ‘lend the same
(B) weakens the author’s view, and is not the answer. type of support’ to the argument of the author as the one
given by him.
5. The main point in the argument is that, because of The other choices are obviously inappropriate.
extensive inbreeding, the elephant seals all have a genetic
uniformity and are thereby in greater danger of becoming 8. (‘Egregious’ means ‘glaring’. ‘Tuition’ means ‘tuition fee’)
extinct (meaning ‘ceasing to exist’) than any other species. The given narrative first says that, in Borkland University,
This implies that genetic uniformity is not desirable in any $ 4.2 million worth of scholarship stipends were unclaimed
species of animals, and that those species, in which members last year, even though a large number of promising students
are genetically different from one another, have a better chancewere in need of scholarships but could not fulfill the criteria
of survival (because a disease which destroys those with a stipulated for them.
particular genetic property cannot kill others without that It therefore draws the conclusion that the qualifying
genetic property). (D) is what states this, and is the answer. criteria of many of these scholarships must be revised.
The other choices are not related to the consequences of We have been asked to identify that choice which will
‘inbreeding’. strongly support this conclusion.
The solution for (A) is that the availability of the
6. We have been asked to identify that choice which scholarships should be given wide publicity. (A) does not
supports the hypothesis that the artist had painted the chiru therefore call for any change in the qualifying criteria, and is
from personal knowledge of its features, and not out of pure not the answer.
196
American Education Aids - LSAT - Logical Reasoning - Analysis
(B), if true, will require an increase in the funds earmarked sentences.
for the scholarships, and not any change in their qualifying We have been asked to identify that phrase among the
criteria. Therefore, (B) does not strengthen the conclusion in choices which could have been the major contributory cause
the narrative. for the introduction of such a community-service sentence
(C) means that the problem mentioned in the narrative is instead of a prison sentence in the state.
not real and there is, therefore, no need for any change in the The direct result of the community-service program is a
qualifying criteria. It weakens, and does not strengthen, the substantial reduction in the number of persons who had to be
given conclusion. accommodated in prisons. Its introduction was, therefore,
If (D) is true, and if those scholarships had earlier been most probably prompted by the overcrowding of prisons in
earmarked for certain fields of study which are no longer the state. So, (D) is the answer.
offered by Borkland University, it is obvious that, for these The other choices do not give a plausible reason why
funds to be utilized, the conditions relating to those community service sentence was awarded instead of the usual
scholarships must be revised. So, it is (D) which strongly prison sentence in the case of this type of offenders.
supports the conclusion in the narrative, and is the answer.
(E) would imply that it is not worthwhile revising the 10. If the trail of volcanic craters and cinder cones were
criteria at a huge cost. This will weaken the conclusion in the formed when the continental mass slid westward over a
narrative, and will not strengthen it. stationary plume, earlier craters must be on the western side
of the trail, while later craters must be on the eastern side of
9. The first sentence says that, instead of the usual prison the trail.
sentences, approximately 5000 people convicted of non-violent So, it is (D) that follows logically from the given passage,
crimes in the state have been given community-service and is the answer.

197
Analysis - Chapter 7
'COMPLETION' QUESTIONS
SET 1 reduction in price would increase the sales, and would not be
Answers an undesirable business strategy.
1.E 2.B 3.B 4.E 5.A 6.E 7.B 8.C 9.B 10.E (D) would also contradict the advice because, if the
Analysis expansion of the market niche will increase profits, one should
1. The use of word “because” preceding the blank implies advocate mass-marketing techniques, and not advise against
that we should identify that choice which gives a logical reason it.
why new managers are more likely to successfully accomplish (E) would also contradict the advice, because achieving
the task of terminating the failing projects than the old more sales of such a premium product through mass-
managers are. marketing techniques would be more profitable, and should
(A) will be a logical conclusion to the sentence if the be supported and not opposed.
intention is to save the project, and not if the intention is to
terminate the project. 3. The first sentence of the narrative says that most
(B) would also be appropriate if the task of the new participating countries favoured uniform control over the
manager is to overcome the problems and enable the project quality of the effluents, whether or not specific environmental
to survive, and not when his task is to terminate the project. damage could be attributed to a particular source of effluent.
The passage is about the appointment of a new manager The second sentence says, “What must, of course, be shown,
after a final decision has been taken to terminate a failing in order to avoid excessively restrictive controls, is that ....”.
project. (C), which talks of action to be taken before the decision The concluding part of the sentence must therefore be that
is taken to terminate a project, cannot therefore be a logical choice which is an illustration of the avoidance of excessively
conclusion to the second sentence. restrictive controls.
(D) gives a possible reason why the project had got into (A) talks of the need for quick implementation of any
difficulties, and may be a possible conclusion to the passage control that is adopted, and does not form an illustration of
if the intention were to save the project (by appointing a more the avoidance of excessive controls. So, (A) is not the answer.
competent manager who can foresee any future problems) (B) says that it must be shown that any substance that is
instead of to terminate it. made the subject of controls can actually cause environmental
When a project shows symptoms of failing, the normal damage. If this stipulation is made, it is only the damage-
reaction of the original managers (who had been in charge of causing effluents that will be subject to control, and harmless
its implementation) will be to put the blame on others, and effluents which do not cause damage to environment will not
ask for additional resources to keep the project alive. These be subjected to the proposed controls. This will certainly
managers will usually advance reasons to justify why the result in the avoidance of excessively restrictive controls. So,
project should be maintained and not terminated. Obviously, (B) is the answer.
a decision taken by the management to terminate the project (C) will merely ensure that even those countries which
will not be implemented by the old managers wholeheartedly. are presently generating the largest quantities of effluents
On the other hand, a new set of managers can successfully are conscious of the need to control environmental damage.
accomplish the task because they have no need to justify the So, (C) will not result in the avoidance of excessively restrictive
earlier decision to maintain the project. So, (E) is the most controls, and is not the answer.
logical completion to the given paragraph, and is the answer. (D) implies that some quantity of a pollutant that is now
generated may not actually be reaching the North Sea at
2. The first two sentences imply that, in buying a premium present, while the remaining quantity is reaching the North
(meaning expensive) product, people are motivated by a desire Sea. If so, what is required, in order to combat environmental
to be associated with something special. threats to the North Sea, is that even that limited quantity of
The first part of the next sentence says that, while trying this pollutant that presently reaches the North Sea must be
to market such premium products, “mass-marketing controlled. So, the statement in (D) that control over any
techniques and price-reduction strategies should not be used”. pollutant must be exercised only if it is shown that the entire
We have been asked to identify the choice which gives a quantity of that pollutant reaches the North Sea is not logical.
valid reason behind this advice. So, (D) is not the answer.
If the motivation that impels people to buy a product, in (E) is obviously irrelevant to the avoidance of excessive
spite of its being expensive, is that they want to be associated restrictive controls on future pollution, and is not the answer.
with something special, we can deduce that many of them
would not buy that product if it becomes commonplace and 4. (The word ‘objectivity’ means ‘truth’ or ‘neutrality’.) In
cheaper. This means that continued sales of such a product the first sentence, the author says that critics often question
depends on preserving the special nature of the product, or on the objectivity of government-owned news media (such as
“the maintenance of an aura of exclusivity” as stated in (B). State-owned TV networks), because such media are usually
So, (B) is the answer. financially dependent on the government. In the next sentence,
(A) would, in fact, contradict the advice because, if affluent he says that the critics’ point is that one cannot take an
purchasers are a shrinking proportion of the population of objective view of one’s patron.
purchasers, reduction of prices and mass-marketing In the first part of the last sentence, he says that the same
techniques would increase the sales of the product, and are point applies in the case of privately owned news media also,
therefore desirable business strategies. because “privately owned news media ....”
(C) would also contradict the advice because, if the We have been asked to spot that choice which best
prospective purchasers are concerned about the price, completes the last sentence.
198
American Education Aids - LSAT - Logical Reasoning - Analysis
We must therefore look for that choice which implies that those who have copied their programs illegally would be
the privately owned news media also depend financially upon purchasing them after paying their price. In the first sentence
the patronage of a limited number of persons or groups, and of the passage, the author says that this estimate is greatly
such news media also cannot be objective while reporting the exaggerated, and goes on to say that most of the illegal copying
news about these persons or groups, or about subjects in which is done by people with no serious interest in the programs.
these persons or groups have vested interests. The obvious inference from this statement is that, if paying
Among the choices, it is only (E) which states this, and is for these programs is the only way of obtaining them, most of
the answer. these illegal copiers (who have no serious interest in them)
(A) would only imply that news gathered by privately will not purchase them at all, and the real loss of revenue to
owned news media may be incomplete, but does not constitute the computer industry because of such illegal copying is,
a reason why privately owned news media may not be therefore, much smaller than estimated. It is (B) which
objective. So, (A) is not the answer. imparts this meaning to the last sentence, and is the answer.
(B) talks of the relative security of employment in privately
owned and government-owned news media, and is not 8. The last sentence starts with the phrase ‘The clearest
relevant to their relative objectivity. So, (B) is not the answer. example of this defensive strategy is the fact that .....”. The
(C) by itself does not constitute a reason why the privately previous two sentences explain the defensive strategy as that
owned news media may not be objective, and is not the answer. of ‘established companies’ who tend ‘not to be innovative
(B) will, in fact, strengthen the critics’ point of view, and themselves and tend to underestimate the effects of the
not the contrary view of the author (as can be guessed from innovations of others’. The author implies by the phrase ‘tend
the use of the adverb ‘However’) in the last sentence, and is to underestimate the effects of the innovations of others’ that
not the answer. such companies would come to grief by such shortsightedness.
Among the choices, the most appropriate illustration of a
5. The last sentence in the paragraph must follow logically company that came to grief on such basis is (C), which talks
from the previous sentence. of a ‘once-successful’ manufacturer of slide rules who reacted
Henry Miller, according to George Orwell, wrote the way to the introduction of electronic calculators by not entering
many ordinary people talked, and, like the ordinary people, the electronic calculator market themselves, but by producing
he shrugged off the contradictions of life. Where he stood better slide rules. (We now know that slide rules have become
apart from them was in his piercing clarity which was museum pieces within two decades.) So, (C) is the answer.
revolutionary. He lived till the ripe old age of eighty-eight. (A) does not imply that the manufacturers of ballpoint pens
During the long years of his writings (which, because they and soft-tip markers are different from the manufacturers of
were written in the same way as was spoken by people), he fountain pens. Nor does it say that fountain pens have become
had converted the people to his ways of thinking, particularly totally outdated. On the contrary, they are now being
in relation to politics. What was revolutionary when he started marketed as ‘luxury’ or ‘prestige’ items at higher prices. So,
writing had thus become the common attitude of the middle (A) is not an illustration of what is stated in the first two
class, mostly because of his writings. sentences of the passages, and is not the answer.
Among the given choices, it is (A) which is the logical last (B) talks of two innovations by the same manufacturer (of
sentence for the paragraph, and is the answer. which one failed), and is not an illustration of one established
(B) is wrong since Orwell has been mentioned only in the manufacturer underestimating an innovation by another new
first sentence, and there is nothing in the subsequent manufacturer, and is not the answer.
sentences which can logically connect him to the last sentence. (D) talks of a product intended for one category of
(C) is wrong since we hear nothing about the contradictions consumers being used also by another category of consumers,
of life after the second sentence, and it cannot suddenly recur and is not an apt illustration of the author’s viewpoint.
in the last sentence. (E) refers to a product aimed at one category of consumers
(D) is wrong because what Orwell said was that Henry that has become much more popular with another category
Miller wrote in the way many ordinary people talked, and of consumers, and is not an apt illustration of the author’s
there is nothing in the subsequent sentences to show that viewpoint.
Miller had changed his style later.
(E) is a factually wrong statement, since the penultimate 9. An obvious illustration of the proposition that ‘greater
sentence says that, by the time Miller was eighty-eight, risk must be offset by the chance of greater rewards’ is the
ordinary people shared Miller’s views. demand of a higher rate of interest on unsecured loans than
on loans backed by collateral. So, (B) is the answer.
6. This is a very clever question and the answer, which at Obviously none of the other choices illustrates the
first glance is surprising, is (E). proposition so aptly.
A moment’s reflection would tell you why the paintings
will be true images of the subjects in spite of the painter’s 10. The first two sentences state that cars fuelled by
astigmatism. methanol have much lower levels of emissions of carbon
This is because, in the eyes of the painter, the paintings monoxide and environmentally harmful hydrocarbons than
will also be as distorted as the subject as seen by him. gasoline-fuelled cars, but have somewhat higher level
Therefore, for the viewers who do not have a defective vision, (meaning only a slightly higher level) of emission of
the painting will be a true image of the original, and will formaldehyde than gasoline-fuelled cars.
reproduce the actual proportions! The first part of the next sentence concludes that,
nevertheless, a methanol-powered car actually produces less
7. In claiming that it is losing millions of dollars through atmospheric formaldehyde pollution than a comparable
illegal and free copying of programs, the computer industry gasoline-powered car.
is obviously assuming that, if such copying is not allowed, all The use of the conjunction ‘because’ implies that the
199
American Education Aids - LSAT - Logical Reasoning - Analysis
missing portion of the sentence should give a valid reason less atmospheric formaldehyde pollution than a comparable
why it is so. gasoline-powered car. So, (E) is the most appropriate ending
(E) points out that, while methanol-powered cars emit to the last sentence of the passage, and is the answer.
formaldehyde directly, most formaldehyde pollution generated You might have been tempted to choose (A) as the answer.
by gasoline-powered cars results from the photochemical But (A) only says that formaldehyde pollution is a less serious
conversion of other hydrocarbon emissions into formaldehyde threat to environment than carbon monoxide and some
in the atmosphere. Combining this information with the hydrocarbons, but does not give a reason why a methanol-
statement that methanol-powered cars emit much lower levels powered car produces less formaldehyde pollution than a
of hydrocarbons than gasoline-powered cars do, and only a comparable gasoline-powered car.
somewhat higher level of formaldehyde, we can come to the You can easily verify that all the other choices are very
conclusion that a methanol-powered car actually produces much wide off the mark.

200
Analysis - Chapter 8
'EVIDENCE ANALYSIS' QUESTIONS
SET 1 What is implied in this criticism is that the law would
Answers result in an increase in the beverage prices.
1.D 2.A 3.A 4.C 5.C 6.E 7.C 8.B 9.B 10.C The third sentence starting with ‘However’ represents the
Analysis view of the author. He counters the argument given in the
1. From the first sentence, we can say that the persons second sentence by pointing out that the quantity of water
who have the least risk of contracting type A influenza are needed for washing the used bottles is much less than the
either (i) those who had already been attacked by it once, or quantity of water needed to manufacture throwaway
(ii) those who had been vaccinated against it. containers, and also that the saving in the cost of this water
(D) mentions the first group, and is the obvious answer. is more than the cost of the gasoline needed to transport the
None of the other choices mentions either of these two used containers.
categories of persons. If what the author states is true, then the use of recyclable
bottles is cheaper than the use of throwaway bottles, and
2. The fear of the insurance companies is that the amount therefore the law objected to by some people will not lead to
covered by the claims may be more than the premium income, an increase in beverage prices. So, (C) is the answer.
leading to losses to them.
The best way to minimize such possible losses is to make 5. (A) is wrong, because it would further deplete the number
the policies attractive enough for a large number of people of young shrimps that go out into the sea, and would reduce
who are not likely to submit claims immediately to join the the shrimp harvest in South Atlantic still further.
scheme. Attracting ‘middle-aged’ customers, as suggested in (B) will not only not add to the number of shrimps that
(A), is the best way to achieve this purpose. So, (A) is the migrate into the sea, but may even reduce the shrimp harvests
answer. in the sea still further, because of the limitation on the number
The proposed policy is specifically intended to cover the of fishing trips.
services required by elderly people who are likely to suffer (E) is wrong, because there has been no ban at any time
from diseases that affect the elderly. The fact that a person against recreational fishermen moving out into the open sea,
had not suffered serious diseases as a child does not mean and so the question of allowing them to move into the South
that, when he grows old, he will not be affected by diseases Atlantic now does not arise.
which generally affect the elderly. So, (B) will have no impact So, the answer is either (C) or (D).
on the losses of the insurance companies either way, and is The passage says that young shrimps usually live in
not the answer. estuaries where they mature, and move into the sea after
(C) and (D) will add to the losses of the companies if their they mature. So, the best method to increase the shrimp
pay-out continues to be more than their premium income. harvest in the sea is to allow all young shrimps to mature in
The question whether the insured persons are rich or poor the estuary and then migrate to the sea, and not allow any of
will make to difference to the quantum of losses incurred by them to be caught by the recreational fishermen. Since even a
the company if its pay-outs are more than its premium fully grown shrimp is only about 4 inches long, the young
income. Moreover, the very purpose of insurance is to help and immature ones will be much shorter. So, a regulation
persons with less income to get the best medical benefit. So, that recreational fishermen should use only large mesh nets
(E) is also wrong. (meaning nets with large holes) in estuaries will help all the
young shrimps to escape from the nets, so that they can
3. The second sentence of the given narrative asserts that subsequently mature and migrate into the sea. So, (C) is the
both a rising standard of living and a balanced trade are best method to help increase the shrimp harvest in the sea,
required for establishing a country’s ability to compete in the and is the answer.
international marketplace. It is only after the fish that are caught are landed that the
We have been asked, based on the above, to identify that recreational fishermen would know what types of fish or
choice which would form a proper test of the country’s ability shrimps have been caught in his net. By that time the fish or
to be competitive. the shrimps would have died. So the suggestion in (D) of
Obviously, the proper test is the country’s ability to achieve ‘putting an upper limit on the size of the shrimp that
both a balanced trade and a rising standard of living recreational fishermen are allowed to catch’, by itself, does
simultaneously. (A) itself states this, and is the answer. not make sense. In fact, (C) mentions a practical way of
All the other choices lack either one or both of these criteria, implementing the suggestion in (D), and is the better answer.
and are wrong.
6. The passage says that the biggest threat to the burn
4. There are two ways of selling beverages (i) in throwaway patients is from various types of bacteria. Therefore the best
bottles, and (ii) in recyclable containers (meaning containers protection is to keep the patient away from any possible
(such as bottles) which can be collected back from the bacterial infection.
consumers and used more than once). So, keeping the burn patients in a maximally sterile
US laws require that beverages must be sold only in environment is the most effective precaution to enhance the
recyclable containers. Some people object to this law on the effectiveness of the standard treatment.
ground that this law makes the manufacture of beverages (E) is what states this, and is the answer.
more expensive, because the collection and transport of the While the suggestions in the other four choices are also
used bottles from the consumers requires gasoline (petrol), desirable ones, they fall short of meeting the ‘biggest threat’
and washing them clean requires water. to the burn patient, namely from the bacteria.
201
American Education Aids - LSAT - Logical Reasoning - Analysis
purchasers of the coupons can still use them on the permitted
7. The given passage means that, if the cost of production days, which also happen to be the busy weekdays.
of a radio in country Y is $100 and its cost of production in (D) and (E) will not also eliminate the problem, because
country Q is only $90, then, even after the transportation fees the illegal purchasers of the coupons could still use them
and tariff charges are added, the radio produced in country subject to these conditions.
Q can be sold in country Y for less than $ 100.
We have been asked to spot that choice which is best 10. We can understand from the given passage that,
supported by the given information. normally, a resident of State X has to pay income-tax for each
The given information means that the tariff charges on year at a specified rate of the total taxable income earned by
the radio produced in country Q, but sold in Y, is less than him during that year. But, if his income in a particular year
$10 (which is 10% of its cost of production in country Y). has exceeded his average income of the previous three years
It is (C) which says this, and is the answer. by over 40%, he has to pay a lower rate of tax on a part of his
income, and the prescribed higher rate only on the remaining
8. The problem faced by the ocean freight industry, part. Such a person will therefore have to pay less tax than
according to the given passage, is that, in spite of the increase another person who had the same amount of income during
in the speed of ships and their reduced fuel consumption while that year, but which did not exceed 40% of his average income
at sea, the industry’s economics has grown worse. during the previous three years. This procedure is given the
The author of the passage says that this is because the name “income-averaging”.
real cost of operation of the ships is not incurred while they We have been asked to identify that group among the given
are travelling in the sea, but when they are remaining idle in choices who would be most seriously affected if the procedure
the ports during loading and unloading of the cargo. of income-averaging is discontinued.
We have been asked to spot that choice which is likely to We should, therefore, look for that choice which mentions
solve the problem faced by the industry. a group of persons whose income in the current year is at
Obviously, the effective solution must be one that reduces least 40% more than their average income during the previous
the time spent by the ships at the ports during the loading three years.
and unloading operations. In the case of the person in (A), this year’s income will be
Among the choices, it is only (B) which would result in an less than the average of the previous three year’s income. Such
improvement in the speed of the loading and unloading of persons will not be entitled to the benefit of “income-
cargo, thereby reducing the time in the port, and is the answer. averaging”, and its discontinuance will therefore have no
The actions in the other choices will not result in the adverse effect on them. So, (A) is not the answer.
reduction of time spent in ports which has been identified as In the case of the persons mentioned in (B), the current
the major cause of the industry’s unsatisfactory economics, year’s income will be the same as the average of the previous
and will not lead to a solution to the problem faced by the three year’s income, and they will therefore not be entitled to
industry. the benefit of “income-averaging”. So, its discontinuance will
not have any adverse effect on them either. So, (B) is not the
9. Bravo Airlines grants free-travel coupons to frequent answer.
travellers as an incentive to retain them as its clients, and (C) refers to persons whose income this year is 100% more
expects the recipients of the coupons to use them for their than their average income during the previous three years.
own flight trips. If these coupons are sold to others at a price So, they will be eligible for the benefit of “income-averaging”,
less than the actual airfare, Bravo Airlines loses its revenue, and its discontinuance will result in their having to pay the
since these travellers would otherwise have purchased tickets higher rate of tax on their full income for the year. So, the
from it paying their full price. group in (C) will be most adversely affected by such
The obvious way of discouraging the sale of such coupons discontinuance. So, (C) is the answer.
by their recipients to third parties is to stipulate that the (D) refers to persons whose income this year is less than
coupons can be used only by those to whom they were awarded, their average income during the previous three years. So, they
or by their close relatives. It is (B) which states this, and is will also not be entitled to the benefit of “income-averaging”,
the answer. and its discontinuance will not affect them adversely. So, (D)
(A) will make the airline unpopular with its regular is not the answer.
passengers, while not eliminating the misuse of the free The current year’s income of the group of persons in (E)
coupons awarded. So, it is not only not a solution to the will be equal to their average income during the previous three
problem, but may result in Bravo Airlines losing even some years. So, the “income-averaging” does not apply to them, and
of its regular frequent-flying clients. its discontinuance will not affect them adversely. So, (E) is
(C) also will not eliminate the problem, because the illegal not the answer.

202
Analysis - Chapter 9
'RELEVANCE' QUESTIONS
SET 1 in measuring the productivity of the postal workers. So, we
Answers can say that he will doubt the truth of the statement that the
quality of the service rendered can be ignored while computing
1.C 2.D 3.B 4.A 5.E 6.C 7.E 8.D 9.E 10.B
productivity. So, (D) is the answer.
Analysis
If the author had doubted the truth of the proposition that
1. (‘Custom-made’ means ‘made especially to suit a
postal workers are representative of service workers in general,
particular person after taking his measurements’ as contrasted
he would not have cited the example of postal workers for
with ‘readymade in standard sizes’.) The first part of the
asserting his view that correctly measuring the productivity
narrative says that custom-made prosthetic bone
of service workers is complex. So, (A) is not the answer.
replacements (produced through a computer-aided design
If the author had doubted the truth of the statement that
process) cost more than twice as much as ordinary
the delivery of letters is the primary activity of the postal
replacements.
service, he would not have said that losing of letters and
The second part of the narrative says that in spite of such
delaying the delivery of letters should also be taken into
higher price, the custom-made replacements are still cost
consideration while assessing the productivity of postal
effective, because they last longer, thereby reducing the need
workers. So, (B) is not the answer.
for further hospital stays.
By citing the example of postal workers as a category, and
We have been asked to identify that choice which contains
not of individual postal workers, the author shows that he
a proposition that should be studied in order to evaluate the
believes, and does not doubt, the statement that productivity
argument given above.
should be ascribed to categories of workers and not to
We can surmise from the given narrative that both the
individuals. So, (C) is not the answer.
custom-made replacement and the ordinary replacement of
The author does not imply that the number of letters
the prosthetic bone last for only limited periods (the former
delivered is not at all relevant to measuring the productivity
for a longer period than the latter), and that the patient will
of postal workers. He only implies that this is only one
have to undergo repeated periodical surgeries for implanting
measure of their productivity, and that two other measures
a fresh replacement. So, if the custom-made replacement is
which should also be taken into consideration are the number
considered to be still cost-effective in spite of its price being
of letters lost and the number of letters delayed. So, we cannot
more than double that of the ordinary replacement, its higher
say that his argument is based on doubts about the truth of
price must be offset by a considerably less number of
(E). So, (E) is not the answer.
additional times that the patient will have to undergo the same
surgery, and be hospitalized therefor. Among the choices, it
3. The first part of the given statement only talks of the
is (C) which states this, and is the answer.
percentage rates of decline of the costs of the logic device and
(A) does not distinguish between custom-made replacement
memory device. On this basis, the conclusion is arrived at
and ordinary replacement, and is not the answer.
that, at the end of three years, the cost of the average
The author’s argument is based on the present high price
computer memory device will have declined by a greater
of the custom-made replacement. (B) and (E), which talk of
amount than the cost of the average computer logic device.
possible future reduction in these prices, are not relevant to
We have been asked to spot that choice which represents
the evaluation of his argument.
an information that is relevant for evaluating the correctness
(D) talks of the relative care with which the ordinary
of this conclusion.
replacements and the custom-made replacements are
If the initial costs of these two devices were $1000 and
produced, and is not relevant to the argument of the author
$10 respectively, the decline in their costs will respectively
regarding the cost-effectiveness of the latter. So, (D) is wrong.
be $(250+187.50+140.64) and $(4+2.40+1.44). In this case,
the former will be greater than the latter.
2. The author first makes an assertion that correctly
If, on the other hand, their initial costs were $10 and $1000
measuring the productivity of service workers is complex. As
respectively, the decline in their respective costs will be
an illustration, he points out that postal workers are often
$(2.5+1.87+1.40) and $(400+240+144). In this case, the former
considered to be more productive if they deliver more letters
will be less than the latter.
per postal worker.
Therefore, unless the actual initial prices of the two devices
He questions the validity of this theory by pointing out
are known, we cannot evaluate the correctness of the given
that it may so happen that, at the same time that a postal
conclusion.
worker delivers more letters than the norm (and is therefore
So, (B) is the answer.
considered productive), he may also lose more letters than
the norm, or may delay more letters than the norm, and
4. The fact that a majority of the population have expressed
therefore cannot be considered to have become more productive.
a particular view can only mean that the legislators, when
We have been asked to identify that choice whose truth is
they vote in favour of that view, are being responsive to the
considered as doubtful by the author while raising the above
will of the public. But the passage also adds “without
objection.
alienating any special interest groups”.
After stating that it is generally believed that the
This will be true only if the special interest groups were
productivity of postal workers can be measured merely by
not part of the 17% who expressed themselves against the
the number of letters delivered per postal worker, the author
renewal of the Act, but formed part of the 80% who were in
implies in the second part of the narrative that the quality of
favour of the renewal.
the service (which is reflected in less number of letters lost,
On the contrary, if the ‘special interest groups’ were part
and less number of letters delayed) is also equally important
203
American Education Aids - LSAT - Logical Reasoning - Analysis
of the 17% who had expressed themselves against the renewal attending secretarial schools is beneficial to one’s health. So,
of the Act, then the legislators who voted for the renewal will the answer to the question in (C) would be very relevant for
be alienating them. an evaluation of the conclusion in the given narrative. So, (C)
So it is (A) which will be useful to us in evaluating the is the answer.
logic of the given argument, and is the answer. (A) refers to the average age at which students of secretarial
The issue is regarding the renewal of the Act for future schools and students of high schools graduate. Knowing the
enforcement, and so, the period it has already been in force is answer of this question cannot by itself enable us to evaluate
of no relevance to the argument. (B) is therefore wrong. the conclusion that attending secretarial schools enables one
Since the economic effect of the Act is not even referred to to live longer. So, (A) is not the answer.
in the passage, (C) is not the answer. The information whether some secretarial school graduates
Even if it is assumed that the author may be trying to subsequently get college degrees will not, by itself, enable us
influence the voting of some legislators, this assumption does to evaluate the claim that attending secretarial school enables
not help us in evaluating the logic of the argument. So (D) is one to live longer. So, (B) is not the answer.
wrong. The given narrative is based on information about the life
(E) would have been a valid answer if the first sentence of expectancy of secretarial graduates vis-a-vis that of graduates
the passage had stated that 80% and 17% of those who from other high schools, and is not based on information about
responded to the survey had voted in favour of and against the life expectancy of women secretarial graduates vis-a-vis
the Act respectively. But this sentence says that 80% and that of other women. So, any additional information on “what
17% of the public gave views as mentioned. So it is only 3% of proportion of all women attend secretarial school?” is not
the public who chose not to respond. So, this small percentage relevant for the evaluation of the given conclusion. So, (D) is
will not affect the validity of the given argument. not the answer.
The given conclusion is based on a differentiation between
5. (Parakeet is a kind of parrot.) From the statement that, secretarial school graduates and high school graduates. So,
in a census of parakeet owners in selected comparable the information on the proportion of high school graduates
countries, the United States ranked second in the percentage going to secretarial schools subsequently is not relevant to
of population owning parakeets, the conclusion is drawn that the given conclusion. So, (E) is not the answer.
‘people’ in the United States are more likely to own parakeets
than are people in most other countries’. 7. From the fact that several of the largest senior citizens’
We have been asked to locate that choice, the information organizations (namely, organisations which represent the
in which will be most useful in judging the accuracy of the interests of old people) are constituted almost exclusively of
conclusion. middle-class elderly people, the critics hold that these
Note that the census was not taken in all, or even in most, organizations are not likely to speak out on behalf of the needs
other countries, but only in selected comparable countries of economically disadvantaged (or poor) elderly people.
(whose number is not given). Therefore, it is not possible to We have been asked to identify that choice which would
straightaway arrive at the conclusion that people in the substantiate, or strengthen, this criticism of the attitude of
United States are more likely to own parakeets than are these organizations.
people in most other countries. To judge whether this We should therefore look for that choice which would imply
conclusion is true or not, we will have to know the pattern of that (i) the needs of middle-class elderly people are different
ownership of parakeets in countries not included in the census from the needs of economically disadvantaged elderly people;
already taken. It is (E) which states this, and is the answer. and (ii) an organization which consists predominantly of
You can easily see that the other choices are not relevant middle-class elderly people is not likely to speak out on behalf
to the conclusion that people in US are more likely to own of the needs of the economically disadvantaged elderly people.
parakeets than are people in most other countries, and cannot If (E) is true, it implies that middle-class elderly people
help us to evaluate this conclusion. have joined these organizations in a big way because the
purpose of these organizations has been perceived to be to
6. (In this narrative, the word “graduate” does not mean a further the economic, political or social interests of middle-
“degree holder”, but a person who has passed out of school.) class elderly people. This choice will also imply that
From the result of a survey which found that, on the average, economically disadvantaged elderly people have not joined
students who had attended secretarial schools lived four years these organizations because they felt that these organizations
longer than those who had attended other high schools, the do not advance their economic, political or social interests.
author comes to a possible conclusion that attending So, it is (E) which would substantiate the criticism that the
secretarial schools is beneficial to one’s health. largest senior citizens’ organizations do not have a
We have been asked to spot that choice the answer to which commitment to speaking out on behalf of the needs of
would be most relevant for evaluating the above conclusion. economically disadvantaged elderly people, and is the answer.
It is known (as is implies by the word “given”) that women Since the passage says nothing about the politics or the
generally have a longer life expectancy than men. So, if a location of the senior citizens’ organizations, (A) neither
preponderant proportion of secretarial school graduates are substantiates nor weakens the criticism against them, and is
women, then the phenomenon of secretarial school graduates not the answer.
living longer may merely be due to the fact that most of them The passage implies a distinction between the needs of
are women. In this case, the conclusion mentioned in the middle-class elderly people and the needs of the needs of poor
narrative, which attributes their longevity only to the fact elderly people. It does not imply a distinction between poor
that they had gone to secretariat schools, would be baseless. elderly people and other poor groups within society. So, (B)
On the other hand, if a majority of secretarial school graduates also does not either substantiate or weaken the criticism in
are men, the phenomenon of secretarial school graduates the passage, and is not the answer.
living longer could possibly reinforce the conclusion that (C) talks of the bargaining strength of groups vis-a-vis that
204
American Education Aids - LSAT - Logical Reasoning - Analysis
of individuals, and does not explain why an organized group relevant consideration for the union for going ahead with the
of middle class senior citizens cannot speak out on behalf of proposed boycott, and is not the answer.
poor senior citizens. So, (C) is not the answer. Gasco customers are not directly involved in the dispute.
(D) is a fact that can be inferred from the given passage, So, if they cannot easily obtain their needs of gasoline
and does not strengthen a criticism of the attitude of the elsewhere, they are not likely to participate wholeheartedly
organizations. in the boycott and, even if they participate initially, the boycott
is likely to fizzle out soon. So, (B) is also a relevant
8. From the fact that, during the period of the Second World consideration for the union leaders to decide whether they
War, 375,000 civilians had died at home as against only should embark on the proposed boycott. So, (B) is not the
408,000 US army personnel who died overseas, the conclusion answer.
is arrived at that it was not much more dangerous to have If other miners’ unions have won contracts similar to the
fought in the war than to have stayed back in US. one proposed by this union, it implies that the union’s demands
The basic absurdity in this argument is that the number are not irrational or without precedent, and therefore, the
of 375,000 civilian deaths was out of a total population of users of gasoline, who are not directly concerned with the
USA running into a few billions (of whom a sizeable number dispute, will lend the union moral support by agreeing to the
might have died of old age), while the number of 408,000 boycott. So, (C) is also a relevant consideration for the union
armed forces deaths was out of a total armed strength of just before finally deciding on the boycott.
a few millions (almost all of them in their prime age) proving If other unions have adopted a similar strategy earlier
that the risk to life was much greater in the army abroad against their managements and have been successful, such
than in civilian life at home. success is also a relevant and encouraging consideration for
In other words, while interpreting such figures, it is not the union leaders for deciding in favour of the proposed
the absolute numbers of deaths that must be taken as the boycott. So, (D) is not the answer.
criterion, but the number of deaths per thousand of each Obviously, the question whether other corporations which
group. own coal companies also own coal stations is a consideration
It is (D) which states this, and is the answer. that is totally irrelevant to the proposed boycott of Gasco’s
The other choices do not bring out the absurdity of the gas stations. So, (E) is the answer.
argument so forcefully.
10. We learn from the passage that there had been an
9. What we learn from the passage is that Energy accident in the factory due to a problem in the control panel,
Incorporated is a corporation which owns both Coalco, a coal and that the problem had been mentioned earlier in a safety
mining company, and Gasco, which owns gasoline stations. inspector’s report.
The miners’ union has been on strike because the But the company says that it should not be held responsible
management of Coalco has not agreed to the union’s new for failing to correct the problem, because Industry Standard
contract proposal. The union leaders are therefore No. 42 requires corrective action only when an accident arising
contemplating organizing a consumer boycott against Gasco’s out of the problem is foreseeable.
gasoline stations in order to pressurize the parent corporation We have been asked to spot that question among the
to force the management of Coalco to agree to their terms. choices, the answer to which is most relevant to decide whether
Of the five choices, four are relevant considerations for the the company had violated Industry Standard No. 42.
union leaders to judge whether a boycott of Gasco’s gas Obviously, the question must be the one that asks whether,
stations will lead to the acceptance of their demands by the in this particular case, the problem in the control panel that
Coalco management. One of the choices is not relevant, and was pointed out by the safety inspector was of a type that is
we have been asked to identify this choice. known to indicate that an accident is likely to happen if it
If the revenue losses suffered by Gasco because of the was not immediately corrected.
proposed boycott seriously affect Energy Incorporated, the It is (B) which asks this question and is the answer.
parent organization, it will sooner or later compel Coalco You can easily see that none of the other choices are
management to yield to the union’s demands. So, (A) is a relevant to Standard No. 42.

205
Analysis - Chapter 10
'ARGUMENTATION' QUESTIONS
SET 1 We have been asked to spot the best logical evaluation of
Answers Edward’s response to Teresa’s statement from among the
choices.
1.C 2.C 3.D 4.A 5.B 6.A 7.B 8.C 9.C 10.D
The statement of Edward neither disproves nor supplies
Analysis
evidence against what Teresa says.
1. We learn from the passage that the international price
So, (A) and (B) are wrong.
of copper is less than the price at which US copper mining
While safety is important in all endeavours, we cannot
companies are able to sell their copper. The mining companies
say that an enterprise must be definitely undertaken,
therefore argue that the Government should restrict the
irrespective of its cost, just because it is safe. So, we cannot
import of copper from abroad so that the users of copper will
conclude that Edward’s statement outweighs Teresa’s
be forced to buy local copper at the higher prices, since
argument. So, (C) is wrong.
otherwise the US mining companies will not be able to stay
(D) is the answer, because, as pointed out above, Edward
in business.
does not meet Teresa’s point that the cost of manned space
On the other hand, a major consumer of copper, namely
flights is their greatest impediment and he assumes that, so
the copper wire manufacturers (who purchase as much as
long as their safety record holds, it is desirable to undertake
70% of all the copper mined by US companies) argue that, if
them.
they are forced to pay a price for local copper which is higher
Teresa’s argument is about the inherent cost of a manned
than the international price, they will not be able to compete
space flight, irrespective of whether it is undertaken with
in the international market for exporting the copper wire
government or with private funds, and she does not refer to
manufactured by them. They will, then, be compelled to reduce
the national budget at all. So, there was no need for Edward
their purchase of raw copper from the US copper mining
to argue as to whether space flight should have priority over
companies even below the present levels.
other items in the national budget. So, (E) cannot be
If both these statements are factually correct, the
considered to be a logical evaluation of Edward’s argument.
argument of the copper wire companies means that the
proposal of the mining company will not only not benefit the
4. The point made by the author in the first two sentences
mining company, and will, on the contrary, have a negative
is that government programs reduce unemployment in two
effect on the mining company’s own existing business.
ways; (i) directly through the jobs created in government itself,
(C) is what states this, and is the answer.
and (ii) indirectly through the jobs created in the private
sector as a consequence of government programs.
2. The passage is: "Many pregnant women suffer from
To prove his point, he gives the example of government’s
vitamin deficiency, but this is not due to vitamin deficiency
mass-transit project which produces jobs not only in the
in their diets; most often,it is because they have higher
project itself, but also in the private sector which has to
requirements for vitamins than do the rest of the population".
produce the tools required for the project.
We have been identify the best criticism of this reasoning
He thus cites a case in which there is a relationship between
from among the given choices.
a cause (mass-transit project), and its effect (jobs created in
(A) is wrong, because the reasoning in the passage does
the private sector). So, (A) is the answer.
not require the specification of the exact percentage of
pregnant women who suffer from vitamin deficiency.
5. The author’s argument is that, since intellectuals no
(B) is wrong, because the reasoning in the passage does
longer act as advocates for oppressed groups, they can be
not require information about why pregnant women require
considered as having ceased to be ‘clarifiers of fundamental
higher does of vitamins.
moral issues’.
(D) is wrong, since the passage is only about pregnant
Obviously, he closely links the clarification of fundamental
women, and has no information about other groups which
moral issues with the advocacy on behalf of oppressed groups.
also require higher vitamin dosages.
So, (B) is the answer.
Since there is no ambiguity about the phrase ‘higher
(A) appears tantalizingly as the correct answer, but the
requirements’, (E) is also wrong.
equation of intellectualism with clarification of fundamental
So, by elimination, (C) is the answer. The phrase ‘vitamin
moral issues is the ‘widely held perception’ but the author
deficiency’ is used twice in the same sentence. In the first
differs from it. The question we have been asked is on the
instance it is used to describe pregnant women having less
author’s view, and not on the view of the general public. So,
vitamins than what they themselves need. In the second
(A) is not the answer.
instance it is used to describe diets which have less vitamins
The other choices do not explain the logical structure of
than what their consumers need. Thus the passage fails to
the passage.
employ the same reference group for both uses of this phrase,
and is its flaw.
6. The author calls the view of some people that the
economic problems of Western Europe have been caused by
3. Teresa’s argument is that manned space flight is too
the Organization of Petroleum Exporting Countries (OPEC)
expensive, and that the objectives of manned space flights
as nonsense, and points out that, though Great Britain is
can be achieved more economically through cheaper
not dependent on OPEC oil, it also suffers the same economic
alternatives.
problems that afflict France and West Germany.
Edward does not touch upon the cost aspect of manned
We have been asked to identify the tactic which the author
space flight at all, but says that manned space flight has a
employs to make his point.
very bright future because it is relatively safe.
206
American Education Aids - LSAT - Logical Reasoning - Analysis
Obviously, the author seeks to make his point by citing 9. The argument consists of only three sentences, and we
Great Britain as a counter-example of a country which is not have been asked to identify the logical weakness in the
dependent on OPEC oil but still suffers from the same argument. We should therefore see whether the last sentence,
economic malaise that affects two other countries which are which incorporates the conclusion, follows necessarily from
dependent on OPEC oil. So, (A) is the answer. the previous two sentences, and if not, why not.
Except stating that France and West Germany face It is well known that the use of fossil fuels (which mean
economic problems (which are similar to those of Great coal and petroleum products) has been ever on the increase,
Britain), the author does not analyze these difficulties. So, and is likely to go on increasing with the increase in
(B) is wrong. population, and with more and more countries on the road to
The author uses just a fact to refute the views of some development. It is also known that the consumption of these
people, and does not point out any misconception in the fuels does produce carbon dioxide. What the ecologists say is
reasoning of those who hold the contrary view. So, (C) is that increased emission of carbon dioxide may result in
wrong. increase in the temperature of the atmosphere.
Since no alternative explanation for the economic Thus, while the ecologists’ prediction is that increased use
difficulties of Great Britain, France or Germany is given in of fossil fuel can result in increase in the atmospheric heat,
the passage, (D) is wrong. the author goes many steps further and concludes that it is
(E) is wrong because France and Germany are just paired only the consumption of fossil fuels that can lead to the
together, and no ‘analogy’ is drawn between them. increase in atmospheric heat. The author does not consider
the possibility, for example, of the atmospheric heat getting
7. The first sentence says that Jones’s new book contains increased as a result of other causes such as repeated nuclear
information that is likely to destroy the reputation of persons tests.
who have held high government responsibility during a period Thus, it is (C) which points out the weakness of the author’s
of national crisis. argument, and is the answer.
Any person who is interested in protecting the reputation (You can also see that the other four choices are views of
of these persons must therefore try to prove either that (i) the ecologists, and not arguments of ‘the author’.)
the information contained in the book is not authentic, or (ii)
that the conclusions drawn from the information are not 10. The first recommendation of the readers, namely,
logical. “newspapers should emphasize events closer to the readers’
But the author of the letter does not talk about the merits lives” really means that newspapers should not be satisfied
of the book at all, but says that Jones’s criticisms should be with purveying international and national news alone, and
dismissed only because of the two reasons that (i) Jones is that sufficient importance should be given to the coverage of
known to be generally anti-government, and (ii) that he himself local news also.
has never been in a responsible position. The second recommendation is that newspapers should
Thus, the letter-writer assumes that personally attacking also periodically publish articles about the reporters
the author of the book is sufficient to invalidate the information themselves.
in his book. This is the questionable technique followed by This has been misinterpreted by the author of this passage
the author. to mean that the readers wanted articles about events in the
It is (B) which states this, and is the answer. readers’ own personal lives to be written by the reporters,
(A) is wrong, because the term “responsibility” has been along with embellishments of the reporters’ own
employed in the same sense in both the places where it occurs autobiographical details.
in the passage. Thus, (D) is the answer.
(C) is a factually incorrect statement, because the letter-
writer merely says that Jones’s criticisms will convince “only SET 2
people like himself”, and does not say that such people are in Answers
a majority.
1.A 2.C 3.C 4.C 5.D
The question is about the argument contained in the letter,
Analysis
and has nothing to do with the person to whom the letter is
1. The first sentence merely states that a program of steady,
addressed. So, (D) is wrong.
moderate exercise coupled with a diet that is low in saturated
The passage refers neither to a “cause” nor to an “effect”,
fats and cholesterol has been associated with reduced risk of
and therefore, the question of a confusion between the two
heart attacks and strokes.
does not arise at all. So, (E) is also wrong.
From this premise, the author comes to a sweeping
conclusion that if one exercises moderately and eats only foods
8. What the author says is that, in the same manner that
that are low in saturated fats and cholesterol he will not get
a mechanical system such as a watch will be damaged by
a heart attack or stroke at all.
interfering with its working, living organisms too will be
By using the phrase ‘reduced risk’, the first sentence, from
damaged when subjected to technological alteration.
which the author draws his conclusion, does not itself rule
He is obviously drawing an analogy between a natural
out the possibility of some people getting heart attacks and
system and a mechanical device. So, (C) is the answer.
strokes regardless of the diet and the level of exercise. The
(A) is obviously wrong, because opinions of informed
author ignores this possibility while coming to his
observers are not referred to at all in the passage.
unwarranted and sweeping conclusion.
The analogy of a watch, cited in the passage, is not
(A) is therefore the best criticism of the author’s conclusion,
representative of a natural system, but is representative of a
and is the answer.
mechanical system. So, (B) is also wrong.
You can see that none of the other choices points out the
(D) is wrong for the same reason.
flaw in the author’s conclusion that he arrives at based on
(E) is far off the mark, and can be rejected straightaway.
207
American Education Aids - LSAT - Logical Reasoning - Analysis
the information in the first sentence alone. (D) and (E) are wrong because Miko’s argument is neither
circular nor a generalization from an atypical case, and Kofi
2. From the statistics that the number of farms in Otsego does not attempt to establish it to be one of these.
county had come down from 800 in 1850 to 81 in 1988, the
author comes to the conclusion that the land in the county 4. X says that people respond irrationally when a serious
that is devoted to farming has dropped by about 90 percent but rare industrial accident occurs by believing that such
in the past 140 years. accidents are becoming more frequent. He draws the analogy
The author’s conclusion would be wrong if the 800 farms of a person who has been dealt four aces in a hand of poker
in the county in 1850 had an average extent of only 10 acres, (which is a very rare event) believing that he will be dealt
while the average extent of the 80 farms in 1988 was 100 such similar hands quite often in the future.
acres each. In this case, there would have been no reduction Y responds to X’s claim by stating that, to the contrary, the
in the land that is devoted to farming in that county. belief of the people (that such accidents are becoming more
So, the major flaw in the author’s argument is that he frequent) is rational because it ‘results in their sensing a
ignores the possibility that the average size of the farms might danger to themselves not sensed before and taking
have changed during this period. precautionary actions to prevent similar accidents in the
(C) is what states this, and is the answer. future’.
We are asked to spot that choice which best describes how
3. Miko argues that since any computer software developed Y responds to X’s claim.
by the faculty of the university is the result of their work for Since, in his response, Y does not even refer to the analogy
which they have already been paid salary, the copyright to of poker cards used by X, (A) is wrong.
the software must belong to the university and not to the Y does not say that people vary in their responses to a
faculty. serious accident. So, (B) is wrong.
Kofi responds to this argument by stating that a copyright While X says that it is irrational to believe that a rare
policy which is so restrictive can impede the university’s accident, once it has occurred, will occur more frequently in
primary mission of generating and disseminating knowledge. future, Y says that, once a rare accident has occurred, it is
We have been asked to spot that choice which mentions only rational on the part of people to take precautionary action
the relationship of Kofi’s response to Miko’s argument. to prevent similar accidents in the future. So, Y is trying ‘to
(A) is not the answer because Miko himself presents only shift the basis for judging rationality to considerations of
an argument and does not provide evidence for it; nor does utility’, as stated in (C), which is the answer.
Kofi contradict him.
(B) is wrong because neither does Miko’s argument have a 5. After stating first, “The burden of taxation on the back
hidden assumption nor does Kofi points it out. of the people is not unlike the burden of weight on the back of
(C) is the answer because what Kofi does is to point out a a horse”, the author proceeds to explain why it is so. Obviously,
problematic consequence (of the university not fulfilling its he makes his point using an analogy.
primary mission) if the copyright policy of the university is So, (D) is the obvious answer.
made as restrictive as is suggested by Miko. You can easily verify that none of the other choices is apt.

208
Analysis - Chapter 11
'PARADOX' QUESTIONS
SET 1 consumption of a half-empty freezer is more than the energy
Answers consumption of a fully stocked freezer.
1.C 2.A 3.E 4.D 5.B 6.C 7.E 8.B 9.C 10.B (D) is what can be inferred from the statement in the
Analysis passage, and does not explain the statement itself.
(E) would contradict and not explain the author’s statement
1. The passage states that, in spite of a sharp reduction in
that fully stocked freezers consume less energy than half-
the average weekly volume of trade for many weeks after
empty freezers and are therefore more energy-efficient.
October, 1987, as compared to the weekly average of 1986,
there was no appreciable shortfall in the annual volume of
3. Car insurance claims are paid on two counts: (1) to
trade in 1987 as compared to the annual volume of trade in
reimburse the cost of treatment of injury to the victims; and
1986.
(2) to reimburse the cost of repairs to the vehicles involved in
We have been asked to spot the choice which would explain
the accident.
this apparent contradiction.
The narrative says that, in 1960, only 10% of the claims
This question is based on a simple principle of arithmetic
was paid towards treatment of injuries but, in 1990, the
and averages. If the total sales volume in 1987 had been equal
percentage went up to 50%, even though the cars were much
to the total sales volume of 1986, the weekly average sales
safer in 1990 than in 1960.
volume in 1987 must have been equal to the weekly average
We have been asked to locate that choice which explains
sales volume in 1986.
this discrepancy.
So, if the weekly average had been less than this figure in
(A) does not explain the discrepancy, because fewer
some weeks, it must have been compensated by the weekly
accidents must normally have resulted in less, and not more,
average being more than this figure in some other weeks. It is
percentage of claims on injuries.
(C) which states this, and is the answer.
(B) would also imply that the insurance compensation for
(A) contradicts the information in the passage that sales
injuries must have been less in 1990 and not more. So, (B)
volume of stocks dropped in spite of a drop in their prices. It
does not explain the discrepancy noticed.
does not explain how the annual volume remained the same
(C) would imply that the percentage of compensation
while the weekly volume dropped in a few weeks. So, (A) is
claims on car repairs, between 1960 and 1990, must have
not the answer.
grown, and not become less, whereas they had actually got
The passage relates to the volume of stocks traded, which
reduced from 90% to 50%. So, (C) does not explain the
could increase or decrease irrespective of the number of stock
discrepancy.
buyers being the same or different. So, (B) does not explain
(D) does not explain why, between claims on injuries and
the contradiction pointed out by the author, and is not the
claims on car repairs, the proportion of the former had grown
answer.
so fast between 1960 and 1990. So, (D) is not the answer.
Since the conclusion in the passage does not relate to the
If, as stated in (E), health care costs had risen sharply
average price per share, but relates to the volume of stocks
between 1960 and 1990, an injury which cost x dollars for
traded, (D) also does not explain the contradiction pointed
treatment in 1960 would now be costing many times that
out, and is not the answer.
figure. If so, even a lesser number of injuries in accidents
(E) does not specifically say that the volume of trade had
could have resulted a much larger payout because of the higher
risen during a corresponding period in 1987, and does not
cost of treatment. So, (E) gives a possible reason why the
therefore adequately explain the contradiction.
percentage of insurance payouts for automobile accident
injuries could have gone up from 1960 to 1990, in spite of
2. The author first says that many people keep their electric
cars having become much safer during the period. So, (E) is
freezers half-empty on the assumption that the process of
the answer.
freezing food consumes energy.
He then points out that freezers that are half-empty often
4. The paradox that is to be explained in this case is why a
consume more energy than they would if they were kept fully
community which consists of retired people and very few
stocked.
families with small children should actively patronize
We have been asked to spot that choice which explains
businesses renting out furniture for infants and small
this apparent discrepancy.
children.
When only half of a freezer is filled with food, the other
Among the choices, the obvious answer is (D) which states
half is not really empty, but is filled with air. (A) states that a
that many residents of these communities must provide for
given volume of air requires more energy to be maintained at
the needs of visiting grandchildren several weeks a year.
a below-freezing temperature than a corresponding volume
None of the other choices explains why families with no
of frozen food. If this is true, it perfectly explains why freezers
children should frequently hire furniture intended for
that are half-empty often consume more energy than freezers
children.
that are fully stocked with food. So, (A) itself is the answer.
(B) is a scientific fact, but is applicable to both fully stocked
5. Two personal battle armours of King Henry VIII, made
and half-empty freezers. It does not explain why a half-empty
in 1510 and 1540 respectively, have been discovered, and the
freezer should consume more energy than a fully-stocked
latter weighs 40 pounds more than the former.
freezer.
The five choices give five possible explanations for this
(C) says that the temperature of a given volume of air
discrepancy, and we have been asked to identify the choice
inside the freezer rises only temporarily. It does not explain
which contributes LEAST to such an explanation.
why, even later, after the food has been frozen, the energy
209
American Education Aids - LSAT - Logical Reasoning - Analysis
(A) says that Henry was slim in 1510 and had grown bulky in the number of passenger trips (because a number of persons
later in his life. So, (A) is a rational explanation for the who had earlier been traveling by car or by train to these
difference in weight between the two armours, and is not the places may now be travelling by airplanes), it does not explain
answer. why the increase in the number of passenger trips was less
If, as stated in (C), Henry VIII had started the fashion of than the increase in the number of boardings. So, (A) is not
decorating battle armour with heavy and elaborate metal the answer.
pieces, it would also explain why the 1540 armour was much (B) states that the number of airlines declined significantly
heavier than the 1510 armour. So, (C) is not the answer. between 1977 and 1987. If, in spite of such decline, the
(D) says that Henry ascended the throne in 1510 while number of passengers had increased by nearly 100%, it only
still an adolescent, and grew three inches during his first 5 implies that a number of smaller airlines had been absorbed
years as king. (D) does not constitute a strong explanation as by the bigger airlines. (B) does not constitute a reason why
to why the armour made for him nearly 25 years later should the increase in the number of airline trips was less than the
weigh 40 pounds more than the one made in 1910. So, (D) is increase in the number of airline boardings. So, (B) is not
a possible answer. the answer.
(E) says that, during the 1530’s, all armour was given a (D) would have been the answer, only if it had been
multi-layered design. Thus (E) is also a rational explanation mentioned that business travellers more often take connecting
for the heaviness of the 1540 armour of Henry VIII. flights at intermediate airports than pleasure travellers do.
(B) says that nearer the close of his reign, Henry VIII had Since this information is not given, (D) by itself does not
increased his arsenal (meaning the total inventory of weapons explain in the discrepancy between the increase in the
in his kingdom) because of his growing unpopularity. This number of trips and the increase in the number of boardings.
statement, by itself, is totally irrelevant to why his own So, (D) is not the answer.
personal armour was heavier by 40 pounds. While (E) may be a cause for the increase in the number
So, we have to choose between (B) and (D). Between the of airline passengers, it does not explain the discrepancy
two, (D) at least refers to Henry VIII’s physical dimensions, between the increase in passenger trips and the increase in
and gives information why his armour, made in the beginning passenger boardings. So, (E) is not the answer.
of his reign, must have been quite light - because he was just
an adolescent whose growth had not yet been completed. So, 7. The passage implies that exposure to polychlorinated
we can eliminate (D) and choose (B) as the answer. biphenyls (PCBs) is harmful to health, and causes a specific
disease. It says that, even though exposure of people to PCBs
6. The narrative first says that passenger boardings on had significantly decreased during 1975 to 1985, the incidence
airlines increased by nearly 100% between 1977 and 1987, of the disease caused by PCB exposure doubled from previous
but the actual number of trips made by passengers increased levels, namely the levels before 1975 which were already high.
by only 40% during this period. We have been asked to identify that choice which resolves
The second sentence in the narrative explains this this apparent contradiction in the passage.
discrepancy by pointing out that, in those cases when a (E) says that the time interval between the time of exposure
passenger has to take more than one flight to complete a trip, to PCBs and the onset of the disease caused by that exposure
the trip is counted as only one, but the number of his boardings is often longer than 10 years. If this is true, the higher
is counted as more than one. (For example, if a passenger incidence of PCB-related disease during 1975-1985 must have
making a trip from New York to Chicago flies first from New been caused by the higher level of exposure during the previous
York to Pittsburgh and transfers there to a flight from decade 1965-1975. This satisfactorily explains the apparent
Pittsburgh to Chicago, he is considered to have made only contradiction in the passage, and is the answer.
one trip (from New York to Chicago), but the number of his (A) and (B) are irrelevant to the observed fact that the
boardings is counted as two.) incidence of the PCB-related disease was inversely
We have been asked to spot that choice which best helps to proportional to exposure to PCB during 1975-1985.
explain why the increase in the number of passenger (C) contradicts the information in the passage that the
boardings had exceeded the increase in the number of airline incidence of the disease was already at a high level in the
passenger trips between 1977 to 1987. decade 1965 to 1975.
(C) states that the proportion of passenger flights requiring The information in (D) relates to the period after 1985,
passengers to change planes en route to their final destinations and it does not give information about the level of incidence
increased significantly between 1977 and 1987. (For example, of the disease during this period. So, it does not resolve the
there might have been 10 direct flights between New York apparent contradiction in the passage.
and Chicago before 1977, but this number might have been
reduced to 5 by 1987, forcing many passengers intending to 8. Zooplanktons are minute organisms growing in sea
fly from New York to Chicago to change planes en route at water and constitute the primary food for many varieties of
Pittsburg.) If this is true, many boardings that had been fishes. In this passage, the author gives information about
counted as only one (at New York) before 1977 would be two species of zooplankton, named D-hyalina and D-galeata.
counted as two (one at New York and one at Pittsburg) in The ‘paradox’ mentioned is that while the latter, which lives
1987, while the trip would continue to be counted as only one near the surface of the sea grows and reproduces much faster,
(namely from New York to Chicago). So, it is (C) which best its population is often less than the population of the former,
helps to explain why the increase in the number of boardings which lives near the bottom of the sea and grows and
exceeded the increase in the number of passenger trips reproduces much slower.
between 1977 and 1987. So, (C) is the answer. One possible explanation for this paradox is that the
(A) merely states that the number of airports in USA population of fish varieties which consume zooplankton is
handling flights by major airlines increased significantly more near the surface of the sea than at the bottom. So, (B) is
during the period. While this fact would explain the increase the answer.
210
American Education Aids - LSAT - Logical Reasoning - Analysis
None of the other choices can explain the paradox in the years were women, and all of them have taken to professional
passage. practice, but women constitute only 5% of the total number
of members in the profession now.
9. The passage says that, though the normal rule of We have been asked to spot the choice which could explain
economics is that the demand for a particular item falls when the difference in these two percentages.
its price increases and vice versa, the price of canned soups (B) points out that the majority of those currently in the
rose in USA in 1980 but the demand for it also rose profession had entered it more than twenty years ago when
simultaneously. no woman had been certified. If this is true, and if the career
We have been asked to spot that choice which satisfactorily span in the profession is 30 years or more, it would
explains this apparent contradiction. satisfactorily explain why women constitute only 5% of its
Among the choices, (C) says that the retail prices of other members now. (If the percentage of certification continues to
foodstuffs rose faster than the price of canned soup, and remain at 11%, it would take a period of time equal to the
canned soup therefore became cheaper relative to other total span in the profession for women to achieve the figure
foodstuffs. This, if true, satisfactorily explains the apparent of 11% among the actual practitioners of the profession) So,
contradiction in the passage, and is the answer. (B) is the answer.
All the other choices, if true, should have resulted in (E) makes the further point that, women form only 2% of
reduced demand for canned soups, thereby contradicting the the supervisory positions, but it does not explain why they
information contained in the passage. form only 5% of the overall membership in the profession.
None of the other choices is relevant to the difference in
10. The passage says that 11% of those who received percentages of the women certified and the women who are
certification in a certain profession during the last twenty actually in the profession

211
Analysis - Chapter 12
'ANALOGY' QUESTIONS
pass for it, this flaw is not similar to the one in the given
SET 1 narrative, because there is no violation of a rule or a law in
Answers this example.)
1.B 2.C 3.B 4.B 5.C (C) is not worded correctly in the manner in which we
Analysis understand it in India. In the given context, the meaning of
1. (A hamster is a type of mouse that is kept as a pet. It is the phrase “must not have turned right” is not “should not
also used in medical experiments.) What the result of the have turned right”, but “has not turned right”.) When you
experiment suggests is that variations in the lighting understand choice (C) in this manner, you can realize that
arrangement in the place where sick hamsters were kept had this choice has the same type of flaw as the one in the given
a positive effect on their rate of recovery. narrative because here also the author assumes that no driver
We have been asked to spot that question among the ever breaks a traffic rule. So, (C) is the answer.
choices for answering which a similar method of research will (D) is a logical statement, and there is no flaw in it. So,
be appropriate. (D) is not the answer.
(B) poses the question, “Can hospital lighting be improved (E) is also a logical statement, and there is no flaw in it.
to promote the recovery of patients?”. This question can be So, (E) is not the answer.
answered experimentally by similarly exposing patients to
continuous fluorescent light and observing the result. So, (B) 3. Robots are an example of ‘intelligent’ machines. The
is the answer. given narration first gives the author’s view on a particular
None of the other choices relates to the health of the item (‘industrial robots’), and then generalizes it to the entire
subjects of the experiment, and can be discarded for that class (‘intelligent’ machines) to which that item belongs.
reason alone. Among the choices, (B) has a similar structure because,
the author’s view about a particular item (‘bacon’) is
2. The first sentence says that, after the amendment to generalized to cover the entire class (‘fatty foods’) to which
the copyright law in 1978, books that are less than 50 years that item belongs.
old should not be photocopied without the publishers’ So, (B) is the answer.
permission. The second sentence draws the conclusion that You can easily verify that none of the other choices has a
any book that has been photocopied after1978 without the similar structure.
publisher’s permission must therefore be at least 50 years
old. 4. The first sentence says that more sophisticated artificial
The question says that there is a flaw in the reasoning in intelligence programs can be developed if only computer
the above conclusion, and asks us to spot that choice which scientists knew more about how human beings think.
has a similar flaw in its reasoning. The author counters this view by pointing out the analogy
We should first try to locate the flaw in reasoning in the that airplane design has not progressed very far in spite of
given narrative. Obviously, the flaw lies in the author’s new insights into how birds fly.
assumption that, if a law has been passed, nobody breaks it. Obviously, the author thinks that ‘artificial intelligence
He does not consider the possibility that some persons might programs’ and ‘airplane designs’ are similar, just as ‘how
have photocopied even books which are less than 50 years old human beings think’ and ‘how birds fly’ are similar.
without the publishers’ permission, thereby deliberately So, (B) is the answer.
violating the copyright law. You can easily verify that none of the other choices explains
There is no logical flaw in (A) (though it is factually not the analogy.
correct). So, (A) cannot be compared to the given narrative
which has a logical flaw which we have identified above. 5. The given passage gives two instances of items which
(B) is also a logical and rational statement, and there is no started as a hobby or means of amusement but later found
flaw in it. So, (B) is not the answer. commercial applications.
(Even if you argue that it is not obligatory on the part of Among the five given choices, it is only (C) which gives
Demetrios to ride on a city bus just because he has bought a another such instance, and is the answer.

212
Analysis - Chapter 13
'MULTIPLE' QUESTIONS
Remember that, in questions of this type, the choices in the couples who are readers of Weddings magazine want a ca-
two questions under each passage are independent of each tered reception, the author is assuming that the readers of
other, and you should not assume the truth of a choice in one the Weddings magazine form a representative sample of all
question for evaluating a choice in the second. the prospective wedding couples. If this is not true and the
percentage of those who prefer to have a catered wedding is
much lower among all prospective wedding couples than
SET 1 among the readers of the Weddings magazine, the author’s
Answers conclusion will be wrong. So, it is (D) which reveals a weak-
1.B 2.B 3.D 4.E 5.A 6.D 7.D 8.B 9.C 10.E ness in the evidence cited by the author, and is the answer.
Analysis
1. The meteorologists’ claim is that, if an accurate math- 4. The investment advice given in the passage is based on
ematical model of the atmosphere could be designed, they (i) an opinion poll among the readers of the Weddings maga-
would be able to predict the weather precisely. zine, 70% of whom stated that they wanted a catered wed-
What the author implies in the second sentence is that, ding reception, and (ii) the fact that the existing caterers are
even if such an accurate mathematical model is made avail- capable of serving only 55% of the estimated weddings in each
able to them, the meteorologists may still commit human year.
errors resulting in wrong predictions, and will then put the We have been asked to pick out that choice which would
blame for the errors on the model itself as being inaccurate. undermine the validity of this investment advice.
He therefore concludes that it is not possible to evaluate Since the investment advice does not relate to the number
the claim of meteorologists that they would be able to accu- of guests at each wedding reception, (A) neither strengthens
rately forecast the whether if they could design an accurate nor weakens the investment advice given in the passage.
mathematical model of the atmosphere. Even if (B) is true, it means that 75% of the weddings take
We have been asked to spot that choice which could form a place with a reception while the existing caterers can serve
basis for arguing against the author’s conclusion. only 55% of them. So, (B) will strengthen, and not undermine,
We should therefore look for that choice which implies that the validity of the investment advice.
greater accuracy in the mathematical models has in the past (C) neither strengthens nor undermines the investment
resulted in greater accuracy in weather prediction. advice, because, for the purpose of the advice, it is immate-
(B) states just this, and is the answer. rial as to who pays the bill.
None of the other choices relates the degree of accuracy of (D) is also irrelevant to the soundness of the advice.
the mathematical model with the degree of accuracy of weather If (E) is true and, even if we assume that the readers of
prediction. Weddings magazine form a representative sample of all pro-
spective wedding couples, it means that, though 70% of them
2. Any mathematical model has to depend on quantifiable said that they preferred a catered reception, only 35% of them
data. will actually hire the service of caterers, as against the ca-
So, if some factors which impact on the constitution of the pacity of the existing caterers to cater to 55% of the prospec-
atmosphere are not quantifiable and are liable to arbitrary tive weddings. This implies that the catering business is al-
changes, then no accurate mathematical model can be made ready overcrowded, and that there is no scope for any fresh
that can help meteorologists to predict the weather accurately. caterer to thrive in the business. If so, the advice that now is
It is (B) which states this, and is the answer. the time to invest in catering business will be seriously un-
dermined. So, (E) is the answer.
3. From the facts that (i) the current number of caterers
can serve only 55% of the weddings likely to occur each year, 5. Blood banks normally screen all prospective donors for
and (ii) 70% of the readers of the Weddings magazine want a various infections. We learn from the passage that NANB
catered marriage reception, the author comes to the conclu- hepatitis was not one of the infections for which donors were
sion that “now is an excellent time to invest in the catering screened till now. The first sentence says that screening for
business”. this disease, before a donor’s blood is accepted by the bank, is
We have been asked to locate the statement which is a going to commence soon.
weakness in this evidence. The second sentence says that the new screening test is
(A) does not by itself constitute a weakness, because the not foolproof, and that, even though 15% of all prospective
cost of paying the labour can be passed on to the clients. blood donors would carry the infection, it would detect its
(B) will, in fact, strengthen the given evidence because it presence in only 5% of them.
will give scope to the prospective investor to choose an area The second sentence then forecasts that, therefore, among
where there is a felt and greater need for new caterers. all actual donors of blood, 10% would still pass on the hepa-
(C) implies that the trend is likely to continue, and this titis-contaminated blood.
will strengthen and not weaken the given evidence. We have been asked to identify the assumption that has
(E) means that, in spite of reading articles about prepar- been made while making this forecast.
ing food oneself for one’s wedding reception, most readers of According to the first part of the second sentence, it is 10%
the Weddings magazine prefer a catered reception. This will of all prospective donors who would be cleared for donation in
also not weaken the evidence cited in the passage. spite of their carrying the hepatitis infection. The second part
In stating that the existing caterers can serve only 55% of forecasts that 10% of the actual donors would pass on the
the likely weddings, and that 70% of the prospective wedding hepatitis-contaminated blood.
213
American Education Aids - LSAT - Logical Reasoning - Analysis
This forecast, therefore, equates the prospective donors as compact discs. So, (E) gives a reason why digital tapes
undergoing the hepatitis test (minus the 5% who would be must be available commercially, and does not explain why
disqualified) with the subsequent actual donors. It thereby they are not. So, (E) is not the answer.
assumes that none of those who is cleared in the hepatitis
test will be disqualified in any of the other routine screening 8. This question is the reverse of the last, and asks us to
tests. identify that choice which would weaken the record compa-
It is (A) which states this, and is the answer. nies’ claim that the non-availability of digital audio tapes in
the commercial market is not because of profit-motivated
6. The passage implies that, with the introduction of the marketing decisions by them.
new test to screen NANB hepatitis, an additional 5% of all The record companies’ claim relates only to the quality of
prospective blood donors will be disqualified. reproduction, and does not refer to the prices or the selec-
The immediate consequence of the introduction of the test tions. So, (A) by itself neither refutes nor strengthens that claim,
will therefore be a reduction in the amount of blood available and is not the answer.
with the blood bank. So, (D) is the answer. (B) implies that the recording companies had invested
None of the other choices is a necessary consequence of the substantial amounts in the development of compact-disc
information given in the passage. manufacturing technology, that the profits so far made by
them through the sales of compact discs are less than such
7. The author of the passage states in the first sentence investments and that they are reluctant to introduce the new
that record companies defend the introduction of the com- digital tapes in the commercial market till they recover the
pact discs in place of the cheaper long-playing vinyl records investments on the development of the compact disc technol-
by claiming that “the audio market is ruled by consumer de- ogy. If this is true, it would strengthen the argument of the
mand for ever-improved sound reproduction rather than by author in the second sentence of the passage that the claim
the record manufacturers’ profit motivated marketing deci- of the companies that profit-motivated marketing decisions
sions”. In the second sentence, the author refutes this claim do not govern the audio-market is not true. So, (B) is the an-
by stating that, if this claim were true, the record companies swer.
should have made digital audio tape commercially available (C) is a normal phenomenon arising out of demand and
to the public, because it produces even better sound than the supply, and only strengthens the claim of the recording com-
compact discs. panies that the audio market is governed only by consumer
The author implies thereby that the non-introduction of demand. So, (C) is not the answer.
the digital audio tape commercially is only because it does Neither (D) nor (E) is relevant to the claim of the recording
not suit the profit-oriented marketing interests of the record companies that the audio market is governed only by the
companies. quality of reproduction, and not by profit-motivated decisions
In this question, we have been asked to identify that choice of the companies.
which would reconcile the record companies’ claim (that the
audio market is governed solely by consumer demand for ever 9. From the information about what happened in the past
improving quality) with the fact that digital audio tapes are when women entered the professions of teachers, bank tell-
not available for the commercial market. We should therefore ers and secretaries, the author predicts similar results if they
look for that choice which gives a valid reason, other than enter the professions like accounting, law and medicine in
profit-related interest of the recording companies, for the fact the future.
that digital audio tapes have not yet been introduced by them Obviously, his argument is based on an analogy between
in the commercial market. the past and the future. So, (C) is the answer.
(A) implies that the consumer preference is for audio tapes You can easily see that none of the other choices is appro-
as compared to long-playing records or CDs. If so, according priate.
to the companies’ own statement that the audio market is
governed by consumer preference, audio tapes must have been 10. In this question, we have been asked to spot the choice
made available commercially, which has, however, not hap- which would refute the conclusion that the entry of women
pened. So, (A) does not constitute an explanation as to why into the professions of accountants, lawyers and medicine will
digital audio tape is not available for the commercial mar- result in the incomes and prestige of these professions drop-
ket, and is therefore not the answer. ping, in the same way that their earlier entry into the profes-
(B) mentions the use of digital audio tapes in recording sions of teachers, bank tellers and secretaries led to a fall in
studios, and does not explain why they have not been made the pay and status of those professions.
available commercially. So, (B) is not the answer. (A) refers to what happened in the past, and does not ei-
If (C) is true, and digital audio tape gives better reproduc- ther confirm or refute the author’s conclusion as to what is
tion of music than compact discs, then digital audio tapes likely to happen in the future when women enter these pro-
must have been made available commercially by now. (C) cer- fessions in large numbers.
tainly does not explain why digital audio tapes are not so (B), in a reverse way, will confirm, and not refute the au-
available, and is not the answer. thor’s conclusion that the same occupation will get higher pay
(D) specifically states that record companies have not yet and status when it is male-dominated than when it is female-
solved several quality control problems involved in the com- dominated.
mercial production of digital audio tapes. If this is true, it (C) makes no reference to the occupations of accounting,
will constitute a valid reason why the record companies have law and medicine, and neither confirms nor refutes the au-
not made available digital audio tapes in the quality-conscious thor’s conclusion.
commercial market. So, (D) is the answer. The author’s argument sets no time limit, and merely says
If (E) is true, and digital audio tapes of acceptable quality that, if women become the majority in the accounting, legal
can be produced commercially, they will sell at least as briskly or medical professions at any time in the future, their pay and
214
American Education Aids - LSAT - Logical Reasoning - Analysis
prestige will drop; so, the statement in (D) that women are the assumptions made by the economist while advancing his
likely to become a majority among lawyers only after two dec- argument.
ades neither confirms nor refutes the author’s argument. Only one argument of the economist, namely that govern-
If (E) is true, and the pay and status of females in the new ment insurance is one of the reasons for bank failures - is
professions are governed by significantly different sociologi- mentioned in the narrative. For advancing this argument, he
cal and economic forces than were the earlier professions, then does not have to assume that bank failures are caused only
the analogy drawn by the author will be inapplicable, and the when big borrowers do not repay the loans. The failure could
results predicted by him will not fructify. So, it is (E) which also be due to a very large number of small borrowers not
would refute the conclusion drawn by the author, and is the repaying the loans. So, (A) is not the answer.
answer. (B) also is not a necessary assumption in the author’s ar-
gument.
SET 2 The argument is about all depositors, and is not limited to
Answers only those who deposit big amounts. So, (C) is also not a neces-
sary assumption of the economist.
1.A 2.C 3.E 4.B 5.A 6.B 7.C 8.E 9.C 10.D
That different banks give different rates of interest is also
Analysis
not relevant to the economist’s argument, and (D) is not a
1. The fear of the private electronic transmittal operators
necessary assumption for him.
is that, if the Postal Service enters their field, it will keep its
The argument of the author is that, because the govern-
electronic transmittal rates lower than theirs, thereby driv-
ment is insuring all deposits, the depositors do not have the
ing them out of business, because it can afford to subsidize
incentive or need to determine which banks are secure against
such lower rates from the profits that it is able to make out of
failure so that they would deposit their savings only in them.
the first-class mail on which it has a monopoly.
He obviously presumes that all depositors have sufficient
We have been asked to spot the choice which would allay
knowledge and access to information that will enable them to
this fear of the private electronic mail operators.
evaluate which banks are secure against failure and which
We should therefore look for that choice which says that,
banks are not. It is (E) which states this, and is the answer.
once the postal department offers electronic mail transmis-
sion, it will not be able to make a profit on first-class mail,
4. The economist’s argument is that one of the reasons for
and therefore no money will be available to it for subsidizing
the high rate of bank failures is the practice of the govern-
the former, implying that it cannot charge a rate less than
ment insuring all individuals’ bank deposits.
what private operators do.
In this question, we have been asked to locate that choice
(A) says just this (because the messages that are capable
which weakens this argument.
of being sent electronically are precisely those that are now
(A) will strengthen, and not weaken, the economist’s argu-
being sent by first-class mail.). So, if the Postal Service’s elec-
ment, and is not the answer.
tronic mail business is successful, its revenue from first-class
(B) states that, before the Government started insuring
mail will dwindle, and it cannot, therefore, use the latter to
deposits, frequent bank failures occurred as a result of de-
subsidize the former.
positors’ fears of losing money in bank failures. (This used to
So, (A) is the answer.
happen because, when the depositors apprehend, rightly or
wrongly, that a bank was becoming weak, all of them would
2. The question implies that the queries in four of the five
demand repayment of their deposits at the same time, thereby
given choices cannot be answered from the information given
causing a run on the bank and its consequential collapse.)
in the passage, and the query in only one of them can be an-
If this is true, the removal of such fear (through insurance
swered. We have been asked to spot that choice which can be
of their deposits by the government) would have removed one
so answered.
major cause of the banks’ frequent failures. In this case, the
You can easily see that none of (A), (B), (D) and (E) can be
insurance of the deposits by the government would have re-
answered from the information available in the given pas-
sulted in averting bank failures and not in causing them. So,
sage.
it is (B) which seriously weakens the economist’s argument,
From the second sentence of the given passage, we learn
and is the answer.
that the electronic transmittal operators believe that the Postal
(C) strengthens the economist’s argument, and does not
Service is presently making profits from the monopoly of first-
weaken it.
class mail, and apprehend that this profit will be used by the
(D) implies that the deposits of most of the depositors are
Postal Service to subsidize electronic transmission, thereby
insured by government. This will only strengthen the econo-
driving them out of business.
mist’s argument, and not weaken it.
So, the question in (C) can be answered in the affirmative
The argument of the economist is based on depositors’ at-
from the information available in the given passage. So, (C)
titudes, and not on the borrowers’. So, (E) is irrelevant to the
is the answer.
issue.
3. The argument in the narrative is that, because govern-
5. According to Roland, the fact that 90% of the people re-
ment has insured all individuals’ bank deposits, there is no
port that they know at least one person who is unemployed is
incentive for depositors to evaluate different banks regarding
‘alarming’, because it implies that unemployment is presently
the relative security of their deposits with them, and that this,
a very serious problem in the country.
in turn, has resulted in banks becoming lax in ensuring that
Sharon’s reply is that unemployment of 5% of the workers
they are secure. The economist therefore argues that insur-
(1 out of every 20) at any point of time may be considered as
ance by government of the depositors’ money has been partly
normal or moderate, and need not be taken as ‘alarming’. She
responsible for the high rate of bank failures.
adds that, if a person knows approximately 50 workers, it is
We have been asked to locate that choice which is one of
quite likely that as many as 5% of them (2.5) are unemployed
215
American Education Aids - LSAT - Logical Reasoning - Analysis
at any given time. So,(E) is the answer.
We have been asked to spot choice which can be arrived as None of the other choices is germane to the conclusion in
a conclusion based on Sharon’s argument. the second sentence of the passage.
Since the Sharon’s contention is that 2.5 out of every 50 9. (In this passage, ‘stock’ means the shares in company
are likely to be unemployed even under a normal and moder- equity, and not the ‘inventory’ of raw materials and finished
ate level of unemployment, she implies that the fact that 90% products.) Professor A says, “We must make a strong moral
of the people know at least one person who is unemployed is statement against Country X’s policies”, while Professor B
not an indication that unemployment is abnormally or alarm- says, “Our aim must be to encourage X to change its policies”.
ingly high. It is (A) which states this, and is the answer. So, they state different goals.
(B) is factually wrong, because that Sharon implies is just Again, while Professor A advocates ‘total divestment’,
the opposite as we have seen in (A). namely the sale of all stocks in companies that have factories
None of the other choices logically follows from Sharon’s or business offices in X, Professor B argues for ‘partial divest-
statement. ment’, namely the sale of stocks only of companies that either
sell goods to X’s government, or do the majority of their busi-
6. By stating that a normal, moderate level of unemploy- ness in X. So, they propose different ways of achieving the
ment is 5%, Sharon argues that one out of 20 workers is likely respective goals.
to be unemployed at any given time, which means that, if a So, (C) is the answer.
person knows approximately 50 workers, 5% of them are likely You can easily verify that the other choices are all inap-
to be unemployed at any given point of time. propriate.
We have been asked to spot that choice which forms a nec-
essary assumption by Sharon while advancing her argument. 10. A University buys stocks in companies with a view to
By stating that 5% is a normal level of unemployment, get dividend income from them. Professor A’s recommenda-
Sharon implies that any percentage higher than 5% can be tion is to totally ‘divest’ (meaning sell) the stocks in compa-
considered to be an abnormal level. She does not presume nies which have factories or business offices in country X.
that the percentage of 5% is never exceeded. So, (A) does not With the funds obtained from the sale of these stocks, the
follow from the passage. University will then buy stocks in other companies which
Sharon’s statement, ‘A normal, moderate level of unem- have no connection with country X.
ployment is 5%, with 1 out of 20 workers unemployed. So, at We have been asked to pick out that choice which would
any given time, if a person knows approximately 50 workers, ensure that the university would not be economically harmed
1 or more will very likely to be unemployed” implies that the by such divestiture.
percentage of 5% is uniform throughout the country. But, if Obviously, the university’s present dividend income will
there are some geographical regions in the country where the be protected if the financial return from the new stocks are
unemployment rate is presently 50% or above, the fact that not less than the financial returns from its current stocks.
the national average is only 5% would imply that there are a (D) is what states this, and is the answer.
number of other geographical areas where the unemployment
rate is usually much lower, say 0 to 1%. So, the ‘normal, mod- SET 3
erate’ level of unemployment in these areas should be con- Answers
sidered to be 0 to 1%, and not 5%. If this is so, then it is out of
1.C 2.B 3.C 4.E 5.E 6.B 7.A 8.B 9.D 10.C
every 100 workers (and not out of every 50), that one is likely
Analysis
to be unemployed during normal times. Sharon’s argument
1. The statistical finding is that children born in Hawaii
will be wrong in this case. So, for her argument to be valid,
have a longer average life-span than children born in Louisi-
she has to make the necessary assumption that unemployment
ana.
is not normally concentrated in certain geographical segments.
If this longevity is due to factors relating to geographical,
So, (B) is the answer.
atmospheric and ecological conditions in Hawaii, the child
None of (C), (D) and (E) is a necessary assumption for vali-
born in Hawaii to the newly-wed Louisiana couple can be
dating Sharon’s argument.
expected to have the prospect of the same longer life that chil-
dren born to permanent Hawaiian parents enjoy.
7. The first sentence says that if, among the private planes,
But, in this question, we have been asked to identify a
only those equipped with radar are allowed to use the air-
possible weakness in this argument.
space around centrally located airports, most of the private-
Therefore, the correct choice must be that statement which
plane traffic would be forced to use outlying fields.
attributes the longevity of Hawaiians to a factor which is not
From this statement, we can draw the ready conclusion
attributable to external conditions in Hawaii, but to some-
that most of the private planes presently using centrally lo-
thing inherent in the people who have always been living there.
cated airports are not equipped with radar now.
It is (C) which gives such a reason and is the answer, be-
So, (C) is the answer.
cause, if the longevity of the current population in Hawaii is
attributable to genetically determined factors, the conclusion
8. The conclusion in the second sentence of the passage is,
must be that Hawaiians would have the same average lon-
“Such a reduction in the amount of private plane traffic would
gevity even if they migrate to other States, and that non-
reduce the risk of midair collision around the centrally lo-
Hawaiians migrating to Hawaii will continue to be governed
cated airports”. From the information in the previous sen-
by the longevity related to their own genetic factors.
tence that most of the private planes presently using the cen-
trally located airports are not equipped with radar, we can
2. This question is the reverse of the last question, but the
draw the conclusion that, at present, private planes not
reasoning is the same. The argument which will strengthen
equipped with radar systems are the cause for a large number
the conclusion that the children of a Louisiana couple mi-
of midair collisions around centrally located airports.
216
American Education Aids - LSAT - Logical Reasoning - Analysis
grating to Hawaii will gain an advantage in longevity is the be seriously weakened. It is (E) which contradicts the author’s
one which states that external conditions favouring such lon- premise, and thereby weakens his conclusion. So, (E) is the
gevity are more abundant in Hawaii than in Louisiana. It is answer.
(B) which states this, and is the answer. You can easily see that the other choices are not related to
If (A) is true, it will foil the hope of the Louisiana couple, the correlation between a lawyer’s decision to advertise his
because the Hawaiian average may in due course fall below services and the fees charged by him.
the Louisiana average.
Even if (C) is true, the greater probability is that the chil- 5. Different companies even in the same industry spend
dren born to the Louisiana couple will be counted in the re- different percentages of their gross receipts towards promo-
maining 75 percent. So, we cannot say that (C) will automati- tional advertisements, though the overall average of the in-
cally strengthen the conclusion in the passage. dustry may be 10%.
(D) will also weaken the argument in the passage, because It is quite likely that one company spent 20% of its in-
it implies that the Louisiana couple would be better off by come, and induced 25% of the smokers of another brand (which
staying in Louisiana itself. had utilised only 5% of its receipts on advertisement) to switch
(E) will also weaken, and not strengthen, the given argu- over to its own brand. If this be the case, the general conclu-
ment, because it implies that the longevity of Hawaii resi- sion that ‘cigarette companies would have been no worse off
dents is attributable to the genetic factors, and not to the eco- economically if they had dropped their advertising’ would be
logical conditions in Hawaii. erroneous. So, the flaw in the argument is the assumption
that figures for the cigarette industry as a whole is applica-
3. (Most countries have imposed restrictions on advertise- ble to each company in the industry.
ment of legal services by lawyers. In the US, a lawyer is per- It is (E) which states this, and is the answer.
mitted to advertise the legal services rendered by him, pro-
vided that he simultaneously specifies in the advertisement 6. (A) deals with the economic welfare of magazine pub-
itself the fees that he will charge for such services. The first lishers, and not of cigarette companies, and is irrelevant to
sentence of the passage states that the lawyers who adver- the conclusion.
tise a specific service usually charge less for that service than If (B) is true, the stoppage of cigarette promotion will re-
lawyers who do not advertise. The passage argues that, if the sult in the dwindling of the number of smokers, thereby ad-
above-stated restriction is removed, and lawyers are allowed versely affecting the economics of the cigarette companies. So,
to advertise their legal services without having to mention (B) is the answer.
the fees they will charge for them, the litigant public will be (C) means that there is no conclusive research that ciga-
benefited.) rette advertising is not related to increases in smoking, and
In this question, we have been asked to spot the choice cannot therefore be chosen as the answer.
which will harmonize with the argument in the passage. (D) is wrong, because the question is not whether ciga-
(A) will result in the litigant public having to pay more for rette manufacturers are likely to drop the practice of adver-
legal services, and will therefore militate against the argu- tising or not, but whether stopping advertising will affect their
ment in the passage, and not harmonize with it. economics.
(B) implies that the liberalization of the rules relating to Since no information has been given of the percentage that
the advertisement of legal services will increase the income the sales of inexpensive brands of cigarettes contribute to the
of lawyers. It does not therefore strengthen the conclusion in total sales income in the cigarette industry, we cannot say
the passage that it will help the litigant public. So, (B) is not that (E) will most seriously weaken the conclusion in the pas-
the answer. sage.
(D) refers to the situation in which the current compul-
sion to state the fees in the advertisement continues to be in 7. The argument of the lobbyists is against legislation ban-
force, and is therefore not relevant to the argument in the ning smoking in workplaces on health grounds, and the rea-
passage. son given by them is that “there is no statistical evidence that
Since the author’s plea is for the removal of only some of breathing other people’s tobacco smoke increases the incidence
the restrictions on legal advertisements, and not all of them, of heart disease or lung cancer in healthy nonsmokers’.
(E) is not relevant to the conclusion in the passage. We have been asked to identify the choice which will
The passage first states that lawyers who advertise a spe- weaken the argument of the lobbyists that smoking in work
cific service usually charge less for their service than lawyers places should not be banned on health grounds.
who do not advertise, meaning that litigant public now pay (A) makes a definitive statement that breathing smoke-
more fees to the lawyers who do not advertise; later, it argues filled air increases the incidence of headaches and coughs in
that, if the compulsion to state the fees in the advertisement healthy non-smokers. If this is true, then, obviously, there is
is removed, the overall legal costs to the litigant public would justification for banning smoking in workplaces on health
be lower. These statements imply that the author believes grounds, and the argument of the lobbyists is weakened. So,
that the removal of the compulsion to state the fees in the (A) is the answer.
advertisement will result in more lawyers advertising their Since the “dislike” of nonsmokers to the odor of smoke-
services; and that, therefore, clients of more number of law- filled air is not an adequate ground for banning smoking on
yers will have to pay less fees, thereby bringing down the over- health grounds, (B) is not the answer.
all legal costs. It is (C) which states this, and is the answer. (C), obviously, is also not a sufficient justification for ban-
ning smoking on health grounds, and is not the answer.
4. The basic premise on which the author bases his conclu- (D) mentions the preference of most workers, and is not a
sion is that a lawyer who advertises his services charges a sufficient justification for banning smoking in work places
lesser fee than a lawyer who does not advertise his services. on health grounds.
If this premise is wrong, the conclusion of the author would (E) may be a reason for banning smoking in workplaces on
217
American Education Aids - LSAT - Logical Reasoning - Analysis
grounds of industrial peace, and would not justify such ban- ers. (For example, they could be due to an outbreak of war or
ning on health grounds. other production problems.) So, an increase in the size of the
oil-tanker fleet will not, by itself, help a nation to achieve
8. This question is also based on the argument of the lob- long term price stability. So, (B) is not the answer.
byists that there is no justification for the banning of smok- The question is about the steps to be taken to achieve long-
ing in workplaces on health grounds, because of the absence term stability in the price of oil. Such a policy must therefore
of statistical evidence that breathing other people’s tobacco operate even during periods when the international oil-prices
smoke increases the incidence of heart disease or lung can- are stable at low levels. Obviously, a country like US cannot
cer in healthy nonsmokers. suspend diplomatic relations with major oil-producing coun-
This question asks us to identify the best criticism of this tries during periods when the international oil prices are low!
argument. So, (C) is not the answer.
(A) is irrelevant to the argument of the lobbyists relating (E) will, in fact, make the nation even more vulnerable to
to the banning of smoking due to health reasons, and is not changes in international prices of oil, and will therefore go
the answer. against the stated objective. So, (E) is not the answer.
The lobbyists’ argument talks of the absence of statistical
evidence connecting inhalation of tobacco smoke with higher 10. The given passage states that, when international oil
incidence of heart disease or lung cancer in healthy nonsmok- prices rise, there is a corresponding increase in domestic oil
ers. The argument does not talk of the existence or otherwise prices of open-market countries even if they are self-sufficient
of statistical evidence relating to the adverse effect of inhala- in oil.
tion of tobacco smoke on nonsmoking workers who are not We have been asked to identify the choice which can be
healthy. considered to be a valid conclusion from this statement.
So, (B) is the best criticism against the argument of the If domestic oil prices in a nation rise or fall with interna-
lobbyists, and is the answer. tional oil prices, the logical conclusion is that its domestic oil
(C) is irrelevant to the argument relating to the banning of market is also closely linked to international oil market in
smoking in workplaces on health grounds, and is not the an- some way even if it neither imports or exports oil. It is (C)
swer. which says this, and is the answer.
Since the fact that nonsmokers inhaling smoke-filled air (A), in fact, contradicts the given statement, because ex-
may become concerned about their health cannot be a reason cluding domestic oil producers from international market will
for banning smoking in workplaces on grounds of health, (D) increase local oil supplies, and will therefore reduce its local
is not as strong an argument as (B) against the lobbyists’ prices.
contention. (B) and (D) clearly contradict the given statement, and are
(E) will in fact support, and not go against, the lobbyists’ not logical conclusions that can be drawn from it.
argument that there is no case for banning smoking in (E), apart from not being a logical conclusion that can be
workplaces on health grounds, and is not the answer. drawn from the given statement, is against commonsense
because, when international oil prices are higher than do-
9. The passage says that if international oil-supply is dis- mestic oil prices, the tendency of oil marketing companies in
rupted resulting in higher international oil prices, even do- open market nations will be to export more, and not to import
mestic oil prices in open-market countries will increase in more.
spite of the fact that they do not import any of the oil required
by them. SET 4
We have been asked to identify the choice which would be Answers
an effective policy in an open-market nation for reducing the
1.B 2.D 3.E 4.C 5.D 6.C 7.E 8.D 9.D 10.B
long-term economic impact of sharp and unexpected increases
Analysis
in international oil prices.
1. The first sentence of the narrative says that, if televi-
An “open-market nation” is one whose government does
sion editors did not exercise control over television commer-
not impose any restriction on either import or export of goods.
cials, many commercials would make exaggerated claims for
So, when the international oil prices rule high, oil producing
their products. The second sentence (starting with “After all
companies in such open-market nations would like to exploit
.....") seeks to substantiate this statement by pointing out
the situation, and sell much more oil in the international
that one third of the commercials reviewed each year by tel-
market, correspondingly reducing the supply of oil for local
evision editors are rejected and returned for revisions, because,
consumption. This will push up the price in the local market
in many cases, claims made in the advertisements are not
also, in spite of the fact that the nation does not import any
sufficiently documented.
oil. So, a long-term strategy for stabilizing oil price in such
We have been asked to identify that choice which consti-
nations will be to reduce the local consumption of oil to the
tutes a basic assumption behind the argument.
minimum, so that, even in times of high international oil
The evidence cited by the author is that many commer-
prices, local companies can meet the local needs fully, and
cials are returned by the television editors because the claims
still exploit the international situation. This would also en-
made in them are not sufficiently documented. The argument
able the nation to continue to rely on its own oil resources for
that he builds on this evidence is that, if there is no such
a longer period. It is (D) which states this, and is the answer.
review, many commercials would make exaggerated claims
(A) will contradict the intended purpose, because import-
for their products. What he obviously assumes is that claims
ing the same quantity of oil even during periods of high in-
are not sufficiently documented only because they are exagger-
ternational oil prices will push up the domestic market price,
ated. It is (B) which states this, and is the answer.
and not stabilize it.
The narrative says that “one-third” of the commercials re-
The passage does not say that international oil-supply in-
viewed are rejected each year, of which many are rejected be-
terruptions are caused only by the non-availability of oil tank-
218
American Education Aids - LSAT - Logical Reasoning - Analysis
cause the claims made in them are not sufficiently docu- nent’ is a person who propagates an idea) The argument of
mented. The narrative neither assumes nor implies that, the proponents of astral projection can be summarized as:
before the review was introduced, “most” commercials (mean- Astral projection is beneficial to social well-being because, in
ing “over 50%” of them) had made exaggerated claims for their areas where more people have taken to astral projection, crime-
products. So, (A) is not the answer. rates have come down.
(C) is also wrong because of the phrase “most of the claims The given argument connects two seemingly unconnected
made in commercials”, which is inconsistent with the fact phenomenon as a cause and its effect.
that only one-third of them are being rejected, whereas a It is choice (E) which has the same reasoning as above,
majority of two-thirds are still being accepted. namely, connecting two apparently unconnected phenomena
The argument is confined to the truth of the claims, but as a cause and its effect, and is the answer.
does not speak of whether the TV viewers believe the claims In all the other choices, the cause and the effect are closely
or not. So, (D) is not a necessary assumption for the author’s related to each other.
argument based on the evidence cited by him.
The second sentence of the narrative states that, of the 4. Assuming that the crime rate had come down in the
one-third commercials reviewed and returned each year, many areas mentioned by the author, it is quite likely that the rea-
(and not all) are returned because the claims made in them sons for such fall are totally different from an increase in the
are not sufficiently documented. This implies that at least number of persons practicing astral projection - such as (a) a
some others are returned because of other reasons (such as strengthening of the police machinery in that area or (b) an
obscenity, violence, poor production values, loud music etc.). improvement in the economic conditions of the people in that
(E), which implies that all of the commercials returned each area etc. The proponents not having considered any other
year by the review committee make exaggerated claims for explanation for the drop in the crime rate is the strongest ob-
their products contradicts the evidence cited, and is not the jection to their theory. So, (C) is the answer.
answer. You can easily see that none of the other choices consti-
tutes a strong objection to the theory of the proponent that
2. From the evidence that many of the commercials which the increase in the number of astral projectors in an area and
had been returned by the review committee had not suffi- the fall in the crime rate in that area are directly related as a
ciently documented the claims made in them, the author ar- cause and its effect.
gues that, if there were no such review, many commercials
would make exaggerated claims about their products. 5. It is quite likely that both B and C were copied inde-
In this question, we have been asked to identify that choice pendently from A, with one mistake having been made in B
which would most strengthen the author’s argument. and two mistakes having been made in C. I is therefore a
(A) merely implies that, while the average of returned com- possibility overlooked by the scholar.
mercials is only around one-third, it is nearly 50% in the case It is also likely that C was first copied from A incorporat-
of some of the advertising agencies. This only means that ing two mistakes in the process and then B was copied from C
some advertising agencies are more prone to exaggerations than correcting one of the mistakes. Thus, II is also a possibility
the others. Therefore, (A), by itself, neither strengthens nor overlooked by the scholar.
weakens the author’s argument, and is not the answer. III is not possible since, as per the given information, A
(B) says that the standards used by television editors for was written at least five years prior to C.
judging whether a commercial’s claims are sufficiently docu- Thus (D) is the answer.
mented have varied widely over a period of time. (The stand-
ards might have become either more strict or less strict dur- 6. Since the scholar’s logic was that a manuscript with two
ing this period.) This statement, by itself, neither strengthens mistakes must have been copied imperfectly from a manu-
nor weakens the author’s argument that, if there had been no script with one mistake, he would most probably conclude that
control by the review committee, many of the commercials a manuscript with three mistakes must have been copied
would make exaggerated claims for their products. So, (B) is imperfectly from a manuscript with two mistakes. So, (C) is
not the answer. the answer.
(C) merely cites one source from which the television edi-
tors get their information while reviewing the commercials, 7. It is easy to see that the author of this statement is
and neither strengthens nor weakens the author’s argument critical of the experts. The question is what his criticism is
that, if such review were not there, many commercials would based on.
make exaggerated claims for their products. So, (C) is not The assumption of the experts is that there is a surplus of
the answer. doctors in the country, and their conclusion (indicated by the
(D) says that many of the commercials that were rejected word ‘consequently’) is that the number of admissions in
and returned by the television editors were subsequently medical colleges must therefore be restricted, so as to main-
resubmitted to them with more moderate claims and were tain the high quality of medical care. We cannot infer from
accepted for broadcast in the revised form. This, if true, cer- this statement that the assumption and the conclusion are
tainly strengthens the author’s argument that, if there had the same. So, (A) is wrong.
been no control by television editors, the commercials would (B) is incorrect, since the author does not mention the
have been broadcast with the exaggerated claims that they names of the experts either directly or indirectly. There is no
had made initially. So, (D) is the answer. ‘personal’ attack of the experts by him, but he only questions
(E) would in fact weaken, and not strengthen, the author’s their conclusion.
argument that many commercials are returned because of It is quite obvious that, if the intake into medical colleges
exaggerated claims. So, (E) is not the answer. is restricted, the number of practicing doctors will progres-
3. (You need not worry about the meaning of ‘astral projec- sively decline. The author does not dispute this obvious in-
tion’. Apparently, it is some form of occult practice. A ‘propo- ference. So, (C) is also incorrect.
219
American Education Aids - LSAT - Logical Reasoning - Analysis
There is no ambiguity in the meaning of the word ‘re- of the proposed higher toll. If this happens, the funds required
stricted’, which obviously means reduced or curtailed. The by the Authority may not be collected in the time-span of 5
author’s criticism of the experts has no reference to this word. years that is projected, and may take a longer time. So, (D) is
So, (D) is not the answer. the answer.
In the last sentence, the author, while conceding that there
is indeed an overabundance of doctors in high-income areas, 2. The plan proposes to raise the required funds for the
mentions the fact (or introduces evidence) regarding the short- improvement of the rail line from road users, and the objec-
age of doctors in the low-income areas. In the light of this tion to the proposal is that it ‘would unfairly force drivers to
evidence, restricting the admission of students in medical absorb the entire cost of something from which they receive
colleges will further aggravate the shortage of doctors in the no benefit’.
low-income areas and, consequently, the quality of medical In this question, we have been asked to spot that choice
care in these areas will deteriorate still further. The aim of which can be the strongest counter-argument against this
the experts will therefore be defeated by their own remedy. objection.
Thus the evidence introduced by the author throws a new light We should therefore look for that choice which points out
on the theory of the experts. So (E) is the answer. that the improvement to the railway line is, in fact, of benefit
to the road users also.
8. The first sentence of the passage states the experts’ claim It is (D) which gives such an argument by saying that the
as “there is a surplus of doctors and that access to medical roads along the route served by the railway line are highly
schools must consequently be restricted so that the high qual- congested now and that the drivers would benefit when com-
ity of medical care could be maintained”. muters are diverted from these congested roadways to the
The obvious assumption of the experts that a surplus of mass transit railways, and is the answer.
doctors undermines the high quality of medical care.
So, (D) is the ready answer. 3. By pointing out that the wholesale prices of most illegal
None of the other choices is a necessary assumption for the drugs dropped substantially in 1987, the author of the pas-
experts’ conclusion. sage argues that the program to control the entry of illegal
drugs into the country was a failure in 1987.
9. The last sentence in the paragraph states that, by imi- We have been asked to spot the choice which is a necessary
tating the poisonous coral snake, the non-poisonous snake assumption made by the author while advancing this argu-
gets a protection that it would not otherwise have. This pro- ment.
tection must obviously be due to its enemies mistaking it for The price of a commodity can drop for either of the two
the poisonous variety and therefore avoiding it. So, (D) is the reasons: (i) increase in its supply; or (ii) decrease in the de-
answer. mand for it.
(A), (B) and (E) are obviously irrelevant to the passage. By basing his argument on the first of these two reasons,
(C) is wrong since the imitation is meant for protection the author is assuming that there was no decrease in demand
from enemies and not for misleading potential victims. for the illegal drugs during this period that could have caused
the reduction in their prices. So, (E) is the answer.
10. (A) is wrong since, according to the second sentence of
the passage, the coral snakes in the dry areas do not have red 4. As pointed out above, the author’s argument is that it
bands. was the entry of larger quantities of illegal drugs into the
(B) is the logical inference from the passage, since, if the country in 1987 that had led to a substantial fall in the prices
non-poisonous snake had to imitate the poisonous ones, the of these drugs. This argument will be most seriously weak-
characteristics of the poisonous ones must have developed ened if it is true that the domestic production of illegal drugs
earlier. So, (B) is the answer. had increased substantially in 1987 (to a level that had more
Neither (C) nor (D) necessarily follows from the given pas- than offset the fall in smuggled illegal drugs).
sage. It is (B) which says this, and is the answer.
(E) also need not be factually correct.
5. According to the author, a study comparing 217 million
SET 5 acres of common grazing land with 433 million acres of pri-
Answers vate grazing land showed that the common land was in bet-
ter condition, and that this falsifies Hardin’s claim that the
1.D 2.D 3.E 4.B 5.C 6.D 7.B 8.A 9.A 10.A
common grazing land will always be used less carefully than
Analysis
private grazing land.
1. The Transportation Authority hopes to raise all the re-
We have been asked to spot that choice the answer to which
quired funds for making improvements to the main commuter
will be most useful in evaluating the significance of the study
rail line by raising automobile tolls on the two highway bridges
with particular reference to Hardin’s claim.
along the rail route for a period of five years.
If the common grazing land had been in a far, far superior
This question asks us to spot the choice which will cast the
condition to the private grazing land before both of them were
most doubt on the success of the financial plan of the Author-
put to grazing use, the fact that the common grazing land
ity.
was in a ‘better condition’ than private grazing land at the
(D) says that, when the tolls were last increased on the
time of the study would prove Hardin’s point. But if both were
same two bridges, almost 20% of the regular commuter traf-
in the same condition before they were put to grazing use,
fic switched to a slightly longer alternative route. Because
but the common grazing land is now in a better condition,
that alternative route has been much improved since then,
then Hardin’s theory will be falsified.
even a higher percentage of drivers are likely to switch over
So, the answer to (C) is most useful in evaluating the sig-
to that alternative route now, thereby avoiding the payment
nificance of the study. So, (C) is the answer.
220
American Education Aids - LSAT - Logical Reasoning - Analysis
are not able to find alternative jobs and, therefore, the gov-
6. Had it been known to the ranchers that, if one user of ernment has to extend to them the unemployment benefits.
common land overuses it even slightly, the other users are It is (A) which says this, and is the answer.
likely to do so even more, with the consequence that the costs
to each user outweigh the benefits, then each of them would 9. The first sentence says that, in order to protect certain
have used the common grazing land even more carefully than new industries in the country, the government banned the
they use their own private grazing land, and this would ex- imports of the types of products which these industries were
plain why the common grazing land is in a better condition starting to make. The second sentence says that, as a conse-
than the private grazing land. quence, the export-dependent industries in the same country
It is (C) which says this, and is the answer. had to pay a higher price for these products.
The obvious inference that can be drawn from this infor-
7. The author points out that corporations which intro- mation is that it was cheaper to import these products than to
duce automated equipment in order to reduce manpower and make them within the country by the fledgling industries.
thereby save money will have to eventually pay for the gov- It is (A) which states this, and is the answer.
ernment unemployment assistance through increased taxes
and unemployment insurance premiums. 10. The last part of the passage says that the cost of these
It is (B) which summarizes this, and is the answer. products to the export-dependent industries went up, thereby
limiting the ability of these industries to compete effectively in
8. The author’s argument is that the government will be the export markets.
forced to extend financial assistance to those who are thrown The conclusion that can be drawn from this information is
out of employment through automation, and will have to raise that the profit margins in these industries were not high
taxes and unemployment insurance premiums for that pur- enough to enable them to absorb the rise in the cost thereby
pose. keeping their export prices at the same level as earlier and
This argument of the author will be weakened if the work- continue to compete successfully in the international mar-
ers who lose their jobs because of automation in their compa- ket.
nies are able to readily find alternative jobs elsewhere. Like- It is (A) which says this, and is the answer.
wise, what strengthens his argument is that these persons

221

Potrebbero piacerti anche